Lange Q&A Psychiatry: 11th Edition

4,218 700 66MB

English Pages 304 Year 2017

Report DMCA / Copyright

DOWNLOAD FILE

Polecaj historie

Lange Q&A Psychiatry: 11th Edition

Citation preview

ELEv ENTH EDITION



LA N G E q &A

PSYCHIATRY Sean M. Blitzstein, MD Staff Psychiatrist, Jesse Brow n VA Med ical Center Director, Psychiatry Clerkship Clinical Associate Professor of Psychiatry University of Illinois at Chicago Chicago, Illinois

N ew York Chicago San Francisco Athens Lond on Mad rid Mexico City Milan N ew Delhi Singap ore Syd ney Toronto

Copyright © 2017 by McGraw-Hill Education. All rights reserved. Except as permitted under the United States Copyright Act of 1976, no part of this publication may be reproduced or distributed in any form or by any means, or stored in a database or retrieval system, without the prior written permission of the publisher. ISBN: 978-1-25-964395-8 MHID: 1-25-964395-6. The material in this eBook also appears in the print version of this title: ISBN: 978-1-25-964394-1, MHID: 1-25-964394-8. eBook conversion by codeMantra Version 1.0 All trademarks are trademarks of their respective owners. Rather than put a trademark symbol after every occurrence of a trademarked name, we use names in an editorial fashion only, and to the bene t of the trademark owner, with no intention of infringement of the trademark. Where such designations appear in this book, they have been printed with initial caps. McGraw-Hill Education eBooks are available at special quantity discounts to use as premiums and sales promotions or for use in corporate training programs. To contact a representative, please visit the Contact Us page at www.mhprofessional.com. Notice Medicine is an ever-changing science. As new research and clinical experience broaden our knowledge, changes in treatment and drug therapy are required. The authors and the publisher of this work have checked with sources believed to be reliable in their efforts to provide information that is complete and generally in accord with the standards accepted at the time of publication. However, in view of the possibility of human error or changes in medical sciences, neither the authors nor the publisher nor any other party who has been involved in the preparation or publication of this work warrants that the information contained herein is in every respect accurate or complete, and they disclaim all responsibility for any errors or omissions or for the results obtained from use of the information contained in this work. Readers are encouraged to con rm the information contained herein with other sources. For example and in particular, readers are advised to check the product information sheet included in the package of each drug they plan to administer to be certain that the information contained in this work is accurate and that changes have not been made in the recommended dose or in the contraindications for administration. This recommendation is of particular importance in connection with new or infrequently used drugs. TERMS OF USE This is a copyrighted work and McGraw-Hill Education and its licensors reserve all rights in and to the work. Use of this work is subject to these terms. Except as permitted under the Copyright Act of 1976 and the right to store and retrieve one copy of the work, you may not decompile, disassemble, reverse engineer, reproduce, modify, create derivative works based upon, transmit, distribute, disseminate, sell, publish or sublicense the work or any part of it without McGraw-Hill Education’s prior consent. You may use the work for your own noncommercial and personal use; any other use of the work is strictly prohibited. Your right to use the work may be terminated if you fail to comply with these terms. THE WORK IS PROVIDED “AS IS.” McGRAW-HILL EDUCATION AND ITS LICENSORS MAKE NO GUARANTEES OR WARRANTIES AS TO THE ACCURACY, ADEQUACY OR COMPLETENESS OF OR RESULTS TO BE OBTAINED FROM USING THE WORK, INCLUDING ANY INFORMATION THAT CAN BE ACCESSED THROUGH THE WORK VIA HYPERLINK OR OTHERWISE, AND EXPRESSLY DISCLAIM ANY WARRANTY, EXPRESS OR IMPLIED, INCLUDING BUT NOT LIMITED TO IMPLIED WARRANTIES OF MERCHANTABILITY OR FITNESS FOR A PARTICULAR PURPOSE. McGraw-Hill Education and its licensors do not warrant or guarantee that the functions contained in the work will meet your requirements or that its operation will be uninterrupted or error free. Neither McGraw-Hill Education nor its licensors shall be liable to you or anyone else for any inaccuracy, error or omission, regardless of cause, in the work or for any damages resulting therefrom. McGraw-Hill Education has no responsibility for the content of any information accessed through the work. Under no circumstances shall McGraw-Hill Education and/ or its licensors be liable for any indirect, incidental, special, punitive, consequential or similar damages that result from the use of or inability to use the work, even if any of them has been advised of the possibility of such damages. This limitation of liability shall apply to any claim or cause whatsoever whether such claim or cause arises in contract, tort or otherwise.

Co nte nts

Contributors ............................................................................................................................................................v Preface ................................................................................................................................................................. vii Acknow ledgments ........................................................................................................................................... viii 1. Child and Adolescent Psychiatry ................................................................................................................. 1 Qu estions ........................................................................................................................................................... 1 Answ ers and Explanations ........................................................................................................................... 17 2. Adult Psychopathology ............................................................................................................................... 33 Qu estions ......................................................................................................................................................... 33 Answ ers and Exp lanations ........................................................................................................................... 58 3. Somatic Treatment and Psychopharmacology ........................................................................................ 81 Questions ......................................................................................................................................................... 81 Answ ers and Explanations ......................................................................................................................... 103 4. Psychological Treatment and Management .......................................................................................... 123 Qu estions ....................................................................................................................................................... 123 Answ ers and Exp lanations ......................................................................................................................... 132 5. Legal and Ethical Issues in Psychiatry and Medicine ......................................................................... 141 Qu estions ....................................................................................................................................................... 141 Answ ers and Exp lanations ......................................................................................................................... 154 6. D ifferential D iagnosis and Management ............................................................................................... 165 Qu estions ....................................................................................................................................................... 165 Answ ers and Explanations ......................................................................................................................... 185 7. Practice Test 1 .............................................................................................................................................. 205 Qu estions ....................................................................................................................................................... 205 Answ ers and Explanations ......................................................................................................................... 225

iii

iv

Conte nts

8. Practice Test 2 ............................................................................................................................................... 247 Qu estions ....................................................................................................................................................... 247 Answ ers and Explanations ......................................................................................................................... 268 Bibliography ...................................................................................................................................................... 287 Index .................................................................................................................................................................... 291

Co ntributo rs

Shetal M. Amin, MD Legacy Com m u nity H ealth Services Clinical Assistant Professor Departm ent of Psychiatry and Behavioral Sciences Baylor College of Med icine H ouston, Texas

Marika Inga Wrzosek, MD Director, Med ical Stu d ent Ed u cation Assistant Professor of Clinical Psychiatry Departm ent of Psychiatry and Behavioral Med icine Med ical College of Wisconsin Milw au kee, Wisconsin

Jennifer Sprague, MD Psychiatry Resid ent Departm ent of Psychiatry University of Illinois at Chicago Chicago, Illinois

Alexander K. Yuen, MD Child and Ad olescent Psychiatry Chief Fellow Institu te of Ju venile Research Departm ent of Psychiatry University of Illinois at Chicago Chicago, Illinois

Kelley A. Volpe, MD Child and Ad olescent Psychiatry Fellow Institu te for Ju venile Research Departm ent of Psychiatry University of Illinois at Chicago Chicago, Illinois

v

This page intentionally left blank

Pre fac e

Welcom e to Lange Q&A : Psychiatry, 11th ed ition. It is m y sincere p leasu re to p rovid e you w ith this ed ition in su p p ort of you r ongoing ed u cation in p sychiatry. Psychiatric research and p ractice are at the forefront of m ed icine, incorp orating p harm acology, p sychology, sociology, and neu rosciences; the resu lt is a genu ine biop sychosocial ap p roach to p atient care. Su ch a sizeable integration yield s a broad field in w hich to stu d y and test. Becau se of this, the 11th ed ition covers the gam u t of top ics, from ad u lt and child p sychiatry, to psychopharm acology and ethics. All m ajor d iagnoses are covered , w ith the em phasis being on d ifferential d iagnosis and treatm ent (w hether p sychological or som atic). In fact, this ed ition consists of eight chapters w ith 810 qu estions, and it has been u pd ated to reflect the Am erican Psychiatric Association’s Diagnostic and Statistical M anual of M ental Disorders, Fifth Edition (DSM -5). As this book is geared tow ard m ed ical stu d ents stu d ying for their Psychiatry clerkship exam inations, as w ell as for their United States Med ical Licensing Exam ination (USMLE) Step 2 CK exam ination, each and every qu estion is clinically focu sed com p rising of a p atient-centered vignette. Consistent w ith the USMLE form at, the answ ers are either m ultip le-choice or extend ed m atching item s. In ord er to p rom ote self-assessm ent and form ative learning, all answ ers inclu d e d etailed explanations regard ing why the incorrect choices are flaw ed . A list of references are also includ ed for those stud ents w ho w ish to further research a particu lar issue. Finally, a few su ggestions on how best to u se this book. This book shou ld be u sed as a su p p lem ent to you r p rim ary learning. It neither rep laces read ing a basic text on Psychiatry, nor seeing live p atients d u ring a rotation. Chap ters 1 throu gh 6 can be u sed to reinforce or com p lem ent certain areas. The final tw o chapters (Chapters 7 and 8) are practice tests, each consisting of greater than 115 rand om ly ord ered qu estions that sim u late an actu al board exam ination. Sean M . Blitzstein, M D

vii

Acknowle dg me nts

Thank you to m y p atient, su p p ortive, and organized ed itors at McGraw -H ill. I also sincerely thank m y contributors, and m y prior m entors and m entees, w ho have all given m e so m u ch. Finally, to Jackie and Anika, w ho are m y hom e. Sean M . Blitzstein, M D

viii

CHAPTER 1

Child and Ado le s c e nt Ps yc hiatry Que s tions

DIRECTIONS (Questions 1 through 75): For each of the multiple-choice questions in this section, select the lettered answ er that is the one best response in each case.

d u ring her p reviou s m ajor d ep ressive ep isod e, she d enies it at this visit. At this tim e, she also d enies self-inju rious u rges. Which of the follow ing w ou ld be the m ost ap propriate p lan for treatm ent?

Questions 1 and 2

(A) Ad m it her to the hosp ital given her history and potential risk. (B) Ask you r p sychiatric colleagu e to assess her at their next available intake. (C) Prescribe antid ep ressant m ed ications w ith a follow -u p ap p ointm ent in 2 w eeks. (D) Refer her to a social w orker for p sychotherapy. (E) Tell the m other you w ill follow u p w ith her at you r next rou tine visit.

A 15-year-old girl w ith a history of m ajor d epressive d isord er (MDD) com es to you r p rim ary care office for a rou tine visit. When you w alk into the exam ining room , you notice that she is w ithd raw n w ith her head bent d ow n. She d isplays p oor eye contact and barely speaks d uring you r interview . She ad m its that she u ses m arijuana ap proxim ately tw ice per w eek and cu ts on her arm w hen stressed . H er m other reveals that she is particu larly w orried as the girl’s paternal u ncle com m itted su icid e 10 years ago. You su spect that she is having a recu rrence of d ep ressive sym ptom s and are concerned about her risk for su icid e. 1. Which of the follow ing factors m ost increases this patient’s risk of com m itting suicid e? (A) (B) (C) (D) (E)

Cu tting behavior Gend er Presence of d ep ression Relative w ho com m itted su icid e Su bstance u se

2. Fu rther history is obtained , and the p atient reveals that she has, in fact, been feeling m ore d ep ressed recently, w ith d ifficu lty sleep ing, low ap p etite, fatigu e, and p roblem s concentrating. While she had su icid al id eation

Questions 3 and 4 A m other brings her 7-year-old son to you becau se she is w orried that he sits u p in bed in the m id d le of the night and scream s. She says that at those tim es he is inconsolable but eventually falls back to sleep. 3. Which of the follow ing is the m ost likely d iagnosis? (A) (B) (C) (D)

Interm ittent exp losive d isord er N arcolep sy N ightm are d isord er N onrap id eye m ovem ent (N REM) sleep arou sal d isord er, sleep terror typ e (E) N REM sleep arou sal d isord er, sleep w alking type

1

2

1: Child a nd Ad ole s c e nt P syc hia try

4. Du ring w hich stage of sleep d o these ep isod es m ost likely occur? (A) (B) (C) (D) (E)

Stage 1 Stage 2 Stages 3 to 4 Rap id eye m ovem ent (REM) stage Any stage

Questions 5 and 6 A 17-year-old girl w ith a history of asthm a p resents for a p hysical exam ination p rior to entering college. You note that she appears angry. Upon fu rther qu estioning, you learn that she has felt irritable for the past 6 m onths since breaking up w ith her boyfriend of 2 years. She says she feels tired all the tim e and com es hom e from school every d ay, lies on the cou ch, and w atches You Tu be vid eos. H er grad es have d rop p ed becau se she cannot concentrate. Despite her fatigue, she com plains of d ifficu lty sleeping. She has lost 12 lb over the last 6 m onths. She rep orts that she quit the senior celebration com m ittee, no longer “hangs ou t” w ith her friend s, cannot im agine things w ill im p rove, and is consid ering not going to college. H er p hysical and laboratory exam ination is norm al. 5. Which of the follow ing m ed ications w ou ld be the m ost app ropriate to treat this p atient? (A) (B) (C) (D) (E)

Carbam azep ine (Tegretol) Im ip ram ine (Tofranil) Lithiu m Olanzap ine (Zyp rexa) Sertraline (Zoloft)

6. Accord ing to the Diagnostic and Statistical M anual of M ental Disorders, Fifth Edition (DSM -5), w hich of the follow ing sym p tom s in this p atient d ifferentiate her d isord er from that in an ad u lt? (A) (B) (C) (D) (E)

Anhed onia Decreased concentration Insom nia Irritable m ood Weight loss

Questions 7 and 8 Parents bring their 6-year-old boy to the clinic at the requ est of the boy’s teachers. The teachers rep ort that he is quiet in class. When he d oes talk, he frequently m akes errors w ith verb tense. H is parents recall that his sp eech w as d elayed . On exam ination, the boy is friend ly and cooperative. H is speech is clear, but he u ses sim ple sentences w ith a lim ited vocabulary. Otherw ise, his physical and laboratory exam ination is norm al. 7. Which of the follow ing is the m ost likely d iagnosis? (A) Child hood -onset flu ency d isord er (B) Langu age d isord er (C) Social (p ragm atic) com m u nication d isord er (D) Sp ecific learning d isability (E) Sp eech sou nd d isord er 8. By w hat age w ou ld failu re to sp eak 200 w ord s be m ost consistent w ith a sp eech d elay in this patient? (A) (B) (C) (D) (E)

1 year 2 years 3 years 4 years 5 years

Questions 9 and 10 A 15-year-old boy w ithou t prior p sychiatric history is on his school concert trip. H e is brought to a local em ergency room becau se of the acu te onset of increased anger, agitation, and p aranoia. On interview , he reports feeling u nsafe because a governm ent agency is spying on him . 9. Which of the follow ing tests w ou ld be the m ost im portant to ord er first? (A) Electroencep halogram (EEG) (B) Glu cose tolerance test (C) Positron em ission tom ograp hy (PET) scan of his head (D) Thyroid fu nction tests (E) Urine toxicology screen

Que s tions : 4–15

10. All approp riate laboratory evaluations and stu d ies com e back w ithin norm al lim its. Which of the follow ing d iagnoses w ou ld be m ost consistent w ith this patient’s presentation? (A) (B) (C) (D) (E)

Anorexia nervosa Bip olar d isord er, m anic Bord erline p ersonality d isord er Generalized anxiety d isord er Major d ep ressive d isord er

Questions 11 and 12 An 8-year-old boy is referred to you by a school nu rse because he has been com p laining of stom achaches every m orning in school. On interview ing the boy’s m other, you learn that he d oes not like to go to school, insists on com ing hom e im m ed iately after school each d ay, and sleep s in his parents’ bed at night. The m other d enies other com p laints. 11. Which of the follow ing is the m ost likely d iagnosis? (A) (B) (C) (D) (E)

Post-trau m atic stress d isord er (PTSD) Reactive attachm ent d isord er Sep aration anxiety d isord er Social anxiety d isord er (social p hobia) Sp ecific p hobia

12. Which of the follow ing w ou ld you m ost likely find in this p atient’s history? (A) (B) (C) (D) (E)

Low levels of p arental control Parent w ith an anxiety d isord er Parents w ith a w arm and accep ting style Secu re attachm ent Tem p eram ent characterized by sociability and extroversion

Questions 13 and 14 A 16-year-old boy is brou ght to the p ed iatric em ergency d ep artm ent by his grand m other w ho rep orts he is not acting like him self. H e rep orts that he feels “incred ibly great” and d oes not need a d octor becau se he has p ow ers to heal him self of all sickness. H e sp eaks rapid ly and his grand m other rep orts he has not slept m ore than 6 hou rs over the p ast 3 d ays.

3

She rep orts that he ap pears extrem ely revved up and hyp eractive. 13. Which of the follow ing is the m ost likely d iagnosis? (A) Attention-d eficit/ hyp eractivity d isord er (ADH D) (B) Bip olar d isord er, m ost recent ep isod e m anic (C) Major d ep ressive d isord er w ith p sychotic featu res (D) Panic d isord er (E) Schizop hrenia 14. Ingestion of w hich of the follow ing su bstances w ou ld m ost likely bring abou t sim ilar sym ptom s? (A) (B) (C) (D) (E)

Alcohol Cannabis Cocaine H eroin Phencyclid ine (PCP)

15. A 9-year-old boy w alks into you r office accom panied w ith his mother. They are argu ing abou t his w anting a new portable vid eo gam ing system . H is m other, exasp erated w ith her son’s behavior, tells you that they w ere late becau se it took him a long tim e to finally agree to get into the car to come to the appointm ent w ith you. Alone w ith you in you r office, he appears irritated and refu ses to answ er your questions or look up from his lap. After 10 minutes, he tells you about the “annoying kid s” in his class and how they “mad e m e get in trouble.” Fu rther history from the m other reveals that, d espite the above behavior, he has never been violent or d estructive, and he has not had any legal problems. Which of the follow ing is the m ost likely d iagnosis? (A) Agorap hobia (B) Attention-d eficit/ hyp eractivity d isord er (ADH D) (C) Cond u ct d isord er (D) Generalized anxiety d isord er (GAD) (E) Op p ositional d efiant d isord er (ODD)

4

1: Child a nd Ad ole s c e nt P syc hia try

Questions 16 through 18 A 15-year-old girl w ho is a com p etitive figu re skater presents w ith concerns about her w eight. She believes that she w ou ld be a better skater if she cou ld lose w eight and feels very u p set and fru strated that she has failed in her attem pts. The girl relu ctantly ad m its that she som etim es eats “a w hole lot” of food at one tim e su ch as a qu art of ice cream , a large bag of potato chips, and a jar of peanu t butter. You also notice abrasions on the back of her right hand . 16. Which of the follow ing opening statem ents w ou ld be the m ost ap p rop riate? (A) “Tell m e abou t the scratches on you r hand .” (B) “I’ve noticed the cu ts on you r hand . Are you trying to hu rt you rself?” (C) “H ow d id the scratches hap p en?” (D) “I see you have scratches on you r hand . Do you have a cat?” (E) “Som etim es I see girls w ho m ake them selves throw up. H ave you ever d one that?” 17. Which of the follow ing laboratory abnorm alities w ould you m ost likely find in this patient? (A) (B) (C) (D) (E)

Elevated iron Elevated p rotein H yp erchlorem ia H yp okalem ia H yp onatrem ia

18. Which of the follow ing typ es of p sychotherap y w ould likely be the m ost effective for this p articular p atient? (A) (B) (C) (D) (E)

Cognitive-behavioral therap y Fam ily therap y Grou p therap y Psychoanalysis Psychod ynam ic p sychotherap y

Questions 19 through 21 An 8-year-old boy w ith a history of major depressive d isorder (MDD) treated w ith fluoxetine (Prozac) is brought to the emergency department after running

into the street in front of a car on the w ay home from school. H e is physically unharmed , but refused to speak to the emergency room d octor. You are asked to consult as you are on your psychiatry rotation. The emergency room doctor is suspicious that the boy’s behavior reflected und erlying suicid al impulses. 19. Which of the follow ing op ening statem ents w ou ld be the m ost valu able in facilitating the boy’s d iscu ssion of the situ ation? (A) “You su re w ere lu cky the car sw erved at the last m inu te.” (B) “You w eren’t trying to actu ally get hit by the car, w ere you ?” (C) “What were you thinking w hen you ran into the street?” (D) “It sou nd s like qu ite a d ay. Can you tell m e abou t w hat hap pened after school tod ay?” (E) “Child ren w ho try to hu rt them selves are very confu sed . Are you confu sed ?” 20. Which of the follow ing sym p tom s of MDD w ou ld be m ore likely in this p atient com p ared to an ad olescent w ith MDD? (A) (B) (C) (D) (E)

Dru g u se H op elessness H yp ersom nia Psychom otor agitation Weight change

21. Which of the follow ing w ou ld be this child ’s m ost likely m ethod of attem p ting suicid e? (A) (B) (C) (D) (E)

Firearm s H anging Ju m p ing from a significant height Stabbing Su bstance ingestion

22. An 11-year-old boy w ith enu resis presents to the clinic for rou tine follow -up. H is bed w etting had not resp ond ed to behavioral interventions, so you had p reviou sly initiated treatm ent w ith intranasal d esm opressin (Dd avp ) after com p letion of a fu ll p hysical and laboratory exam ination. Which of the

Que s tions : 16–27

follow ing signs/ sym ptom s w ou ld be the m ost likely ad verse effect? (A) (B) (C) (D) (E)

H ead ache H ypotension Liver toxicity Sed ation Trem or

Questions 23 and 24 A 9-year-old boy w ith a history of p anic d isord er treated w ith cognitive-behavioral therapy is brought to your office by his m other becau se he has been irritable and d ep ressed . On p hysical exam ination, the boy appears d epressed bu t otherw ise norm al. Laboratory exam ination is norm al. 23. What w ou ld be the likelihood of this p atient having a com orbid m ajor d ep ressive d isord er (MDD)? (A) (B) (C) (D) (E)

5% 15% 25% 50% 75%

24. After a thorou gh history and m ental statu s exam ination, you d iagnose the boy w ith MDD and d ecid e to initiate treatm ent w ith flu oxetine. You inform the boy and his m other of possible ad verse effects of flu oxetine. Which of the follow ing w ou ld be the m ost likely sid e effect? (A) (B) (C) (D) (E)

H yp otension Liver toxicity N au sea Sed ation Weight gain

Questions 25 and 26 A 30-month-old girl is brou ght to the clinic by her m other for a rou tine visit. The m other tells you that the girl seem s to avoid affection, often d oes not look her in the eye, has stopp ed speaking in preschool, and d oes not really engage w ith her p reschool peers nor w ith her 5-year-old brother. She continu es to

5

m ake od d , rep etitive m ovem ents w ith her hand s, and p refers to line her toys u p . She becom es acu tely d istressed w hen rou tines are changed . The child ’s physical exam ination is u nrem arkable, bu t you note that she d oes not seem to cry w hen she trip s over you r exam ination stool, keep ing her attention on aligning you r colored p encils. 25. Which of the follow ing is the m ost likely d iagnosis? (A) (B) (C) (D) (E)

Au tism sp ectru m d isord er (ASD) Intellectu al d isability (ID) Rett synd rom e Selective m u tism Social (p ragm atic) com m u nication d isord er

26. The m other asks you abou t p rognosis for her d au ghter ’s cond ition. Which of the follow ing is the m ost p ositive p rognostic factor? (A) (B) (C) (D) (E)

Com orbid ep ilep sy Engagem ent in social skills training Fu nctional language by 5 years of age Intellectu al d isability Presence of catatonic sym ptom s

Questions 27 and 28 A 12-year-old boy is referred by the cou rt for evalu ation. H e skip s school, stays ou t late at night, and verbally abu ses his parents. H e has run aw ay from hom e on three sep arate occasions, p rom p ting his p arents to call the p olice. H e has been cau ght shoplifting and has been in nu m erou s p hysical fights w ith his p eers. 27. Up on fu rther history, w hich of the follow ing w ou ld m ost likely be fou nd in this p atient? (A) (B) (C) (D) (E)

Absence of a biological father Absence of a biological m other Mother w ith an anxiety d isord er Patient being an only child Parents w ho d o not u se corp oral p u nishm ent

6

1: Child a nd Ad ole s c e nt P syc hia try

28. Which of the follow ing p ersonality d isord ers is this boy m ost likely to d evelop ? (A) (B) (C) (D) (E)

Antisocial p ersonality d isord er Avoid ant p ersonality d isord er Paranoid p ersonality d isord er Schizoid p ersonality d isord er Schizotyp al p ersonality d isord er

Questions 29 and 30 A 10-year-old girl w ho has recently been d iagnosed w ith d iabetes m ellitu s typ e I is referred to you by her ped iatrician for an evalu ation. You notice that she seem s sad . H er parents are concerned about her being d ep ressed . Consid eration is given for d iagnosing ad justm ent d isord er w ith d epressed m ood versu s m ajor d epressive d isord er (MDD). 29. Which of the follow ing criteria for the d iagnosis of ad ju stm ent d isord er m ost d istinguishes it from MDD? (A) Sym p tom s cau se m arked d istress or significant im pairm ent in fu nctioning. (B) Sym p tom s d evelop follow ing an id entifiable stressor. (C) Sym p tom s d evelop w ithin 3 m onths of the onset of the stressor. (D) Sym p tom s d o not p ersist for m ore than 6 m onths follow ing term ination of the stressor. (E) Sym p tom s d o not rep resent norm al bereavem ent. 30. Approxim ately w hat p ercentage of child ren w ho are d iagnosed w ith d iabetes m ellitu s typ e I d evelop ad justm ent d isord er follow ing their m ed ical d iagnosis? (A) (B) (C) (D) (E)

1% 5% 10% 33% 75%

Questions 31 and 32 A 9-year-old boy is referred to you for evaluation because of repeated teasing at school related to his inappropriate peer interactions. The teachers report

that at any time, w ithout w arning, the boy w ill make a d isruptive sound or shout out in class. They d escribe him as polite and neat but restless and jumpy. 31. Which of the follow ing is the m ost likely d iagnosis? (A) (B) (C) (D) (E)

Cond u ct d isord er Op p ositional d efiant d isord er Panic d isord er Sep aration anxiety d isord er Tou rette d isord er

32. Which of the follow ing m ed ications w ou ld be the m ost appropriate to prescribe initially? (A) (B) (C) (D) (E)

Bu p rop ion (Wellbu trin) Clonid ine (Catap res) H alop erid ol (H ald ol) Paroxetine (Paxil) Venlafaxine (Effexor)

Questions 33 and 34 A 7-year-old boy w ith leukem ia is referred to you becau se of concerns abou t his m ood . H is p arents rep ort that he flu ctu ates betw een app earing d epressed and acting angry. At tim es, he plays quietly in his room , bu t at other tim es he d isp lays anger outbu rsts, often hitting his 4-year-old brother. H is m other ad m its that she has d ecreased her expectations of him , and feels that since he is ill he should not receive any p u nishm ents. 33. Which of the follow ing m ethod s w ould be the m ost effective w ay to engage his m other in a d iscussion regard ing the role of her actions on the boy’s behavior? (A) Acknow led ge her gu ilt and anxiety abou t her son’s illness and explain the im p ortance of p rovid ing lim its and stru cture for his em otional w ell-being. (B) Em p athize w ith the trau m a of having a sick child . (C) Refer her to a p arent su p p ort grou p . (D) Sit qu ietly and m ake no com m ents abou t her p arenting style. (E) Tell her that treating her son like a baby is hu rting him em otionally.

Que s tions : 28–38

34. It is d eterm ined that the boy is su ffering from m ajor d epressive d isord er. You d iscu ss both p sychopharm acologic and psychotherapeu tic treatm ent options, but she is concerned abou t his ongoing leukem ia treatm ent. Which of the follow ing approaches w ou ld be the m ost ap propriate regard ing treating both his leu kem ia and m ajor d epression? (A) Treat the d ep ression p rior to the leu kem ia. (B) Treat the d ep ression after the leu kem ia. (C) Treat the d ep ression concu rrently w ith the leu kem ia. (D) Treat the leu kem ia first and the d epression w ill resolve. (E) Treat the leu kem ia as treating the d epression w ill not be effective in the setting of a m alignancy. Questions 35 and 36 An 8-year-old boy p resents to you r office for a rou tine visit. One m onth earlier, you d iagnosed him w ith Tou rette d isord er and p rescribed m ed ication. H e and his p arents rep ort that the m ed ication has been help fu l. 35. Which of the follow ing d isord ers w ou ld this p atient be m ost likely to d evelop ? (A) (B) (C) (D) (E)

Au tism sp ectru m d isord er Bip olar d isord er Langu age d isord er Obsessive-com p u lsive d isord er (OCD) Sep aration anxiety d isord er

36. Prior infection w ith w hich of the follow ing w ou ld be m ost likely fou nd in the history of this patient? (A) (B) (C) (D) (E)

Haemophilus influenzae Influ enza viru s Parainflu enza viru s Resp iratory syncytial viru s Streptococcus pyogenes

7

Questions 37 and 38 An 8-year-old boy w ith a family history of tic d isord ers is referred to you for an evaluation of behavioral d ifficulties in school. H is teachers report that he is unable to sit still, constantly fid gets, and is unable to complete class w ork becau se he is so easily d istracted . When he com pletes his w ork, it is often d one carelessly and is frequently not tu rned in. The teachers also note that he blurts ou t replies and has a hard tim e d u ring quiet tim e. The boy’s m other reports that he has alw ays had a lot of energy. Preparing to leave for school in the m orning is challenging because of her son’s d isorganization and forgetfulness. Otherw ise, she has no complaints. She d enies that her son prod uces any repetitive m ovem ents or sou nd s. 37. Which of the follow ing is the m ost likely d iagnosis? (A) Attention-d eficit/ hyp eractivity d isord er (ADH D) (B) Bip olar d isord er (C) Cond u ct d isord er (D) Op p ositional d efiant d isord er (E) Unsp ecified d isru p tive behavior d isord er 38. The p atient is su bsequ ently treated for the above cond ition. H e retu rns to an app ointm ent after several w eeks, now w ith rep etitive grim acing and blinking m ovem ents, w hich have resu lted in his getting teased in school. Which of the follow ing classes of m ed ications is m ost likely to be resp onsible? (A) (B) (C) (D)

Benzod iazep ines D 2 antagonists Monoam ine oxid ase inhibitors (MAOIs) Selective serotonin reu p take inhibitors (SSRIs) (E) Stim u lants

Questions 39 and 40 A 6-year-old boy is referred to you by his school to evalu ate his d ifficu lty w ith keep ing u p w ith read ing and m ath d esp ite his above average intelligence. It is su sp ected that he su ffers from a learning d isord er, so further testing and evaluation is ind icated .

8

1: Child a nd Ad ole s c e nt P syc hia try

39. Which of the follow ing find ings w ou ld be required for a d iagnosis of sp ecific learning d isord er w ith im p airm ent in read ing? (A) The child has an above average IQ (intelligence qu otient) bu t below average read ing achievem ent. (B) The child has an average IQ and below average read ing achievem ent. (C) The child ’s read ing achievem ent is substantially below the child ’s IQ. (D) The child ’s read ing achievem ent is substantially above the child ’s IQ. (E) The child ’s read ing achievem ent and IQ are both below average. 40. What w ou ld be the ap p roxim ate risk of this child having a com orbid psychiatric d isord er? (A) (B) (C) (D) (E)

5% 10% 25% 50% 75%

Questions 41 and 42 An 8-year-old child is referred to you for an evaluation of bed -w etting. Several behavioral interventions have been attem pted , inclu d ing eliminating fluid intake in the evening, sched uled aw akenings at night to use the bathroom, and a urine alarm (a bell and pad ). These techniqu es have been u nsu ccessful, and the child continues to u rinate in bed every night. 41. What is the likelihood of this p atient having a com orbid m ental illness? (A) (B) (C) (D) (E)

5% 10% 20% 50% 75%

42. Which of the follow ing m ed ications w ou ld be the m ost ap propriate to prescribe to treat this p atient? (A) Benztrop ine (Cogentin) (B) Desm op ressin (Dd avp )

(C) Methylp henid ate (Ritalin) (D) Paroxetine (Paxil) (E) Trazod one (Desyrel) Questions 43 through 45 A 10-year-old boy is referred to you becau se his ped iatrician su spects that he m ay have attentiond eficit/ hyperactivity d isord er (ADH D). After a thorou gh history, physical exam ination, and laboratory investigation you m ake the d iagnosis of ADH D. After d iscu ssing the ad verse effects of m ed ications, you p rescribe m ethylp henid ate to be taken in the m orning and at lu nch on school d ays. 43. Which of the follow ing w ou ld be the m ost likely sid e effect of m ethylp henid ate? (A) (B) (C) (D) (E)

H yp otension Insom nia Liver toxicity Trem or Weight gain

44. What is the likelihood that this patient w ill significantly benefit from the m ethylp henid ate? (A) (B) (C) (D) (E)

10% 25% 33% 50% 70%

45. Despite education and reassurance, the mother remains opposed to stimulant medications but still w ishes her son to receive pharmacologic treatment for his ADH D. Which of the follow ing med ications w ould be the most appropriate to prescribe? (A) (B) (C) (D) (E)

Arip ip razole (Abilify) Atom oxetine (Strattera) Citalop ram (Celexa) Mixed am p hetam ine salts (Ad d erall) Valp roic acid (Dep akote)

Questions 46 and 47 Du ring a rou tine office visit, the m other of a 37-m onth-old girl tells you that she is concerned

Que s tions : 39–51

abou t her d au ghter’s behavior. Since the birth of her son 4 m onths earlier, the m other states that her d aughter has been m ore irritable and angry. The child has told her m other that she d oes not w ant the baby anym ore and to take him back. The m other is esp ecially concerned becau se her d au ghter tried to bite the baby the w eek before. 46. Which of the follow ing statem ents w ou ld be the m ost approp riate response to this m other? (A) “If you sim p ly ignore you r d au ghter’s behavior, it w ill p ass.” (B) “It is u nd erstand able that you r d au ghter is angry and exp eriences jealou sy w ith the new baby joining the fam ily.” (C) “The next tim e she tries to bite him , you should bite her back so she know s how it feels.” (D) “Tell you r d au ghter she is being a very bad girl and you w on’t love her if she bites the baby.” (E) “Tell you r d au ghter that she need s to love the baby and be a w ond erfu l big sister.” 47. Which of the follow ing tasks w ou ld this girl be able to p erform at her cu rrent age? (A) Acknow led ge her angry and com petitive feelings tow ard her sibling. (B) Be able to state her age and gend er. (C) Cou nt to 50. (D) Recognize that w ater p ou red from one glass into another of a d ifferent size is the sam e volu m e. (E) Rid e a bicycle. Questions 48 and 49 A 4-year-old boy is referred to you for evalu ation becau se he has p oor social related ness. Up on interview , he appears healthy and w ell-kem pt. H e grabs you r office key off you r d esk. At you r office d oor, he takes the key and locks and u nlocks you r d oor rep eated ly. Desp ite attem p ts to red irect and d istract him , he rem ains preoccu pied w ith this task. After abou t 10 m inu tes, you attem p t to take the key aw ay from him and he becom es extrem ely u p set, m aking an insistent, p iercing cry.

9

48. In which of the following areas would you most likely expect additional difficulties in this patient? (A) (B) (C) (D) (E)

Attention Fine m otor skills Gross m otor skills Im aginary p lay Potty training p rogress

49. Which of the follow ing qu alities w ou ld be m ost associated w ith a m ore favorable p rognosis in this child ? (A) (B) (C) (D) (E)

Easily toilet trained Interested in m echanical toys Organized in p lay Recip rocal conversation Reciting songs and p oem s from m em ory

Questions 50 and 51 A 17-year-old girl com es to you r office for a rou tine visit. She states that she feels fine and offers no com p laints. On p hysical exam ination, you find that her w eight is 92 lb and her height is 65 in. One year earlier, her w eight w as 126 lb, and height 65 in. After fu rther d iscu ssion, you learn that she is terrified of gaining w eight and believes that she is fat and need s to lose m ore w eight. She ad d itionally reports that she has not m enstru ated in the past 6 m onths. 50. Which of the follow ing laboratory abnorm alities are you m ost likely to find in this p atient? (A) (B) (C) (D)

H yp ercholesterolem ia H yp erkalem ia H yp ocarotenem ia Increased thyroid -stim u lating horm one (TSH ) (E) Leu kocytosis

51. Which of the follow ing com p lications w ould be the m ost likely ind ication for ad m itting this p atient to the hosp ital? (A) (B) (C) (D) (E)

Anem ia Arrhythm ia Brad ycard ia H yp otension Lanu go

10

1: Child a nd Ad ole s c e nt P syc hia try

Questions 52 and 53 An 8-m onth-old boy is brought to the clinic by his m other. She com plains that her son has been experiencing scream ing and crying fits w hen she leaves him w ith a babysitter. She says that in the p ast he d id not object to being left w ith a babysitter and asks you w hy he becom es so u p set now , and w hat she can d o about it. 52. Which of the follow ing statem ents w ou ld be the m ost app ropriate response? (A) “This behavior is characteristic of au tistic child ren.” (B) “It is p ossible that you r son has separation anxiety d isord er.” (C) “This behavior su ggests that you ’re not spend ing enou gh tim e w ith you r son.” (D) “It sou nd s as thou gh you r son is overly attached to you .” (E) “This behavior is norm al at you r son’s age and w ill p ass w ith tim e.” 53. Which of the follow ing d iagnoses w ou ld be m ost ap prop riate if the boy in the p reviou s qu estion (Qu estion 52) w ere an 8-year-old boy w ith sim ilar behavior? (A) (B) (C) (D) (E)

Agorap hobia N orm al behavior Obsessive-com p u lsive d isord er Sep aration anxiety d isord er Social anxiety (social p hobia)

Questions 54 and 55 A 10-year-old boy is referred to you d u e to extrem e d ifficulties in school. H e has been held back a grad e and is still not p assing his classes. Du ring the cou rse of your evalu ation, you learn that the boy hears voices telling him that he is stupid and to leave the classroom . Afraid to d isobey, he goes to the bathroom frequ ently. H e also has d ifficu lty falling asleep at night becau se the voices keep him aw ake. In ad d ition, you learn that the boy believes others can read his thou ghts. Physical and laboratory exam inations are norm al. You su sp ect that the boy m ay be su ffering from schizop hrenia.

54. Which of the follow ing item s in his history w ou ld be m ost consistent w ith you r p rovisional d iagnosis? (A) Alw ays social and ou tgoing (B) Father w ith schizotyp al p ersonality d isord er (C) Parents getting d ivorced (D) Recently transferred schools (E) Recently u sed m ariju ana 55. Which of the follow ing featu res w ou ld ind icate a poorer p rognosis in this patient? (A) (B) (C) (D) (E)

Acu te onset Affective sym p tom s Good p rem orbid ad ju stm ent Onset before the age of 13 years Well-d ifferentiated sym p tom s

Questions 56 through 58 A 9-year-old girl w ith a fam ily history of bipolar d isord er is referred to you by her school because of d isruptive behavior in class that has been w orsening over the past 3 m onths. H er teachers report that her energy level is high, and she is m arked ly more d istractible. She is also sleeping more poorly and is increasingly m ore intrusive into her siblings’ personal space. Both attention-d eficit/ hyp eractivity d isord er (ADH D) and a m anic episod e are consid ered . 56. Which of the follow ing sym p tom s w ou ld be m ore consistent w ith ADH D rather than m ania? (A) (B) (C) (D) (E)

Distractibility Im p u lsivity Low self-esteem Motoric hyp eractivity Pressu red sp eech

57. If this child ’s school is u nable to m anage her behavior in her classroom d esp ite ou tp atient treatm ent and m ed ication, w hich of the follow ing long-term op tions w ou ld be the m ost optim al school placem ent? (A) H om e schooling (B) Parochial school

Que s tions : 52–63

(C) School for child ren w ith learning d isabilities (D) Resid ential treatm ent (E) Therap eu tic d ay school 58. Further history and evaluation over time result in the d iagnosis of bipolar d isord er, most recent episod e manic. Which of the follow ing med ications w ou ld be the m ost appropriate treatm ent? (A) (B) (C) (D) (E)

Bu p rop ion (Wellbu trin) Du loxetine (Cym balta) Methylp henid ate (Ritalin) Mixed am p hetam ine salts (Ad d erall) Valp roic acid (Dep akote)

Questions 59 and 60 An 8-year-old boy is brou ght to you r office by his m other for evaluation of an u pp er resp iratory infection. The m other m entions that her son has started w etting the bed again. In ad d ition, she m entions that the boy’s grand m other d ied recently and w ond ers if this is affecting him . 59. At w hich of the follow ing ages w ou ld a child norm ally be able to ap preciate that d eath is irreversible? (A) (B) (C) (D) (E)

2 years 3 years 5 years 7 years 12 years

60. Which of the follow ing d efense m echanism s is the boy m ost likely em p loying w hen he is w etting the bed ? (A) (B) (C) (D) (E)

Acting ou t Denial Regression Repression Som atization

11

Questions 61 and 62 A 12-year-old boy w ith Tou rette d isord er com es to you r office for a rou tine visit. Tw o w eeks earlier, you had p rescribed clonid ine for his illness. The boy rep orts that his tics have su bsid ed slightly since starting the clonid ine, bu t he com plains abou t the m ed icine. 61. Which of the follow ing ad verse effects is this boy m ost likely exp eriencing? (A) (B) (C) (D) (E)

Dry m ou th H yp otension N au sea Sed ation Trem or

62. The p arents bring in the boy’s 7-year-old brother for evalu ation. After fu rther history is obtained , he is d iagnosed w ith attentiondeficit/ hyperactivity d isord er (ADHD). Which of the follow ing classes of med ications w ould be the most appropriate choice for the brother? (A) Antip sychotic (B) Monoam ine oxid ase inhibitor (MAOI) (C) Serotonin-sp ecific reu p take inhibitor (SSRI) (D) Stim ulant (E) Tricyclic antid epressant (TCA) 63. A 6-year-old boy is brou ght to the em ergency d ep artm ent by his m other, w ho reports that he w as p laying on som e step s in front of the hou se w hen he slip p ed and fell. She tells you that she is concerned that he m ight have broken his arm . An x-ray of the boy’s arm show s a fracture of the u lna, as w ell as signs of several old fractures of varying ages. Which of the follow ing is the m ost app rop riate cou rse of action? (A) Recom m end calcium su pp lem ents and a m ultivitam in d aily. (B) Refer the boy to an orthoped ist for fu rther evalu ation. (C) Set the cu rrent broken bone in a cast and have the boy see his p ed iatrician for follow -u p care.

12

1: Child a nd Ad ole s c e nt P syc hia try

(D) Tell the boy that you notice that he has had m u ltip le broken bones and ask him how each of these fractures happened . (E) Tell the m other that you notice that the boy has had m ultiple broken bones and recom m end that she lim it the boy’s sports activities. 64. Th e m oth er of a 6-year-old boy calls an d asks you for ad vice. She says th at h er son still su cks his th u m b, an d sh e is concerned abou t th is beh avior. Wh ich of th e follow in g su ggestion s for h er to d o is th e m ost ap p rop riate? (A) Ask the d entist to construct a m outh ap p liance that w ill d eter su cking. (B) Coat her son’s thum b in hot pepper sauce. (C) Give him gu m frequ ently. (D) Ignore the behavior. (E) Im p lem ent a behavioral system to rew ard stop p ing. Questions 65 and 66 A 14-year-old girl presents to her pediatrician complaining that she has been “freaking out.” The girl describes episodes of shaking, gasping for air, and feeling like she is going to die. The feelings intensify for a few minutes and resolve spontaneously. These episodes have occurred at various times, in various situations, and the girl is worried that she is going crazy. A complete history and physical examination does not reveal any further relevant symptoms or signs. 65. Which of the follow ing is the m ost app rop riate p harm acologic treatm ent? (A) (B) (C) (D) (E)

Arip ip razole Carbam azep ine Du loxetine Sertraline Valp roic acid

66. Prior to prescribing m ed ications, w hich of the follow ing shou ld the p ed iatrician ord er next? (A) Electrocard iogram (ECG) (B) Electroencep halogram (EEG)

(C) N eu rology consu lt (D) Pu lm onary function tests (E) Routine laboratory stu d ies 67. A 5-year-old girl d iagnosed w ith lu pu s is seen by her fem ale p ed iatrician for a rou tine visit. After retu rning hom e from the clinic, the girl asks her friend to “p lay d octor.” Which of the follow ing d efense m echanism s best d escribes this behavior? (A) (B) (C) (D) (E)

Displacem ent Dissociation Id entification Rationalization Reaction form ation

Questions 68 and 69 A 10-year-old girl w ith a history of asthm a is brou ght to the clinic after a recent increase in her asthm a sym p tom s. Du ring the visit, you learn that she is being physically beaten by her m other’s boyfriend on a regu lar basis. 68. Und er w hich of the follow ing circu m stances d oes the law requ ire m and atory rep orting by a p hysician of su sp ected child abu se? (A) In all cases (B) Only in cases in w hich the child show s behavioral m anifestations of abu se. (C) Only w hen consent of a p arent or gu ard ian is obtained . (D) Only w hen the p hysician believes it is in a child ’s best interest. (E) Only w hen the p hysician has exam ined all child ren in the fam ily. 69. Which of the follow ing m anifestations w ou ld be the m ost likely ou tcom e of the abu se? (A) (B) (C) (D) (E)

Aggression Dissociative d isord er Generalized anxiety d isord er Major d ep ressive d isord er Post-trau m atic stress d isord er

Que s tions : 64–75

70. A 4-year-old boy is referred to you becau se he w ill not sp eak in p reschool. Over the cou rse of abou t 2 m onths, he grad u ally stop ped talking. H is m other rep orts that he initially objected to going to preschool, but now no longer com plains. She states that at tim es her son is qu iet and stays in his room , bu t that she has not otherw ise noticed a significant change in his sp eech or behavior. Which of the follow ing is the m ost likely d iagnosis? (A) Major d ep ressive d isord er (MDD) (B) Persistent d ep ressive d isord er (d ysthym ia) (C) Selective m u tism (D) Sep aration anxiety d isord er (E) Social anxiety d isord er (social p hobia) 71. A 2-year-old boy is referred to you for evalu ation d u e to the su sp icion that the child is the victim of abu se second ary to factitiou s d isord er im posed by another. Which of the follow ing fam ily m em bers is the m ost likely p erp etrator fabricating the illness? (A) (B) (C) (D) (E)

Father Brother Mother Sister Uncle

72. A fru strated m other brings her 14-year-old son to a child p sychiatrist after he is expelled from three high schools in 1 year. She reports the boy has tried tw ice to set his school on fire, has slashed school bus tires, and has broken into the principal’s office to steal athletic trophies. In ad d ition, he has been su sp end ed nu m erou s tim es for getting into fights w ith other stu d ents. She shu d d ers and tearfully relates that she recently caught him singeing one of the fam ily cats w ith a cigarette butt. Which of the follow ing p ersonality d isord ers is this boy m ost at risk of d evelop ing in the fu tu re? (A) (B) (C) (D) (E)

Antisocial Bord erline H istrionic Obsessive-com p u lsive Schizotyp al

13

73. An 8-year-old boy is brou ght in by his m other w ho com p lains that she cannot get her son to listen to her. She is fru strated becau se he frequently ignores her requ ests and instru ctions. Consid eration is given tow ard the d iagnosis of oppositional d efiant d isord er (ODD). Which of the follow ing featu res w ou ld best sup port the d iagnosis in this child ? (A) (B) (C) (D)

Aggression to p eop le Dep ressive sym p tom s Disobed ience tow ard teachers Lack of p articip ation in tasks requ iring attention (E) Violation of ru les

74. A 13-year-old girl is seen by her p sychiatrist 1 year after an au tom obile accid ent. She d em onstrates intact langu age ability and com plex m otor skills. She has no id entifiable abnorm alities in the perception of stim uli, bu t she has lost the ability to read since the accid ent. Which of the follow ing d eficits is she m ost likely d em onstrating? (A) (B) (C) (D) (E)

Agnosia Alexia Anom ia Ap hasia Ap raxia

75. A 10-year-old girl w ithou t significant m ed ical history is brou ght by her father to the p ed iatrician for evaluation. Over the past school year, she has been having increasing d ifficu lties going to sleep. Althou gh she has “alw ays had bed tim e ritu als,” they have extend ed in com plexity and length. Most of her tim e in the evening is now spent going arou nd the hou se nu m erou s tim es, locking and u nlocking the d oors and w ind ow s. While she know s the chances of a bu rglary are slim , she is extrem ely anxiou s abou t her safety, and she “can’t stop ” the u rges to perform these behaviors. As a resu lt, she only obtains 5 hou rs of sleep, and she has been falling asleep in class w ith d im inishing grad es. Which of the follow ing therap eu tic interventions is consid ered the first-line treatm ent for this d isord er?

14

1: Child a nd Ad ole s c e nt P sychia try

(A) (B) (C) (D) (E)

Cognitive-behavioral therapy (CBT) Fam ily therapy Grou p therap y Short-term psychod ynam ic therapy Supp ortive therapy

D IRECTION S (Questions 76 through 93): The follow ing group of numbered items are preceded by a list of lettered options. For each question, select the one lettered option that is most closely associated w ith it. Each lettered option may be used once, multiple times, or not at all. Questions 76 through 79 Match the severity level of intellectual d isability w ith the patient’s ad aptive functioning. (A) (B) (C) (D) (E)

N o intellectu al d isability Mild intellectu al d isability Mod erate intellectu al d isability Severe intellectu al d isability Profou nd intellectu al d isability

76. A 14-year-old boy w ho has lim ited attainm ent of conceptual skills, has sp oken language lim ited to single w ord s or p hrases, requ ires su p p ort for all activities of d aily living, requ ires sup ervision at all tim es, and w ho cannot m ake resp onsible d ecisions regard ing the w ell-being of others. 77. An 8-year-old girl w ho has no obviou s d eficits in learning, is equ ally m atu re as her p eers, is able to perform d aily living tasks w ithout su p p ort, and has sou nd ju d gm ent. 78. An 11-year-old boy w ho has d ifficu lties in acad em ic skills of w riting, read ing, and m ath, is im m atu re in social situ ations, and w ho has d ifficulties w ith regu lating em otions and behavior in age-ap p rop riate fashion. While he is able to p rovid e his ow n p ersonal care, he requ ires su p p ort in m ore com p lex d aily living tasks, ju d gm ent, and organization. 79. A 17-year-old boy w hose concep tu al skills consistently lag behind those of his peers, and , w hile he has d ifficu lties p erceiving or

interp reting social cu es and u tilizes less com p lex sp oken langu age, is able to retain basic langu age for social com m u nication. H e can care for his basic personal need s after extensive teaching, bu t w ou ld eventu ally requ ire consid erable su pport from co-w orkers and sup ervisors to m anage responsibilities in the fu tu re. Questions 80 through 83 Match the m ost likely d isord er w ith the appropriate p atient. (A) (B) (C) (D) (E) (F) (G) (H ) (I) (J) (K) (L)

Anorexia nervosa Au tism sp ectru m d isord er Bu lim ia nervosa Binge eating d isord er Illness anxiety d isord er Interm ittent exp losive d isord er Obsessive-com p u lsive d isord er Panic d isord er Pica Pyrom ania Tou rette d isord er Trichotillom ania

80. A 14-year-old girl w ith ep isod es of palpitations, chest p ain, shortness of breath, and d iaphoresis w ho has a norm al physical and laboratory exam inations. 81. A 10-year-old overw eight girl feels a loss of control w hen she qu ickly consum es large am ou nts of food , d enies heavy exercise, vom iting or laxative u se afterw ard s, and feels significant guilt and sham e regard ing her behavior. 82. An 8-year-old boy w ith erythem atou s, chapped hand s, and an otherw ise norm al p hysical and laboratory exam ination. 83. A 13-year-old girl w ith a bald p atch on the back of her head and an otherw ise norm al p hysical and laboratory exam ination.

Que s tions : 76–94

Questions 84 through 87 Match the m ost likely d isord er w ith the appropriate patient. (A) Attention-d eficit/ hyp eractivity d isord er (ADH D) (B) Au tism sp ectru m d isord er (C) Generalized anxiety d isord er (D) Langu age d isord er (E) Selective m u tism (F) Social anxiety d isord er (G) Social (p ragm atic) com m u nication d isord er 84. A 6-year-old girl whose parents are going through a divorce will not speak while at school. 85. A 9-year-old boy w ho frequ ently blu rts ou t com m ents in class w ithou t w aiting his turn to be called on. 86. A 7-year-old boy w ho p erform s w ell in school althou gh seem s to talk as if reciting a m onologu e rather than interacting in conversation and generally avoid s other child ren, bu t d oes not d isplay rep etitive or restricted interests. 87. An 8-year-old boy w ho is having d ifficu lty in school and avoid s interactions w ith his classm ates and others for fear of em barrassm ent. Questions 88 through 93 Match the age range w ith the corresp ond ing d evelopm ental m ilestone. (A) (B) (C) (D) (E)

Infant (0–18 m onths) Tod d ler (18–36 m onths) Preschool age (3–6 years) School age (7–12 years) Ad olescence (13–17 years)

88. Focu s on follow ing the ru les. 89. Establishing self as au tonom ou s, sep arate from caregiver, by p racticing leaving and retu rning to the caregiver.

15

90. Establishing tru st in the w orld throu gh resp onsiveness and em p athy of a caregiver. 91. Preoccup ation w ith su perheroes w ho rep resent id ealized caregivers as a resu lt of conflicted feelings tow ard caregivers. 92. The d evelop m ent of the ability to think abou t and m anipu late id eas abstractly. 93. The d evelop m ent of the ability to ap p ly reasoning so that the child is not lim ited only by p ercep tions. DIRECTIONS (Questions 94 through 99): For each of the multiple-choice questions in this section, select the lettered answer that is the one best response in each case. 94. A 16-year-old girl is brou ght to you by her m other because of d ropping grad es, apathy, and p oor m otivation. You learn that she has recently started sm oking m ariju ana on a regu lar basis. Which of the follow ing p atterns w ou ld m ost su p p ort the d iagnosis of cannabis u se d isord er, severe? (A) Over 12 m onths, u sing m ore than intend ed and being u nsu ccessfu l at stopp ing, w ith a loss of her sum m er job and legal consequ ences from her u se. (B) Over 12 m onths, having cravings to u se, spend ing significant tim e obtaining m arijuana, and m issing school in ord er to sm oke. (C) Over 12 m onths, continu ing to use d esp ite recu rrent argu m ents, being “high” w hile d riving, u sing d espite know ing it is affecting her m otivation, need ing increased am ounts to get the sam e “high,” and having ongoing cravings w hen not u sing. (D) Over 18 m onths, sm oking by herself bu t need ing to sm oke m ore to achieve the sam e effect. (E) Over 18 m onths, sm oking m ost d ays p er w eek, m ultip le tim es p er d ay, and d eveloping w ithd raw al sym ptom s once the m ariju ana is stopped .

16

1: Child a nd Ad ole s c e nt P sychia try

95. As the school p sychologist, you are asked to see a fou rth grad er w ho has been consistently acting out in class. H e often lies abou t things he has d one in class, such as trying to cheat on tests, and he physically bu llies you nger child ren. You learn that at hom e he practices shooting his BB gu n at squirrels as w ell as at the fam ily d og. H e exp resses no concern for these creatu res nor rem orse at his behavior. Which of the follow ing is the m ost likely d iagnosis? (A) (B) (C) (D) (E)

Au tism sp ectru m d isord er Bip olar d isord er Child hood onset schizop hrenia Cond u ct d isord er Op p ositional d efiant d isord er

96. An 8-year-old boy is brou ght to the p ed iatrician for concerns that he has significant tantru m s. The tantru m s occur at least three tim es p er w eek (bu t frequ ently m ore) and inclu d e p hysical aggression tow ard inanim ate objects to the extent that he has d estroyed property. H is parents note that the fam ily w alks on eggshells to p revent setting him off, and they are starting to feel “held hostage” by his chronically irate m ood . They believe this change in behavior started a year ago, bu t has m arked ly w orsened in the p ast several w eeks. Which of the follow ing is the m ost likely d iagnosis? (A) Bip olar d isord er (B) Disru p tive m ood d ysregu lation d isord er (DMDD) (C) Generalized anxiety d isord er (D) Major d ep ressive d isord er (MDD) (E) Op p ositional d efiant d isord er (ODD) 97. A healthy 16-year-old stu d ent, w ho started school in the United States after fleeing a w ar-engulfed region 1 year ago, is struggling to keep up in her classes, m ore so w ith English than m ath. Which of the follow ing best explains w hy her d ifficu lty is not d u e to a learning d isability? (A) H earing d eficit (B) N eu rocognitive d isord er d u e to traum atic brain injury

(C) N orm al variant in acad em ic attainm ent (D) Post-trau m atic stress d isord er (PTSD) (E) Vision d eficit 98. A 7-year-old girl is brou ght to the p ed iatrician d u e to w eight loss. She has been grad u ally refu sing food s over the p ast several w eeks to the point she is now barely even d rinking su pplem ental shakes. This has resu lted in significant w eight loss and fatigu e. The girls tells you she is aw are she is losing w eight, that she w ou ld like to fit into the ou tfits she has, bu t that she is afraid of vom iting if she eats the w rong thing. Physical exam ination reveals a child in the 5th p ercentile of w eight, w hile her baseline had p reviou sly been in the 30th percentile, and basic laboratory stu d ies reveal a m ild m icrocytic anem ia. Which of the follow ing is the m ost likely d iagnosis in this case? (A) Anorexia nervosa (B) Avoid ant/ restrictive food intake d isord er (ARFID) (C) Binge eating d isord er (D) Bu lim ia nervosa (E) Sp ecific p hobia 99. A 7-year-old boy is brou ght to the p ed iatrician becau se, for the p ast 6 m onths, he has been telling his m other that he is a girl. Initially his parents felt this w as “a p hase,” bu t the patient has becom e increasingly d istressed and m ore insistent that he hop es his breasts w ill grow d u ring p u berty. H e has been p u tting on his old er sister ’s clothing, and prefers p laying w ith her d olls rather than engaging in rou ghhou sing w ith his p eers. H e has rep eated ly stated he d islikes his p enis and w ishes he never had one. Physical exam ination reveals norm al m ale genitalia of the app rop riate Tanner stage. Based on this inform ation, w hich of the follow ing is the m ost likely d iagnosis? (A) (B) (C) (D) (E)

Bod y d ysm orp hic d isord er Delu sional d isord er Gend er d ysp horia in child ren N onconform ity to gend er roles Transvestic d isord er

Ans we rs a nd Expla na tions

1. (C) Su icid e is a consid erable risk in d ep ressed ad olescents, and shou ld be sp ecifically ad d ressed d u ring an interview w ith a p atient w ho ap p ears d ep ressed or agitated , or has a history of a su icid e attem pt. The ad olescent su icid e rate has increased substantially d u ring the p ast few d ecad es. Male gend er, a p rior su icid e attem pt, history of psychiatric illness, fam ily history, and su bstance u se are all risk factors for a com p leted su icid e. A history of a p rior su icid e attem p t is the largest risk factor for suicid e for both m ales and fem ales in all age grou ps, and the m ajority of those w ho com plete su icid e have attem pted su icid e in the p ast. Greater than 90% of you th w ho com m it suicid e have com orbid psychiatric illness, frequ ently m ajor d ep ression. Althou gh cu tting behavior is concerning in ad olescents, it is not necessarily associated w ith the intent to kill oneself. More girls than boys d em onstrate su icid al behavior and attem pts, bu t at least three tim es m ore teenage boys com p lete su icid e than d o teenage girls. This is becau se boys m ore frequ ently u se gu ns and other violent m ethod s to attem p t suicid e. 2. (C) Given the ad olescent’s history of d ep ression and risk factors of su bstance use and fam ily history, it is im p ortant to initiate ap prop riate treatm ent for her qu ickly. Given the severity of her d epression in the p ast, and d ecline now , it is reasonable to initiate antid ep ressant treatm ent at this visit w ith close follow -u p . While it w ou ld be op tim al for a p sychiatrist to evalu ate her as soon as possible and a referral is ap propriate, the w ait in getting to a specialist (psychiatrist) shou ld not d elay care. She d oes not necessarily need to

be ad m itted to a hosp ital, since she d oes not ap p ear at im m inent risk of self-harm . Certainly, w aiting m ore than that w ithou t any intervention w ou ld be inad equ ate and inap p ropriate. She w ou ld likely benefit from m ed ication m anagem ent, so a p sychiatrist w ou ld be p referable to a social w orker in this situ ation, bu t therapy w ou ld also be appropriate concurrent treatm ent. 3. (D ) The Diagnostic and Statistical M anual of M ental Disorders, Fifth Edition (DSM -5) recognizes N REM sleep arou sal d isord ers of tw o types: sleep w alking typ e and sleep terror type. Both are characterized by recu rrent episod es of incom plete aw akenings from sleep, u su ally in the first third of the m ajor sleep ep isod e, w ith am nesia for the ep isod es, no or little recollection of d ream im agery, and resu ltant im pairm ent or d istress. Sleep terror type involves rep eated ep isod es of su d d en aw akening from sleep accom p anied by p anic sym p tom s that begin w ith a scream ; it is associated w ith u nresp onsiveness to com fort or attem p ts to aw aken the child . Attacks typically last just a few m inu tes and are often m ore d istressing to the caregiver, as the patient d oes not recall them in the m orning. N REM sleep arou sal d isord er, sleep -w alking typ e, involves the child sitting up or leaving the bed but is not accom p anied by terror or au tonom ic arou sal. N ightm are d isord ers occu r in the latter third of the night and d u ring REM sleep , in contrast to sleep terrors. When aw akened , the ind ivid u al qu ickly becom es oriented , u nlike other parasom nias w here the ind ivid ual is d isoriented , confu sed , and d ifficu lt to arou se. Interm ittent exp losive d isord er d oes not occu r 17

18

1: Child a nd Ad ole s c e nt P sychia try

d uring sleep and involves outbursts of anger and som etim es violent behavior. N arcolep sy is characterized by the triad of sleep attacks, cataplexy (sud d en loss of m uscle tone), and hyp nopom pic/ hypnagogic hallu cinations or sleep paralysis, as w ell as a hypocretin d eficiency. 4. (C) Ep isod es of sleep terror, as w ell as of sleep w alking, occur d u ring d eep sleep (stages 3–4). N ightm are d isord er occurs d uring REM sleep. 5. (E) This patient is su ffering from a m ajor d ep ressive d isord er (MDD), single episod e. A selective serotonin reu ptake inhibitor (SSRI) such as sertraline is a first-line agent for MDD in child ren and ad olescents. Tricyclic antid ep ressants (TCAs) su ch as im ip ram ine cau se m ore ad verse effects than SSRIs typically d o, and in overd ose are m u ch m ore likely to be lethal. Mood stabilizers su ch as carbam azep ine and lithiu m are u sed for bipolar d isorder and as adjuncts to the treatment of MDD refractory to antidepressant medications alone. Antipsychotics such as olanzapine are usually reserved for use as adjuncts when psychosis develops or in bipolar disorder. 6. (D ) In the DSM -5, m ajor d epressive d isord er requ ires a d epressed m ood for at least 2 w eeks for ad ults to receive the d iagnosis, but in the case of child ren and ad olescents, an irritable mood may substitute for having a d epressed mood . The required symptoms that involve sleep, appetite, anhed onia, concentration, and psychomotor functioning are the same for ad ults and ad olescents w hen diagnosing major d epressive d isord er. 7. (B) This boy m ost likely su ffers from langu age d isord er. Langu age d isord er is characterized by p ersistent d ifficu lties in the acqu isition and u se of langu age d u e to d eficits in com prehension (receptive) or prod u ction (expressive). Typ ical sym p tom s inclu d e having a m arked ly lim ited vocabu lary, m aking errors in tense, and having d ifficulty recalling w ord s or prod u cing sentences w ith d evelopm entally ap propriate length or com p lexity. In ad d ition, the boy’s d ifficulties are interfering w ith

acad em ic fu nctioning. There is no ind ication that this child has d ifficulties w ith speech p rod u ction (i.e., he can clearly articu late phonem es of sp eech), so d iagnosing sp eech sou nd d isord er w ou ld be inappropriate. As there are no d istu rbances in the flu ency and p attern of speech, a d iagnosis of child hood -onset flu ency d isord er (stu ttering) is inap p rop riate. Social (pragm atic) com m u nication d isord er is characterized by persistent d ifficu lties in the social use of verbal and nonverbal com m unication, for instance im p airm ents in greetings, inability to change com m u nication to a given context, d ifficu lties follow ing ru les for conversation or story-telling, or d ifficu lties in u nd erstand ing w hat is inferred but not overtly stated . This child is able to appropriately m od u late verbal com m u nication, bu t his d ifficu lty is in the expression of verbal cues. There m ay be a com orbid learning d isord er, but at this time, there is insufficient information to m ake that d iagnosis. 8. (C) Sp eech d elay refers to exp ressive langu age d evelop m ent and nu m ber of w ord s spoken. The average nu m ber of w ord s a 2-year-old sp eaks is 200 w ord s. Therefore, the inability to sp eak 200 w ord s by age 3 w ould constitu te a speech d elay. 9. (E) It is m ost likely that the boy u sed illicit substances d u ring his trip that caused him to experience the acute psychotic sym ptom s. While it is im p ortant to ru le ou t other m ed ical cau ses, su bstance use w ould be the m ost likely. Seizu re d isord ers, head inju ries, d iabetes, and thyroid d isease often w ou ld be accom p anied by other sym p tom s in ad d ition to the new onset psychosis. 10. (B) While a su bstance-ind u ced state rem ains p ossible d u e to the large nu m ber of su bstances that cannot be confirm ed by u rine toxicology screens, it is im portant to consid er that this instance of p sychosis is a m anifestation of an u nd erlying m ood d isord er, m ost likely bipolar d isord er. In ad d ition to an elevated or irritable m ood , the d iagnosis of bip olar d isord er necessitates that ind ivid uals d isplay three of the follow ing (four if irritable m ood ): grand iosity,

Answe rs : 4–15

d ecreased need for sleep, increase in talkativeness, objective or su bjective flight of id eas, d istractibility, increase in goal-d irected behaviors, and engagem ent in p otentially high-risk behaviors. Mania in bip olar d isord er can often p resent concu rrently w ith psychotic sym p tom s includ ing hallu cinations, d elu sions, and d isorganized thinking. Anorexia nervosa (an eating d isord er) and generalized anxiety d isord ers d o not p resent w ith psychotic sym ptom s. Ind ivid u als w ith bord erline personality d isord er m ay have p sychotic-like sym ptom s, esp ecially w hen they are regressed , bu t they are m u ch m ore likely to p resent w ith d ep ression, suicid al thinking, and su bstance u se. While p sychotic featu res can accom p any m ajor d ep ressive d isord er, the d ep ressive sym ptom s occu r first, and psychotic sym ptom s are m ore acu te and frequ ent in bipolar d isord er 11. (C) This boy’s behavior and sym p tom s are m ost consistent w ith sep aration anxiety d isord er, characterized by d evelop m entally inapp rop riate and excessive anxiety concerning separation from the hom e or from those to w hom the ind ivid u al is attached . Consistent w ith this d iagnosis, the boy d oes not like to go to school, com es hom e im m ed iately after school, sleeps in his parents’ bed at night, and has repeated physical sym ptom s w hen at school. The boy is not suffering from PTSD becau se there is no evid ence of a trau m atic event that is p ersistently re-exp erienced or has cau sed sym ptom s of increased arou sal and avoid ance of associated stim u li. For a d iagnosis of reactive attachment d isord er, the boy w ould need to have su ffered m arked ly d istu rbed social related ness, in m ost contexts, beginning before the age of 5 years, and display a pattern of inhibited , w ithd raw n behavior toward caregivers. A related disorder, d isinhibited social engagem ent d isord er, also stem s from significant neglect or d eprivation and results in child ren d isp laying a m arked ly overfam iliar ap p roach and com fort w ith strangers. The boy’s sym p tom s are not consistent w ith social anxiety d isord er (social p hobia) becau se he d oes not have a m arked and p ersistent fear of social or p erform ance

19

situ ations, w ith exp osu re causing intense anxiety. Finally, a d iagnosis of specific phobia w ould require the display of marked and persistent fear cued by the presence of anticipation of a specific object or situation. 12. (B) Child ren w ith p arents w ho have a history of an anxiety d isord er are at increased risk for d evelop ing an anxiety d isord er them selves. Other risk factors for d eveloping a child hood anxiety d isord er inclu d e p arents w ho have an anxiou s, overly controlling, or rejecting style, an insecu re attachm ent w ith one’s p rim ary caregiver, and an inhibited and shy tem peram ent. 13. (B) These sym p tom s are consistent w ith a m anic ep isod e of bip olar d isord er. Attentiond eficit/ hyperactivity d isord er is associated w ith hyp eractivity and im p u lsivity bu t not grand iosity and d elu sional thinking. Major d ep ression m ay have psychotic sym ptom s if severe, bu t it w ou ld not p resent w ith an inflated m ood or increased energy. Panic d isord er w ou ld present w ith recu rrent panic attacks and significant anxiety. Although schizophrenia can present w ith d elusional sym ptom s, there are often other sym ptom s such as grossly d isorganized speech and / or behavior, or flat affect; in ad d ition, this p atient has significant m ood sym ptom s. 14. (C) Cocaine is a stimu lant and can prod u ce both manic and psychotic sym ptoms. Alcohol, cannabis, heroin, and PCP ingestion can ind uce a psychotic state includ ing hallucinations and paranoia, bu t it w ou ld not classically be accompanied by manic symptom s. 15. (E) This boy likely has ODD. ODD is an externalizing behavior d isord er that involves a p attern of hostile and d efiant behavior. Child ren w ith ODD are often angry, argu m entative, and easily annoyed by others. In ord er to confirm the d iagnosis, you w ou ld need to gather ad d itional inform ation regard ing the length of tim e the behavior has been present (at least 6 m onths is requ ired ), as w ell as rule ou t a m ood , p sychotic, or su bstance u se d isord er. Althou gh anxiety d isord ers can p resent w ith

20

1: Child a nd Ad ole s c e nt P syc hia try

tension, irritability, and even noncom pliance w hen a child is p laced in a new situ ation (su ch as a d octor’s office), this child d oes not ap p ear to have anxiety in places w here escape m ay be d ifficu lt in the event of a p anic attack (su ch as in agorap hobia), nor excessive w orry abou t a nu m ber of events (su ch as in generalized anxiety d isord er). While the p atient’s p resenting sym p tom s are not p rim arily involving inattention, d istractibility, or hyp eractivity consistent w ith ADH D, there is consid erable com orbid ity betw een ADH D and ODD. H is lack of violence or seriou s violation of ru les is not consistent w ith cond u ct d isord er; how ever, ODD (esp ecially if u ntreated ) m ay lead to cond uct d isord er over tim e. 16. (A) This patient appears to be suffering from bulim ia nervous, an eating d isord er characterized by episod es of bingeing and compensatory behaviors (e.g., purging). Signs of bulim ia nervosa includ e erosion of tooth enamel caused by acid ic vomitu s as w ell as abrasions on the d orsu m of the hand d u e to scraping by the u pper teeth as the ind ivid ual pushes the hand to the back of the throat to ind u ce vom iting. As it is not a secret that the abrasions are there, but the patient has not mentioned them, a d irect bu t open-end ed statement, such as, “Tell me about the scratches on your hand ,” is most likely to be helpful in this situation. Usually, a patient appreciates a d irect question rather than w ond ering if you ignored or d id not notice something obvious. Questions su ch as, “Do you scratch you rself?” or, “Do you have a cat?” are not open-end ed ; they require “yes” or “no” answ ers and are unlikely to yield new information. “Are you trying to hurt you rself?” or “H ow d id the scratches happen?” m ay sound accu satory; patients are more likely to offer information if you seem nonjud gmental. If this girl d oes not volu nteer inform ation that confirm s to you that she is not ind ucing vom iting, you may need to u se a more d irect bu t reassuring statement su ch as, “Som etimes I see girls w ho make them selves throw up. H ave you ever d one that?” 17. (D ) Med ical com p lications can arise for bu lim ic p atients w ho engage in com p ensatory

behaviors su ch as ind u ced vom iting, laxative, or d iu retic use. A hypokalem ic-hypochlorem ic alkalosis (d u e to vom iting) is a p ossible seriou s find ing that can contribute to a card iac arrhythm ia. Anem ia (as op p osed to increased iron) can be seen, as w ell as d ecreased p rotein. The red u ced chlorid e can lead to elevated sod ium d u e to sod ium resorption. Becau se of these changes, it is im portant to m onitor bu lim ic p atients for electrolyte and acid –base im balances. 18. (A) Bulimia nervosa is most effectively treated w ith cognitive-behavioral therapy. Family therapy, group therapy, and longer-term, insightoriented therapies (such as psychoanalysis and psychod ynamic psychotherapy) may be used as adjuncts to cognitive-behavioral therapy, but they have not been demonstrated to be as effective in changing the behaviors associated w ith bulimia nervosa. 19. (D ) When engaging a child in a clinical setting, it is im portant to choose w ord s carefu lly in ord er to establish an op en, tru sting connection, especially w ith a troubled and resistant child . It is p referable to initiate the interview w ith m ore op en-end ed rather than closed end ed qu estions. The qu estions shou ld op tim ally reflect em p athy for the child and their situ ation w ithout ap pearing too sentim ental or jud gm ental. Also, it is im portant to avoid assu m ptions abou t the p atient’s feelings w ithou t checking w ith the patient first. The statem ent, “You su re w ere lu cky the car sw erved at the last m inute” assu m es the boy w anted to avoid being hit by the car, and also is a com m ent that d oes not lead to any elaboration from the p atient. Statem ents (B) and (C) are both ju d gm ental and m ore likely to shut d ow n conversation than facilitate it. Also, asking a child if they are confu sed is a close-end ed qu estion that com es across as cond escend ing. 20. (D ) Psychom otor agitation is m ore com m only seen in child ren w ith MDD com pared to ad olescents w ith MDD. Child ren w ith MDD m ay ap p ear m ore anxiou s and irritable than sad and d ep ressed . H yp ersom nia, hop elessness, w eight change, and illicit d ru g u se are m ore

Answe rs : 16–27

com m only seen in ad olescents w ith MDD com pared to child ren w ith MDD. 21. (B) Suffocation (e.g., hanging) is the most common m ethod for attem pting suicid e in ind ivid uals u nd er 15 years of age. Su bstance ingestion is another comm on m ethod . While firearms are used less frequently, they are more often lethal. Other m ethod s of suicid e frequently attempted by child ren inclu d e stabbing, cutting, jum ping from build ings, ru nning in front of vehicles, and gas inhalation. Some su icid e attempts m ay be m istaken for accid ents, so it is im portant to d irectly ask child ren if they intend ed to hurt or kill themselves. 22. (A) H ead aches and nau sea are com m on ad verse effects associated w ith the synthetic antid iuretic horm one, Dd avp. H yp otension, liver toxicity, sed ation, and trem or are not typically associated w ith d esm opressin. 23. (B) Approxim ately 10% to 15% of child ren w ith anxiety d isord ers d evelop MDD. 24. (C) The m ost com m on ad verse effects of flu oxetine inclu d e gastrointestinal sym p tom s (e.g., nausea, loose stools), insom nia, agitation, and head aches. In general, hyp otension, liver toxicity, sed ation, and w eight gain are not sid e effects associated w ith flu oxetine. 25. (A) This girl’s history and presentation are consistent w ith ASD. ASD is characterized by d eficits in social comm unication and social interaction, as w ell as restricted , repetitive patterns of behavior, interests, or activities (e.g., stereotyped or repetitive movements, inflexible rigid ity to routines, fixated or restricted interests, sensory input abnorm alities). Child ren w ith ASD have d ifficulties w ith socialemotional reciprocity and have d ifficulty engaging in prosocial behavior. Fu rther, they often have imp aired eye contact and have d ifficulty und erstand ing nonverbal com munication or cues. They also have abnormal social attachments, su ch as app earing to have no interest in peers (as in this case). The DSM -5 significantly revised the category of ASD, so that verbal language abilities no longer represent

21

a separate criterion. While ASD is more common in males, it d oes exist in fem ales. ID m ay be comorbid w ith ASD, but m ust be separately assessed . ID is characterized by intellectu al and functioning d eficits in conceptual, social, and practical d omains. Rett synd rome is seen only in fem ales and is characterized by normal prenatal and perinatal d evelop ment, normal head circum ference at birth, and normal p sychom otor d evelopment throu gh the first 5 m onths of life. Betw een the ages of 5 and 48 months, there is d eceleration of head grow th, loss of hand skills w ith d evelop ment of stereotyped hand m ovem ents su ch as hand w ringing, loss of social interaction (w hich m ay improve later), appearance of poorly coord inated gait or tru nk m ovem ents, and severely im paired exp ressive and recep tive language d evelopment w ith severe psychomotor retard ation. Consequ ently, child ren w ith Rett synd rom e, d uring the period of regression, m ay appear to meet criteria for ASD. In this case, there is no evid ence of the typical physical examination abnormalities. Of note, Rett synd rome no longer has its ow n d iagnostic criteria in the DSM -5 bu t rem ains d escribed in the d ifferential d iagnoses for several d isord ers. Selective m u tism is characterized by a failu re to sp eak, d esp ite norm al ability to d o so, in social situ ations w here speaking is exp ected , su ch as at school; ou tsid e of this social situation, they d isplay norm al speech and interactions. Child ren w ith social (pragm atic) com m u nication d isord er d isplay persistent d ifficu lties in how they use verbal and nonverbal com m unication in social settings, bu t lack the restricted / rep etitive p atterns of behaviors and interests that are present in ASD. 26. (C) ASD is d iagnosed four times more in boys than girls, and w hen it is present in girls, intellectual d isability is often comorbid . The presence of functional language by age 5 is a positive prognostic sign, and it also allow s child ren to engage more effectively in social skills training. The remaining listed factors negatively affect the prognosis of patients w ith ASD. 27. (A) This patient’s presentation is consistent w ith conduct disord er (CD). Factors associated

22

1: Child a nd Ad ole s c e nt P syc hia try

w ith an increased risk of a child d evelop ing CD inclu d e a low IQ, low school achievem ent, p oor parental su p ervision, p u nitive or erratic p arental d iscip line, child hood physical abu se, p arental conflict, the child ’s biological father being absent, antisocial p arents, and having a large fam ily. 28. (A) Child ren and ad olescents w ho are d iagnosed w ith cond u ct d isord er are at increased risk for an tisocial p ersonality d isord er as ad u lts. Antisocial p erson ality d isord er is not d iagnosed u ntil after th e age of 18, and on e of the criteria is evid ence of con d u ct d isord er p rior to the age of 15. Earlier onset of con d u ct d isord er is associated w ith an increased risk of d evelop ing antisocial p ersonality d isord er. Cond u ct d isord er is not as closely associated w ith avoid ant p erson ality d isord er, p aranoid p ersonality d isord er, schizoid p erson ality d isord er, or schizotyp al p ersonality d isord er. 29. (D ) By d efinition, sym ptom s of ad ju stm ent d isord er d o not last longer than 6 m onths after a stressor or the term ination of its consequences. If d epressive sym ptom s persist, MDD m ay be d iagnosed . In both ad justm ent d isord er and MDD, sym ptom s m ust cau se m arked d istress or significant im pairm ent in fu nctioning, m ay d evelop follow ing a stressor, m ay d evelop w ithin 3 m onths of the onset of a stressor, and m u st not represent norm al bereavem ent. Ad ju stm ent d isord er m ay also be given sp ecifiers d ep end ing on the p resence of anxiety, d ep ressed m ood , and cond u ct d istu rbances. Ad ju stm ent d isord er follow ing bereavem ent m ay be given p rovid ed that the ind ivid ual d oes not m eet criteria for fu ll MDD, and the intensity of qu ality of the grief reaction exceed s w hat is exp ected for cu ltu ral, religiou s, or age-m atched norm s. 30. (D ) Follow ing a d iagnosis of d iabetes m ellitu s type I, approxim ately 33% of child ren d evelop sym ptom s of ad justm ent d isord er. 31. (E) Tou rette d isord er is the m ost likely d iagnosis becau se the boy’s ou tbu rsts are consistent w ith vocal tics, an d th e rep ort of

“restless an d ju m p y” beh avior is consistent w ith m isinterp retation of m otor tics. Tou rette d isord er m ost com m only d evelop s in grad e school-aged boys, an d the involu ntary tics m ay be m isinterp reted as p u rp osefu lly d isru p tive behavior. It is not u n u su al for these child ren to have d ifficu lty w ith social and p eer interactions. Of note, there is frequ ent com orbid ity w ith ADH D. Th is boy d oes not su ffer from con d u ct d isord er or op p osition al d efian t d isord er becau se he is p olite and d oes not d isp lay hostile, d estru ctive, or angry behaviors. Th e ou tbu rsts are n ot typ ical of p an ic d isord er, in w hich there are d iscrete p an ic attacks, p eriod s of in tense fear, or d iscom fort, w ith p hysical m anifestations su ch as p alp itation s and su bjective d ifficu lty breathing. There is no evid ence that this boy exp eriences d istress an d w orry w h en sep arated from an im p ortant attach m en t figu re as in sep aration an xiety d isord er. 32. (B) Clonidine has become the first-line treatment for Tourette disorder. It has a limited side-effect profile and helps control symptoms of a frequently associated comorbid disorder, attention-deficit/ hyperactivity disorder (ADHD). Tricyclic antidepressants (TCAs) have been shown to be effective in the treatment of Tourette disorder, but other antidepressants such as bupropion, paroxetine, and venlafaxine are not known to be effective. High-potency antipsychotics such as haloperidol and pimozid e were trad itionally the first-line agents for Tourette’s, but are more likely to cause significant side effects. Recently, the newer atypical (or second-generation) antipsychotics such as risperid one and olanzapine have also been used to treat the disorder. 33. (A) It is not u nu su al for p arents of a seriou sly ill child to w ant to try to p rotect them from any fu rther su ffering or d istress. H aving an ill child is overw helm ing and anxiety-provoking. Parents m ay end u p treating su ch child ren as if they w ere you nger than their actual age and m ay be m ore reluctant to set ap prop riate lim its. When calling a p arent’s attention to the potential harm of su ch interaction, a clinician m u st be tactfu l and em p athic. By gently

Answe rs : 28–38

acknow led ging this m other’s d istress, she w ill be m ore recep tive to hearing feed back abou t her interactions w ith her child , inclu d ing that her relu ctance to set lim its and stru ctu re m ay inad vertently be d etrim ental. While it is essential, em p athizing alone w ill be insu fficient to set the stage for a d iscussion of p arent–child interactions. Referring her to a p arent su p p ort grou p m ight be u sefu l follow ing you r initial d iscu ssion w ith her. Telling her she is treating her son like a baby is overly harsh and m ore likely to m ake her d efensive rather that recep tive. 34. (C) A psychiatric illness in a child or ad olescent w ith a m ed ical illness shou ld be treated aggressively u sing the type of treatm ent m ost effective for the sp ecific p sychiatric illness, w hile keep ing in m ind p otential interactions of treatm ents. Med ical illnesses can exacerbate p sychiatric illness and vice versa. As su ch, effective treatm ent of a m ental illness m ay p ositively affect a m ed ical illness if im p rovem ent of psychiatric sym p tom s enables the child to m ore fully participate in treatm ent of the m ed ical cond ition. In ad d ition, em otional state is related to im m u ne response. Althou gh there is a specific stressor in this case, it is u nclear w hether the d ep ressive sym p tom s w ou ld resolve if the leu kem ia goes into rem ission; in ad d ition, given the im p airm ent in this boy’s fu nctioning and that it is u nclear if/ w hen the leu kem ia w ou ld rem it, not treating the d ep ression w ou ld be inap p rop riate. Ap p rop riate p harm acologic treatm ent shou ld be avoid ed only if there are sp ecific contraind ications based on the m ed ical treatm ent the child is receiving. In ad d ition, the effectiveness of psychotherapy is related to the specific m ental illness, not to the p resence or absence of a m ed ical illness. 35. (D ) It is not u nu su al for child ren w ith Tou rette d isord er to have com orbid p sychiatric d isord ers. Obsessive-com pu lsive d isord er is a very com m only associated d isord er, often presenting in ad olescence. Other anxiety d isord ers, attentional d isord ers, and learning d isord ers can be seen as w ell. Autism spectru m d isord ers are not com m only associated w ith Tou rette’s.

23

Mood lability can be an associated sym p tom , althou gh the incid ence of bip olar d isord er is not p articularly increased . Langu age or separation anxiety d isord ers are also not increased in ind ivid u als w ith Tou rette d isord er. 36. (E) A su bset of child ren w ith prior grou p A beta-hem olytic strep tococcal infections have d evelop ed Tou rette d isord er after the infection. Other d isord ers associated w ith su ch an infection inclu d e OCD and Syd enham chorea. This grou p of d isord ers are referred to as p ed iatric au toim m u ne neu rop sychiatric d isord ers associated w ith strep tococcal infections, or PAN DAS. The other infectiou s agents listed are not associated w ith Tou rette d isord er. 37. (A) This boy’s history is typical of ADH D, com bined presentation. H e d oes not comp lete tasks, is careless, loses necessary item s, is d istracted and forgetful, is energetic, impu lsive, and cannot tolerate playing quietly. Symptom s are p resent before age 12 and are occu rring in at least tw o settings, and are causing im pairment and d istress. Increased energy and im pulsivity can be a symptom of both ADH D and bipolar d isord er, but to be an exacerbation of bipolar d isord er, those symptom s m ust be associated w ith energy and im pulsivity beyond baseline, inflated mood , grand iosity, increase in goal d irected activity, and a d ecreased need for sleep. This boy d oes not d isplay excessive aggression, d estru ction of prop erty, d eceitfu lness, theft, or serious violations of rules, as seen in cond u ct d isord er. H is behaviors are not negativistic, hostile, or d efiant, so he d oes not suffer from oppositional d efiant d isord er. Therefore, he d oes not have characteristics of a d isruptive behavior d isord er. 38. (E) Stim u lant m ed ications are the first-line treatm ent for ADH D bu t have been associated w ith an increased risk of d eveloping tics. In general, if a child suffers from tics or has a fam ily history of tics, stim u lant m ed ications shou ld be avoid ed , and an alternate m ed ication shou ld be u sed to treat ADH D if necessary. Benzod iazep ines, D 2 antagonists, MAOIs, and SSRIs d o not com m only exacerbate tics and are not u sed to treat ADH D.

24

1: Child a nd Ad ole s c e nt P syc hia try

39. (C) To be d iagnosed w ith a learning d isord er w ith im p airm ent in read ing, there m u st be evid ence of an ind ivid u als’ p erform ance in a given acad em ic area (in this case, read ing) falling below w hat is exp ected for age. Psychom etric testing, su ch as IQ testing, is one m easu re to assess the d isparity betw een exp ected achievem ent and actu al achievem ent. A learning d isord er can also be d iagnosed if the ind ivid ual is u sing significant and excessive com p ensatory levels of effort or su pp ort to sustain average achievem ent. 40. (D ) At least 50% of child ren w ith learning d isord ers have a com orbid p sychiatric d isord er. The m ost com m on com orbid cond itions inclu d e ADH D, anxiety, and d epressive d isord ers. 41. (C) Most child ren w ith enu resis are boys, and overall anyw here betw een 20% and 40% have a com orbid m ental d isord er. 42. (B) Of the choices p rovid ed , Dd avp is the treatm ent of choice for enu resis; it is a variation of the antid iu retic horm one vasop ressin and is given intranasally. Benztrop ine is u sed to p revent extrap yram id al sym p tom s cau sed by antip sychotic (neu rolep tic) u se. Methylp henid ate is a stim u lant com m only u sed in the treatm ent of ADH D. Paroxetine is a serotonin-sp ecific reu p take inhibitor u sed to treat d ep ression and anxiety. Trazod one is an antid ep ressant m ost often u sed for insom nia. 43. (B) The m ost com m on ad verse effects of m ethylp henid ate and other stim u lant m ed ications inclu d e insom nia, d ecreased ap p etite, w eight loss, d ysp horia, and irritability. Trem or, hyp otension, w eight gain, and liver toxicity are not com m on sid e effects. 44. (E) Stim u lant m ed ications, su ch as m ethylp henid ate, are effective in d im inishing sym ptom s of ADH D in ap p roxim ately 70% of p atients. 45. (B) Atom oxetine is a nonstim u lant norepinephrine reu ptake inhibitor, w hich has been fou nd to be help fu l in treating both child hood and ad ult ADH D. Mixed am p hetam ine salts

(Ad d erall) are u sed to treat ADH D, bu t they are also stim u lants and w ill have sim ilar sid e effects to the m ethylp henid ate-based m ed ications. Aripip razole is a second -generation (atyp ical) antip sychotic, citalop ram is an antid ep ressant, and valp roic acid is a m ood stabilizer u sed in bip olar d isord er; none are u sed to treat ADH D. 46. (B) The clinician can be help fu l to p arents by exp laining and norm alizing the child ’s behavior. H ow ever, it is still im p ortant for the p arent to resp ond to or correct this behavior. In the case of this you ng girl, jealou sy of her you nger sibling is entirely norm al and is m ore d ifficu lt for her, since as a p reschooler her langu age skills m ay not be d evelop ed su fficiently to articu late her d istress. Once the m other accep ts this child ’s behavior as u nd erstand able and in the norm al range, she can resp ond in an em p athic w ay w hile still setting lim its on her aggressive behavior. Ignoring the behavior w ill not ad d ress the aggression and is not em p athic. Und er no circu m stances is biting the child back help fu l. Threatening to w ithd raw love is m anip u lative and anxiety-p rovoking to the child . By telling the child that she m u st love the baby, the m other is not tru ly em p athizing w ith her and m ay elicit sham e and fu rther resentm ent tow ard the baby. 47. (B) A 37-m onth-old girl is a you ng preschool age child . At this stage, she shou ld be able to state her age and gend er and be involved in or have recently com p leted p otty training. Althou gh she has significant langu age, it w ou ld be very u nu su al for her to u se her langu age sp ontaneou sly to id entify and articu late her feelings abou t her you nger sibling. Althou gh a p recociou s 3-year-old m ay be able to cou nt to 50, it w ou ld not necessarily be exp ected of her u ntil she is 5 years. She is cu rrently consid ered to be in the p reop erational stage of cognitive d evelopm ent and cannot yet ap p ly concep ts of conservation of m atter (concrete op erations). H er gross m otor skills and balance are likely not d evelop ed enou gh to rid e a bicycle. She shou ld be able to rid e a tricycle, how ever.

Answe rs : 39–55

25

48. (D ) This p atient’s p resentation is su sp iciou s for au tism sp ectru m d isord er (ASD). ASD is characterized by d eficits in social com m unication and social interaction as w ell as restricted , rep etitive p atterns of behaviors, interests, or activities. Of the choices listed , the d ifficu lty w ith im aginary p lay rep resents a d eficit in u nd erstand ing relationships, w hich m ore broad ly falls u nd er the d iagnostic criterion of d eficits in social interaction. The langu age im p airm ent criterion of the DSM -IV -TR w as elim inated in the DSM -5. Attentional d ifficu lties, im pairm ent in m otor skill d evelopm ent, or d elay in potty training are not necessarily featu res of ASD.

52. (E) Stranger anxiety occurs as part of norm al child d evelopm ent and is evid ence of the d evelopm ent of a secure attachm ent; it d oes not su ggest that a p arent is inattentive. Stranger anxiety u su ally ap p ears by 7 to 8 m onths and generally resolves w ith tim e. Typ ically, stranger anxiety is stronger tow ard com pletely u nknow n persons than tow ard those w ho are m ore fam iliar. Becau se stranger anxiety at this age is d evelop m entally app ropriate, this boy cannot be d iagnosed w ith sep aration anxiety d isord er, and he is not overly attached to his m other. Child ren w ith au tism spectrum d isord er, in fact, often lack this d evelop m ental m arker.

49. (D ) Child ren w ith au tism spectrum d isord er (ASD) have m ore favorable p rognoses if they are able to converse m eaningfu lly w ith others; though no longer p art of the DSM -5 d iagnostic criteria, the acqu isition of fu nctional language by the age of 5 years rem ains a p ositive prognostic factor. It is not u nu su al for child ren w ith ASD to be easily toilet trained , enjoy m echanical toys, be rigid ly organized in their play, and be capable of m em orizing or reciting p oem s, d ialogu e from a TV show or m ovie. These latter characteristics are not necessarily associated w ith a good prognosis.

53. (D ) This boy w ou ld be d iagnosed w ith sep aration anxiety d isord er. While an 8-m onth-old child w ou ld typically d isplay stranger anxiety, an 8-year-old w ou ld not. This behavior w ou ld have norm ally gone aw ay for m ost child ren by age 3 or 4 years. School age child ren are exp ected to be able to sep arate from the parent/ caregiver in ord er to attend school and other activities. Ind ivid u als w ith agoraphobia often refu se to leave their hom e d ue to the fear of d eveloping a panic attack. Obsessive-com pu lsive d isord er is characterized by obsessions and com pu lsions; the focu s is not related to sep aration. Social anxiety d isord er (social p hobia) involves anxiety in a social or perform ance situ ation.

50. (A) This girl likely has anorexia nervosa. H yp ercholesterolem ia is com m on in anorexia nervosa. Other find ings associated w ith the starvation state are m ild norm ocytic norm ochrom ic anem ia and leu kop enia. If vom iting is ind u ced , hyp okalem ia, hyp ochlorem ia, and m etabolic alkalosis m ay be seen. H ypercarotenem ia, cau sing yellow ing of the skin, m ay be seen if m any carrots are eaten in an attem p t to satisfy the appetite w ith a low -calorie food . TSH is not typ ically altered . 51. (B) While all of the choices are complications resulting from anorexia nervosa, a card iac arrhythmia is consid ered a major com plication and therefore alone justifies an inpatient ad mission. If the patient has mu ltip le minor com plications (e.g., anem ia, brad ycard ia, hypotension, lanu go), strong consid eration for ad mission to the hospital shou ld also be given.

54. (B) Ind ivid uals w ho d evelop schizophrenia are m ore likely to have a p arent w ith a p sychotic or schizotyp al d isord er. Ind ivid uals w ho d evelop schizop hrenia are m u ch m ore likely to have a history of social w ithd raw al and introversion rather than extroversion. Recent stressors su ch as p arental d ivorce or m oving m ay contribu te to the onset or exacerbation of som e p sychiatric d isord ers bu t not sp ecifically schizophrenia. Finally, recent d ru g u se w ou ld be m ore likely to su p p ort a d iagnosis of su bstance-ind u ced p sychosis. 55. (D ) Early-onset schizop hrenia occu rs before age 18 years, w hile child hood onset schizophrenia has an onset before the age of 15 years. Child hood onset of schizop hrenia is rare and

26

1: Child a nd Ad ole s c e nt P syc hia try

associated w ith a poorer ou tcom e. A better p rognosis is associated w ith an acu te onset, m ore affective sym ptom s, old er age at onset, good p rem orbid fu nctioning, w ell-d ifferentiated sym ptom s, and lack of a fam ily history of schizophrenia. 56. (C) Child ren w ith ADH D often suffer from low self-esteem , w hile child ren w ith m ania are m ore likely to be eu phoric. Distractibility, im p u lsivity, m otoric hyp eractivity, and p ressured sp eech m ay be seen in child ren w ith both d isord ers. 57. (E) If a school is unable to manage a child’s behavioral or emotional difficulties despite ad ded support and special placement in a separate classroom, the most appropriate next step is therapeutic d ay school placement. Therapeutic schools are d esigned for stud ents w ith psychiatric disorders that are severe enough that their home schools cannot keep the child or other stud ents safe and manage the child in a learning environment. H ome schooling, a parochial school, or a school for stud ents w ith learning d isabilities are not therapeutic in nature and d o not have teachers and staff w ho are trained to w ork w ith students w ith emotional and behavioral disord ers. Resid ential placement involves the child resid ing at the school w hile receiving therapeutic treatment and attend ing classes, and is reserved for students w hose needs cannot be met by therapeutic schools. 58. (E) Valp roic acid is an anticonvu lsant, w hich is also a m ood stabilizer u sed to treat bip olar d isord er. Du loxetine and bu propion are both antid epressants and not ap propriate for treating m ania as they m ay w orsen the sym p tom s or prom ote rapid -cycling. Methylphenid ate and m ixed am phetam ine salts are stim u lants u sed in attention-d eficit/ hyp eractivity d isord er, and are contraind icated in m ania because they m ay actu ally exacerbate m ania. 59. (D ) Typ ically, child ren are 7 or 8 years old w hen they begin to und erstand the irreversibility of d eath. To und erstand that d eath is irreversible, child ren m ust have com pleted the cognitive stage of preoperational thou ght and

d eveloped the ability to have concrete thinking. Usually, this level of cognitive m aturity is achieved betw een the ages of 6 and 10. 60. (C) This boy is displaying regression, a defense mechanism in w hich there is an attempt to return to an earlier d evelopmental phase to avoid the tension and conflict at the present level of development (e.g., d istress over the grandmother’s death). Acting out is a defense mechanism in w hich an unconscious w ish or impulse is expressed through action to avoid an accompanying affect. Denial is a d efense mechanism in w hich the aw areness of a painful aspect of reality is avoided by negating sensory d ata. Repression is a d efense mechanism in w hich an idea or feeling is expelled or w ithheld from consciousness. Somatization is the presence of physical symptoms that are a manifestation of emotional or psychological d istress. 61. (D ) Sed ation is a frequ ent ad verse effect of clonid ine u pon initiating treatm ent. With continu ed treatm ent, the sed ation usu ally subsid es. Dry m outh is a less com m on ad verse effect. H yp otension is a p ossible sid e effect as clonid ine is also used as an anti-hypertensive m ed ication. The red u ction in blood pressu re u su ally d oes not resu lt in significant sym p tom s, how ever. In general, nausea and trem or are not seen. 62. (D ) Stim u lants are the m ost com m only u sed m ed ication in the treatm ent of ADH D, and althou gh they can u nm ask tic d isord ers, ADH D treatm ent gu id elines state that stim u lants can still be u sed in ind ivid u als w ith tics. In fact, a recent stu d y fou nd that stim u lants d id not w orsen tics in those w ith tic d isord ers. Antip sychotics and MAOIs are not u sed for ADH D. SSRIs are infrequ ently u sed to au gm ent treatm ent in som e cases and are not consid ered a first-line treatm ent. Althou gh TCAs m ay be as effective as stim u lants, their p otential for ind u cing a card iac arrhythm ia or, less likely, su d d en d eath p reclu d es tricyclics from being u sed as first line for ADH D. 63. (D ) The find ing of m u ltiple fractu res, especially w hen they are of d ifferent ages, is a

Answe rs : 56–69

red flag for p hysical abu se. Even thou gh the boy m ay be scared to rep ort w hat hap p ened for fear of punishm ent, it is im portant to try to talk w ith him alone and find out as m u ch as possible. Recom m end ing extra vitam ins or lim ited sp orts d oes not ad d ress the qu estion of w hether the boy is safe at hom e. It is not ap propriate to refer the boy to an orthop ed ist or his ped iatrician prior to investigating the p ossibility of p hysical abu se. 64. (E) In ad d ressing the d elicate task of helping a child stop or alter their method of self-soothing, sensitive and collaborative approaches are much more preferable to abrupt and harsh interventions. Encouraging a child to stop thumb sucking with a more flexible reward system that is enticing for the child engages the child in making choices about their own body, rather than imposing a harsh intervention. Both hot pepper sauce and a mouth appliance are punitive in nature. However, more passive approaches like ignoring or offering gum are also not likely to be successful. Because prolonged thumb sucking can result in significant dental problems, helping a child stop thumb sucking is eventually valuable. 65. (D ) This teenager is likely su ffering from p anic d isord er. Serotonin-specific reu ptake inhibitors (SSRIs) su ch as sertraline are effective in the treatm ent of p anic d isord er. Antipsychotics, su ch as arip ip razole, are not ind icated for the treatm ent of panic d isord er. While d u loxetine, a serotonin-norep inep hrine reu p take inhibitor, has an ind ication for the treatm ent of generalized anxiety d isord er in child ren over age 7 years, it has not been stu d ied extensively in p anic d isord er. Mood stabilizers, such as carbam azep ine and valproic acid , are not as effective in p anic d isord er. 66. (E) Althou gh, the sym p tom s that the p atient d escribes are consistent w ith p anic attacks, it is im p ortant to ru le ou t any other m ed ical causes that could be em u lating that p resentation (e.g., m etabolic problem s such as thyroid d ysfu nction, electrolyte abnorm alities, anem ia). An ECG w ou ld be help fu l if she rep orted card iac sym p tom s as p art of her p resentation. This optional test can be d one after

27

the blood w ork is com plete. An EEG and referral to a neu rologist is not necessary u nless one has su sp icions of a seizu re d isord er. A p u lm onary fu nction test is used to d iagnose asthm a, w hich is a m u ch less likely etiology of this p atient’s sym ptom s. 67. (C) Identification is the process of adopting other people’s characteristics. Identification w ith a parent is important in personality formation. This girl’s behavior may occur as an attempt to imitate the doctor because she ad mires her, or it may represent an effort to cope w ith anxiety about the d octor because she fears her. Displacement is a d efense mechanism in w hich emotions are shifted from one idea or object to another that resembles the original but evokes less distress. Dissociation is a d efense mechanism in w hich a person’s character or sense of id entity is temporarily but d rastically mod ified in ord er to avoid emotional d istress. Rationalization is a d efense mechanism in w hich rational explanations are offered in an attempt to justify unacceptable attitudes, beliefs, or behaviors. Reaction formation is a d efense mechanism in w hich an unacceptable impulse is transformed into its opposite. 68. (A) In all ju risd ictions in the United States, clinicians are m an d ated rep orters of su sp ected child abu se and are requ ired to rep ort all cases. The law d oes not leave th e issu e to the clin ician’s d iscretion or p arental/ gu ard ian consent. Once the rep ort is m ad e, child p rotective services (CPS) m ay screen th e in cid ent “ou t,” if the incid ent d oes not m eet the state’s d efinition of m altreatm ent or if insu fficient inform ation is p rovid ed , CPS m ay screen th e case “in,” w hich p rom p ts fu rther investigation and p ossibly rem oving the child from the hom e u ntil an investigation is com p leted . 69. (D ) Although all of the listed choices are possible ou tcom es associated w ith child hood abuse, d ep ression is the m ost com m on one. In p articu lar, abu se of a chronic natu re can have a significant effect on p ersonality d evelop m ent, and , in ad d ition, p red isp ose the child to significant psychop athology as an ad u lt.

28

1: Child a nd Ad ole s c e nt P syc hia try

70. (C) This boy’s history is m ost consistent w ith selective m u tism . H e d oes not sp eak at school but continu es to talk at hom e. Consistent failure to sp eak in a specific social situ ation d espite sp eaking in other situ ations is not characteristic of MDD, persistent d epressive d isord er (d ysthym ia), sep aration anxiety d isord er, or social anxiety d isord er (social p hobia). Social p hobia can be d iagnosed along w ith selective m u tism . 71. (C) The m other is the m ost com m on perpetrator of intentionally p rod u cing physical or psychological sym ptom s in her child in ord er to assu m e the sick role by p roxy (factitiou s d isord er im posed on another). The victim is usu ally a preschool child , and im p osing an illness on a child is consid ered to be physical abuse w hich requ ires rep orting. 72. (A) The case represents a child w ith cond u ct d isord er. Ind ivid u als w ith cond u ct d isord er are at increased risk of later d evelop m ent of antisocial p ersonality d isord er; in fact, DSM -5 criteria for antisocial personality d isord er requ ire evid ence of a cond u ct d isord er before the age of 15 years. 73. (C) Disobed ience tow ard authority figu res is a characteristic of ODD. Aggression and violating ru les are sym p tom s consistent w ith cond u ct d isord er. If the hostile and negativistic behavior consistent w ith ODD occu rs only in the context of a m ood d isord er, the child cannot be d iagnosed w ith ODD. Relu ctance to p articip ate in tasks that requ ire ongoing m ental attention is a com m on sym ptom of ADH D. 74. (B) Alexia is the inability to read . Agnosia is the inability to recognize objects d espite intact senses. Anom ia is the sp ecific inability to nam e objects even though the object is recognizable and can be d escribed by the p atient. Ap hasia is m ore global than alexia and is an abnorm ality in either the exp ression or the com p rehension of langu age. Apraxia is an inability to p erform learned m otor skills d espite norm al strength and coord ination. 75. (A) This p atient’s sym p tom s are consistent w ith obsessive-com p u lsive d isord er (OCD).

CBT for OCD is the nonp harm acologic therap eu tic treatm ent of choice. The CBT often involves exp osu re and resp onse p revention to help extinguish com pulsions. Fam ily therapy can be help ful in ad d ressing fam ilyrelated issu es contribu ting to the child ’s sym ptom s. Grou p therapy can be p art of an OCD treatm ent, particu larly CBT. Short-term p sychod ynam ic and sup portive therap ies are not consid ered ad equ ate treatm ents alone, althou gh they m ay be help fu l in ad d ressing related issu es su ch as self-esteem and relationship s. 76–79. [76. (D ), 77. (A), 78. (B), 79. (C)] The DSM -5 revised the intellectu al d isabilities (form erly m ental retard ation, MR) severity scale to be based on the d egree of d eficits in fu nctional im p airm ent; su ch ad ap tive fu nctioning involved ad ap tive reasoning in the follow ing d om ains: concep tu al, social, and p ractical. Ind ivid u als m ay fu nction at d ifferent levels betw een each d om ain. Clinical assessm ent (i.e., d egree of fu nctional ability) is necessary to assess ad ap tive fu nctioning and therefore severity of intellectu al d isability; p sychom etrics su ch as IQ testing shou ld be taken into accou nt, bu t m ay not accu rately assess reallife fu nctioning and attainm ent of p ractical, d aily tasks. The d eficits in ad ap tive fu nctioning m u st be related to the intellectu al im p airm ents and occu r in the d evelop m ental p eriod . The rep resentative abilities for m ild , m od erate, and severe intellectu al d isability are listed in the qu estion stem . Ind ivid u als w ith p rofou nd intellectu al d isability generally are lim ited to concep tu al skills that involve the p hysical w orld , rather than sym bolic p rocesses (concep tu al d om ain), have lim ited u nd erstand ing of sym bolic com m u nication in sp eech or gestu res bu t m ay u nd erstand som e sim p le instru ctions (social d om ain), and are d ep end ent on others of all asp ect of d aily p hysical care, health, and safety (p ractical d om ain). Of note, the range of IQ for severe intellectu al d isability (form erly MR) w as historically 25 to 39, for average (norm al) intelligence w as historically 90 to 110, for m ild intellectu al d isability w as historically 55 to 70, for m od erate intellectu al

Answe rs : 70–93

d isability w as 40 to 54, for p rofou nd intellectu al d isability w as below 25, and bord erline intellectu al fu nctioning w as consid ered an IQ betw een 71 and 84. 80–83. [80. (H), 81. (D ), 82. (G), 83. (L)] Palp itations, chest pain, shortness of breath, and d iap horesis com m only occu r d u ring p anic attacks, a com p onent of p anic d isord er (H). Binge eating d isord er (D ) is characterized by sham e and loss of control over eating large am ou nts of food , w ithou t com p ensatory behaviors, and w ithou t bod y im age d istortions. Chap p ed hand s and other d erm atologic p roblem s are often p resent in obsessive-com p u lsive d isord er (G) d u e to excessive w ashing w ith w ater or cau stic cleaning agents. Trichotillom ania (hair-p u lling d isord er) (L) is m u ch m ore com m on in girls, w ith the onset frequ ently starting w ith, or shortly after, p u berty. The rep eated p u lling ou t of hair resu lts in d ecreased or com p lete loss of hair in a sp ecific area. The scalp , eyebrow s, and eyelashes are the sites m ost com m only involved . Au tism sp ectru m d isord er (B) is characterized by im p airm ents in social interactions and the d evelop m ent of stereotyp ed or rep etitive p atterns of behaviors, bu t w ithou t significant d elay in langu age skills or cognitive fu nction. Bu lim ia nervosa (C), seen in 1% to 1.5% of ad olescent girls and you ng w om en, is a bingeing and p u rging typ e of eating d isord er; anorexia nervosa (A) is an eating d isord er w ith significant w eight loss and fear of w eight gain. Illness anxiety d isord er (E), form erly hyp ochond riasis, is the p reoccu p ation that one has a seriou s d isease based on m isinterp retation of sym p tom s, d esp ite ap p rop riate m ed ical evalu ation and reassu rance. Interm ittent exp losive d isord er (F) is characterized by recu rrent ou tbu rsts of behavior, m anifesting as either verbal or behavioral aggression, that are ou t of p rop ortion to p rovocation or p recip itating stressors. Pica (I) is the eating of nonnu tritional su bstances su ch as d irt or p aint. Pyrom ania (J) requ ires several criteria, inclu d ing d eliberate and p u rp osefu l fire-setting and tension or affective arou sal before the act, w hich is not better accou nted for by cond u ct d isord er, a m anic ep isod e, or antisocial

29

p ersonality d isord er. Eye tics su ch as blinking and eye rolling are the m ost com m on initial sym p tom s in Tou rette d isord er (K). Facial tics su ch as grim acing or licking m ovem ents and vocal tics su ch as throat clearing or gru nting are the next m ost com m on initial sym p tom s. Whole-bod y tics, su ch as bod y rocking or p elvic thru sting, and self-abu sive tics, su ch as hitting, m ay d evelop later. 84–87. [84. (E), 85. (A), 86. (G), 87. (F)] Selective m u tism (E) is rare, and child ren w ho su ffer from it are only m u te in certain situ ations (e.g., school). It often involves a stressful life event such as parents’ d ivorce and the child ren are m u te by choice. Blurting ou t in class is a sym ptom of im pu lsivity that is often seen w ith ADH D (A). N o langu age im pairm ent is seen as p art of ADH D. Au tism sp ectru m d isord ers (B) have both d ifficu lties in social interaction as w ell as restricted / rep etitive interests, w hile child ren w ith social (pragm atic) com m unication d isord er (G) stru ggle w ith the practical ap p lications and nu ances of both verbal and nonverbal com m u nication. Child ren w ith langu age d isord ers (D ) u nd erstand social conventions, bu t stru ggle w ith the acqu iring and u sing langu age d u e to problem s w ith com p rehension or p rod u ction. Child ren w ith generalized anxiety d isord er (C) d isp lay excessive, d ifficu lt to control w orry, abou t a m ultitu d e of events; the anxiety or w orry also m anifests w ith at least one (in child ren) p hysical sym p tom . Social anxiety d isord er (social phobia) (F) is characterized by anxiety in situ ations w here one is exp osed to be scru tinized , ju d ged , or observed . 88–93. [88. (D ), 89. (B), 90. (A), 91. (C), 92. (E), 93. (D )] As children grow , they pass through d ifferent stages of d evelopment that represent emotional and cognitive maturation. A vital task of infancy (A) is to establish a secure sense of trust that occurs in the relationship w ith one’s responsive caregiver. Tod dlers (B) struggle w ith the increasing understanding that they are a separate being from their caregiver, and they practice separating and reuniting w ith their caregiver as a w ay to consolid ate their sense of separateness and autonomy. Preschool

30

1: Child a nd Ad ole s c e nt P syc hia try

(C) child ren’s identification w ith strong and pow erful characters such as superheroes helps them cope w ith their ow n feelings of smallness and inadequacy (as compared to their parents). This stage is also called the Oedipal phase; at this time child ren typically experience longings tow ard the parent of the opposite sex and jealousy and hostility tow ard the same sex parent. School age (D ) child ren d evelop the ability to think more logically and concretely and thus can id entify that equal amounts of liquid in tw o d ifferently shaped containers hold the same amount; once they are able to grasp this concept of conversation of liquid s, child ren have progressed into the stage of concrete operations, w here they can consistently hold onto more than one d imension at a time. Ad olescents (E) further advance in cognitive abilities, finally d eveloping the ability to think in an abstract fashion (reaching the stage Piaget d escribed as formal operations). 94. (C) The DSM -5 rates the severity of su bstance u se d isord ers on the basis of how m any criteria are p resent, w ith “m ild ” severity need ing tw o to three criteria, “m od erate” severity need ing four to five criteria, and “severe” su bstance u se d isord er requ iring m ore than six criteria. Substance u se criteria includ e trou blesom e behaviors and consequ ences associated w ith the u se, inclu d ing a failu re to m eet w ork or school expectations (su ch as failing grad es), p utting oneself at risk (su ch as d riving w hile u sing) or legal p roblem s. The criteria also incorp orate evid ence of p hysical and / or p sychological d epend ence on the su bstance; this inclu d es tolerance, w ithd raw al, heavier u se than intend ed , and d isp laying an ongoing d esire or u nsu ccessful attem pts to cut back on su bstance u se. N either u sing a su bstance w hile alone nor the frequ ency of u se are sym ptom s w hen d iagnosing a substance u se d isord er; how ever, using a su bstance m ost d ays per w eek and / or u sing alone ind icate the person’s use is m ore extensive and potentially m ore concerning, w hich shou ld prom pt fu rther qu estioning. 95. (D ) This child fu lfills criteria for cond u ct d isord er, nam ely, three of the fou r follow ing

acts: aggression tow ard p eop le and anim als, d estru ction of p rop erty, d eceitfu lness, and seriou s violations of ru les. There is nothing to su ggest an au tism sp ectru m d isord er, p articu larly becau se it w ou ld have likely been d iagnosed m u ch earlier; m ore im p ortantly, his actions seem w illfu l and not rep etitive or stereotyp ed . A child w ith bip olar d isord er m ay d isp lay im p u lsivity and aggressive behavior, how ever, the child m aintains an overall sense of right and w rong and often exp eriences regret for the ou t-of-control behavior once sym p tom s are im p roved . Also, bip olar child ren have significant m ood sym p tom s that u nd erlie their behavior. Child ren w ith cond u ct d isord er m ay also have a com orbid m ood d isord er, bu t m ood sym p tom s are not necessarily part of the cond uct d isord er constellation. There is no evid ence of psychotic symptom s, su ch as hallu cinations or d elusions that w ou ld be characteristic of child hood schizophrenia. Oppositional d efiant d isord er requ ires 6 months of negativistic, hostile, and d efiant behavior d irected mostly at au thority figures. While it m ay be a p remorbid cond ition of cond uct d isord er, it is not as severe. 96. (B) This child is exhibiting chronic irritability that is im pairing to his fu nctioning, a hallm ark of DMDD. The DSM -5 classifies DMDD, a new d iagnosis, in the d epressive d isord ers segm ent, and it is therefore im p ortant to d istingu ish it from MDD. This p atient d oes not have a p red om inantly d ep ressed m ood and neurovegetative sym ptom s (e.g., insom nia, anergia, change in ap petite, poor concentration) that w ou ld be consistent w ith MDD. Criteria For DMDD inclu d e that the sym p tom s have been consistently p resent (3 or m ore per w eek) for at least 12 m onths, w ithou t a sym ptom -free p eriod m ore than 3 w eeks; sym p tom s m ust be p resent by age 10 years, and no earlier than age 6 years; sym p tom s m u st occu r in at least tw o of three settings and m u st be severe in at least one. There is no evid ence of m anic sym p tom s (e.g., d ecreased need for sleep, elevated m ood , p ressu red sp eech, flight of id eas, and grand iosity) consistent w ith bip olar d isord er. H e d oes not ap pear to exhibit anxiety over

Answe rs : 94–99

d ifferent areas of his life to ind icate generalized anxiety d isord er. While there is som e overlap betw een DMDD and ODD (both m ay be d iagnosed ), p atients w ith ODD d o not have the pred om inate irritable m ood sym ptom s intersp ersed w ith the anger ou tbu rsts in m u ltiple settings. 97. (C) This stu d ent is m ost likely struggling d ue to variations in her acad em ic environm ent, inclu d ing external factors su ch as learning in English (not her native langu age), likely having a gap w ith d isru p tion of her schooling d ue to w ar, and possibly a lack of equ ivalent ed u cational op p ortu nity in her hom e cou ntry. Learning d ifficu lties d ue to neu rological (e.g., stroke, traum atic brain injury) or sensory (e.g., hearing or vision im pairm ent) cau ses are d istinct from specific learning d isord ers. It is im p erative that sensory d eficits are screened and treated before giving a psychiatric d iagnosis; in this case, there is no ind ication of su ch a d eficit, w hich, even if corrected , still w ou ld not accou nt for the variance of this child ’s acad em ic background . Learning d isord ers are not m u tu ally exclu sive from PTSD, bu t there is no evid ence of re-exp eriencing, arou sal, or avoid ance sym p tom s seen in PTSD. 98. (B) This ch ild is exhibiting sym p tom s of ARFID, w h ich is ch aracterized by avoid ing food or restricting food intake w ithou t intentional efforts to lose w eight and w ithou t bod y shap e concerns, that resu lt in nu tritional d eficiencies and w eight loss. Patien ts w ith ARFID d o not have the sam e

31

bod y im age concerns as in anorexia, and are often aw are they are losin g w eigh t. Th ese p atients tend n ot to binge eat as in bin ge eatin g d isord er, n or engage in com p ensatory m echanism s to lose w eight, su ch as in bu lim ia nervosa. While this child d oes have a fear of vom iting, w hich cou ld be a resu lt of a sp ecific p h obia, the overarch ing clin ical concern is her n u tritional statu s, w orsened by the d istress of eating. For som e child ren, th e avoid ance m ay be based on the food ’s sen sory characteristics. Patients w ho lack an interest in eating or have a low ap p etite m ay also qu alify for a d iagnosis of ARFID. 99. (C) This child is struggling with gender d ysphoria of childhood. The core feature of gender dysphoria (formerly called gender identity disorder) is an experience of incongruence between one’s experienced gender and one’s assigned gender. Presentations of exhibited behaviors differ pre- and post-puberty, but the core features remain. In body dysmorphic disorder, the main feature is perception of a specific body part as abnormal, not the sense that one is the wrong gender. In transvestic disorder, cross-dressing behavior generates sexual excitement in heterosexual individuals (most often) and does not challenge primary gender. Insistence on being of some other gend er is not considered delusional in the absence of other psychotic symptoms. Nonconformity to gender roles should be distinguished from gender dysphoria by the degree of distress and pervasiveness of gender-variant activities in the latter, driven by the desire to be a different gender.

This page intentionally left blank

CHAPTER 2

Adult Ps yc ho patho lo g y Que s tions

DIRECTIONS (Questions 1 through 120): For each of the multiple-choice questions in this section select the lettered answer that is the one best response in each case. Questions 1 and 2 A 22-year-old single m an is referred to you for a 1-year history of strange behavior characterized by talking to the television, accu sing local p olice of bu gging his room , and carrying on conversations w ith him self. H is m other d escribes a 3-year history of progressive w ithd raw al from social activities, and reports the p atient d ropp ed ou t of college and since has been living in his room at hom e. Attem pts to hold a job as a bu sboy at a local restaurant have abruptly end ed after d isp u tes w ith the em p loyers. 1. What is the p revalence of this patient’s likely illness in the general p op u lation? (A) (B) (C) (D) (E)

0.1% 1% 2% 3% 5%

2. The p atient’s m other inform s you that he has an id entical tw in brother. What is the likely chance of the patient’s tw in also having the sam e illness? (A) 1% (B) 10% (C) 20%

(D) 50% (E) 100% Questions 3 and 4 A 40-year-old man w ith schizophrenia comes for his regular outpatient med ication management appointments. H e reports that over the last w eek his intestines and heart have been removed . H e has subsequently w ithd raw n and been staying in his basement apartment, avoid ing friend s and family members. When asked about his lack of getting out in the w orld , he responds, “What w orld? There is no w orld!” 3. Which of the follow ing term s best d escribes this sym p tom ? (A) (B) (C) (D) (E)

Cap gras synd rom e Cotard synd rom e Folie à d eu x Fregoli d elu sion Major d ep ressive d isord er

4. Up on retu rning for a follow -u p visit 15 d ays later, the p atient now claim s that “cyborg alien robots” that look id entical to his p arents have recently rep laced his m other and father. Which of the follow ing term s best d escribes this sym ptom ? (A) (B) (C) (D) (E)

Cap gras synd rom e Cotard synd rom e Delu sional d isord er Folie à d eu x Fregoli d elu sion

33

34

2: Ad ult P sychop a thology

5. A 32-year-old w om an w ith a history of d ep ression as a teenager, now 6 d ays p ostp artu m , is brou ght into the em ergency room by her hu sband . She has not slep t for the p ast several d ays, even w hile the new born is nap p ing. She has been irritable, and has been pacing in the m id d le of the night and w eeping, w hile talking to no one in particular. Yesterd ay she began to ignore the infant, bu t tod ay she volu nteered that the child “is the Antichrist and m ust be d estroyed .” She ad m its to you that she w ants to sm other the infant in a hu m ane w ay to p revent the ap ocalyp se. Which of the follow ing d iagnoses is the m ost likely? (A) Bip olar d isord er (B) Delu sional d isord er (C) Major d ep ressive d isord er (MDD) w ith p sychotic featu res (D) Schizoaffective d isord er (E) Schizop hrenia Questions 6 through 8 An 18-year-old college freshm an w ithou t p rior p sychiatric history is brought to the em ergency room after being fou nd on her d orm itory roof d ressed only in her u nd erw ear, d esp ite freezing tem p eratu res. Cam pu s p olice report she w as flapping her hand s and clim bing the banister on the roof, stating to an u nseen other, “I w ill d o as you com m and —soar to m y d eath to fu lfill your p rophecy!” She accu sed the p olicem en of being “Satan’s horsem en” and cu rsed as she w as being taken d ow n. H er room m ates confirm that for the p ast 5 w eeks the patient has been acting bizarrely, and her speech has been increasingly d isorganized . You p lan to obtain collateral inform ation from her fam ily. 6. Which of the follow ing features w ou ld you be m ost likely to find in her history? (A) (B) (C) (D) (E)

H ead trau m a Low intelligence N eglectfu l m other Physical or sexu al abu se Progressive social w ithd raw al

7. She is ad m itted to the psychiatric unit. H er p hysical exam ination is unrem arkable, and

her blood alcohol is und etectable. H er urine toxicology results com e back positive for cannabis, w hich she confirm s she started sm oking abou t 3 m onths ago. Which of the follow ing w ou ld be the m ost ap p rop riate p rovisional d iagnosis? (A) (B) (C) (D) (E)

Brief p sychotic d isord er Schizop hreniform d isord er Schizop hrenia Su bstance-ind u ced m ood d isord er Su bstance-ind u ced p sychotic d isord er

8. She w as started on appropriate treatm ent. One year later this w om an retu rns to you r office w ith her m other for follow -u p . H er sym p tom s rem itted w ithin a m onth. H ow ever, she has not d one w ell in her freshm an year and for the p ast several m onths has continu ed to exp erience w orsening social isolation and am otivation. While she has not u sed any su bstances since she last saw you , she relu ctantly ad m its to occasionally hearing the d evil com m u nicating w ith her. She tries to ignore the com m u nication, and has taken to arranging her books in a certain m anner to p revent his controlling her thoughts. On her m ental statu s exam ination she m akes poor eye contact and her affect is blu nted . H er m other rep orts that the p atient now rarely calls hom e, thou gh before she’d d o so tw ice w eekly. Which of the follow ing is the m ost likely d iagnosis? (A) (B) (C) (D) (E)

Brief p sychotic d isord er Schizop hreniform d isord er Schizop hrenia Su bstance-ind u ced m ood d isord er Su bstance-ind u ced p sychotic d isord er

9. A 42-year-old w om an p resents to a therap ist w ith a history of d ram atic m ood sw ings since early ad olescence, w here she w ill qu ickly becom e d eep ly d ep ressed for hou rs to d ays, u su ally in resp onse to sep aration from a loved one. She also ad m its to “rage attacks,” w here she w ill break item s, scream , or scratch herself su p erficially on her arm s. She interm ittently binge d rinks and has frequ ently engaged in u np rotected sexu al intercou rse w ith new p artners. Others d escribe her as

Que s tions : 5–13

“reactive” and intense. Which of the follow ing d efense m echanism s d oes this p atient m ost likely em p loy? (A) (B) (C) (D) (E)

Altru ism Intellectu alization Sp litting Sublim ation Und oing

10. The fam ily of a 26-year-old patient w ith schizophrenia brings him in for follow -u p. H e w as initially d iagnosed at age 25 after a psychotic break that, in retrosp ect, follow ed a protracted cou rse of increasing isolation and am otivation. Desp ite d ifficu lties, he w as able to grad u ate from college and hold a fu ll-tim e job by age 23. H e reports interm ittent hallu cinations, bu t has been able to m aintain ind epend ent living and p art-tim e em p loym ent. On exam ination, he is a d isheveled m an w ho articu lates a m u ltitu d e of d elu sional beliefs w ith a sophisticated vocabu lary. Which of the follow ing characteristics in this patient is m ost strongly associated w ith a better overall prognosis? (A) (B) (C) (D) (E)

Age at presentation Gend er of p atient Insid iou s sym p tom onset Pred om inantly p ositive sym p tom s Prem orbid fu nctioning

Questions 11 and 12 A 36-year-old patient w ith no previou s psychiatric history is brou ght to the em ergency room by his fam ily. For the p ast m onth he has not been eating regu larly and has isolated him self in the ap artm ent w here he lives alone. Tw o m onths ago he stop p ed fishing, and rep orted feeling like a bu rd en on his fiancée. On m ental statu s exam ination, he d isp lays p sychom otor retard ation and rarely blinks. H e resp ond s in the negative to any qu estion asked . On physical exam ination, he app ears in no acute d istress, is afebrile, m arginally hypotensive, and m ild ly tachycard ic. H e resists you r m otions w ith strength proportional to w hat you exert and crud ely m im ics your m ovem ents. H is laboratory stu d ies are u nremarkable.

35

11. Which of the follow ing d iagnoses is the m ost likely? (A) Catatonia associated w ith m ajor d ep ressive d isord er (B) Catatonia associated w ith schizop hrenia (C) H yp othyroid ism (D) N eu rolep tic m alignant synd rom e (E) Unsp ecified catatonia 12. Recognizing the clinical situ ation in front of you , you ad m it the p atient to the p sychiatric w ard for inability to care for self. Which of the follow ing treatm ents w ou ld be best started im m ed iately? (A) (B) (C) (D) (E)

Am itriptyline Electroconvulsive therapy (ECT) Lithiu m Lorazep am Sertraline

13. A 49-year-old bank teller w ith no know n psychiatric history is referred to your office by her internist for an evaluation. For the p ast 2 m onths, she has been increasingly convinced that a w ell-know n m usic star is in love w ith her and that they have had an ongoing affair. She is w ell-groom ed , and there is no evid ence of thou ght d isord er or hallucinations. She has been fu nctioning w ell at w ork and in other social relationships. Which of the follow ing is the m ost likely d iagnosis? (A) (B) (C) (D) (E)

Brief psychotic d isord er Delusional d isord er Paranoid p ersonality d isord er Schizop hrenia Schizop hreniform d isord er

Questions 14 and 15 A 46-year-old d ivorced w om an w ith a history of m ajor d ep ressive d isord er is ad m itted to you r inp atient p sychiatric u nit follow ing an intentional acetam inop hen overd ose. She has had m u ltip le psychiatric hospitalizations w ith similar presentations. The patient reports a 3-w eek history of insom nia, d ifficulty concentrating, low energy, hop elessness, and a d ecreased app etite. She has been

36

2: Ad ult P sychop a thology

unable to w ork recently because of her d epression and has lost interest in activities she once enjoyed . She has no history of manic episod es. H er past psychiatric history is significant for a prior episod e of d epression after the birth of her second child . She has u nd ergone treatment w ith several ad equ ate trials of m ed ications, inclu d ing au gm entation w ith lithium, yet she has continued to have resid ual sym ptoms of d epression. She has never been treated w ith psychotherapy. In the past she has experienced au d itory hallucinations w hen her d epression w as most severe. H er med ical history is significant for hyp othyroid ism , w hich is ad equately m anaged w ith levothyroxine. She also had prior surgery after a leg fracture from a suicid e attem pt w here she jumped ou t of a w ind ow . 14. Which of the following factors would be most influ ential in you r recom m end ing electroconvu lsive therapy (ECT) for this patient? (A) Concu rrent thyroid d ysfu nction w ith ad equate treatm ent (B) H istory of associated p sychotic sym ptom s w ith prior d epression (C) H istory of p erip artu m d ep ression (D) Severe d ep ression that has not resp ond ed to several m ed ications (E) Treatm ent-resistant d ep ression w ith recu rrent su icid al id eation 15. The ind ications for ECT are d iscu ssed w ith the patient, as w ell as the risks, benefits, and sid e effects. She asks appropriate qu estions and consents to ECT. Fu rther m ed ical history is exp lored p rior to beginning the p roced u re. Which of the follow ing cond itions w ou ld be a relative contraind ication to p roceed ? (A) Coronary artery d isease, w ith a m yocard ial infarction 2 years ago (B) Im p lanted p acem aker for p eriod ic arrhythm ia (C) Incid entally fou nd frontal m eningiom a m easu ring 10 cm in d iam eter (D) Second -trim ester p regnancy (E) Trau m atic brain inju ry su stained in teenage years

Questions 16 and 17 A 34-year-old w hite m ale is referred by his p rim ary care physician for d epression. Upon initial interview , he com plains of feeling “blue.” H is m ental statu s exam ination reveals a d isheveled ap pearance, d ep ressed m ood , p sychom otor retard ation, and su icid al id eation w ithout plan. H is thought processes are significant for thou ght blocking and som e slow ing. Deficits w ith rem ote and short-term m em ory are noted . Ju d gm ent and insight are also im p aired . You r provisional d iagnosis is m ajor d epressive d isord er. 16. Which type of sleep d istu rbance you w ould m ost exp ect to see in this patient? (A) Decreased resp onse to sed ative d ru gs (B) Early m orning aw akening (C) Increased rap id eye m ovem ent (REM) stage latency (D) Sleep ing too d eep ly (d ifficu lty being aw akened ) (E) Sleep ing too lightly (aw akened too easily) 17. Which of the follow ing m etabolic changes w ou ld be m ost likely fou nd in this p atient? (A) Decreased m onoam ine oxid ase (MAO) activity (B) Increased catecholam ine activity (C) Increased cortisol secretion (D) Increased sex horm ones (E) Increased im m u ne fu nctions Questions 18 and 19 A 35-year-old m an is being treated for m ajor d ep ressive d isord er w ith paroxetine. H e has m issed his last tw o appointm ents, leaving m essages telling you that “I’ve been sp ectacular!” H is w ife has since called you to rep ort that her hu sband has been sp end ing m oney on frivolou s item s and overd rew the cou p le’s bank account. She states that her hu sband now quickly becom es agitated and angry. During the interview in you r office, the p atient qu estions you r cred entials and accu ses you of being m ore loyal to his w ife than to him . Most of the interview is spent interru p ting the p atient as you try to d ecip her his

Que s tions : 14–23

rap id sp eech. In his rant he threatens to cu t the brake lines on his m other-in-law ’s car becau se he feels she has been intru d ing in his m arriage. 18. Which of the follow ing is the next m ost ap propriate step ? (A) Discharge the p atient hom e as he refu ses ad m ission, bu t see him tom orrow (B) Escort the p atient (w ith p olice assistance as need ed ) to the nearest em ergency room (C) Inform the m other-in-law that she is in d anger (D) Inform the p olice that a threat has been m ad e against the m other-in-law (E) Tell the w ife to have the m other-in-law stay w ith her 19. After ad d ressing the above situ ation, the p atient is su bsequently started on valp roic acid . Which of the follow ing ad d itional p harm acologic interventions w ould be the m ost ap prop riate? (A) Check a seru m p aroxetine level (B) Cross tap er the p aroxetine to nortrip tyline (C) Discontinu e the p aroxetine (D) Initiate bu p rop ion (E) Initiate lorazep am Questions 20 and 21 A new patient com es to you r office for a p sychiatric evalu ation. H e d escribes m any years w here he experiences episod es of 5 to 7 d ays of feeling very d epressed , w ith insom nia, low energy, and poor concentration. H e d enies any suicid al id eation or prior attem p ts. Up on further history he reveals ad d itional period s w here “I feel the opp osite,” w ith d ecreased sleep, elevated energy, feeling “on top of the w orld ,” and increased sex d rive. These tim es last 1 to 2 w eeks, and he d enies any p aranoia, d elu sions, or hallu cinations. Desp ite these recu rrent ep isod es, he has alw ays been able to fu nction ad equ ately, although they rem ain d istressing to him . H e d enies any d ru g or alcohol u se, and he has no significant m ed ical p roblem s.

37

20. Which of the follow ing d iagnoses is m ost likely? (A) (B) (C) (D) (E)

Bip olar I d isord er Bip olar II d isord er Cyclothym ic d isord er Dou ble d ep ression Persistent d ep ressive d isord er

21. The patient returns w ithin 4 w eeks, and he now d escribes w orsening d epression for the past 3 w eeks, w ith ongoing insomnia, poor appetite, little energy, anhed onia, and poor concentration. H e also ad mits to passive suicidal ideation w ithout plan. Which of the follow ing w ould be the most likely d iagnosis? (A) (B) (C) (D) (E)

Bip olar I d isord er Bip olar II d isord er Cyclothym ic d isord er Dou ble d ep ression Persistent d ep ressive d isord er

Questions 22 and 23 A 26-year-old fem ale p resents to the psychiatric em ergency d ep artm ent in an acu tely d istressed , nervou s state. She com p lains of terrible anxiety, and the em ergency d ep artm ent staff is u nable to calm her d ow n or gain an ad equ ate history from the patient. On physical exam ination, she is slightly d iaphoretic, tachycard ic, and her pup ils are m ild ly d ilated . She is on no m ed ications. 22. Which of the follow ing tests w ould be the m ost useful in d eterm ining the etiology of her sym ptom s? (A) (B) (C) (D) (E)

Blood glu cose Catecholam ine m etabolites Electrocard iogram (ECG) Thyroid fu nction Urine toxicology

23. Which of the follow ing su bstances w ou ld be m ost likely to app ear on her urine toxicology resu lts? (A) Caffeine (B) Cannabis

38

2: Ad ult P sychop a thology

(C) Cocaine (D) Op iates (E) LSD Questions 24 through 26 A 46-year-old m an is ad m itted to the hospital for elective cholecystectom y. On hosp ital d ay 4, he is noted to be afebrile, but acutely d iaphoretic, tachycard ic, hypertensive, trem ulou s, and agitated . H e tears ou t his sutures and insists on leaving against m ed ical ad vice (AMA). H e is ap parently hallu cinating, ju d ging from his insistence that he be allow ed to “squ ash those bu gs on the w all” (there are none). 24. Which of the follow ing d iagnoses is the m ost likely? (A) (B) (C) (D)

Brief psychotic d isord er Delirium Delu sional d isord er Fu nctional neu rological sym p tom d isord er (E) Postoperative sepsis

25. Which of the follow ing m ed ications w ou ld be m ost app rop riate to treat his cond ition? (A) (B) (C) (D) (E)

Diazep am Disulfiram Lorazep am Phenobarbital Phenytoin

26. If u ntreated , w hat w ou ld be his m ost likely mortality rate? (A) (B) (C) (D) (E)

5% 10% 30% 50% 60%

Questions 27 and 28 A 62-year-old w om an presents to the nu rsing home w here you w ork as a consulting psychiatrist. She has a history of a bilateral tem poral lobectomy for intractable seizures. After a few w eeks at the new facility, in ad d ition to her short-term m em ory d ifficulties,

the staff reports that she has becom e extremely d ocile and d isplays very little emotion. She has a large appetite and compu lsively puts both food and nonfood item s in her mouth. She also d isplays sexual d isinhibition, often w alking out of her room w ithout her pants on. 27. Which of the follow ing clinical cond itions best d escribes her behavior? (A) (B) (C) (D) (E)

Arnold –Chiari synd rom e Klü ver–Bu cy synd rom e Möbiu s synd rom e Pick d isease Pu nch-d ru nk synd rom e

28. N eu roim aging w ou ld m ost likely show d am age to w hich tem p oral lobe stru ctu re? (A) (B) (C) (D) (E)

Am ygd ala H ip p ocam p u s Inferior horn of the lateral ventricle Insu la Su p erior tem p oral gyri

Questions 29 and 30 A 22-year-old w oman presents w ith fatigue for 4 months. She also reports irritability and poor energy, w ith dismal sleep and poor concentration. She has maintained a rigorous exercise routine, as she states it makes her feel good to run off her boyfriend’s cooking. Her oral intake has sharply d ropped and she has lost a significant amount of w eight, but she explains that she “hasn’t been as hungry” d ue to increased stress. H er thyroid stud ies are normal, her CBC reveals anemia, and she is not pregnant, in fact stating that she hasn’t had a period in several months. 29. Up on fu rther qu estioning, w hich of the follow ing qu alities w ou ld you m ost exp ect to find in her social history? (A) H er p arents likely p raise her su ccesses. (B) She has a legal history significant for reckless d riving. (C) She has asp ired to be a m od el. (D) She is of low socioeconom ic statu s. (E) She is a scholastically gifted perfectionist.

Que s tions : 24–34

30. Which of the follow ing d iagnoses is the m ost likely? (A) Anorexia nervosa (B) Avoid ant/ restrictive food intake d isord er (ARFID) (C) Bu lim ia nervosa (D) Exercise-ind u ced am enorrhea (E) Obsessive-com p u lsive d isord er (OCD) 31. A 23-year-old college stu d ent has been su ffering w ith frequ ent ep isod es of feeling “u tter d oom ” for the p ast 3 m onths. Du ring these p eriod s, he also experiences trem u lou sness, sw eating, d izziness, and tingling in his extrem ities. H e rep orts having these attacks at least once a w eek and is now becom ing fearfu l of attend ing classes lest he has an ep isod e. Which of the follow ing m ed ications w ou ld be the m ost appropriate for im m ed iate relief of his sym ptom s? (A) (B) (C) (D) (E)

Alprazolam (Xanax) Chlord iazep oxid e (Libriu m ) Divalp roex sod iu m (Dep akote) Fluoxetine (Prozac) Phenelzine (N ard il)

Questions 32 and 33 A 36-year-old grad u ate stu d ent com es to you r office because of d ifficulty sleep ing since breaking up w ith his fiancée 5 m onths ago. H e is sp end ing at least 2 hours in bed thinking abou t his ex-fiancée and w hat he cou ld have d one d ifferently. H is concentration is w orsening and he’s having d ifficu lties com pleting his coursew ork. While he feels d ow n, he is not su icid al, and he is seeking su p p ort of friend s in the p ost-breaku p p eriod . H e is attend ing m ost classes. H e d escribes feeling tenser overall, especially w hen he is at a bar. H e is having m ore trouble relaxing and has noted the new onset of low back and should er pain. 32. Which of the follow ing is the m ost likely d iagnosis? (A) Acu te stress d isord er (B) Ad ju stm ent d isord er (C) Generalized anxiety d isord er

39

(D) Major d ep ressive d isord er (E) N orm al reaction 33. Seven m onths later, your patient’s m ood has w orsened , and he feels w orthless and hop eless that he w ill never find another girlfriend . H e has lost a few p ou nd s and has not resu m ed d ating, p referring to isolate him self in his stu d io apartm ent. H e is still feed ing his d og, bu t he has had to m iss increasing am ou nts of w ork becau se he ju st can’t get him self to get read y for class. Which of the follow ing is the m ost likely d iagnosis? (A) (B) (C) (D) (E)

Acu te stress d isord er Ad ju stm ent d isord er Generalized anxiety d isord er Major d ep ressive d isord er N orm al reaction

34. A 21-year-old college stu d ent is brou ght to you r office by her p arents after com p leting her spring sem ester w ith u ncharacteristically low grad es. H er p arents rep ort that since the sp ring session end ed 2 m onths ago their d au ghter has been staying in her room , bu t is irritable w hen they try to engage her in conversation. She is not interested in fam ily barbecu es or being a cou nselor for the p ark d istrict soccer cam p . After her p arents leave the exam ination room , your patient d iscloses she has been feeling d epressed since a m onth before final exam inations, and her concentration and sleep have been “horrible.” She had su icid al thoughts after she got her report card back, but d enies them cu rrently. She also ad m its to d rinking alcohol to m ake herself feel better, and her intake ranges from 3 to 4 beers or shots of liqu or m ost d ays of the w eek. Asid e from likely alcohol u se d isord er, w hich of the follow ing d iagnoses is the m ost ap p rop riate at this tim e? (A) (B) (C) (D) (E)

Acu te stress d isord er Ad ju stm ent d isord er Am otivation synd rom e Major d ep ressive d isord er (MDD) Su bstance-ind u ced m ood d isord er

40

2: Ad ult P sychop a thology

35. A 37-year-old w om an, w ho w orks the night shift at a local grocery store taking inventory, reports that her child hood and college years w ere u neventfu l bu t hap p y. She sp end s m ost of her tim e alone w hen she is not at w ork. She d oes not ventu re out of her house and her social contacts are lim ited to w ork-related interactions w ith cow orkers. She is an avid p lant lover, and she spend s m ost of her free tim e taking care of her ind oor nursery. She reports that she is quite content w ith her life. Which of the follow ing is the m ost likely d iagnosis for this p atient? (A) (B) (C) (D) (E)

Agorap hobia Au tism sp ectru m d isord er Avoid ant p ersonality d isord er Schizoid p ersonality d isord er Schizotyp al p ersonality d isord er

36. An 18-year-old , p regnant, hu m an im m u nod eficiency virus (H IV)–p ositive w om an presents for the treatm ent of op iate u se d isord er. She reports u sing heroin for the last 8 m onths w ith su bstantial (bu t u nsu ccessfu l) efforts to qu it for the last 4 m onths. She is now hom eless and has recently been arrested for shop lifting. Which of the follow ing pharm acological therap ies w ould be the m ost ap prop riate at this tim e? (A) (B) (C) (D) (E)

Bu prenorphine Clonid ine H alop erid ol Methad one N aloxone

37. A 47-year-old w om an transfers care to you r office. She has a long history of recu rrent d epression. She also has a history since her early tw enties that is significant for interm ittent com m and au d itory hallucinations and p ersecu tory d elu sions that are often p resent even w hen her affective sym p tom s are in rem ission. She hold s a p art tim e job and lives ind ep end ently. She has been com p liant w ith m ed ications and has not been hosp italized since a su icid e attem p t over 7 years ago. Which of the follow ing is the m ost likely d iagnosis?

(A) Bipolar d isord er w ith psychotic features (B) Major d ep ressive d isord er w ith p sychotic featu res (C) Schizoaffective d isord er (D) Schizop hrenia (E) Schizop hreniform d isord er 38. A 42-year-old bu siness execu tive p resents for his first contact w ith a m ental health provid er. H e rep orts that for the last 4 m onths he has been feeling d ep ressed . H is low -energy level and p oor m otivation are affecting his job p erform ance and the CEO of his com p any ad vised him to “take a cou p le of w eeks off.” The p atient rep orts that he started feeling d ow n w hen his w ife d iscovered that he w as involved in his third extram arital affair. Since then he has m oved into a sm all apartm ent by him self. H e is sleep ing alm ost 12 hou rs every night, has a p oor app etite, and is exp eriencing financial d ifficu lty d u e to ind iscrim inate p u rchases. H e lam ents the loss of his form er self. H e reports that he u sed to have several p eriod s of tim e p er year, lasting for u p to 1 w eek, w hen he only need ed 4 to 5 hou rs of sleep, had large am ou nts of energy, spoke qu ickly, and cou ld “p arty all night and w ork all d ay.” Which of the follow ing d iagnoses is m ost likely in this case? (A) (B) (C) (D) (E)

Bipolar I d isord er Bip olar II d isord er Bord erline p ersonality d isord er Major d ep ressive d isord er N arcissistic p ersonality d isord er

39. A 52-year-old w om an w ho has been treated w ith m ed ication for 3 years for a chronic m ood d isord er reports d ry m ou th, trou ble u rinating, and occasional d izziness w hen she gets ou t of bed . Which of the follow ing m ed ications is she m ost likely being prescribed ? (A) (B) (C) (D) (E)

Divalp roex sod iu m Flu oxetine Im ip ram ine Lithiu m Phenelzine

Que s tions : 35–45

Questions 40 and 41 A 72-year-old m an is brou ght in by his w ife to you r geriatric p sychiatry clinic. The p atient’s w ife is concerned abou t his progressive confu sion over the last year. She is p articu larly d istressed that he rep eated ly asks the sam e qu estions throu ghou t the d ay. H er hu sband has becom e increasingly u nstead y on his feet and need s to u se a w alker w hen they go ou t. She w ond ers if these sym ptom s m ay be related to the m eningitis he su ffered from 3 years ago. 40. Which of the follow ing sym p tom s or signs w ou ld m ost likely be fou nd in this p atient? (A) Elevated opening p ressu re u p on lu m bar p unctu re (B) Frontal release signs (C) H istory of incontinence (D) Ocu lom otor d ifficu lties (E) Perseveration 41. Which of the follow ing w ou ld m ost likely be fou nd on neu roim aging w ith com p u ted tom ography (CT) in this p atient? (A) (B) (C) (D) (E)

Cerebellar atrop hy Dilated lateral ventricles Focal su bcortical hyp ointensities Frontoparietal atrop hy Frontotem p oral atrop hy

Questions 42 through 44 A 69-year-old w om an is brou ght to you r geriatric psychiatry clinic by her hu sband . She d enies anything is w rong, but he is concerned that for the past year or so she has been less able to recall things she read s abou t in her m agazines. She u sed to be in charge of grocery shopping, but over the past year or so she has been leaving things off the list, and her husband has taken over the job since the p atient can’t seem to organize it anym ore. She is no longer able to keep track of the cou p le’s finances, and there have been several occasions w hen her son fou nd her w and ering ou tsid e, a nu m ber of hou ses aw ay. She is otherw ise healthy, and takes a m ultivitam in d aily. H er physical exam ination is unrem arkable. Mental status exam ination reveals a healthy ap pearing w om an w ho is coop erative w ith you r qu estions,

41

and lau ghs w hen she cannot recall any of the three item s you ask her to rep eat. H er attention is intact. She has som e d ifficulties nam ing objects. She states her m ood as “good ” and her affect is euthym ic and fu ll range. 42. Which of the follow ing is the m ost ap propriate p rovisional d iagnosis? (A) (B) (C) (D) (E)

Deliriu m Major d ep ressive d isord er Major neu rocognitive d isord er Mild neu rocognitive d isord er N orm al p ressu re hyd rocep halu s

43. A w orkup is p erform ed . H er laboratory valu es are u nrem arkable, and a CT scan of her brain d em onstrates d iffu se cortical atrop hy and norm al ventricles. Which of the follow ing d iagnoses is m ost likely? (A) Major neu rocognitive d isord er d u e to Alzheim er d isease (B) Major neu rocognitive d isord er d u e to Lew y bod y d isease (C) Major neu rocognitive d isord er d u e to trau m atic brain inju ry (D) Major neu rocognitive d isord er d u e to vascular d isease (E) Unsp ecified m ajor neu rocognitive d isord er 44. If the p atient’s CT scan revealed a p rep ond erance of atrophy in the frontal and tem p oral regions, w hich of the follow ing behavioral m anifestations w ou ld be m ost likely? (A) (B) (C) (D) (E)

Decline in object nam ing Mem ory p roblem s Poor organization Social d isinhibition Word find ing d ifficu lty

45. A 48-year-old m an has been d rinking u p to 6 beers p er night d u ring the w eek and u p to 12 beers a night on the w eekend . A year ago, he had his d river ’s license su spend ed for d ru nk d riving. H is m arriage is failing becau se of these d ifficulties. Last m onth, he w as d iagnosed w ith a gastric u lcer as a result of alcohol

42

2: Ad ult P sychop a thology

consum ption. H e ad m its to an alcohol p roblem and has tried to stop on nu m erou s occasions. H e find s that he experiences insom nia if he d oes not d rink for m ore than 2 d ays. Which of the follow ing featu res of this case su ggests a severe versus m od erate alcohol u se d isord er? (A) H igh qu antity of alcohol consu m ed on a regu lar basis (B) H istory of legal problem s (C) Inability to stop d rinking d esp ite know ing the harm fu l effects (D) Marital conflict d u e to d rinking (E) N u m ber of d ifficu lties he is experiencing 46. A 26-year-old com pu ter p rogram m er w ithou t p reviou s p sychiatric history has been m arried for 4 years. H is w ife is exp ecting their first child . She rep orts that 3 m onths ago the p atient becam e p reoccup ied w ith the id ea that she becam e pregnant by another m an. Du ring this tim e, he began m issing w ork and isolated him self in his bed room . H is affect has p rogressively becom e m ore blu nted . Recently, he believes that his w ife is carrying a child conceived by extraterrestrial forces. H e u rged her to have an abortion and she refu sed . The p atient d enies any history of significant alcohol or illicit su bstance u se and his recent m ed ical evalu ation w as w ithin norm al lim its. Which of the follow ing is the most app ropriate d iagnosis? (A) Brief p sychotic d isord er (B) Delu sional d isord er (C) Major d ep ressive d isord er w ith p sychotic featu res (D) Schizop hreniform d isord er (E) Schizop hrenia 47. A 27-year-old w om an w as involved in a train d erailm ent 2 w eeks ago. Since that event, she has felt d ow n, has not slept w ell, has exp erienced rep eated and intru sive thou ghts of the accid ent, and has recu rrent nightm ares. Lately, she has changed her com m u te to avoid the train, even thou gh this ad d s 3 hours to her com m ute d aily. When on the train she has an acu te increase in her anxiety. She also often

becom es “ju m p y” w henever she hears the train going by her hom e. Which of the follow ing d iagnoses is the m ost ap p rop riate for this p atient? (A) (B) (C) (D) (E)

Acu te stress d isord er Ad ju stm ent d isord er Generalized anxiety d isord er (GAD) Posttrau m atic stress d isord er (PTSD) Major d ep ressive d isord er (MDD)

Questions 48 and 49 A 40-year-old w om an w ith a 20-year history of schizophrenia p resents to the p sychiatric em ergency d ep artm ent after a su icid e attem p t by carbon m onoxid e poisoning. She has ongoing com m and hallu cinations to harm herself, and has acted on them at least 10 tim es since her initial d iagnosis. She also has p ersistent d elu sions that she is resp onsible for w orld d isasters, w hich is w hy she m u st elim inate herself as the sou rce of pain and su ffering in the w orld . She has been tried on both typ ical and atyp ical antipsychotics, thou gh none have been effective in fu lly elim inating her p sychotic sym p tom s. H er level of fu nctioning rem ains p oor, and she p resently lives in a grou p hom e. 48. Which of the follow ing is this p atient’s m ost likely lifetim e risk of su icid e? (A) (B) (C) (D) (E)

1% 5% 10% 30% 50%

49. Which of the follow ing m ed ications w ou ld be the m ost ap p rop riate to p rescribe for this p atient? (A) (B) (C) (D) (E)

Clozap ine Flu p henazine H alop erid ol Lithiu m Zip rasid one

50. A 27-year-old internal m ed icine resid ent generally d islikes w orking in an ou tp atient clinic. Tod ay, how ever, he is looking forw ard to his

Que s tions : 46–55

clinical w ork becau se one of his app ointm ents is a follow -u p visit for a single, attractive 31-year-old w om an w ho is finishing her antibiotic regim en for treatm ent of p neu m onia. Which of the follow ing term s bests d escribes this d octor ’s resp onse to his patient? (A) (B) (C) (D) (E)

Cou ntertransference Em pathy Id entification Projection Transference

Questions 51 through 53 A 31-year-old w om an w as ad m itted to a p sychiatric u nit after attem pting suicid e by overd ose. She had recently broken u p w ith her boyfriend of 4 m onths. She also d escribes episod es of m ood lability, m arked by feelings of d ep ression and anger d irected tow ard the psychiatric resid ent w ho com pleted the rotation 5 d ays after her ad m ission. When the resid ent left, she rep orted that she w as having u rges to cu t her w rists. She has had nu m erou s ad m issions for su icid al gestu res and a lifelong history of tu m u ltu ou s rom antic relationships. As the patient nears her d ischarge d ate, she rep orts that “all the staff hates m e except for Dr. Johnson.” Dr. Johnson, a m ed ical stu d ent, had a recent d ifference of opinion w ith the nursing staff regard ing the patient’s d ischarge. 51. Which of the follow ing d iagnoses is the m ost likely for this patient? (A) (B) (C) (D) (E)

Bord erline personality d isord er Cyclothym ic d isord er H istrionic p ersonality d isord er Major d ep ressive d isord er (MDD) Schizoaffective d isord er

52. Which of the follow ing w ou ld be the m ost ap p rop riate ou tp atient treatm ent for this p atient? (A) (B) (C) (D) (E)

Antid epressants Benzod iazepines Grou p p sychotherap y Ind ivid u al psychotherap y Mood stabilizers

43

53. In this p atient, w hich typ e of p sychotherap y w ou ld be the m ost efficaciou s? (A) (B) (C) (D) (E)

Cognitive-behavioral therap y (CBT) Dialectical behavioral therap y (DBT) Grou p therap y Psychoanalysis Psychod ynam ic therap y

Questions 54 and 55 A 34-year-old w om an p resents for the treatm ent of her severe, m ed ication-refractory, m ajor d epressive d isord er. After review ing her p ast p sychiatric history and interview ing the patient, you conclu d e that she w ou ld be app ropriate for ECT. 54. In d iscu ssing the effects of ECT w ith the patient, w hich of the follow ing should you tell her is the m ost likely sid e effect? (A) (B) (C) (D) (E)

Am nesia Asp iration Card iac arrhythm ias Convu lsion fractu res Psychosis

55. For the best p ossible ou tcom e, how m any treatm ents is this patient likely to requ ire? (A) (B) (C) (D) (E)

2 4 10 15 20

Questions 56 and 57 A 38-year-old w om an p resents to your clinic telling you that she has had d istu rbing, recu rrent thou ghts abou t harm ing her 7-m onth-old infant. She im agines u sing a knife to stab her child , bu t she has no d esire to hu rt her child . As a resu lt of having these d istressing thou ghts, she has rem oved all sharp objects from her kitchen. Because of this, she has not been able to p rep are m eals at hom e and has chosen to buy fast food or take ou t for the fam ily m eals. She feels very anxiou s regard ing these, althou gh she has not shared these thou ghts w ith her hu sband .

44

2: Ad ult P sychop a thology

56. Which of the follow ing d iagnoses w ou ld be the m ost likely for this patient? (A) Delusional d isord er (B) Obsessive-com p u lsive d isord er (C) Obsessive-com p u lsive p ersonality d isord er (D) Schizophrenia (E) Schizotyp al p ersonality d isord er 57. Which of the follow ing w ould be the m ost ap propriate first-line p harm acotherap y for this cond ition? (A) (B) (C) (D) (E)

Lithium Lorazep am Flu voxam ine H alop erid ol N ortrip tyline

Questions 58 through 61 A 72-year-old w om an is brou ght to the em ergency d epartm ent from a nu rsing hom e for poor oral intake. She is afebrile, has a pu lse of 95, and a blood pressu re of 90/ 60. Mental statu s exam ination (MSE) reveals an aw ake and alert, bu t frail, m alnou rished and d ehyd rated w om an w ho is oriented to p erson only. She rep orts that the p resid ent is George Bu sh. She is easily d istracted and cannot recall any of three item s after a few m inu tes. She is irritable and sw ings at the staff w hen they try to insert an IV. The team starts IV flu id s as blood and u rine are sent to the laboratory. A chest x-ray is u nrem arkable, as is the head CT. One hou r later, she is calm er and rep orts the correct d ay, tim e, and p lace; she is less d istractible. 58. Labs are rem arkable for leu kocytosis and d irty u rinalysis. Asid e from cystitis, w hich of the follow ing is the m ost ap p rop riate d iagnosis for the p atient? (A) Deliriu m (B) Major d ep ressive d isord er (C) Major neu rocognitive d isord er d u e to Alzheim er d isease (D) Substance-ind u ced neu rocognitive d isord er (E) Unsp ecified neu rocognitive d isord er

59. The p atient is su bsequ ently ad m itted , bu t tries to hit several staff as they tend to her; she also rep eated ly tries to get ou t of bed and d em and s to be let go. Which of the follow ing w ou ld be the m ost appropriate im m ed iate pharm acologic m anagem ent for this p atient? (A) (B) (C) (D) (E)

Dip henhyd ram ine H alop erid ol Lorazep am Phenobarbital Valp roic acid

60. Which of the follow ing w ou ld be the m ost ap p rop riate long-term treatm ent of this p atient? (A) (B) (C) (D) (E)

Antibiotic therap y Chest x-ray every 6 m onths CT head every 6 m onths Ind w elling Foley catheter Intravenou s flu id s

61. Which of the follow ing is the m ost likely 6-m onth m ortality of the d iagnosis in Qu estion 58? (A) (B) (C) (D) (E)

5% 15% 20% 35% 45%

Questions 62 and 63 You are a research psychiatrist cond u cting a d ou bleblind , p lacebo-controlled trial of a new antid ep ressant. You have enrolled 200 p atients in the stud y, all of w hom m eet the criteria for u ncom plicated m ajor d ep ression. You p lan to rand om ize 100 p atients to a p lacebo m ed ication and the other 100 p atients to the exp erim ental antid ep ressant. 62. Of the 100 p atients taking the placebo, ap p roxim ately how m any p atients w ou ld be exp ected to im p rove after 6 w eeks? (A) 5 (B) 10 (C) 30

Que s tions : 56–68

(D) 50 (E) 70 63. Of the 100 p atients taking the exp erim ental antid ep ressant (assu m ing this d ru g is as efficacious as stand ard antid epressants), ap proxim ately how m any p atients w ou ld be exp ected to im p rove after 6 w eeks? (A) (B) (C) (D) (E)

10 30 50 60 80

Questions 64 and 65 You r p atient is a 38-year-old d ivorced Catholic m ale w ith a m onth long history of d ep ressed m ood , anhed onia, initial insom nia, low energy, and p oor app etite. H e ad m its to su icid al id eation w ith a plan to overd ose for the past several d ays, and he has gathered p ills this m orning. Up on fu rther history, he ad m its to sim ilar sym ptom s 5 years prior, also w ith a p rior su icid e attem p t by overd osing. H e d rinks 1 beer w eekly and d enies illicit d ru gs. H e is on no other m ed ications except for a m u ltivitam in. H e is su bsequ ently ad m itted after being m ed ically cleared . 64. What p ercentage of p atients w ith this illness eventu ally com m it su icid e? (A) (B) (C) (D) (E)

1% 5% 10% 20% 30%

65. Which of the follow ing characteristics of this patient is the nu m ber one p red ictor of a fu tu re com p leted su icid e? (A) (B) (C) (D) (E)

Age Gend er Previous attem pt Relationship statu s Religion

45

66. A 32-year-old single su ccessfu l Wall Street execu tive tells you that on w eekend s he likes to visit a d om inatrix. H is regular, paid app ointm ent w ith this p erson is d escribed as hum iliating and som ew hat p ainfu l bu t also very sexually arou sing. While he ad m its that this behavior “m ay be w eird to som e p eop le,” he enjoys it, can afford it, and it d oesn’t interfere w ith his w ork or other hobbies. Which of the follow ing d iagnoses is the m ost ap p rop riate? (A) (B) (C) (D) (E)

Fetishistic d isord er Frotteu ristic d isord er N o d iagnosis Sexu al m asochism d isord er Sexu al sad ism d isord er

Questions 67 and 68 A 28-year-old w om an com p lains of falling asleep d u ring the d ay. This problem has been occu rring for 3 m onths and is now interfering w ith her w ork as a telephone operator as she falls asleep tw o or three tim es a d ay w hile speaking w ith cu stom ers. At tim es, she find s herself falling asleep at her d esk, and she is aw akened w hen her head hits the com p u ter console in front of her. Od d ly enou gh, she reports, this can happen w hen she becom es particularly stressed ou t, for exam ple, if she is m anaging m any calls. The p atient also states that this d istu rbance has not im p roved d esp ite her sleep ing 8 hou rs each night. 67. Which of the follow ing is the m ost likely d iagnosis? (A) (B) (C) (D) (E)

Circad ian rhythm sleep–w ake d isord er H yp ersom nolence d isord er Insom nia d isord er N arcolep sy N ightm are d isord er

68. Which of the follow ing is the m ost app rop riate p harm acotherap y for this cond ition? (A) (B) (C) (D) (E)

Bu p rop ion (Wellbu trin) Flu oxetine Lorazep am Methylp henid ate (Ritalin) Phenelzine

46

2: Ad ult P sychop a thology

69. A 28-year-old w om an presents for her annu al gynecology ap p ointm ent. She com p lains that in the w eek before her period , she often exp eriences m arked anger and irritability and argu es m ore w ith her boyfriend . She also reports d im inished energy and concentration, and is sleep ing m ore than is u sual for her. These sym p tom s, in ad d ition to breast tend erness and head aches, alw ays rem it in the w eek after her m enses is finished . Which of the follow ing is the m ost likely d iagnosis? (A) (B) (C) (D)

Major d ep ressive d isord er N o d iagnosis Persistent d ep ressive d isord er Prem enstru al d ysp horic d isord er (PMDD) (E) Prem enstru al synd rom e (PMS)

70. A 37-year-old accou ntant presents to the p rim ary care clinic w ith com p laints of insom nia. H e ad m its to feeling “blu e” for 6 w eeks since getting p assed over for prom otion. Since that tim e, he has had poor sleep, often aw akening early in the m orning. H e also has had a d ecreased ap p etite w ith a 15-lb w eight loss, p oor energy, gu ilt over “not being good enou gh,” and he has been d istracted at w ork. H e ad m its to passive su icid al thoughts w ithou t a plan. Which of the follow ing w ou ld be the likely cou rse of this patient’s illness if not treated ? (A) (B) (C) (D) (E)

3 to 6 m onths 3 to 7 m onths 6 to 13 m onths 9 to 15 m onths 12 to 18 m onths

Questions 71 and 72 A 36-year-old m an is brou ght to the em ergency d epartm ent in respiratory arrest. On exam ination, he is brad ycard ic and u nresponsive, w ith constricted p u pils bilaterally. There are no other obviou s inju ries on the p atient, bu t no one is im m ed iately available to p rovid e collateral history. There is susp icion that the patient’s cond ition m ay be the result of an overd ose.

71. Which of the follow ing d ru gs is m ost likely to cau se this p resentation? (A) (B) (C) (D) (E)

Alcohol Cocaine H eroin Inhalants Phencyclid ine (PCP)

72. Which of the follow ing w ould be m ost im portant to ad m inister im m ed iately? (A) (B) (C) (D) (E)

Acetylcysteine Deferoxam ine Methad one Methylene blu e N aloxone

73. A 30-year-old male veteran from Operation Iraqi Freed om / Operation End uring Freed om presents to the m ental health clinic at the urging of his w ife. While stationed in Iraq on his third d eployment, his u nit w as hit by a road sid e bom b. H e saw a fellow Marine killed , w hile he and several other members of the comm and sustained nonlethal injuries. Since that time, he has had chronic insom nia w ith ongoing nightmares of the event as w ell as occasional flashbacks. H e d escribes alw ays feeling “on ed ge,” avoid ing crow d s, and becoming easily startled w ith loud noises. H e ad m its to regular alcohol use, especially w hen his sym ptoms are w orse. H e feels d etached from events w ith his family, preferring to isolate himself. Which of the follow ing is the most appropriate treatment to begin for this patient? (A) (B) (C) (D)

Atyp ical antip sychotic Benzod iazep ine Lithiu m Serotonin-sp ecific reu p take inhibitor (SSRI) (E) Valp roic acid

74. A 40-year-old m arried w om an is referred by her internist to a p sychologist for fu rther treatm ent. She p resents an 8-m onth history of recu rrent bou ts of “terror,” associated w ith chest p ain, tachypnea, trem ors, flushing, nausea, and fears of im p end ing d oom . These ep isod es

Que s tions : 69–79

last for ap proxim ately 15 m inu tes and d o not have a particu lar trigger. As a resu lt, she has had increasing d ifficu lty traveling far from her hom e d ue to concerns over having fu rther attacks in p u blic. Despite ad equate treatm ent w ith sertraline, she rem ains sym p tom atic and in significant d istress. Which of the follow ing psychotherapies would be the most appropriate for her cond ition? (A) Cognitive-behavioral therap y (CBT) (B) Eye m ovem ent d esensitization and rep rocessing (EMDR) (C) Insight-oriented therap y (D) Interpersonal p sychotherap y (E) Su p p ortive p sychotherap y Questions 75 and 76 You are treating a 48-year-old m arried fem ale on the inpatient m ed ical u nit for pyelonephritis; she has resp ond ed w ell to app ropriate antibiotic therapy and has been afebrile for the last 24 hou rs. You inform her of likely d ischarge if she continu es to im p rove. The next m orning, how ever, she com p lains of feeling feverish and achy, and having d ysu ria again. The nu rsing staff reports that she has a su d d en fever of 103°F. You treat the fever w ith acetam inophen and p erform a physical exam ination, ord er chest x-rays, d raw blood , and ord er a u rinalysis w ith cu ltu re. While you are aw aiting these resu lts, the nu rse inform s you that she w itnessed the patient d ipping her therm om eter into a hot cup of tea before her tem p erature w as taken. 75. Which of the follow ing d iagnoses m ost likely accou nts for this w om an’s behavior? (A) Factitiou s d isord er (B) Fu nctional neurological sym ptom d isord er (C) Illness anxiety d isord er (D) Malingering (E) Som atic sym p tom d isord er 76. Which of the follow ing is the m ost likely m otivator of this patient’s behavior? (A) Conscious d esire to assu m e sick role (B) Consciou s d esire to avoid w ork

47

(C) Unconsciou s d esire to assu m e sick role (D) Unconsciou s d esire to avoid w ork (E) Unconsciou s conflict p rod u cing sym p tom s Questions 77 and 78 A 32-year-old m an is brou ght to the psychiatric em ergency d epartm ent by the p olice after having been arrested for p u blic nu d ity. On m ental statu s exam ination, the p atient cannot sit d ow n and is only partly cooperative. H e interrup ts the interview several tim es d em and ing to contact his law yer, “becau se m y rights given to m e by God and ord ained by the Jeffersonians have been infringed .” H is sister ind icates that this p atient has been treated for at least tw o ep isod es of m ajor d ep ression in the p ast, one of w hich resu lted in a su icid e attem pt. H e is cu rrently a com p u ter p rogram m er, bu t has been increasingly stressed at w ork. One m onth ago, his girlfriend broke u p w ith him , and since then he has been increasingly irritable. Over the p ast 2 w eeks he has m axed ou t his cred it card from online gam bling sites, w hich he has continu ed to visit over the past fou r consecu tive nights. Laboratory tests show a negative d ru g screen and alcohol levels. 77. Which of the follow ing is the m ost likely d iagnosis? (A) (B) (C) (D) (E)

Ad ju stm ent d isord er Bip olar d isord er Brief p sychotic d isord er Cyclothym ic d isord er Major d ep ressive d isord er (MDD)

78. Which of the follow ing w ould be the m ost ap p rop riate pharm acologic treatm ent for this p atient? (A) (B) (C) (D) (E)

Carbam azep ine H alop erid ol Lam otrigine Lithiu m Sertraline

79. A 68-year-old m an w ithou t p rior p sychiatric history, bu t w ith a history of hypertension, hyperlip id em ia, and arthritis is ad m itted for a new left m id d le cerebral artery stroke. Which

48

2: Ad ult P sychop a thology

of the follow ing psychiatric sym ptom s w ou ld be the m ost likely as a resu lt? (A) (B) (C) (D) (E)

Anxiety Depression Mania Obsessions Panic attacks

80. A 45-year-old w om an w ith no p reviou s p sychiatric history is ad m itted to neu rology for the treatm ent of an acu te m ultiple sclerosis flare. She d oes not sm oke, d rink alcohol, or u se other illicit d ru gs. She is started on ap p rop riate therap y and by the third d ay her initial low er extrem ity w eakness has resolved . H ow ever, she also begins to act strangely, and asks you to change her room to p revent the “ninjas ou tsid e from creeping in and stealing m y sou l.” She also tells you the nurses have been p oisoning her m ed icine. Which of the follow ing d iagnoses w ou ld be the m ost likely? (A) Ad ju stm ent d isord er (B) Bipolar d isord er (C) Psychotic d isord er d u e to m u ltip le sclerosis (D) Schizophrenia (E) Substance-ind uced psychotic d isord er 81. A 42-year-old m an p resents w ith a history of recu rrent ep isod es of m ajor d ep ression, w ith one prior hospitalization. For the past several m onths he has felt increasingly d ep ressed , w ith insom nia, low ap p etite, little energy, crying sp ells, and p oor concentration. H e is not taking any current m ed ications. H e states that he has been on fluoxetine, p aroxetine, sertraline, and venlafaxine in the p ast, and , w hile they’ve all been efficaciou s, he has stop ped them d u e to significant erectile d ysfu nction. Which of the follow ing m ed ications w ou ld be the m ost ap propriate to prescribe? (A) (B) (C) (D) (E)

Aripip razole Bup ropion Citalop ram Lithium Sertraline

82. A 33-year-old male w ith a history of frequent fighting, aggression, impulsivity, and suicid e attempts is referred to a research facility. H e receives multiple tests and studies. Upon cerebrospinal fluid (CSF) analysis, d ecreased metabolites of w hich of the follow ing neurotransmitters w ould most likely be seen as compared to ind ivid uals w ithout his problems? (A) (B) (C) (D) (E)

Dop am ine Gam m a-am inobu tyric acid (GABA) Glu tam ate N orep inephrine Serotonin

83. A 38-year-old w om an w ith m u ltip le sclerosis of m od erate severity has had sym ptom s of d ep ression and m em ory loss increasing over the last year. On m ental statu s exam ination, you notice a blu nted affect and d ecreased sp eed of m ental processing. A m agnetic resonance im aging (MRI) exam ination is m ost likely to reveal w hich of the follow ing? (A) (B) (C) (D) (E)

Global cerebral atrop hy Mu ltip le p laqu es in frontal w hite m atter N orm al brain Periventricu lar lacu nar infarcts Ventricular enlargem ent

84. An 80-year-old w id ow ed w om an is ad m itted to the hospital for “confu sion.” On exam ination she is som nolent, thou gh earlier in the m orning she had been alert and aw are. She is u ncoop erative, hostile, and hallu cinating. H er insight and m em ory are p oor. The p rim ary team w ishes to know if she is “d eliriou s or d em ented .” Which of the follow ing signs/ sym ptom s in this p atient is the m ost sp ecific for d eliriu m ? (A) (B) (C) (D) (E)

Com bativeness Flu ctu ating consciou sness Poor m em ory Psychosis Uncoop erativeness

85. A happily m arried grad uate stu d ent com es to you r clinic w ith com p laints of insom nia. She has been u nable to fall asleep because

Que s tions : 80–88

she ru m inates abou t grad es, m oney, her relationship , and her you ng child ren. She has alw ays been concerned abou t these, bu t lately her w orries have gotten ou t of control. She is u nable to relax and feels constant tension in her m uscles. While she d enies sym p tom s of p anic attacks, she has noticed an increase in head aches and gastrointestinal d istu rbances over the p ast few m onths. She d enies any p roblem s w ith m ood . She d enies any recent stressors, changes to her rou tine, or changes to her hu sband ’s rou tine. Which of the follow ing is the m ost likely d iagnosis? (A) (B) (C) (D) (E)

Ad ju stm ent d isord er Generalized anxiety d isord er Obsessive-com p u lsive d isord er Panic d isord er Social anxiety d isord er (social p hobia)

86. You are a research p sychiatrist w ho is stu d ying signs and sym p tom s associated w ith certain p sychiatric d isord ers, and notice a category of patients w ho have sensory gating d eficits, short-term m em ory d ifficulties, and abnorm alities in sm ooth-p u rsu it eye m ovem ents. Which of the follow ing d iagnoses is m ost likely to be fou nd in this p atient p op u lation? (A) Attention-d eficit/ hyp eractivity d isord er (ADH D) (B) Major d ep ressive d isord er (MDD) (C) Obsessive-com p u lsive d isord er (OCD) (D) Posttrau m atic stress d isord er (PTSD) (E) Schizop hrenia 87. A 23-year-old single m ed ical stu d ent com es to you r office com p laining of d ifficu lty sleeping, excessive w orry abou t his stu d ies, his relationship w ith his p arents, and that his girlfriend of 2 years is going to break u p w ith him d espite being happ y w ith their relationship. H e feels irritable at tim es, and has significant m u scle tension. Because of these sym p tom s, his grad es have su ffered . H e d oes not rep ort a d epressed m ood . You d iagnose him w ith generalized anxiety d isord er, and p rescribe a com bination of psychotherapy and a selectiveserotonin reuptake inhibitor (SSRI). Three m onths later, the patient com es back to you r

49

office reporting that his m ood has been d ow n in the d u m p s, and he feels like he m ay never feel better. Recently, he has been thinking that life w as not w orth living anym ore and has p assive thou ghts of su icid e. H e reports ongoing thou ghts of overd osing on p ills. Which of the follow ing w ould be the m ost appropriate next step ? (A) Call his p arents and arrange a fam ily m eeting (B) Discontinu e his SSRI and have him retu rn in 1 w eek (C) Refer the p atient to the p sychiatric em ergency d ep artm ent (D) Sched u le w eekly ou tp atient visits (E) Sw itch the SSRI and have him follow -u p w ith you in 1 m onth 88. A 36-year-old m arried w hite fem ale p resents to the em ergency room w ith a 2-m onth history of d epression, term inal insom nia, fatigu e, d ecreased ap p etite, anhed onia, and excessive guilt. She feels hopeless and relu ctantly ad m its to su icid al id eation for the p ast w eek, w ith thou ghts of “taking all of m y m ed icines.” After fu rther qu estioning, she states that “I w ou ld never d o it” as she is a d evou t Catholic w ho attend s chu rch regu larly. Which of the follow ing characteristics m ost increases this p articu lar patient’s risk of su icid e? (A) (B) (C) (D) (E)

Age Gend er H op elessness Marital statu s Religion

Questions 89 and 90 A 29-year-old m arried w hite w om an w ith a p ast m ed ical history of recu rrent m igraines is brou ght to the psychiatric em ergency d epartm ent by her hu sband w ho rep orts that, d esp ite feeling d ep ressed 2 m onths ago and being com p liant w ith p rescribed treatm ent, she now has been acting bizarre for several d ays. On initial interview , the p atient states, “I feel su p erbly su p rem e, and you have no id ea w hat an am azing person I am !” The p atient is talking so rap id ly that you cannot interru p t her. H er hu sband

50

2: Ad ult P sychop a thology

rep orts that the p atient has not slep t in over a w eek, and d u ring the sam e tim e p eriod , she has p u t a d ow n p aym ent for a car, has p urchased a d iam ond tennis bracelet, and has booked an extravagant vacation. 89. Which of the follow ing is the m ost likely d iagnosis? (A) (B) (C) (D) (E)

Anxiety d isord er Bip olar and related d isord er N eu rocognitive d isord er Psychotic d isord er Som atic sym p tom d isord er

90. You suspect a medication is the cause for her current condition, but neither she nor her husband recall the medication prescribed 2 months ago. Which of the following medications is the most likely etiology? (A) (B) (C) (D) (E)

Am itriptyline Clonazep am Flu oxetine Lithium Sertraline

91. A 62-year-old m ale patient w ith schizop hrenia is brou ght into the em ergency d ep artm ent by the police for trespassing. Up on interview, he tells the p hysician that over the past 3 w eeks his television has been giving him cod ed m essages. Which of the follow ing term s best d escribes this p henom enon? (A) (B) (C) (D) (E)

H allu cination Id ea of reference Illu sion Thou ght broad casting Thou ght insertion

92. A 45-year-old p atient tells her d octor that after hearing that her hu sband d ied , she cou ld not rem em ber leaving her office and going hom e. In every other resp ect, her m em ory is intact. Which of the follow ing types of am nesia is this an exam ple of? (A) Continuou s (B) Generalized (C) Localized

(D) Retrograd e (E) Selective Questions 93 and 94 A 75-year-old man comes to you r office com plaining of poor sleep since his w ife’s d eath 1 m onth ago. Since that time, he has been unable to fall asleep, and he has felt “d ow n.” H e is slightly more isolative now , as many of the activities he enjoyed d oing w ere w ith his w ife. H is appetite is d ecreased , but he is still bathing and cooking. H e sometim es feels guilty that she d ied before him , and is angry w ith God that he is alive w ith no “soul mate” anymore. H e sometim es hears the voice of his d eceased w ife encouraging him to move on, and he d enies suicid al id eation. 93. Which of the follow ing is the m ost ap p rop riate d iagnosis at this tim e? (A) Ad ju stm ent d isord er w ith d ep ressed m ood (B) Bereavem ent (C) Major d ep ressive d isord er (D) Persistent com p lex bereavem ent d isord er (E) Schizop hrenia 94. Which of the follow ing w ou ld be the m ost app rop riate treatm ent for this p atient? (A) (B) (C) (D) (E)

H osp italize the p atient im m ed iately Prescribe an antid ep ressant Prescribe an antip sychotic Refer for brief su p portive therap y Refer for cognitive-behavioral therap y

95. When asked abou t his level of ed u cation, a 48-year-old m an w ith a history of schizoaffective d isord er, d ep ressed typ e, sp end s the next 5 m inutes d escribing his high school bu ild ing, friend s he had at the tim e, clu bs he joined , and eventu ally his high school grad u ation cerem ony. H e conclud es by saying, “And that w as the end of m y schooling.” Which of the follow ing term s d oes this response best d em onstrate? (A) Circu m stantiality (B) Loosening of associations

Que s tions : 89–100

(C) Perseveration (D) Pressured sp eech (E) Tangentiality 96. In her p sychiatrist’s office, a p atient su d d enly low ers herself to the floor, begins flailing abou t w ild ly, then flings a garbage p ail against the w all, and runs out of the office. Im m ed iately afterw ard , she returns. She is alert and oriented , yet d oes not rem em ber the incid ent. Which of the follow ing typ es of seizu res is the m ost likely? (A) (B) (C) (D) (E)

Com p lex p artial seizu re Jacksonian seizure N onep ilep tic seizu re Tem p oral lobe seizu re Tonic–clonic seizu re

97. A 26-year-old m an new ly d iagnosed w ith narcolep sy exp lains that he has ep isod es of falling d ow n w ithou t any loss of consciou sness, p recip itated by lau ghter or anger. Which of the follow ing term s best d escribes this sym p tom ? (A) (B) (C) (D) (E)

Catalepsy Catap lexy H yp ersom nia H yp nagogic hallu cinations Sleep p aralysis

98. A 21-year-old m an w ith new ly d iagnosed schizophrenia has been com p liant w ith his med ications and is less p sychotic. At his next follow -u p appointm ent, he is noted to be restless and constantly m oving. H e states that he feels as if he has to be m oving all the tim e and is u ncom fortable if he sits still. Which of the follow ing sid e effects is he m ost likely exp eriencing? (A) (B) (C) (D) (E)

Akathisia Akinesia Dystonia Rabbit synd rom e Tard ive d yskinesia

51

99. A 21-year-old w om an com p lains of recu rrent ep isod es w here she has extrem e anxiety, along w ith p alp itations, d izziness, nau sea, abd om inal cram p s, d iarrhea, tingling of her fingers, shortness of breath, and fearing she m ay d ie. These p eriod s last u nd er 30 m inu tes, and they’ve been increasing in frequency. She is u nable to p red ict w hen they w ill occu r, bu t, as a resu lt, she is often w orried abou t having another attack, and she has been u nw illing to leave her ap artm ent for several w eeks. She d rinks 1 to 2 d rinks of alcohol p er m onth and d enies illicit d ru g u se. She has no m ed ical problem s, is only on oral contraceptives, and her physical and laboratory w orku p are negative. Which of the follow ing w ould be the m ost app rop riate treatm ent? (A) (B) (C) (D) (E)

Bu sp irone Lithiu m Lorazep am Valp roic acid Venlafaxine

Questions 100 and 101 A 46-year-old m an w ith a history of hyp ertension and hyperlip id em ia p resents to his p rim ary care d octor w ith com plaints that “I’m d rinking too m u ch.” H e gives a long history of binges over 4 to 5 d ays at a tim e, w here he w ill d rink 1 to 2 pints of gin per d ay. H e has had several DUIs and is at risk of losing his job. In ad d ition, his d rinking has put a strain on his m arriage. Unfortunately he has been u nable to maintain sobriety for any significant period of tim e. H e is “stressed out” because of the consequences of his d rinking, and he ad mits to chronic, intermittent insom nia. H e has a good appetite, how ever, and his energy is ad equate u nless he is significantly hungover. H e feels helpless but d enies suicid al id eation. 100. Which of the follow ing m ed ications w ou ld be m ost appropriate to prescribe for this patient? (A) (B) (C) (D) (E)

Acam p rosate Bu p rop ion Citalop ram Lithiu m Olanzap ine

52

2: Ad ult P sychop a thology

101. Tw o w eeks later this sam e patient is seen in the em ergency room after being p icked u p by the police for d istu rbing the peace and w and ering. On exam ination he app ears confu sed , has nystagm us, and an unstead y gait. Which of the follow ing shou ld be ad m inistered im m ed iately? (A) (B) (C) (D) (E)

Glucose H alop erid ol Lorazep am N altrexone Thiam ine

102. A 36-year-old man presents to the em ergency d epartm ent after being found w ithout clothing in the street. H e has m ultiple excoriations all over his bod y, is paranoid and agitated , and states that bugs are craw ling all over him. H is vitals are as follow s: T-99.1, BP-160/ 93, P-105, R-20. Which of the follow ing substances w ould be m ost likely found in his laboratory results? (A) (B) (C) (D) (E)

Alcohol Cannabis Cocaine Opiates PCP

Questions 103 and 104 A 72-year-old m arried m ale w ith a history of hypertension, bord erline d iabetes, hyperlipid em ia, and coronary artery d isease is brou ght in by his w ife. Over the p ast year he has becom e increasingly forgetfu l, m isp lacing his keys, getting lost w hile d riving, and starting to w and er. When confronted w ith these situations he becom es d efensive, m aking up excu ses for his behavior. H is sleep is erratic, bu t he is eating w ell and enjoys w atching his favorite m ovies on the television. H is w ife d escribes extrem e m ood lability, w here he w ill be lau ghing and then crying. H e d enies suicid al id eation. H e d oesn’t d rink alcohol and d enies d ru g u se. H e w as a p rior sm oker bu t qu it 20 years ago. H is p hysical exam ination and labs are u nrem arkable. 103. As p art of his w orku p , an MRI of his brain is ord ered . What w ou ld be the m ost likely find ing?

(A) (B) (C) (D) (E)

Atrop hy of brainstem and cerebellu m Atrop hy of frontal and tem p oral lobes Enlarged ventricles Global atrop hy Periventricu lar w hite-m atter lesions

104. Which of the follow ing w ou ld best d escribe his clinical course? (A) (B) (C) (D) (E)

Continu ed im p rovem ent N o im p rovem ent or w orsening Rap id d ecline Stead y d ecline Step -w ise d eterioration

105. An 86-year-old w om an w ith m u ltip le m ed ical problem s and a recent hip fractu re is ad m itted to the intensive care u nit. While in the u nit, she aw akens at night and m istakes her intravenou s (IV) p ole for a fam ily m em ber com ing for a visit. She then becom es agitated , attem p ting to pu ll out her IV and get ou t of the hospital bed . Which of the follow ing ap p roaches is the m ost im p ortant in the u ltim ate m anagem ent of this patient? (A) (B) (C) (D) (E)

Ad m inister d ip henhyd ram ine Ad m inister halop erid ol Ad m inister lorazep am App ly soft restraints Determ ine and correct the u nd erlying cond ition

106. A 24-year-old grad u ate stu d ent in p hilosop hy is referred by his stu d ent health center for a p sychiatric evalu ation. Althou gh he claim s to have had sim ilar bu t attenu ated sym p tom s in ch ild hood , since beginn ing his thesis, he d escribes an acu te w orsening of fears that he w ill contract H IV. While he u nd erstand s the m od es and risks of contraction and p ractices safe sex, he is u nable to “get rid of these thou ghts.” As a resu lt, he feels com p elled to w ash his hand s m any tim es p er d ay, even to the p oint of their becom ing raw an d bleed ing. Desp ite his insigh t that his concerns are irrational, he is not able to stop the behaviors. A p ositron em ission tom ograp hy (PET) scan of this p atien t’s brain w ou ld

Que s tions : 101–111

m ost likely d em on strate increased activity in w hich of the follow ing stru ctu res? (A) (B) (C) (D) (E)

Am ygd ala Cau d ate nu cleus Cerebellu m H ip p ocam p u s Parietal lobes

Questions 107 and 108 A 70-year-old w id ow ed m an w ithou t significant m ed ical history is brou ght in by his d au ghter d ue to concerns abou t his being able to ad equately care for him self. H e has been fou nd lost in the d ow ntow n area on several occasions, attem p ting to w alk into strangers’ hou ses, as w ell. H e is unable to cook for him self bu t is unable to calculate the correct am ou nt for p u rchases. H is d au ghter has noticed that he is d izzy w hen stand ing u p and has a w id e-based , slow w alk. Up on interview , he ap pears d isheveled and smells of u rine. H e is friend ly and coop erative overall, althou gh easily d istracted and confu sed . While he know s his nam e, he believes the year is 1989. When confronted abou t his m em ory d ifficu lties, he states that there are sm all gnom es living in his furnitu re, w ho play tricks on him by m oving item s arou nd in his cond om iniu m . Accord ing to the d au ghter, he rarely d rinks alcohol, d oes not sm oke tobacco, and has never u sed illicit d ru gs. 107. Which of the follow ing d iagnoses is m ost likely? (A) Major frontotem p oral neu rocognitive d isord er (B) Major neu rocognitive d isord er d u e to Alzheim er d isease (C) Major neu rocognitive d isord er w ith Lew y bod ies (D) Major vascu lar neu rocognitive d isord er (E) Su bstance-ind u ced m ajor neu rocognitive d isord er 108. The above p atient is ap p rop riately d iagnosed and then placed in an interm ed iate care facility. Eight m onths later he is brou ght by the d aughter becau se the nu rsing hom e is having increasing d ifficu lty caring for him . H e is m ore confused , resp ond ing to internal stim uli, and

53

easily agitated , striking at nu rsing staff and his d au ghter. Which of the follow ing m ed ications w ou ld be m ost appropriate to prescribe in this case? (A) (B) (C) (D) (E)

Am itriptyline Clonazepam Dip henhyd ram ine Donep ezil H aloperid ol

109. A 36-year-old w om an presents w ith com p laints of a d epressed m ood for the p ast m onth. She reports p oor sleep, little app etite w ith w eight loss, low energy, d ecreased concentration, and little libid o. She ad m its to feeling hop eless and su icid al, althou gh she d enies a sp ecific p lan or intent. She is su bsequently begu n on p aroxetine 20 m g at bed tim e. Which of the follow ing sym p tom s w ou ld be m ost likely to im p rove the earliest? (A) (B) (C) (D) (E)

Decreased libid o Dep ressed m ood H op elessness Poor sleep Su icid al id eation

110. A 35-year-old m an com p lains to his therap ist that his new p artner enjoys sexual activity only w hen inflicting pain on him . This d isturbs and fru strates the patient. Which of the follow ing best d escribes the behavior exhibited by his p artner? (A) (B) (C) (D) (E)

Exhibitionistic d isord er Frotteu ristic d isord er Sexu al m asochism d isord er Sexu al sad ism d isord er Transvestic d isord er

111. An anxiou s 23-year-old Asian m ale u niversity stu d ent p resents to stu d ent health services claim ing that his p enis is shrinking into his abd om en. Desp ite reassu rances from the staff and the p hysician, he rem ains convinced of this belief. Which of the follow ing synd rom es is this p atient m ost likely su ffering from ?

54

2: Ad ult P sychop a thology

(A) (B) (C) (D) (E)

Cap gras synd rom e Koro Ku ru Taijin-kyofu sho Zar

112. A 21-year-old m ale college stu d ent is evalu ated by his college stu d ent health center after being arrested for m astu rbating outsid e of a sorority w ind ow late at night. H e ad m its to having w atched a particu lar fem ale stu d ent insid e the build ing over a period of several m onths. Which of the follow ing is the m ost likely d iagnosis? (A) (B) (C) (D) (E)

Exhibitionistic d isord er Frotteu ristic d isord er Gend er d ysp horia Transvestic d isord er Voyeuristic d isord er

113. A 45-year-old sep arated fem ale is brou ght by her brother into the em ergency room w ith the chief com p laint of “strange behavior.” She has been living w ith her brother for the p ast 2 years, and she stop ped her m ed ications 12 m onths ago. The brother states that over the past 6 m onths she has becom e increasingly paranoid and d elu sional, believing that the neighbors are sp ying on her. She insists that the Chinese governm ent is using her as an agent to com bat terrorism , and that they com m unicate their instru ctions via a w ireless signal transm itted d irectly to her brain. She is often noticed to be talking w hen no one else is in her room . In ad d ition, over the past m onth she has been staying u p m ost of the night, p acing arou nd the hou se and attem p ting to construct an anti-terrorism m achine, u sing p arts of various household electronics. She states that it is im portant that she alone com p letes her m ission, that she is the m ost senior sp y in the organization w ith a top-secret clearance “that only the Presid ent and I p ossess.” H er brother has had a hard tim e calm ing her d ow n as she sp eaks too qu ickly to follow, goes from one top ic to another, and “w on’t sit still.” Which of the follow ing d iagnoses w ou ld be the m ost likely for this patient?

(A) Bip olar d isord er (B) Delu sional d isord er (C) Major d ep ressive d isord er w ith psychotic featu res (D) Schizoaffective d isord er (E) Schizop hrenia Questions 114 and 115 A 63-year-old Vietnam veteran w ith p osttrau m atic stress d isord er (PTSD) p resents to the ou tpatient m ental health clinic. H e has been prescribed nu m erous m ed ications over the years, and he has recently been taking citalop ram 40 m g d aily. H e d enies sid e effects and strongly believes this has been the “m ost effective” m ed ication he has taken. While he d enies intru sive thou ghts or overt flashbacks, he continu es to have severe nightm ares several tim es p er w eek w hich significantly interfere in his sleep . H e rem ains “ju m p y” at tim es, and becau se of this d oesn’t attend fu nctions w here there are large crow d s. H e has not u sed alcohol or d rugs in 18 years, and he d enies any su icid al id eation. 114. Which of the follow ing m ed ications w ou ld be the m ost beneficial to p rescribe for au gm entation? (A) (B) (C) (D) (E)

Alp razolam Bu p rop ion Prazosin Risp erid one Valp roic acid

115. In ad d ition to m ed ications, this p atient w ishes to pursu e “talk therapy.” Which of the follow ing form s of p sychotherap y w ou ld be the m ost app rop riate to recom m end ? (A) (B) (C) (D) (E)

Behavior therap y Cognitive p rocessing therap y Cognitive therap y Dialectical behavioral therapy Psychod ynam ic p sychotherap y

116. A 23-year-old grad u ate stu d ent com es to the psychiatric em ergency room com p laining of anxiety. She has never been seen by a psychiatrist before and is not taking any m ed ications.

Que s tions : 112–120

H er vital signs are notable for a heart rate of 110 beats/ m in. She is also slightly d iap horetic and has m ild ly d ilated p u pils. Given her p resent state, w hich area of this p atient’s brain w ou ld m ost likely d em onstrate increased activity? (A) (B) (C) (D) (E)

Am ygd ala Basal ganglia H ip p ocam p u s Locu s ceru leu s Thalam u s

117. You are asked to give a p sychiatric consu ltation on a 28-year-old w om an w ith system ic lu pu s erythem atosu s w ho w as ad m itted to the m ed ical service. After you see her, one of you r m ed ical colleagu es tells you that she w ill no longer sp eak to any of them becau se she “hates all of them ” and now insists on seeing you becau se you are the “best d octor in the hospital.” Which of the follow ing term s best d escribes the patient’s behavior? (A) (B) (C) (D) (E)

Acting out Externalization Regression Sp litting Su blim ation

118. A 42-year-old w oman w ith recurrent episod es of major d epression is ad mitted to a med ical unit after a car accident that rend ered her unconscious. The patient regains consciousness after 3 days and corroborates that she w as, ind eed , on an antid epressant, but she says she also cannot remember w hich one. She is started on paroxetine (Paxil) for her d epression. Tw o days after beginning this med ication, she develops tachycard ia, d iaphoresis, and myoclonic jerks. The neurotransmitter most likely associated w ith the above reaction is synthesized in w hich of the follow ing central nervous system structures? (A) (B) (C) (D) (E)

Cau d ate nu cleu s Locu s ceru leus N u cleu s accu m bens Rap he nu cleu s Su bstantia nigra

55

119. A 26-year-old m an w ith schizophrenia is being d ischarged from his third ad m ission to the p sychiatric u nit in 3 years. This m ost recent stay w as p rom p ted by acu te exacerbation of com m and aud itory hallu cinations and paranoia that resulted in bizarre behaviors su ch as lying in the m id d le of the road and berating cu stom ers that cost him his restau rant job. H is sym p tom s are im p roved w ith pharm acotherap y, w hich he is tolerating, w ith m ild sid e-effects of sed ation that are grad u ally im p roving. Which of the follow ing tim e p eriod s m ost accu rately d escribes w hen his risk of su icid e is the highest? (A) At his first p sychotic break (B) Du ring his first hosp italization (C) In the w eek p reced ing this hosp italization (D) In the w eek w hile he is hosp italized (E) In the w eeks follow ing hosp italization 120. You are asked to review neu rop sychological testing for a 19-year-old p atient w ho is failing classes at his local com m u nity college. H is resu lts ind icate an intelligence qu otient (IQ) of 55. H e is has a grou p of p eers bu t is often im m atu re, and his friend s often p revent him from being taken ad vantage of as he is very gu llible. H e takes care of his p hysical need s bu t p arents ensu re that he has enou gh groceries and need to rem ind him to check his bank accou nt balance. Which of the follow ing is the m ost ap propriate d iagnosis? (A) Mild intellectu al d isability (B) Mod erate intellectu al d isability (C) N o d iagnosis (norm al intellectu al fu nctioning) (D) Severe intellectu al d isability (E) Profou nd intellectu al d isability D IRECTION S (Questions 121 through 145): The follow ing group of numbered items are preceded by a list of lettered options. For each question, select the one lettered option that is most closely associated w ith it. Each lettered option may be used once, multiple times, or not at all.

56

2: Ad ult P sychop a thology

Questions 121 through 130: Match each scenario w ith its most likely description. (A) (B) (C) (D) (E) (F) (G) (H ) (I) (J) (K) (L) (M) (N ) (O)

Alcohol intoxication Alcohol w ithd raw al Am p hetam ine intoxication Am phetam ine w ithd raw al Caffeine intoxication Caffeine w ithd raw al Cannabis intoxication Cannabis w ithd raw al Cocaine intoxication Cocaine w ithd raw al H allu cinogen intoxication Inhalant intoxication Opioid intoxication Opioid w ithd raw al Phencyclid ine intoxication

121. A 32-year-old single, m ale w ith injected conju nctiva can’t concentrate at w ork, lau ghs read ily at his cow orkers’ d ood les, feels “relaxed ,” and sp eaks slow ly w hile seem ingly focu sed on the air in front of him . 122. A 45-year-old separated male ad mitted for d epression and suicid al id eation is irritable, asks for extra food , and spend s m ost of the first d ay sleeping. The chest pain he had on ad mission has subsid ed and he has no ECG changes. 123. After recess, a ju nior high school stu d ent sm ells “fu nny,” is stu m bling, feels d izzy and nauseated , yet rem ains sm iling and says she feels “su ch a ru sh.” By the m id d le of her next class she has a head ache but otherw ise feels like she d id this m orning. 124. A 17-year-old high school stu d ent attend s a college fraternity party hosted by his eld er brother. After several hou rs, he feels m ore courageou s and approaches lad ies he w ou ld norm ally be too shy to engage, his w ord s are slightly slu rred , and he has d ifficulties m oving in a straight line. H e is slightly flu shed and notes m ild m em ory problem s for events earlier in the night.

125. A 27-year-old grad u ate stu d ent w oke u p late and skip p ed breakfast. She now has a m assive head ache and is irritable w hen she w alks into her first m orning class. She feels like falling asleep and as if she “has the flu ” by the end of the m orning. 126. A veterinarian technician is brou ght to the em ergency d epartm ent after attacking w hat he thought w as a cougar (it w as a hou secat). In the em ergency room he is febrile, ap p ears p anicked , has nystagm u s, and d em onstrates u nexpected strength and rage. 127. A 35-year-old p ilot is brou ght in for evalu ation becau se he has not slep t for d ays, and now he is anxiou s, tachycard ic, trem u lou s, and u nable to give coherent history. H is pu pils are d ilated and his blood p ressure is high, d esp ite norm al valu es 2 w eeks ago on a flight physical. 128. A 54-year-old w om an w ith arthritis com p lains of yaw ning, d iarrhea, abd om inal cram ps, and nausea. H er pup ils are d ilated and she has notable piloerection. 129. A 23-year-old m an is fou nd u nresponsive w ith slow ed breathing. H e has m ultiple scars on his arm s and he has severe m iosis. 130. On the third postop erative d ay you r 63-yearold patient becom es agitated , d em and s you rem ove the snakes from his room , and asks w hy it is so lou d at night (it is d aytim e). H e is tachycard ic, hypertensive, and trem u lous. Questions 131 through 136: Match each scenario w ith the most likely receptor responsible from the follow ing list. (A) (B) (C) (D) (E) (F)

5-H T2 A recep tor Alp ha recep tor Beta recep tor Dop am ine recep tor H istam inergic recep tor Mu scarinic recep tor

Que s tions : 121–145

131. A 32-year-old m an on clozapine exp eriences constipation. 132. A 29-year-old w om an on qu etiap ine experiences d izziness on stand ing. 133. A 52-year-old m an on risperid one exp eriences a m ilky d ischarge from his breasts. 134. A 41-year-old m an on haloperid ol experiences resolu tion of his au d itory hallu cinations and his d elu sions. 135. A 22-year-old w om an on olanzap ine exp eriences significant sed ation and w eight gain. 136. A 53-year-old m an on quetiap ine exp eriences im p rovem ent and stabilization of m ood . Questions 137 through 145: Match each scenario w ith its most likely defense mechanism from the follow ing list. (A) (B) (C) (D) (E) (F) (G) (H ) (I)

Altru ism Denial Disp lacem ent H u m or Projection Projective id entification Reaction form ation Rationalization Splitting

137. A 37-year-old married male is just told by his wife that she has been having an affair. He immediately hugs her and tells her he loves her.

57

138. You r p atient is angry w ith you and claim s you are the w orst d octor ever, as op p osed to her form er clinician, w ho actu ally listened to her. 139. You r bord erline p atient m isses seven ap p ointm ents in a row resu lting in you r term inating her treatm ent contract. 140. After slip p ing in front of you r boss on a frozen pu d d le, you exclaim that “in a form er life I w as actu ally an elite figu re skater!” 141. You r 69-year-old p atient states he ju st has a bad cold after being d iagnosed w ith m etastatic lung cancer, and states all he need s is som e hot tea and rest. 142. You argu e w ith the boss at w ork, and w hen you com e hom e you harshly groom you r cat so that she actu ally w riggles aw ay ou t of your arm s. 143. A 68-year-old w id ow volu nteers at the local veteran’s nursing hom e. 144. After rep eated ly failing to bring you r p ortion of the grou p project to class, you accu se the group lead er of forgetting to e-m ail you a rem ind er. 145. After not m atching into you r chosen sp ecialty, you say that it w as fu ll of boring nerd s anyw ay and that you really have w ay too m u ch personality to be p art of them .

Ans we rs a nd Expla na tions

1. (B) The patient’s 3- to 4-year history of bizarre behavior, d elu sions, and d ecline in social fu nctioning strongly su ggest that he has schizop hrenia. The p revalence of schizophrenia in the general p op u lation is ap p roxim ately 1%. Schizophrenia is fou nd in all societies and geograp hical areas arou nd the w orld , and its p revalence is rou ghly equ al in m ales and fem ales. 2. (D ) In tw in stu d ies, schizop hrenia’s m onozygotic concord ance is 40% to 50%, suggesting that there is a strong genetic com ponent to the illness. The p revalence of schizop hrenia in the follow ing p op u lations is: 10% to 15% in the nontw in sibling of a patient w ith schizop hrenia, 12% in the child of one p arent w ith schizop hrenia, and 40% in the child of tw o p arents w ith schizop hrenia. 3. (B) Cotard synd rome d escribes nihilistic d elu sional content; in ad d ition to lost possessions, patients m ay feel they have lost blood , heart, intestines, as w ell as believe that the w orld beyond them has been red uced to nothingness. This psychotic/ d elusional theme can be seen in m any psychotic illnesses. Capgras synd rom e is a d elusion of d oubles characterized by the belief that people have been replaced by id entically appearing im posters. Folie à d eux, or shared psychotic d isord er, is w hen a sim ilar d elusion is aroused in one person by the close influence of another; both ind ivid uals are usually closely associated for a prolonged period of tim e. The Fregoli d elusion is a variation of the d elusion of d oubles, and is the belief that fam iliar people assume the guise of strangers. While m ajor d epressive d isord er can manifest 58

w ith psychotic featu res, the prominence of psychosis w ith bizarre (not possible) d elusions and w ithou t clear d epressed m ood m akes this d iagnosis less likely. Further, though nihilistic themes and negativism can be observed in d epression, the d elusions are usually nonbizarre (potentially feasible). 4. (A) Th e belief that p eop le h ave been rep laced by im p osters is th e h allm ark of Cap gras synd rom e. Delu sion al d isord er is ch aracterized by th e p resen ce of (u su ally) n on bizarre d elu sion s. See exp lan ation s to Qu estion 3 for fu rth er d efin ition s. 5. (A) This w om an’s presentation is consistent w ith p sychosis w ith p erip artu m onset, characterized by d elu sions and hallu cinations d u ring p regnancy or w ithin 4 w eeks of d elivery. She fu rther exhibits insom nia, irritability, and m ood lability. This, cou pled w ith previou s early d ep ressive ep isod es, strongly su ggests an ep isod ic m ood d isord er. Thou gh p ostpartu m p sychosis can be d ue to d ifferent etiologies, m ost cases eventu ally m anifest as bip olar d isord er. H ow ever, only a sm all fraction of w om en w ith bip olar d isord er w ill m anifest w ith p ostp artu m p sychosis. A less frequ ent u nd erlying etiology of postpartu m psychosis is MDD. Prim ary p sychotic illnesses, su ch as d elu sional d isord er, schizoaffective d isord er, and schizop hrenia, are even m ore rare. 6. (E) Progressive social w ithd raw al is com m only seen as part of the prod rom e of schizop hrenia. All other choices—head trau m a, low intelligence, a neglectfu l m other, and abu se— have not been p roven in any conclu sive w ay

Answe rs : 1–12

to be significantly linked to schizophrenia, althou gh early theories held that a history of one or m ore w as a pred isposing factor. 7. (E) While her history strongly su ggests a p rim ary psychotic d isord er such as brief p sychotic d isord er (lasting 1 d ay–1 m onth), schizop hreniform d isord er (lasting 1–6 m onths), or schizophrenia, the presence of cannabis on her toxicology screen preclud es su ch a d iagnosis at this tim e, especially since the onset of her cannabis u se pred ates the positive sym ptom s by a few w eeks. Unless her sym p tom s p ersist after sobriety from cannabis is attained , given the prom inence of her psychotic (rather than m ood ) sym ptom s, her m ost ap prop riate d iagnosis at this tim e is a su bstance-ind u ced (specifically cannabis-ind u ced ) p sychotic d isord er. Fu rther, if her sym p tom severity rem ains ou t of proportion, she m ay eventu ally m eet criteria for a com orbid p rim ary p sychotic d isord er as w ell as a cannabis u se d isord er. 8. (C) The p atient m eets criteria A for schizop hrenia: for at least 1 m onth she has exhibited hallu cinations, d elu sions, and d isorganized sp eech. Fu rther her sym p tom s are occu rring ou tsid e of an acu te m ood ep isod e and are not d u e to another m ed ical cond ition. They have p ersisted for over a year, d esp ite sobriety from cannabis. Since her p sychotic sym p tom s have lasted longer than 6 m onths, her m ost ap p rop riate d iagnosis is schizop hrenia. Brief p sychotic d isord er is characterized by p sychotic sym p tom s lasting 1 d ay to 1 m onth. Schizop hreniform d isord er refers to sym p tom s lasting m ore than 1 m onth bu t less than 6 in the absence of concu rrent m ood d isord er, su bstance u se, or another m ed ical cond ition. 9. (C) The patient meets the criteria for bord erline personality d isord er characterized by rapid mood sw ings, efforts to avoid aband onment, chronic feelings of emptiness, intense anger outbursts, impulsivity, fluctuations betw een idealization and devaluation, and recurrent self-mutilation or suicid ality. People w ith this personality d isord er commonly employ primitive d efense mechanisms, such as d enial,

59

projective id entification, and splitting. Splitting is d ividing external objects (ind ivid uals) into “all good” or “all bad” categories. Altruism (living vicariously by helping others) and sublimation (gratifying urges in socially acceptable w ays) are mature d efenses, w hile intellectualization (using intellectual processes to avoid feelings) and und oing (acts performed to und o obsessional thoughts) are consid ered neurotic d efenses. 10. (E) Good p rem orbid fu nctioning p ortend s a better p rognosis for this p atient. Other featu res of schizophrenia that pred ict a better p rognosis inclu d e later age at p resentation, fem ale gend er, acute and rap id onset of sym ptom s (as opposed to insid ious onset), and the p resence of m ood sym p tom s. While pred om inantly p ositive sym p tom s also p red ict a m ore favorable prognosis, this patient has significant negative sym p tom s, as evid enced by his isolation and am otivation. 11. (A) This patient is presenting w ith several catatonic featu res, includ ing negativism, psychom otor slow ing, and echopraxia, w ith slight vital sign flu ctuations. Catatonia is often und erd iagnosed . While catatonia w as formerly consid ered a subtyp e of schizophrenia, it is found more frequently in affective d isord ers, especially m ajor d epressive d isord er. This patient’s history of isolation, anhed onia, psychom otor retard ation, d ecreased appetite, and gu ilt (burd en on fiancée) are m ore su ggestive of major d epressive d isord er. While hyp othyroid ism shou ld be ru led ou t, catatonia is not a com mon presentation. There is no evid ence to su ggest that he is faking symptoms for second ary gain, as in m alingering. The lack of prior psychiatric history or psychosis m akes a d iagnosis of catatonia associated w ith schizophrenia unlikely. There is no evid ence of exposu re to antipsychotics nor significant autonomic instability, so neuroleptic malignant synd rom e is u nlikely. 12. (D ) The m ainstay of treatm ent for catatonia is p harm acotherap y w ith benzod iazep ines or electroconvu lsive therap y. In this case, it is reasonable to attem p t a trial of lorazep am

60

2: Ad ult P sychop a thology

and assess for resp onse; if the patient begins to becom e u nstable or requ ires high d oses of lorazepam (>20 m g/ d ), or if his blood p ressure/ pu lse d o not tolerate the titration of lorazep am , ECT w ou ld be the ap p rop riate cou rse of action. Am itrip tyline, sertraline, or lithium —d ep end ing on w hether his d iagnosis is u nipolar or bip olar d ep ression—w ould be ap p rop riate m aintenance therap y once the catatonia is treated . 13. (B) This w om an’s presentation is m ost consistent w ith a d elusional d isord er. She has no other psychotic sym p tom s, her fu nctioning is still good , and the d elu sion is isolated to one specific belief. In ad d ition, this patient’s d elu sion is nonbizarre (i.e., cou ld possibly occu r). While d elu sional d isord er m ay consist of a bizarre (cou ld not occu r) d elu sion, it is not as com m on. She d oes not exhibit other psychotic sym ptom s, m aking brief psychotic d isord er u nlikely; fu rther, in brief p sychotic d isord er, sym ptom s m u st resolve w ithin 1 m onth w hereas hers have been occu rring for at least 2 m onths. There is no evid ence she has a p aranoid personality d isord er. Schizop hrenia and schizophreniform d isord er are unlikely since her sym p tom s are lim ited to one d elu sion, and she has no hallu cinations, negative sym p tom s, nor im p airm ent in fu nctioning. In ad d ition, the tim e cou rse is insufficient for these d iagnoses. 14. (E) This w om an has a profou nd d epression w ith m any lifetim e ep isod es, inclu d ing several severe su icid e attem p ts. She has not resp ond ed to several m ed ications or au gm entation strategies. The p ersistence of su icid ality in light of the treatm ent resistance is the m ost pressing reason to p ursu e ECT. ECT can ind u ce a rap id resp onse, and has excellent rem ission rates (u p to 80%), m arked ly m ore than for m ed ication alone. If her thyroid d ysfu nction w ere untreated , this w ould be a reasonable place to initiate treatm ent, how ever her thyroid abnorm ality has been stabilized . H er prior history of psychotic sym ptom s d oes not necessarily im p ly she w ou ld be a good ECT cand id ate, bu t it is fu rther evid ence of the severity of her illness. Perip artu m d epression

w ou ld sp u r the d ecision for ECT if she w ere currently in the postpartu m state, acutely suicid al, and / or a threat to her infant. 15. (C) There is no absolu te contraind ication to ECT. H ow ever, a sp ace-occu p ying lesion can cause d angerou s increases in intracranial pressure. Since ECT cau ses a tem porary increase in cerebral p erfu sion and the sku ll is a fixed space (volum e), the increase in blood volu m e cau ses increased intracranial p ressu re; a sp ace-occu p ying lesion red u ces the p otential reserve the skull has to offset the increase in p ressu re. Du e to the transient tachycard ia that occu rs w ith seizures, a recent m yocard ial infarction is also a relative contraind ication to ECT becau se of excess d em and s on the m yocard iu m . After at least several m onths, how ever, card iac risks of ECT are the sam e as those for general anesthesia. Pacem akers and a history of traum atic brain inju ry are not contraind ications. Pregnancy is also not a contraind ication to ECT, and the risks are the sam e as those of general anesthesia. With recent ad vances in anesthesia, fractu res are u ncommon since paralysis is attained for the d uration of the ind uced seizu re. 16. (B) While m any sleep d istu rbances have been noted in m ajor d ep ressive d isord er (as d escribed by the other choices), early m orning aw akening has been the m ost consistently linked w ith m ajor d ep ression. The d ecrease in REM sleep latency and slow w ave sleep d eficits often persist, even after treatment. 17. (C) Increased cortisol secretion in patients w ith d epression w as one of the earliest observations in biological psychiatry, and has been w ell borne out in subsequent stud ies. Levels of MAOs are unknow n. Catecholamines, sex hormones, and imm une function are d ecreased in d epression. 18. (B) The m ost ap p rop riate next step is to ensure the safety of the patient and others, and escorting the p atient to the em ergency room w ith p olice assistance gu arantees this is p reserved . Discharge is inap p rop riate at this tim e as the p atient has m ad e an active threat; w hile this

Answe rs : 13–24

p atient is likely in a m anic episod e and the hom icid al threat is related to the d elu sion of his m other-in-law , the threat m u st be taken seriously. States d iffer in the d u ty to w arn, bu t at this tim e in ord er to m aintain safety, ad m ission is necessary. If on d ischarge the threat rem ains, the target and law enforcem ent m ay need to be notified (there is a “d u ty to w arn”). It w ou ld be inap p rop riate to have the w ife stay w ith the m other-in-law as this w ould be p utting her in harm ’s w ay. 19. (C) This patient is su ffering from a m anic ep isod e, and given his anteced ent treatm ent w ith an antid ep ressant, p aroxetine, it is likely the antid ep ressant ind u ced the m anic sw itch. In ad d ition to achieving m ood stabilization through valp roate, it is critical to stop the offend ing agent. While SSRIs are occasionally u sed in bipolar p atients w ith d ep ressive ep isod es, their u se is not recom m end ed as m onotherapy d ue to this p otential to ind uce m ania. Paroxetine levels are not u sed clinically. N ortriptyline w ould be less appropriate than paroxetine, as tricyclic antid ep ressants are more likely than SSRIs to ind uce mania. Bupropion is also an antid epressant and w ould not be appropriate at this time. Lorazepam m ay help w ith agitation, bu t the more imp ortant step is to quickly stop the offend ing agent. 20. (C) This p atient gives a history consistent w ith cyclothym ic d isord er, characterized by ep isod es of hyp om ania and d ep ressive ep isod es not m eeting criteria for MDD. Treatm ent is generally the sam e as for bip olar d isord er. For a d iagnosis of bip olar I, only a m anic ep isod e is necessary, thou gh m ost p atients w ill su ffer from d ep ressive ep isod es as w ell. Bip olar II d isord er is characterized by m ajor d ep ressive ep isod es and hyp om anic ep isod es. Persistent d ep ressive d isord er (d ysthym ia) is a chronic d ep ression that m ay also m eet severity for a m ajor d ep ressive ep isod e; sym p tom s p ersist for 2 years, w ith no m ore than 2 m onths of sym p tom free p eriod s. “Dou ble d ep ression” is an old er term referring to ind ivid u als w ith p ersistent d ep ressive d isord er d evelop ing an MDD, as w ell. Accord ing to the DSM -5, the d iagnosis

61

w ou ld be p ersistent d ep ressive d isord er w ith interm ittent d ep ressive ep isod es. 21. (B) This patient is now experiencing a m ajor d ep ressive episod e, w hich, cou pled w ith his p rior hyp om anic ep isod es, w ould be consistent w ith the d iagnosis of bipolar II d isord er. See exp lanation for Qu estion 20 for rem aining d efinitions. 22. (E) In this you ng p atient on no m ed ications, a u rine toxicology screen is m ost likely to d eterm ine the cau se of her current state, p robably d u e to d ru g intoxication. Blood glu cose is im p ortant to test since hyp oglycem ia can p resent w ith m ental status changes and d iap horesis; how ever, it w ou ld be u nlikely in this otherw ise healthy w om an. Catecholam ine m etabolites w ou ld be p rem atu re at this tim e. An ECG w ou ld be p art of a w orku p to ru le ou t any arrhythm ias, bu t any abnorm alities w ou ld not necessarily id entify a sp ecific etiology or accou nt for her d ilated pu pils. Thyroid fu nction tests are im portant, as thyrotoxicosis can m anifest w ith panic-type sym p tom s, bu t again, in a healthy w om an w ith an acute onset of sym ptom s, the u rine d rug screen is m ore likely to yield an etiology. 23. (C) This p atient is m anifesting signs and sym ptom s of acute stim u lant (cocaine) intoxication, includ ing tachycard ia, d iap horesis, and p u p illary d ilation. Caffeine is not rou tinely tested on urine toxicology screens and w ou ld not cau se su ch a severe reaction. Cannabis intoxication m ay p resent w ith anxiety and p aranoia, as w ell as conju nctival injection, bu t it d oes not resu lt in d ilation of the p u p ils. Intoxication w ith heroin and other op iates w ill u su ally p resent w ith sed ation, resp iratory d ep ression, and constricted (pinpoint) pu pils. LSD w ill often cau se p rom inent hallu cinations d uring intoxication. 24. (B) This p atient has acu te m ental statu s changes, associated w ith vital sign abnorm alities and hallu cinations. This presentation is su sp iciou s for acu te d eliriu m , sp ecifically d elirium trem ens (DTs) d ue to alcohol w ithd raw al. Untreated , DTs can be lethal. The

62

2: Ad ult P sychop a thology

likelihood for onset is highest in the third to fifth d ay after the last d rink. While postoperative sep sis is another cause of d eliriu m , lack of fever m akes this less likely than alcohol w ithd raw al. Brief psychotic d isord er, functional neu rological sym ptom d isord er (conversion d isord er), and d elu sional d isord er d o not fit the acu te onset of sym ptom s w ith alterations in the level of consciou sness and vital sign abnorm alities. In ad d ition, fu nctional neu rological sym p tom d isord er d oes not m anifest w ith hallu cinations. 25. (C) Benzod iazepines are the m ainstay of treatm ent for d eliriu m trem ens (DTs). Of the choices, only d iazepam and lorazepam are benzod iazep ines; d iazep am is less id eal than oxazepam becau se it has active m etabolites and u nd ergoes extensive m etabolism in the liver. Oxazepam and lorazepam are not d ep end ent on liver function for their m etabolism ; they are therefore id eal to treat DTs in p atients w ith u nd erlying liver d isease. Disu lfiram is not appropriate in the acute treatm ent of DTs, bu t m ay have a role in su stained abstinence. Phenobarbital m ay be requ ired to ind u ce sed ation in p atients w ho d evelop seizu res d espite benzod iazepine treatm ent or those w ho requ ire high d oses of benzod iazepines (e.g., enou gh to requ ire intu bation). Phenytoin d oes not have a role in acu te DTs treatm ent. 26. (C) Deliriu m trem ens, if untreated , has a m ortality rate of close to 30%. Becau se of early recognition and im p roved treatm ents, the m ortality rate of DTs is now app roxim ately 5%. 27. (B) Klü ver–Bu cy synd rom e presents w ith d ocility, lack of fear resp onse, anterograd e am nesia, hyp erphagia, and hypersexu ality. Arnold –Chiari synd rom e d escribes a cond ition w ith hyd rocephalu s and cerebellar anatom ic and functional abnorm alities. Möbiu s synd rom e is a congenital absence of the facial nerves and nu clei w ith resu lting bilateral facial p aralysis. Pick d isease is a form of m ajor frontotem p oral neu rocognitive d isord er (d em entia), often ind istinguishable from

m ajor neu rocognitive d isord er (d em entia) d u e to Alzheim er d isease, in w hich the frontal and tem poral lobes are preferentially atrophied . Pu nch-d ru nk synd rom e d escribes an acquired m ovem ent d isord er associated w ith trau m atic d am age to the su bstantia nigra, for exam ple, from boxing. 28. (A) Klü ver–Bucy synd rom e is m ost closely associated w ith severe d am age to, or d isconnectivity of, the am ygd alae bilaterally. The hipp ocam p u s, although im p ortant to shortterm m em ory, is not d irectly involved w ith regu lating aggressive d rives, sexu al behaviors, or fear resp onses. The lateral ventricle is a CSF-containing sp ace that has no d irect neu rop sychiatric fu nctional role. The insu la, fou nd d eep w ithin the central sulcus, is m ed ial to the tem p oral lobe and not involved in Klü ver–Bu cy synd rom e. Su p erior tem poral gyri are m ore generally involved w ith p rocessing com plex aud itory inform ation su ch as the u nd erstand ing of language. 29. (E) Th is p atien t is su fferin g from an eatin g d isord er, p ossibly an orexia n ervosa. Man y p atien ts w ith eatin g d isord ers h ave stron g d rives for p erfection . Psych od yn am ically, p atien ts often feel a lack of con trol an d seek to con trol th e on e th in g th ey p erceive th ey can , th at is, food . Fu rth er, p atien ts also lack a sen se of valid ation from caregivers. Th ey d on ’t ch aracteristically h ave legal p roblem s, an d often are of a h igh socioecon om ic statu s. 30. (A) This patient’s m ost likely d iagnosis is anorexia nervosa. She exhibits intense food restriction as w ell as com p ensatory behaviors of over exercising to m ake u p for any caloric intake she m ay have had . Anem ia is not u ncom m on in patients w ith anorexia, and fatigu e is a frequ ent com p laint. The p resentation is not consistent w ith avoid ant/ restrictive food intake d isord er, w hich d escribes an eating d istu rbance regard ing food ’s sensory characteristics or the concern abou t ad verse consequences of intake not related to bod y im age. In bu lim ia nervosa, p atients engage in food binges, consum ing large qu antities of food and later

Answe rs : 25–35

feeling intense guilt and sham e. Patients w ith bu lim ia m ay also engage in com p ensatory m echanism s, such as laxatives, ip ecac, selfind u ced vom iting, and over exercise to “m ake u p” for the caloric intake. Exercise-ind u ced amenorrhea is possible, but less likely given this patient’s distorted necessity to compensate for any food intake. Many patients with anorexia also have obsessive-compulsive tendencies (perfectionism), but there is no indication that the patient has OCD. 31. (A) Benzod iazep ines are effective in acu tely stopping a p anic attack su ch as d escribed in the vignette. Alp razolam has the shortest halflife as w ell as the m ost rap id onset of action, and is therefore the agent m ost likely to abort the panic attack. H ow ever, because of these sam e qu alities, it also has significant abu se potential and should only be used for shortterm relief. Chlord iazepoxid e is a longer-acting benzod iazep ine bu t d oes not have as rap id an onset of action. Divalproate is not ind icated for this p atient in the absence of m ood instability. Flu oxetine, an SSRI, and p henelzine, an MAOI, both are ind icated for the longterm treatm ent of panic d isord er, bu t onset of sym ptom relief m ay take w eeks. As su ch, m any clinicians w ill start an SSRI bu t w ill also ad d an ad ju nctive benzod iazepine for the first few w eeks of treatm ent, until the SSRI starts to take effect. 32. (B) At this tim e his d iagnosis is m ost consistent w ith an ad ju stm ent d isord er, specifically w ith anxiety. Ad ju stm ent d isord er notes the onset of m ood and behavioral changes follow ing an acu te stressor. The sym p tom s rem it w ithin 6 m onths after the stressor. Ad ju stm ent d isord er can be sp ecified as occu rring w ith d ep ressed m ood , w ith anxiety, w ith m ixed anxiety and d ep ressed m ood , w ith d istu rbance of cond u ct, w ith m ixed d istu rbance of em otions and cond u ct, and as w ell as rem aining u nsp ecified . In acu te stress d isord er, the p atient is exp osed to a trau m atic event and exp eriences reexp eriencing (e.g., flashbacks), num bing, and increased arou sal sym p tom s for u p to 1 m onth after the event. A breaku p is not typically “trau m atic” in the sense that life

63

is not u su ally threatened . While this p atient has an inability to relax an associated m uscle tension, sym p tom s com m only associated w ith generalized anxiety d isord er, his w orries are lim ited to the recent relationship , and his sym ptom s have not been long-stand ing. The p atient d oes not m eet criteria for m ajor d epressive d isord er at this tim e, bu t if his sym ptom s p ersist or w orsen, this d iagnosis shou ld be consid ered . This p atient’s sym ptom s are m ore than w hat w ou ld be exp ected for a norm al reaction—his sleep has been d isru p ted and it is starting to affect his schoolw ork for the p ast 5 w eeks. 33. (D ) At this tim e the p atient’s cond ition has w orsened , and he now has a p ervasive d ep ressed m ood and d istu rbance in sleep, ap p etite, concentration as w ell as associated hop elessness and w orthlessness. In ad d ition to w orsening self-care, his sym p tom s are now significantly affecting his schoolw ork and qu ality of life. H e therefore now m eets criteria for m ajor d epressive d isord er, single episod e, and w arrants ap p rop riate treatm ent, w hether w ith m ed ication, therap y, or both. 34. (E) While this p atient’s sym p tom s are consistent w ith MDD, her ongoing alcohol use preclu d es the d iagnosis at this tim e; therefore, substance-ind u ced m ood d isord er is the m ost likely d iagnosis. If she rem ains sober for a p eriod of tim e and her d epressive sym ptom s continu e, she w ou ld then be d iagnosed w ith MDD. Am otivation synd rom e is a controversial synd rom e associated w ith cannabis u se, and is not app rop riate in this case given the lack of cannabis u se. In acu te stress d isord er, the patient is exp osed to a trau m atic event and exp eriences reexp eriencing (e.g., flashbacks), nu m bing, and increased arou sal sym p tom s for u p to 1 m onth after the event. Ad ju stm ent d isord er is characterized by the onset of m ood and behavioral changes follow ing an acu te stressor that are out of proportion to a typical resp onse. 35. (D ) Persons w ith schizoid p ersonality d isord er are d etached , reclusive, d em onstrate a restricted range of affect, and d o not m ind the

64

2: Ad ult P sychop a thology

lack of social interaction. Agorap hobia is tied to the fear of having panic-type sym ptom s in p ublic. Su ch sym p tom s are not m entioned in this case. Autism spectru m d isord er is a d evelopm ental d isord er characterized by d eficits in social com m u nication and w ith rep etitive, restricted behaviors and interests. Persons w ith avoid ant p ersonality d isord er are shy and fearful of social rejection. H ow ever, their lack of socialization is d istressing to them . Schizotypal ind ivid uals can have schizoid featu res but they also have bizarre thinking and an od d or eccentric m anner. 36. (D ) Methad one m aintenance is the m ost ap p rop riate p harm acological therap y for the treatm ent of op iate (heroin) u se d isord er, even in p regnancy. Methad one m aintenance, esp ecially w hen com bined w ith p sychosocial services and obstetric m onitoring, significantly im p roves neonatal ou tcom es and obstetric ou tcom es for w om en ad d icted to heroin. The low est effective d ose shou ld be u sed d u ring p regnancy. There are no know n teratogenic effects from m ethad one. Wom en m ay requ ire higher d oses of m ethad one in the third trim ester d u e to increased m etabolism of the d ru g at that tim e. Bu p renorp hine is a p artial op iate agonist that has a role in m aintenance treatm ent, bu t its safety d u ring p regnancy and breastfeed ing has not been d efinitively established , and its u se in p regnancy m ay lead to w ithd raw al sym p tom s in op ioid -d ep end ent fetu ses. Clonid ine can be u sed to treat hyp ertension and other sym p tom s d u ring acu te op iate w ithd raw al. While halop erid ol (a first-generation antip sychotic) m ay be u sed safely in p regnancy, it has no u se for the treatm ent of severe op iate u se d isord er, u nless agitation from acu te w ithd raw al is p roblem atic. N aloxone is an op iate antagonist, u sed for op iate overd ose, not ap p rop riate in this case. 37. (C) This p atient exhibits ep isod ic m ood sym p tom s as w ell as chronic p sychotic sym p tom s ou tsid e of those m ood ep isod es, m aking her m ost likely d iagnosis schizoaffective d isord er. There is no evid ence of p rior m anic ep isod es, m aking bip olar d isord er u nlikely.

If the hallu cinations and d elu sions only occu rred in the context of her d ep ressive ep isod es, the m ost likely d iagnosis w ou ld be m ajor d ep ression w ith p sychotic featu res. While schizop hrenia w ou ld be on the d ifferential, her affective (d ep ressive) sym p tom s are p rom inent. Schizop hreniform d isord er is characterized by sym p tom s of schizop hrenia, w hich last from 1 to 6 m onths, so it is not ap p rop riate given the chronicity of her sym p tom s. 38. (B) This p atient’s history is consistent w ith a m ood p attern d efined by p eriod s of hyp om ania (sym p tom s of m ania not severe enou gh to cau se occu p ational d ysfu nction or p sychiatric hosp italization) and cu rrently a m ajor d ep ressive ep isod e. H yp om ania w ith m ajor d ep ression d efines bip olar II d isord er. In bip olar I d isord er, the m ania is m ore severe, cau sing notable occu p ational d ysfu nction, p sychotic sym p tom s, or hosp italization. While this p atient is in the m id st of a m ajor d ep ressive ep isod e, his history of hyp om ania ind icates a bip olar d iagnosis; this is an im p ortant d istinction becau se im p rop er treatm ent w ith antid ep ressants can p recip itate a m anic ep isod e. Bord erline p ersonality d isord er is characterized by a p ersistent p attern of u nstable relationship s, m ood states, and self-im age. N arcissistic p ersonality d isord er is characterized by a p ervasive p attern of grand iosity, need for ad m iration, and lack of em p athy. 39. (C) Dry m ou th, d izziness (associated w ith hypotension), and u rinary hesitancy are d u e to anticholinergic and antiad renergic effects of tricyclic antid ep ressants (TCAs) su ch as im ip ram ine. Divalp roex sod iu m is a m ood stabilizer that m ay cau se gastrointestinal u p set and is com m only associated w ith sed ation and trem or at higher d oses. Fluoxetine, a serotonin-sp ecific reup take inhibitor (SSRI), is m ost often associated w ith gastrointestinal u p set, sexu al d ysfu nction, and activation/ agitation. Lithiu m is another m ood stabilizer u sed in bip olar d isord er, m ost often causing p olyu ria, p olyd ip sia, trem or, and m ental confusion at higher d oses. Phenelzine, a

Answe rs : 36–44

m onoam ine oxid ase inhibitor (MAOI), can be associated w ith hypotension bu t is less likely to have anticholinergic effects. 40. (C) This patient likely has normal pressure hydrocephalus (NPH), one of the few potentially reversible causes of neurocognitive disorder (dementia). The classic triad of NPH is confusion, gait apraxia, and incontinence. Elevated opening pressure is not found in NPH (thus the “normal pressure” part of the d iagnosis). Frontal release signs and perseveration are nonspecific findings common in demented patients but not specific to NPH. Oculomotor difficulties are a part of the Wernicke–Korsakoff syndrome. 41. (B) In N PH , CT scan com m only reveals d ilated ventricles thought to be the resu lt of increased p ressu re w aves im p inging w ithin the ventricu lar system . Cerebellar atrop hy is seen m ost often in congenital d isord ers and alcoholism . H ypointensities found in subcortical areas are often ind icative of lacu nar strokes. Frontoparietal atrop hy is seen in m ajor neu rocognitive d isord er d ue to Alzheim er d isease. Frontotem poral atrophy is fou nd in Pick d isease, an u ncom m on type of frontotem p oral neu rocognitive d isord er (d em entia). 42. (C) This p atient is m ost likely su ffering from m ajor neu rocognitive d isord er (d em entia), characterized by progressive im p airm ent of cognitive fu nction in the absence of d eliriu m . Mem ory im pairm ent and loss of fu nction in at least one other cognitive d om ain are hallm arks. While som e loss of cognitive fu nction is exp ected w ith age (m ild neu rocognitive d isord er d oes not interfere w ith capacity for ind ep end ence), it is too severe and this p atient is no longer able to fu nction ind ep end ently. Deliriu m is characterized by acu te m ental statu s change w ith d eficits in attention (i.e., fluctu ating levels of consciousness). While m ajor d ep ressive d isord er can present w ith neurocognitive d eficits (“pseu d od em entia”), there is no ind ication of d ep ression. This p atient d oes not d em onstrate the gait apraxia and u rinary incontinence characteristic of norm al p ressu re hyd rocephalu s.

65

43. (A) The most common type of major neurocognitive d isord er (d ementia) is d ue to Alzheimer d isease, w hich demonstrates diffuse atrophy on neuroimaging. Major neurocognitive d isord er d ue to Lew y bod y d isease is clinically similar to Alzheimer’s, but also includ es hallucinations, parkinsonian features, and extrapyramid al signs. Major neurocognitive disorder d ue to traumatic brain injury is characterized by emotional lability, dysarthria, and impulsivity. Major neurocognitive d isord er due to vascular d isease (formerly multi-infarct d ementia) is the second most common type of dementia, and involves small w id espread lacunar infarcts. It is seen in those w ith hypertension or other card iovascular d isease; this patient d oes not have either cond itions. Unspecified major neurocognitive d isord er is reserved for patients in w hom the precise etiology cannot be determined w ith sufficient certainly to make the appropriate attribution. All the major neurocognitive d isord ers must be d istinguishes from major d epression; “pseud od ementia” refers to a cond ition w here the patient demonstrates symptoms consistent w ith memory d ifficulties, but in the absence of a dementia; it is often seen in d epression. Patients w ith pseud od ementia usually have more d epressive symptoms, have more insight into their symptoms than d emented patients, and w ill often have a prior history of d epression. 44. (D ) Major frontotem p oral neu rocognitive d isord er (form erly Pick d isease) is characterized by p referential atrop hy of the frontotem p oral region, as op p osed to the p arietotem p oral d istribu tion seen in Alzheim er d isease. Patients m ay exhibit either a behavioral variant or langu age variant, bu t m any have featu res of both. Patients w ith the behavioral variant m ust show three or m ore behavioral sym ptom s (behavioral d isinhibition, ap athy or inertia, loss of sym p athy or em p athy, p erseverative/ stereotyped / ritualistic behaviors, or hyperorality and d ietary changes) or a p rom inent d ecline in social cognition and / or execu te abilities. Patients w ith the langu age variant display a decline in language ability (difficulties in speech production, naming, word finding, comprehension).

66

2: Ad ult P sychop a thology

45. (E) The DSM -5 d istingu ishes the severity of alcohol (and other substance) u se d isord ers on the basis of how m any criteria are m et. Criteria A includ e evid ence of a problem atic pattern of alcohol u se that resu lts in im pairm ent and / or d istress. Patients m eeting 2 to 3 sym p tom s m eet criteria for m ild alcohol u se d isord er; those that have 4 to 5 sym ptom s m eet criteria for m od erate alcohol u se d isord er; those that m eet 6 or m ore of the possible listed 11 sym ptom s can be d iagnosed w ith severe alcohol u se d isord er (form erly alcohol d ep end ence). The DSM -5 d oes not d istingu ish abu se from d ep end ence, and requ ires the nu m ber of p roblem atic sym ptom s to d eterm ine severity. This p atient d rinks alcohol in large am ou nts, has had unsu ccessfu l efforts to cu t d ow n, has cravings, has had persistent interpersonal p roblem s d ue to d rinking, has had recu rrent p hysical problem s d ue to d rinking, and has had w ithd raw al sym p tom s; he m eets at least six criteria and therefore can be d iagnosed w ith alcohol u se d isord er, severe. 46. (D ) This p atient w ould m eet the criteria for schizophrenia excep t that the d u ration of his illness has been less than 6 m onths. Brief p sychotic d isord er m ay present w ith psychosis bu t the d uration of the d istu rbance m ust be 1 m onth or less. This patient’s d elu sions are too bizarre to typ ically be consistent w ith d elu sional d isord er, and the age of onset for d elu sional d isord er is u su ally m id d le age. There is no evid ence of a d ep ressive d isord er at this tim e. 47. (A) This patient has sym p tom s consistent w ith PTSD or acute stress d isord er (i.e., trau m atic event, intrusive m em ories of trau m a, avoid ance of rem ind ers, hypervigilance), bu t the fact that the trau m a w as only 3 w eeks p rior m eans that she w ou ld be d iagnosed w ith acu te d istress d isord er. For PTSD, there m u st also be evid ence of negative cognitions or m ood s. If her sym ptom s d id p ersist after 4 w eeks, then she w ou ld be d iagnosed w ith PTSD. Ad justm ent d isord er m ost often represents a change in m ood , anxiety, or cond u ct that hap p ens after a nontraum atic event. In GAD, there is no associated trau m a, and the sym ptom s of

anxiety last for at least 6 m onths. The patient has no evid ence of a primary d epressive illness such as MDD. 48. (C) The lifetim e incid ence of su icid e in p atients w ith schizop hrenia is ap p roxim ately 10%, compared w ith less than 1% in the general population. 49. (A) Clozapine, the first atypical antipsychotic, has been show n to d ecrease su icid ality in patients w ith schizop hrenia and is p articu larly effective in treatm ent-refractory p atients, such as the one in this exam p le. Clozap ine requires regu lar blood m onitoring to screen for the potentially seriou s sid e effect of agranu locytosis. N one of the rem aining antip sychotics, such as flu phenazine, haloperid ol, or ziprasid one, have been show n to be effective in treatm ent-refractory schizophrenia or to d ecrease su icid ality. While lithiu m has also been show n to d ecrease su icid ality, its u se is not ind icated in this case given the lack of any affective sym p tom s. 50. (A) Feelings and attitudes originating from the clinician, evoked by the patient, are called countertransference. Transference d enotes feelings and attitud es about the physician coming from the patient. Empathy is the ability of the clinician to psychologically “put oneself in the patient’s shoes” and thereby und erstand the patient’s thinking, feelings, or behavior. Id entification is a d efense mechanism characterized by the unconscious incorporation of someone else’s traits into one’s ow n manner—for example, ad olescents having hairstyles similar to ad mired rock stars. Projection is a primitive d efense mechanism w hereby one assigns emotions to another person in an attempt to psychologically d efend against (cover up) the presence of those emotions w ithin oneself; for example, the d octor in the case may tell his fellow resident that he believes that his female patient is attracted to him, w hen in actuality he is attracted to her. 51. (A) This patient d isplays symptom s of fear of loss and aband onment, intense interpersonal relationship s, recurrent suicid al behavior or

Answe rs : 45–56

threats, affective instability, and d ifficulty controlling intense anger, consistent w ith bord erline personality d isord er. Patients w ith cyclothymic d isord er often present w ith similar sym ptoms of mood changes d u e to hyp om anic ep isod es and m inor d epressive episod es, bu t they are not u sually as frequent, short-lived , or intense, and they are not associated w ith the other symptom s of bord erline personality d isord er above. Patients w ith histrionic personality d isord er, like those w ith bord erline personality d isord er, m ay d isplay excessive emotionality and attention seeking, but their core symptoms center around superficial sed uctiveness and theatricality. While patients w ith bord erline personality d isord er may have d ep ressive sym ptoms that d evelop into MDD, this patient’s d epressive sym ptoms are fleeting and in resp onse to stressors such as perceived aband onment. Althou gh this patient ap pears p aranoid that “all the staff hates me,” there is no other evid ence for a primary psychotic illness su ch as schizoaffective d isord er. Of note, how ever, patients w ith bord erline personality d isord er may experience tem porary psychotic symptoms, such as paranoia, or severe d issociation u nd er times of perceived stress or substance u se. 52. (D ) Psychotherap y and stead y social su p p ort rep resent the best long-term m ethod for the m anagem ent of bord erline p ersonality d isord er. Antid ep ressants, antip sychotics, and m ood stabilizers d o have som e efficacy in treating target sym p tom s in som e p atients w ith bord erline p ersonality d isord er, bu t the long-term m ainstay treatm ent is p sychotherap y. Med ication can be a u sefu l ad ju nct in m anaging sym p tom s su fficiently so that the p atient can later engage m ore effectively in p sychotherap y. Benzod iazep ines m ay treat anxiety in these p atients, bu t they have p articu lar abu se liability in these im p u lsive p atients. 53. (B) Psychotherapy is the mainstay of bord erline personality disord er treatment. Of the available types of therapy, DBT has show n to be particularly effective w ith borderline individ uals, especially w hen used in conjunction

67

w ith groups. DBT is an offshoot from CBT and focuses on mind fulness and distress tolerance. CBT is often a short course of therapy focused on d ysfunctional thoughts w ith their resulting behavioral and emotional manifestations. Group therapy is a component of DBT, but should be used in conjunction w ith, not in place of, ind ivid ual therapy. Psychoanalysis and psychod ynamic therapies utilize psychoanalytic principles and focus on formative child hood experiences and the recurrent patterns of behaviors in interpersonal relationships, unresolved feelings, and build ing of ego d efenses. While these therapies may be of benefit, the regression and intense emotions evoked may be too d ifficult for the patient to handle. 54. (A) Amnesia is a comm on sid e effect of ECT, presenting in the form of short-term memory d eficits. Aspiration, fractures, and arrhythmias are rare sid e effects. Psychosis can be improved , rather than w orsened , w ith ECT; in fact, ECT is a treatm ent of choice in patients w ith m ajor d epressive d isord er w ith psychotic features. 55. (C) For m ajor d ep ressive d isord er, 6 to 12 sessions are generally optim al for a good risk–benefit ratio of p ositive treatm ent effects versus m em ory im p airm ent (w hich is m ore likely w ith greater than 20 treatm ents). For catatonic cond itions, tw o or fou r ECT treatm ents m ay be effective. When ECT is u sed for psychosis or m ania, a larger num ber of treatm ents m ay be necessary. 56. (B) This p atient has d istressing recu rrent and intru sive thou ghts (obsessions). She attem p ts to ignore or su p p ress her sym p tom s by not exp laining to her hu sband how she feels and by getting rid of all the knives in her kitchen. Obsessions are the m ental acts, w hereas com p u lsions are p hysical actions p erform ed to d ecrease anxiety associated w ith the obsessions. Obsessive-com p u lsive p ersonality d isord er is a p ervasive character style m arked by a p attern of p reoccu p ation w ith ord erliness, p erfectionism , and m ental and interp ersonal control. These ind ivid u als are often d escribed as “control freaks” by layp ersons and are

68

2: Ad ult P sychop a thology

noted to be p articu larly inflexible. Delu sional d isord er or schizop hrenia are inap p rop riate d iagnoses becau se this p atient is not p sychotic; her w orries are conceived of as a p rod u ct of her m ind and not as im p lanted m essages, as m ay occu r in schizop hrenia. There is no m ention of a history of od d beliefs or strangeness p rior to the birth of her child , w hich w ou ld be consistent w ith schizotyp al p ersonality d isord er. 57. (C) Flu voxam ine and other serotonin-sp ecific reu p take inhibitors (SSRIs) (e.g., p aroxetine, flu oxetine, sertraline, citalop ram ) are know n for their antiobsessional effects as w ell as for their antid ep ressant and antianxiety effects. Patients m ay requ ire higher d oses than those need ed for d ep ression. Lithiu m is a m ood stabilizer u sed for bip olar d isord er. Lorazep am , a benzod iazep ine, m ay d ecrease anxiety bu t has no d irect antiobsessional effects. H alop erid ol is a typ ical high-p otency antip sychotic (neu rolep tic) that m ay be u sefu l in severe form s of OCD in w hich there is a p sychotic com p onent. H ow ever, it d oes not am eliorate p u rely obsessional thinking. Becau se of the im p ortance of serotonin in the treatm ent of obsessive-com p u lsive d isord er, of the tricyclic antid ep ressants (TCAs), clom ip ram ine w ou ld be p referred if this class w ere selected . H ow ever, TCAs are not first-line treatm ent for OCD. 58. (A) This patient’s presentation and examination are m ost consistent w ith d eliriu m, as evid enced by a fluctuating mental status w ith attentional d eficits. Any num ber of m ed ical cond itions can cau se d eliriu m, and the eld erly are particularly p rone. This w oman is eld erly, d ehyd rated , and w ith an active infection; any one of these can increase the risk of d elirium. Urinary tract infections are a frequ ent cau se of altered m ental statu s in the eld erly. A chest x-ray can also elucid ate the presence of infection, and a head CT can rule ou t acute intracranial pathology, both of w hich can cause d eliriu m. While major d epressive d isord er can cause cognitive d eficits on MSEs that are reversible w ith antid epressant treatment, these “pseud od ementias” usu ally

present w ith a more stable MSE than is illu strated in this case; in ad d ition, there is no evid ence that this w om an is d epressed . Although this p atient w ou ld be likely to have a neurocognitive d isord er (d em entia) as she cam e from a nu rsing hom e, her flu ctu ating m ental statu s is ind icative of a d elirium . Dem entias are d iagnosed in the context of a relatively stable set of d eficits on MSE, and the u nd erlying causes are rarely associated w ith acu te m ed ical insu lts. There is no evid ence of a su bstance or other cause of a m ajor neurocognitive d isord er (d em entia). Im portantly, how ever, the p resence of a m ajor neu rocognitive d isord er (d em entia) d oes p red ispose a patient to a d elirium . 59. (B) Polypharm acy can w orsen d eliriu m . Antip sychotics su ch as halop erid ol can be used to calm an agitated patient w ith d eliriu m . Anticholinergic agents like d ip henhyd ram ine shou ld be avoid ed as they can w orsen or cause d elirium , and sed ating m ed ications should be u sed at low est p ossible d oses. Benzod iazep ines su ch as lorazep am shou ld be avoid ed in m ost cases of d eliriu m (w ith the excep tion of sed ative-hypnotic/ alcohol w ithd raw al) as they can d isinhibit patients and w orsen the behavioral p roblem s. In cases of d eliriu m w ith very severe agitation, p henobarbital m ay be u sed , bu t this w ou ld not be the ap p rop riate m anagem ent in this case. Valp roic acid , an anticonvu lsant and m ood stabilizer used in bip olar d isord er, is not ind icated in this p atient. 60. (A) The d efinitive treatm ent for d eliriu m is to treat the und erlying cau se, in this case, treatm ent of the u rinary tract infection w ith antibiotics. Mental statu s need s to be m onitored carefu lly as som e antibiotics can also w orsen d elirium . Sched u led chest x-rays or CT scans have no role in treatm ent of this patient’s cond ition and carry u nnecessary rad iation exposure. An ind w elling Foley catheter w ould increase the risk of d evelop ing a u rinary tract infection, and unless there is ind ication, shou ld be avoid ed . Any associated electrolyte abnorm alities that m ay have resulted from the p atient’s p oor oral intake need to be corrected ,

Answe rs : 57–69

bu t intravenou s flu id s w ill not, by them selves, treat the infection or the d eliriu m . 61. (C) Deliriu m carries a high-m ortality rate and is a poor p rognostic sign. The 1-m onth m ortality rate of p atients w ho have had one ep isod e of d eliriu m is u p to 14%, and the 6-m onth m ortality rate is app roxim ately 22%. 62. (C) Abou t one-third of all p atients w ith d ep ression respond to the su rp risingly pow erfu l p lacebo effect. H ow ever, this resp onse is not as w ell su stained as in p atients placed on antid epressant m ed ications. 63. (D ) Stand ard antid epressant therapies, inclu d ing SSRIs and TCAs, have a p ositive resp onse in 55% to 65% of patients. 64. (C) This p atient likely has m ajor d ep ressive d isord er. The rate of com pleted suicid e in hosp italized p atients w ith m ajor d ep ression is app roxim ately 10% to 15%. Most p atients w ith m ajor d ep ression w ill have som e d egree of su icid al id eation, and u p to 40% of p atients w ith m ajor d ep ression w ill attem p t su icid e. Screening for su icid e shou ld be perform ed in any p atient w ith d ep ression, and stu d ies have show n that asking abou t su icid e d oes not p lant the thou ght in p atients; m ost p atients w ho have attempted suicide have seen a doctor w ithin 4 w eeks of the attempt. 65. (C) The nu m ber one p red ictor of a com p leted suicid e is a previous attem pt. Old er single (or d ivorced ) m ales are m ore likely to attem p t. Religion, good su pport, and child ren can be p rotective factors. Concu rrent su bstance u se and anxiety d isord ers can increase the risk of suicid e. 66. (C) There is no evid ence of occupational or social d ysfu nction or marked d istress caused by this patient’s activities. Virtually all DSM -5 d iagnoses are qualified by these criteria. If the patient d id d escribe su ch concerns, the d iagnosis w ould m ost likely be sexu al masochism d isord er, in w hich patients are aroused by psychologically or p hysically pu nishing acts by another person (or fantasies of pu nishm ent). In

69

sexu al sad ism d isord er, the p atient is aroused by giving such punishment (or fantasies of giving it) to others. Fetishistic d isord er involves sexu al arousal connected to nonliving objects. In frotteuristic d isord er, the patient is sexually arou sed by touching or ru bbing u p against a nonconsenting person. All are examples of parap hilias—persistent sexual interest other than sexual interest in genital stimu lation or preparatory fond ling w ith norm al, m atu re, consenting hu man partners. 67. (D ) N arcolepsy is a d isord er often affecting p ersons in their teens or tw enties. It is a d isord er of REM sleep m echanism s, characterized by catap lexy (su d d en loss of m u scle tone follow ing intense em otion), hyp nopom pic or hypnagogic hallu cinations, or sleep p aralysis (an inability to p erform volu ntary m ovem ents either at sleep onset or aw akening that can be terrifying). Circad ian rhythm sleep –w ake d isord er is a d isord er cau sed m ost often by sleep-sched u ling changes su ch as night-shift w ork. H yp ersom nolence d isord er d escribes self-reported excessive sleepiness d espite m ain sleep p eriod of su fficient tim e. Insom nia d isord er notes a com plaint of d issatisfaction w ith sleep qu ality and qu antity. N ightm are d isord er is d iagnosed in the absence of other p red isp osing m ental illness (e.g., PTSD), in w hich the p atient has rep eated nightm ares causing significant d istress. 68. (D ) Methylphenidate, a stimulant, has been shown to be useful in inhibiting the onset of narcoleptic episodes, likely owing to the capacity for the drug to enhance CNS arousal mechanisms that inhibit REM-related mechanisms. Antidepressant treatment with tricyclic antidepressants has been found to be useful in combination with methylphenidate in some patients with narcolepsy. However, the efficacy of bupropion, fluoxetine (a serotonin-specific reuptake inhibitor), and phenelzine (a monoamine oxid ase inhibitor) has not been well studied in narcolepsy. Benzodiazepine regimens are not particularly effective for narcolepsy. 69. (D ) This p atient’s sym p tom s fit DSM -5 criteria for PMDD. The rem ittance of sym p tom s in

70

2: Ad ult P sychop a thology

the w eek after m enses is typical of PMDD. In ad ults, persistent d epressive d isord er is characterized by su bthreshold d epressive sym p tom s, for m ore d ays than not, over a 2-year p eriod . H er sym p tom s are not severe enou gh nor of sufficient d u ration (2 or m ore w eeks) to d iagnose MDD, and her sym p tom s are m ore d isru ptive and cau se m ore d istress than p urely PMS. 70. (C) This p atient is d isp laying sym p tom s consistent w ith m ajor d epressive d isord er, single ep isod e. The average u ntreated d ep ressive episod e lasts anyw here from 6 to 13 m onths; w ith treatm ent, the d u ration d ecreases to ap proxim ately 3 m onths. Treatm ent can also d ecrease the risk of recu rrence in the future. 71. (C) H eroin overd ose is m ost likely to have caused the clinical situ ation d escribed , inclu d ing the d ecreased level of consciou sness, resp iratory d ep ression, and p inp oint (constricted ) pu pils. Alcohol intoxication and PCP m ay cause com a bu t both are associated w ith nystagm u s rather than pu p illary size changes. Cocaine u se causes pu pillary d ilatation, not constriction, and is usu ally associated w ith agitation in large am ou nts. Inhalants m ay rarely cau se com a bu t are not classically associated w ith pu pillary constriction. 72. (E) N aloxone is u sed to reverse the acu te effects of opiate overd ose by blocking CN S opioid recep tors. Acetylcysteine is ad m inistered in acetam inop hen overd ose and d eferoxam ine is u sed in iron overd ose. Methad one is u sed for the long-term m aintenance of opiate ad d iction and w ou ld only w orsen the sym p tom s of heroin overd ose. Methylene blue is u sed to treat m ethem oglobinem ia. 73. (D ) This p atient is su ffering from sym p tom s of posttrau m atic stress d isord er (PTSD), m anifesting in som e p atients follow ing the exp osu re to threatened or actu al d eath, severe inju ry, or violence, and consisting of intru sion sym ptom s, avoid ance of related stim uli, negative alterations in m ood and cognitions, and increased au tonom ic arou sal. The best stu d ied and m ost efficaciou s m ed ications are

consid ered to be the SSRIs. Antip sychotics, inclu d ing the atyp icals, shou ld not be u sed as m onotherap y, althou gh m ay be used in conjunction, especially if there are associated p sychotic sym p tom s. Benzod iazep ines shou ld be avoid ed in this p atient p op u lation not only becau se of the significant com orbid ity w ith substance ad d iction bu t also as they have not been fou nd to be p articu larly effective. Mood stabilizers, su ch as lithium and valproic acid , are occasionally u sed in ad d ition to SSRIs in ord er to target the m ood lability or aggression som etim es seen in PTSD, althou gh they are not as effective as SSRIs. 74. (A) Th is case is an exam p le of p an ic d isord er and agorap hobia inad equ ately treated w ith ap p rop riate m ed ication, n am ely sertraline. Man y m ental illnesses, inclu d in g an xiety d isord ers, are best treated w ith a com bination of p harm acology an d p sych oth erap y. The best-stu d ied p sychotherap y for p anic d isord er is CBT. Eye m ovem ent d esensitization and rep rocessing (EMDR) is a sp ecific th erap y that has been d evelop ed for PTSD. Insight-oriented therap y is a long-term d ynam ic therap y. Althou gh it m ay be u sefu l in som e cases of p anic d isord er, it has not been as valid ated as CBT. Interp ersonal therap y ad d resses relation ship s as a contribu tor of d ep ression and is u sed to treat ind ivid u als w ith m ajor d ep ression. The p u rp ose of su p p ortive p sychoth erap y is to stren gthen th e p atien t’s d efen se m ech anism s in ord er to retu rn them to a p reviou s level of fu n ctioning. It has not been ad equ ately stu d ied for p anic d isord er. 75. (A) This case d ep icts a patient w ho is feigning or p rod u cing sym p tom s of an illness to gain gratification by assu m ing the sick role. Factitiou s d isord er is som etim es referred to as Mu nchau sen synd rom e; w hen caretakers intentionally cau se illness in their charges, such as child ren, the d isord er is called factitiou s d isord er im posed on another. In fu nctional neu rological sym ptom d isord er (conversion d isord er), p atients present w ith neu rologic sym p tom s that are not consistent w ith know n anatom y or p hysiology, and the

Answe rs : 70–80

concern is u sually w ith a specific sym ptom , not having a p articu lar d isease. In illness anxiety d isord er, p atients have few som atic com p laints bu t are p rim arily concerned w ith the id ea of being sick and having or acqu iring a serious illness; the intensity is not so severe as to w arrant a d iagnosis of d elu sional d isord er. Som atic sym ptom d isord er is characterized by m u ltip le, cu rrent, som atic sym p tom s that are d istressing or result in significant d isruption. There is no reason to believe this act cou ld help her achieve financial or other m aterial gain as in m alingering. 76. (C) Patients w ith factitious d isord er have an u nconscious d esire to assu m e the sick role, althou gh their sym p tom p rod u ction is fu lly conscious. In m alingering, both sym ptom p rod u ction and the d esire for second ary gain (w ork avoid ance) are consciou s. In functional neu rological sym ptom d isord er, the sym ptom p rod u ction is believed to be d u e to u nconscious conflicts. 77. (B) This patient m eets criteria for a current m anic episod e w ith sym p tom s of p sychom otor agitation, p ressu red speech, grand iose d elusions, flight of id eas, and history of excessive spend ing as evid enced by his cred it card d ebt. H is history of good prem orbid functioning and a rem itted m ajor d ep ression are also consistent w ith a d iagnosis of bipolar d isord er. Ad justm ent d isord ers can p resent follow ing an id entifiable stressor and m ay m anifest w ith m ild m ood , anxiety, or behavioral d istu rbances, bu t not overt m ania. Brief p sychotic d isord er is characterized by psychotic sym ptom s lasting 1 d ay to 1 m onth, bu t w hich are not better accou nted for by another m ood d isord er w ith psychotic featu res (su ch as bipolar d isord er) or psychotic d isord er. Cyclothym ic d isord er is incorrect as this patient has had both m anic and d ep ressive ep isod es, w hereas in cyclothym ia the highs d o not exceed a hyp om anic ep isod e, and the low s d o not extend to a m ajor d epressive episod e. The patient’s cu rrent m anic ep isod e ru les ou t MDD; in fact, althou gh he m ay have futu re m ajor d ep ressive episod es, he should never be d iagnosed w ith MDD.

71

78. (D ) Lithium and valproic acid , along w ith atyp ical or second -generation antip sychotics (e.g., olanzap ine) are consid ered first-line treatm ents for acute m ania, as w ell as prevention of fu tu re m ood episod es. Carbam azepine is also an effective m ood stabilizer, bu t d u e to sid e effects is not u su ally first line. While halop erid ol can be u sed to treat the acu te agitation and p sychotic sym ptom s in bip olar d isord er, it is not as effective for the m ood com p onent or in preventing ep isod es of m ania, and its sid e effects (e.g., extra-pyram id al sym p tom s, tard ive d yskinesia) lim it its use as m onotherap y. Lam otrigine and lithiu m are both u sed in the treatm ent of acu te d epressive episod es of bip olar d isord er. Antid ep ressants su ch as sertraline should be avoid ed in this case as they can w orsen m ania and likely prom ote rapid cycling of m ood episod es. 79. (B) Classically, infarcts of the left frontal hemispheres (part of left mid d le cerebral artery d istribution) present w ith d epression, w hereas those of the right frontal hem isphere present w ith euphoria, inappropriate ind ifference, or m ania (althou gh recent research tend s to refute this). Obsessive-com pulsive behaviors present occasionally after d iffu se bilateral frontal injury. Anxiety and panic sym ptoms have not been d escribed as having any particular association w ith left mid d le cerebral artery strokes, although su ch comorbid cases m ay exist. 80. (E) This w om an has d eveloped psychosis m arked by hallu cinations, d elusions, and paranoia. She w as m ost likely given high-d ose steroid s to treat her acute multip le sclerosis flare, and the steroid s ind uced a psychotic state. H igh-d ose steroid s can cause m ood d isturbance in many patients, and florid psychosis in a sm aller subset. The symptom s u sually entirely remit once steroid s are stopped or tapered appropriately. While most symptom s occur at high d oses, such as w ith IV steroid s, steroid psychosis can occur in patients treated w ith chronic low er level oral steroid s as w ell. H er psychotic symptom s rule out an ad justm ent d isord er, and her age and lack of psychiatric history w ould not be consistent w ith a first episod e of bipolar d isord er or schizophrenia.

72

2: Ad ult P sychop a thology

While m ultiple sclerosis can, by itself, prod u ce d epression and affective lability, it d oes not usu ally cau se psychotic sym ptoms. 81. (B) Sexu al d ysfu nction is a com m on sid e effect of m any psychotropics, especially antid epressants such as SSRIs. Wellbu trin, a non-SSRI antid epressant, likely cau ses a low er incid ence of sexu al d ysfu nction. Given his num erous trials and incid ence of erectile d ysfu nction w ith SSRIs, it is d oubtfu l that retrying sertraline or sw itching to another SSRI (e.g., citalopram ) w ill be less likely to cau se sexu al d ysfu nction. Aripip razole (an atypical antip sychotic) and lithium (a m ood stabilizer) can be u sed in au gm enting p artially effective antid ep ressants, bu t w ould not be app ropriate first-line m onotherapy in this case. 82. (E) The serotonin m etabolite d esignated 5-hyd roxyind ole acetic acid (5-H IAA) m easured in the CSF has been show n to be low er in p ostm ortem analysis of victim s of su icid e and patients w ith im pu lsivity and violence or aggression w hen com pared w ith control grou p s. N o su ch associations have been established w ith the other neu rotransm itters listed . 83. (B) Depression is the m ost com m on presenting p sychiatric sym p tom in p atients w ith m u ltiple sclerosis. It m ay present d u ring the cou rse of m ild progressive cognitive d ecline that is consistent w ith a subcortical neurocognitive d isord er (d em entia) (i.e., slow ing of m ental p rocessing, m otor d ifficu lties, blu nted or inappropriate affect, w ith relative preservation of langu age-d epend ent skills). N euroim aging of these p atients u su ally show s d iffuse w hite m atter plaqu es affecting the frontal lobes. Cerebral atrophy and ventricular enlargem ent are associated w ith the m ost com m on form of m ajor neu rocognitive d isord er (d em entia), Alzheim er d isease. Periventricu lar w hite-m atter changes are observed in m ajor vascu lar neu rocognitive d isord er (form ally m u lti-infarct d em entia). 84. (B) The patient appears to be suffering from d elirium. Although aggressiveness or combativeness, m emory d eficits, psychotic symptom s,

and u ncooperativeness m ay be seen in either d elirium or major neurocognitive d isord ers (d em entias), a fluctuation in the level of consciou sness (i.e., from alertness to som nolence) w ith d eficits in attention are the hallm arks for d elirium. Attention is usually intact in neurocognitive d isord ers (d em entias). Fu rther, the acute onset of sym ptoms is more su ggestive of d eliriu m. It is important to remember that having a major neu rocognitive d isord er pred isposes a patient to d elirium , so both m ay be seen in the sam e patient. 85. (B) This p atient is su ffering from generalized anxiety d isord er. H er sym ptom s inclu d e rum inations abou t m any asp ects of her life, d espite there not being any d isruption in those areas. She also has physical sym ptom s associated w ith anxiety. There is no p articu lar stressor to qu alify for ad ju stm ent d isord er. H er ru m inations m ay have an obsessional qu ality, bu t they are not lim ited to a specific thou ght nor associated w ith any com p ensatory com p u lsions, therefore m aking obsessive-com pu lsive d isord er u nlikely. She d oes not have specific p anic attacks or fears of fu tu re attacks consistent w ith panic d isord er. H er anxiety is not related to fears of hu m iliation or scru tiny in a p erform ance situ ation, w hich w ou ld be consistent w ith social anxiety d isord er (social p hobia). 86. (E) There are a nu m ber of stigm ata associated w ith schizop hrenia that are not inclu d ed in DSM -5 d iagnostic criteria. These inclu d e soft neu rologic signs su ch as short-term m em ory d eficits, u nstable sm ooth-p ursu it eye m ovem ents, and d ecreased ability to habitu ate to rep eated sensory stim u li (sensory gating abnorm alities). In ad d ition, p atients w ith schizophrenia have d ifficu lty in conceptualizing com p lex visu al com p ositions. Patients w ith ADH D, MDD, OCD, and PTSD have also d em onstrated short-term m em ory d ifficu lties, bu t not the other signs m entioned . 87. (C) The m ost ap propriate next step is to send the patient to the p sychiatric emergency d epartment. This patient is experiencing an acute episod e of major d epression and , becau se

Answe rs : 81–92

of his suicid al id eation w ith active plan and means to overd ose, he should be referred to the emergency d epartment for further assessment w hile ensu ring his safety. Stopping his prescription w ill not ad d ress the im med iate d anger to self, and having him follow u p in 1 w eek or for w eekly outpatient visits are also inappropriate given the imm ed iate safety concerns. While his p arents may be called for collateral information in the fu tu re, the more pressing step is to ensu re the patient’s safety. H e may not have respond ed to the SSRI or have been in the minority of you ng ad u lts w ho exp erience w orsening of su icid al thou ghts on the SSRIs; either w ay, a sw itch in pharmacotherapy is likely necessary, bu t w ill not resolve his acu te suicid ality, and is therefore inappropriate as the next imm ed iate step. 88. (C) The w om an p resents w ith a classic p resentation for m ajor d ep ressive d isord er. H aving a d epressive d isord er significantly increases one’s risk for suicid e, and feelings of hopelessness have been fou nd to be a particu lar risk factor. Many other factors also increase the risk for com p leted su icid e su ch as w hite race, m ale gend er, old er age, single/ d ivorced , and having poor social su p p ort. 89. (B) The presentation exemplifies a manic episod e, w ith symptoms that includ e inflated self-esteem and grand iosity, pressured speech, a d ecreased need for sleep, and an impulsive shopping excursion w ithout consid ering possible consequences. The most likely d iagnosis is a type of bipolar d isord er, as this patient has a history of d epression and now is manic. How ever, before diagnosing bipolar d isord er type I, the contribution of mania ind uced by an antid epressant must be ruled out. If this manic episod e occurred because of an antid epressant, a more appropriate d iagnosis w ould be substance-induced mood disorder. Anxiety d isorders and neurocognitive d isord ers do not present w ith the classic signs of mania. While this patient has psychotic symptoms (grand iose d elusions), a psychotic d isord er such as schizophrenia cannot be d iagnosed as there is no ind ication of prominent psychotic symptoms (hallucinations, d isorganized thought

73

processes and speech) in the absence of mood symptoms. H er primary symptoms d o not involve physical or somatic symptoms, w hich w ould be seen in a somatic symptom disord er. 90. (A) This p atient w as m ost likely p rescribed an antid ep ressant since her initial presentation w as for a d ep ressive ep isod e 2 m onths ago. Since she also has m igraines, and tricyclic antid ep ressants are often u sed in their treatm ent, this w as m ost likely the agent she has been taking. While all antid ep ressants have the potential to ind u ce a m anic sw itch in suscep tible patients, the rate is slightly higher for tricyclic antid epressants than for serotoninspecific reu ptake inhibitors or serotoninnorep inep hrine reup take inhibitors, such as fluoxetine or sertraline, respectively. Benzod iazepines (e.g., clonazepam ) are not know n to cau se m ood sw itches into m ania, and m ood stabilizers such as lithiu m are a treatm ent for bip olar d isord er. 91. (B) The specific phenomenon described here is an id ea of reference. The patient interprets an event as relating to him, even though it clearly d oes not. A hallucination is a perception in the absence of a clearly defined stimulus, w hereas an illusion is a misinterpretation of an evident stimulus (for instance, IV tubing appearing like snakes). Thought broad casting is a d elusional belief that one’s thoughts can be heard or somehow know n by others w ithout d irect communication. Thought insertion is another d elusion w here the patient believes thoughts from some external entity are placed in his mind. 92. (C) The choices in this qu estion d escribe d ifferent patterns of am nesia. Localized am nesia refers to m em ory loss su rrou nd ing a d iscrete p eriod of tim e, typ ically occu rring after a trau m atic event; in this case, the patient’s am nesia resu lts from learning of her hu sband ’s d eath. Rarely, a patient m ay forget his or her entire p reced ing life (generalized am nesia) or forget all events follow ing a trau m a, excep t for the im m ed iate p ast (continu ou s am nesia). Retrograd e am nesia is any am nesia for events that com e before a trau m atic event. Selective am nesia involves the inability to recall certain

74

2: Ad ult P sychop a thology

asp ects of an event, thou gh other m em ories of the event m ay be intact. 93. (B) This p atient is su ffering from bereavem ent. H is w ife has d ied w ithin the last 2 m onths, and his sym p tom s inclu d e an ap prop riately d ep ressed m ood , althou gh he is still able to fu nction and is taking care of his basic need s. It is not u ncom m on to have au d itory or visu al hallu cinations of the d eceased d u ring the grieving p eriod . Also, m any ind ivid u als exp erience survivor’s gu ilt as part of their grieving. While this patient had a significant stressor (d eath of w ife), his sym p tom s are better accou nted for by bereavem ent rather than ad ju stm ent d isord er, since the sym ptom s are not ou t of prop ortion to w hat w ou ld be expected . If he had suicid al id eation, severe loss of functioning, significant d istressing au d itory hallu cinations or other sym ptom s of psychosis, a d iagnosis of m ajor d epression w ith or w ithou t p sychotic featu res w ou ld be m ore appropriate. Persistent com plex bereavem ent d isord er is prop osed as a cond ition for fu rther stu d y, and w ou ld d escribe severe bereavem ent p ersisting over a year after d eath. Besid es the hallu cinations relating to the d eceased w ife, there is no other evid ence of a prim ary psychotic illness such as schizop hrenia. It is im portant to note that bereavem ent is not a psychiatric d isord er. 94. (D ) The m ost ap p rop riate treatm ent at this stage is su pp ort. If sym ptom s persist, and he begins to exp erience loss of fu nctioning or significant d istress, an antid epressant m ay be w arranted . H osp italization w ou ld be ind icated if this p atient had active thou ghts of su icid e and w as at risk of harm ing him self or others. At this tim e, he is exp eriencing su rvivor’s gu ilt bu t is not actively su icid al. There is no ind ication for an antip sychotic, and cognitivebehavioral therap y is not ind icated in this case. 95. (A) Circum stantiality involves a circuitou s, over-inclu sive answ er that only eventu ally gets to the p oint as the p atient d em onstrates in this exam p le. Loosening of associations is a resp onse com p osed of a series of d isconnected id eas. The p ointless rep etition of a w ord ,

p hrase, or id ea is perseveration. Pressured speech is a continuou s, usu ally rap id , u ninterru p tible stream of id eas that m ay m ake sense bu t is often hard to follow . While this p atient’s speech m ay be pressured , this is unable to be d eterm ined by the vignette alone. Tangentiality occu rs w hen the patient strays off the p oint entirely, never retu rning to the original intention of the answ er. Generally, the sp eech or thou ght p rocesses of a schizophrenic or schizoaffective patient m ay d em onstrate any of these characteristics. 96. (C) A nonepileptic seizu re (formerly called “p seu d oseizu re”) is a psychogenically ind u ced behavior that resembles epileptic activity externally, bu t the EEG is entirely norm al. Althou gh the synd rom e may in some cases be motivated by second ary gain (as in m alingering), it most often occurs in reaction to stress and represents a type of functional neurological sym ptom (conversion) d isord er. The other choices, w hich each represents true epileptic seizu res, rarely, if ever, includ e the purposeful, com plex activity as is seen in this case. N ot uncomm only, patients w ith conversion d isord er (functional neurological symptom d isord er) w ith seizures also have a concurrent seizu re d isord er, and it is quite d ifficult to sort ou t the epileptic seizu res from the nonepileptic ones. Diagnosing a nonepileptic seizu re often requ ires d emonstrating a norm al EEG at the time of the event. The prognosis for nonepileptic seizures is qu ite p oor, w ith frequent exacerbations. 97. (B) N arcolep sy is a sleep d isord er characterized by the tetrad of hyp ersom nia (excessive d aytim e som nolence), cataplexy (transient loss of m otor tone associated w ith strong em otions, as d em onstrated in this case), sleep p aralysis (total or p artial p aralysis in sleep – w ake transition), and hyp nagogic or hyp nop om p ic hallu cinations (vivid d ream -like hallu cinations occurring in the w ake/ sleep transition). Catalepsy is a state of im m obility som etim es seen in catatonic states. 98. (A) All of the choices are possible sid e effects, either short term or long term, of antipsychotic

Answe rs : 93–103

med ication. Akathisia is the su bjective sensation of motor and mental restlessness. Akinesia is a d ysfunction of slow ed or absent m ovement that can be associated w ith p seud op arkinsonism in the setting of antipsychotics (neuroleptics). A d ystonia, like akathisia and akinesia, can occu r acutely and involves mu scle rigid ity and spasticity. Rabbit synd rome is a late-onset sid e effect that involves fine, rhythmic m ovements of the lips. Tard ive d yskinesia is another late-onset neurologic effect of antipsychotics (neuroleptics) and can includ e choreoathetoid movements of the trunk or limbs, as w ell as other areas su ch as the tongue and lips. 99. (E) This p atient is likely su ffering from both p anic d isord er and agorap hobia. The m ost effective and tolerated m ed ications for p anic d isord er are SSRIs (e.g., sertraline) or SN RIs (e.g., venlafaxine). Busp irone, a 5-H T1 A recep tor p artial antagonist, can be effective in generalized anxiety d isord er bu t not p anic d isord er. Lithiu m and valproic acid are m ood stabilizers m ost often u sed in bipolar d isord er. Lorazep am and other benzod iazep ines can be help ful in treating anxiety, bu t should be m inim ized long-term d u e to the risk of ad d iction. 100. (A) This patient is suffering from alcohol u se d isord er. While rehabilitation is the m ainstay of treatm ent for ad d ictions, several m ed ications have been beneficial in red ucing alcohol u se. Acam p rosate, a glu tam ate recep tor m od u lator, has show n efficiency in red u cing craving and alcohol u se. Antid ep ressants (e.g., bu p rop ion, citalop ram ), m ood stabilizers (e.g., lithiu m ), and antip sychotics (e.g., olanzap ine) have not been d em onstrated to red u ce alcohol u se and are not ind icated in the absence of a com orbid m ood or psychotic d isord er. 101. (E) This p atient ap pears to be experiencing Wernicke encephalop athy, a resu lt of chronic, heavy, alcohol u se, cau sed by acu te thiam ine d eficiency. It p resents classically w ith the triad of confu sion, oculom otor abnorm alities (e.g., nystagm u s), and ataxia. The treatm ent is IV thiam ine. While glu cose is often given in an em ergency setting, as w ell, it is im p erative to give thiam ine first, as giving glucose before

75

thiam ine m ay w orsen the encep halopathy. H aloperid ol, a typical antipsychotic, is not ind icated as the p atient is neither p sychotic nor agitated , and w ill not help the u nd erlying cond ition. N either lorazep am , a benzod iazepine u sed in acu te alcohol w ithd raw al, nor naltrexone, an op iate antagonist u sed to red u ce alcohol craving, w ill ap p rop riately treat Wernicke encephalopathy. 102. (C) This p atient p resents w ith d isorganized behavior, agitation, p aranoia, and form ication (a particu lar type of tactile hallu cination in w hich one has the sensation of bu gs craw ling on or u nd er the skin), as w ell as elevated vital signs and d ilated p upils. These are all consistent w ith acute cocaine intoxication. Intoxication w ith alcohol p resents w ith d ecreased level of consciou sness, d ecreased vital signs, slurred speech, and poor coord ination. Cannabis intoxication can p resent w ith p aranoia and tachycard ia, althou gh not u sually agitation or form ication. Opiate use classically p resents w ith a d ecreased level of consciou sness, d im inished vitals, p inp oint p u pils, d ry skin, and constipation. Patients w ho use PCP m ay be qu ite agitated and p aranoid , as w ell as have signs of sym p athetic stim u lation, bu t they often p resent w ith ataxia, fever, and nystagm u s (vertical, horizontal, or rotary). 103. (E) Given his card iovascu lar history, risk factors, and clinical presentation, this patient m ost likely has m ajor vascu lar neu rocognitive d isord er (d em entia), the second m ost com m on cau se of d em entia after Alzheim er d isease. Becau se the etiology is com m only d ue to m u ltiple infarcts, periventricu lar w hite m atter (lacu nar) infarcts m ay be seen on MRI. Atrop hy of the brainstem and cerebellu m m ay be seen in cases of Parkinson d isease. Frontotem poral neu rocognitive d isord er (d em entia), com m only referred to as Pick d isease, is a com m on cause of d em entia in you nger ind ivid u als and d isp lays a p referential atrophy of the frontal and tem p oral lobes on MRI. Enlarged ventricles on MRI of the brain are consistent w ith neu rocognitive d isord er (d em entia) d u e to norm al p ressu re hyd rocephalu s, w hereas cases of m ajor neurocognitive d isord er d ue

76

2: Ad ult P sychop a thology

to Alzheim er d isease m ay show general or global atrop hy of the cerebru m . 104. (E) The cou rse of m ajor vascu lar neu rocognitive d isord er (d em entia) is typically a stepw ise d eterioration, corresp ond ing w ith the continu ed infarcts occu rring to the patient. Most, although not all, d em entias are irreversible, and so continued im p rovem ent is u nu su al; how ever, m ajor neurocognitive d isord er (d em entia) d ue to trau m atic brain inju ry m ay not show im p rovem ent nor w orsening over tim e (assu m ing no fu rther insu lts). Rapid d ecline m ay be seen in cases of m ajor neu rocognitive d isord ers d u e to p rion d isease (e.g., Creu tzfeld t–Jakob d isease, ku ru ), w hile stead y d ecline is typical for m ajor neurocognitive d isord er (d em entia) d u e to Alzheim er d isease. 105. (E) This p atient is exp eriencing d eliriu m . Deliriu m is a synd rom e characterized by a d istu rbance in attention and cognition. While d elirium is a m ed ical em ergency and behavioral control is im p ortant, the m ost im p ortant app roach is to id entify and correctly the u nd erlying cau se(s). Com m on etiologies inclu d e m etabolic, infectiou s, hyp oxia, organ failure, and d rug intoxication/ w ithd raw al. Behavioral control m ay be necessary w ith a high-potency antip sychotic su ch as halop erid ol or soft restraints. Dip henhyd ram ine and other anticholinergic m ed ications shou ld be avoid ed as they can w orsen d eliriu m . Benzod iazepines shou ld also be avoid ed (excep t in cases of alcohol or sed ative/ hypnotic w ithd raw al-ind uced d eliriu m ) as they can cau se d isinhibition and w orsening agitation in m any p atients. 106. (B) This p atient m ost likely su ffers from obsessive-com pulsive d isord er (OCD), a d isord er characterized by recu rrent obsessions (e.g., fear of contracting H IV) and / or com p ulsions (e.g., frequent hand w ashing), w hich cause significant d istress or im p airm ent in fu nctioning. The neu rophysiologic nature of this illness can be d em onstrated throu gh neuroim aging, w hich show s increased activity (m etabolism ) in the cau d ate nu cleus, frontal

lobes, and cingu lu m . These d ifferences are can be reversed after ad equ ate p harm acologic or cognitive-behavioral therapy. The other areas of the brain are not believed to be as involved in OCD. 107. (C) This p atient likely has m ajor neu rocognitive d isord er (d em entia) w ith Lew y bod ies, a com m on cau se of d em entia that is sim ilar to Alzheim er d isease. Com m on sym p tom s are typical for and overlap w ith Alzheim er’s, but as in this case they have ad d itional sym ptom s of urinary incontinence, orthostatic hypotension, visu al hallucinations, and parkinsonism . Major frontotem poral neurocognitive d isord er (d em entia) w as originally called Pick d isease. While not rare, it is a com m on cau se of early-onset d em entia, and often p resents first w ith behavioral d ifficu lties, inclu d ing ap athy, m ood lability, d isinhibition, and d epression. As ind icative of the nam e, im aging d em onstrates a p referential atrophy of the frontal and tem p oral lobes. The above associated sym ptom s (e.g., visu al hallucinations and p arkinsonism ) are not u su ally seen. Patients w ith m ajor vascu lar neu rocognitive d isord er (form erly vascu lar d em entia) have a history of card iovascular d isease, hypertension, strokes, etc., and often have ad d itional neu rological find ings not consistent w ith Lew y bod y d isease. There is no evid ence in the history for any su bstance-ind u ced m ajor neu rocognitive d isord er (d em entia). 108. (D ) The pharm acologic behavioral m anagem ent of m ajor neu rocognitive d isord er (d em entia) w ith Lew y bod ies is tricky. Anticholinesterase inhibitors su ch as d onepezil have d em onstrated efficacy in these p atients and are w ell tolerated . While antip sychotics are often u sed (d esp ite the increased risk of m ortality) to treat agitation and / or p sychosis in p atients w ith m ajor neu rocognitive d isord ers, high-p otency agents like haloperid ol are p articu larly concerning given the sensitivity and p otential for w orsening p arkinsonian sid e effects. Med ications w ith anticholinergic sid e effects, such as tricyclic antid epressants (e.g., am itrip tyline) and antihistam ines (e.g., d iphenhyd ram ine) shou ld be avoid ed as they

Answe rs : 104–114

m ay w orsen the d em entia. Clonazep am and other benzod iazepines shou ld also be avoid ed given the significant risk of d isinhibition. 109. (D ) While antid epressants su ch as paroxetine help all the sym p tom s of d ep ression, som e respond qu icker to m ed ications. Sleep, energy, and ap p etite changes are u su ally the first to resp ond , follow ed later by libid o, hop elessness/ helplessness, and su icid al id eation. Because of the d ifferential resp onse tim es, it is im portant to m onitor a suicid al p atient as their energy and m otivation m ay im p rove before the d ep ression and su icid ality, and thu s, the p atient m ay have the energy to com m it su icid e. 110. (D ) Sexu al sad ism d isord er and sexu al m asochism d isord er are, respectively, the d erivation of sexu al pleasu re from causing or receiving m ental/ p hysical abu se. Exhibitionistic d isord er is exp osu re of the genitalia in p ublic to an u nw illing p articip ant, and u su ally occu rs in m en. Frotteu ristic d isord er, the ru bbing of genitals against another to achieve arou sal, is also u su ally seen in m en and p erform ed in crow d ed places. Patients w ith transvestic d isord er are arou sal by cross-d ressing. 111. (B) Koro, taijin-kyofu sho, and zar are all exam p les of cu ltu re-bou nd d elu sions. Koro is the w orry that the penis is shrinking into the abd om en and is fou nd in South and East Asia. Taijin-kyofu sho is the belief that one’s bod y is offensive to others, and zar is the d elu sional belief of p ossession by a sp irit. Cap gras synd rom e is the d elu sional id ea that im posters have replaced once fam iliar p ersons. Ku ru is a slow ly progressive neurologic d isease w hich lead s to d eath, sim ilar to Creu tzfeld t–Jakob d isease, cau sed by prions. 112. (E) Paraphilias encom p ass a w id e variety of m alad ap tive sexu al behaviors and fantasies. Voyeu ristic d isord er is d eriving sexu al pleasure from w atching another person or persons involved in the act of und ressing or other sexu ally oriented activity. Exhibitionistic d isord er is a paraphilia involving exposing one’s genitalia to an u nsu sp ecting au d ience. Frot-

77

teuristic d isord er is the synd rom e of recu rrent, intense sexu al fantasies and behaviors involving tou ching or ru bbing against a nonconsenting ad u lt. Gend er d ysphoria is a cond ition w here there is incongru ence and d istress betw een one’s exp ressed gend er and assigned gend er. Transvestic d isord er involves being d ressed in clothing of the opposite sex for sexual excitem ent; it is often present in heterosexual m en and d iffers from gend er d ysphoria (transsexu alism ) in that the person is u sually com fortable and content w ith his gend er id entity. It is, how ever, p ossible to see these d isord ers concom itantly. 113. (D ) This p atient’s p resentation is m ost consistent w ith schizoaffective d isord er, bip olar type, characterized by the presence of psychotic sym p tom s (e.g., paranoia, d elu sions, hallu cinations) concu rrent w ith a m anic episod e (e.g., d ecreased need for sleep, increased energy, grand iosity, increased goal-d irected activity, flight of id eas). In this p atient, her p sychotic sym ptom s occu rred in the absence of m ood sym p tom s, as w ell, w hich d istinguishes schizoaffective d isord er from a prim ary m ood d isord er, su ch as bipolar or m ajor d epressive d isord er. Patients w ith d elu sional d isord er also have a d elu sion, althou gh the d elu sion is u su ally m ore circu m scribed than in this case, and they d on’t have the au d itory hallucinations or significant affective sym ptom s (i.e., m anic ep isod e) as seen in this case. Patients w ith schizop hrenia obviou sly have sim ilar sym ptom s su ch as paranoia, d elu sions, hallu cinations, and d isorganized behavior, bu t the m ood sym p tom s are not as prom inent or enou gh to be d iagnosed w ith a m anic ep isod e. 114. (C) This p atient w ith PTSD continu es to be sym p tom atic, sp ecifically w ith nightm ares, insom nia, increased startle, and avoid ance. Given the p artial efficacy and tolerability of the citalop ram (an SSRI), au gm entation w ith another agent m akes the m ost clinical sense. Alp ha-ad renergic blockers like p razosin are p articu larly efficaciou s in treating PTSD sym p tom s, p articu larly nightm ares and insom nia. Benzod iazep ines, inclu d ing alp razolam , have not d em onstrated efficacy

78

2: Ad ult P sychop a thology

in PTSD; in ad d ition, they shou ld be avoid ed given the risk of d ep end ency, a p articu lar concern in this case. Bu p rop ion, an antid ep ressant w ith norep inep hrine and d op am ine reu p take blockad e activity, has not been show n to be beneficial in treating PTSD. Antip sychotics (e.g., risp erid one) and m ood stabilizers (e.g., valp roic acid ) are often u sed ad ju nctively in the treatm ent of PTSD, esp ecially for m ood lability, agitation, flashbacks, and p sychotic sym p tom s, althou gh the evid ence for their benefit is not as conclu sive as for p razosin. 115. (B) Cognitive processing therapy is an evidencebased form of cognitive-behavioral therap y that has d em onstrated efficacy in the treatm ent of PTSD. It incorp orates both a cognitive com ponent and exposu re to the trau m a through rem em bering, d iscussion, and cues. N either p ure behavior nor cognitive therapy alone is as effective. Dialectical behavioral therapy is another form of CBT, specifically d evelop ed for treating bord erline personality d isord er, and it u tilizes ind ivid ual therap y, grou p s, safety-m anagem ent, and hom ew ork. Psychod ynam ic p sychotherapy has not been ad equately studied to recommend as a primary treatment for PTSD. 116. (D ) The locu s ceruleu s is the “alarm ” center of the brain and is hyperactive in anxiety states. It is the location of m ost of the norep inep hrinecontaining neurons in the brain. The am ygd ala and hippocam pu s, both part of the lim bic system , are involved in fear/ anger responses and m em ory form ation, resp ectively. The basal ganglia coord inate m otor activity, w hile the thalam u s is the brain’s relay center. 117. (D ) Sp litting is a prim itive d efense m echanism , m anifested by view ing others as either all good or all bad . It is com m on in p atients w ith bord erline p ersonality d isord er althou gh there is no other evid ence of bord erline p athology in this case. Acting out refers to p erform ing an action in contrast to bearing and m anaging the im p u lse to p erform it. Externalization rep resents the tend ency to p roject one’s ow n internal characteristics onto

others. Regression is a retu rn to patterns of relating, thinking, or feeling that had com e before one’s cu rrent d evelop m ental stage; for exam p le, m any m ed ical stu d ents w ho retu rn hom e act as if they are teenagers w ith regard to their parents or other hom etow n friend s. Sublim ation is the channeling of d rives or conflicts into goals that are gratifying but socially accep table (e.g., ind ivid u als afraid of blood and hospitals w ho becom e hosp ital w orkers or physicians). 118. (D ) This p atient’s reaction to p aroxetine is consistent w ith serotonin synd rom e, a potentially life-threatening cond ition caused by the interaction of a serotonin-sp ecific reu p take inhibitor (SSRI), su ch as p aroxetine, w ith a m onoam ine oxid ase inhibitor (MAOI), w hich resu lts in increased am ou nts of both catecholam ines and ind olam ines. Featu res of m ild serotonin synd rom e includ e tachycard ia, flushing, fever, hypertension, ocular oscillations, and m yoclonic jerks. Severe serotonin synd rom e m ay result in severe hypertherm ia, com a, au tonom ic instability, convu lsions, and d eath; therefore, clinicians m u st w ait at least 14 d ays after d iscontinu ing an MAOI before starting a serotonergic agent. Most serotonin in the CN S is synthesized in the d orsal and m ed ial raphe nu cleu s of the brain stem . The caud ate nucleu s is located in the brain stem and concerned w ith m em ory form ation. The nu cleu s accu m bens is located in the striatu m and thou ght to be involved w ith sensations of rew ard and pleasu re, as w ell as ad d ictive behaviors. The locu s ceru leu s synthesizes norep inep hrine, and the su bstantia nigra synthesizes d opam ine. 119. (E) This m an’s risk of suicid e is highest after d ischarge from the hospital. Up to 10% of p atients w ith schizop hrenia d ie by su icid e. Up to 50% of su icid es am ong ind ivid u als w ith schizophrenia occu rs shortly after d ischarge from the hospital. Many sink into a d epressive-like illness follow ing stabilization of their p sychotic sym p tom s. Risk factors inclu d e being m ale, u nm arried , u nem p loym ent, social isolation, d ep ression, and p reviou s su icid e attem p ts.

Answe rs : 115–130

120. (A) This patient likely su ffers from m ild intellectu al d isability (form erly m ental retard ation). The DSM -5 specifies the level of severity on the basis of intellectu al and ad aptive fu nction d eficits. Deficits occu r in the conceptu al d om ain, the social d om ain, and the p ractical d om ain. Psychom etric testing, su ch as m easu rem ent of IQ, is no longer su fficient to d eterm ine severity of im p airm ent, bu t is often a com ponent to assess the concep tu al d om ain d eficit. This p atient has d eficits in acad em ic skills, has im m ature social jud gm ent that p laces him at risk for being taken ad vantage of, and need s help w ith com plex d aily living tasks (m oney and groceries) even thou gh he can care for his p hysical need s. In the past, an IQ of 50 to 69 w ould be sufficient to d iagnose m ild intellectual d isability; betw een 35 and 49 as m od erate; betw een 20 and 34 as severe; and less than 20 as profound . 121–130. [121 (G), 122 (J), 123 (L), 124 (A), 125 (F), 126 (O), 127 (C), 128 (N), 129 (M), 130 (B)] Cannabis intoxication (G) inclu d es increased heart rate for several hou rs, p ostu re-d ep end ent changes in blood pressu re, injected conju nctivae, a “high” w ith m ild eu phoria, feeling of relaxation, p ercep tu al changes (tim e d istortion, intensified ord inary experiences), and increased sociability. Cocaine w ithd raw al (J) is characterized by onset of d ysp horic m ood , hyp ersom nia, increased ap petite, and fatigue. Inhalants have becom e increasingly com m on d ru gs of abu se. Signs and sym p tom s of intoxication (L) are inhalant-sp ecific, bu t generally inclu d e rapid onset of effects, d izziness, nausea, then slow ing, ataxia, slu rred speech, and d isorientation. Bronchospasm m ay also occur. Intoxication usu ally im p roves w ithin an hou r of abstinence because the su bstances are u su ally volatile hyd rocarbons. Treatm ent of inhalant w ithd raw al is su pportive. In acu te alcohol intoxication (A), as blood concentration of alcohol increases, there is w orsening m otor p erform ance, incoord ination and jud gm ent errors, m ood lability, nystagm u s, slu rred sp eech, potential blackou ts, altered vital signs, and possibly d eath. Caffeine w ithd raw al (F) can occu r w hen p eop le abru p tly stop their accu stom ed intake. Sym p tom s inclu d e

79

head ache, sleepiness, irritability, concentration problem s, vom iting, and m u scle aches/ stiffness. Phencyclid ine (PCP) (O) and other d issociative anesthetics can ind u ce vertical or horizontal nystagm us (rotary w ith ketam ine), hyp ertension (at least m ild , bu t hypertensive crisis can occur), tachycard ia, ataxia, nu m bness and high pain tolerance, ataxia, hyperacu sis, hypertherm ia, m u scle rigid ity, seizu res inclu d ing statu s ep ilep ticu s, and d eath. The p resence of nystagm u s helps to d istinguish PCP intoxication from other form s of psychosis. Behavioral m anifestations can be severe, and severe agitation, rage, and p anic cou pled w ith d isinhibition m ay cau se a d angerou s clinical situ ation. Am phetam ine intoxication (C) is characterized by both behavioral and p hysiological changes. Patients m ay exp erience eu p horia, interp ersonal sensitivity, anxiety, tension, or anger, im p aired ju d gm ent, and im p aired social and occu p ational fu nctioning. Fu rther, they m ay have tachycard ia or brad ycard ia, p u p illary d ilation, insom nia, blood p ressu re changes, sw eating or chills, nau sea or vom iting. Chronic u se m ay resu lt in d ry skin, acne-like lesions, or chronic nose bleed s. Op iate w ithd raw al (N ) is characterized by d ysp horia, nau sea, vom iting, m u scle aches, lacrim ation, rhinorrhea, p iloerection, sw eating, d iarrhea, yaw ning, fever, and insom nia. Op iates can be illegal or p rescrip tion, su ch as narcotic analgesics. Sym p tom s of acu te intoxication (M) includ e pup illary constriction (can be d ilation once anoxic brain inju ry has occurred from respiratory d epression), d row siness, slu rred sp eech, and p ossibly p u lm onary ed em a. At higher d oses, com a or d eath w ith resp iratory d ep ression m ay occu r. Alcohol w ithd raw al (B) can start hou rs after the last d rink, and is characterized by au tonom ic instability (sw eating, tachycard ia), trem ors, insom nia, nau sea/ em esis, transient hallu cinations or illu sions, psychom otor agitation, anxiety, and seizu res. Deliriu m trem ens can d evelop , and can be d ead ly. Am p hetam ine w ithd raw al (D ) is characterized by fatigue, vivid d ream s, sleep d istu rbances, increased ap p etite, and p sychom otor retard ation or agitation. A caffeine intoxication synd rom e (E) is possible, especially in ind ivid u als w ho

80

2: Ad ult P sychop a thology

consum e a large am ou nt or w ho are u naccustom ed to it. In stud ies, participants report sym ptom s of jitteriness, d iuresis, irritability, insom nia, p sychom otor agitation, and nau sea. There is evid ence of a m ild cannabis w ithd raw al (H) synd rom e lasting several w eeks, characterized by d ecreases in m ood and appetite, w ith increases in irritability, anxiety, and tension. Cocaine intoxication (I) is sim ilar to am phetam ine intoxication. At higher d oses, signs of toxicity m ay d evelop and includ e seizu res, chest p ain, hyp erp yrexia, and d eath. Susceptible u sers m ay also experience paranoia, ranging from m ild hypervigilance to frank paranoia and p ersecu tion. H allu cinogens inclu d e su bstances su ch as LSD, m escaline, and ecstasy. They prim arily affect p erceptions and in intoxication (K) can result in m ood changes, p aranoia, id eas of reference, d ep ersonalization, hallucinations, and synesthesia. Au tonom ic sym ptom s m ay also occu r, and inclu d e blu rry vision, tachycard ia, sw eating, trem ors, and p u p il d ilation. 131–136. [131 (F), 132 (B), 133 (D ), 134 (D ), 135 (E), 136 (A)] Blockad e at the 5-H T2 A (serotonin) recep tor (A) is thou ght to im p rove cognitive and affective sym ptom s in schizophrenia and other psychotic d isord ers. Blockad e at the alp ha-1 ad renergic recep tor (B) is resp onsible for the d izziness, sed ation, and orthostatic hyp otension associated w ith m any of the low p otency typical and atyp ical antip sychotics. The antip sychotics are not know n to affect beta recep tors (C). Blockad e at the d op am inergic D2 recep tors (D ) in the m esolim bic and m esocortical areas of the brain are responsible for red u cing the positive psychotic sym p tom s in schizop hrenia. Blockad e of the D2 recep tor in other p athw ays is resp onsible for som e u nd esirable sid e effects: blockad e in the nigrostriatal pathw ay is associated w ith m ovem ent d isord ers su ch as extrapyram id al sym ptom s, w hereas blockad e in the tuberoinfu nd ibu lar p athw ay is resp onsible for the p rolactinem ia seen w ith som e antipsychotics; d op am ine inhibits the release of p rolactin, hence blockad e of the d opam ine receptor resu lts in p rolactin release. Blockad e at the histam ine-1 receptor (E) is believed to contribu te

to sed ation and cau se w eight gain. By blocking m uscarinic-1 receptors (F), antip sychotics cause anticholinergic sid e effects of sed ation, d ry m ou th, and constipation. 137–145. [137 (G), 138 (I), 139 (F), 140 (D ), 141 (B), 142 (C), 143 (A), 144 (E), 145 (H)] Defense mechanisms are conceptualized as the mind ’s (m ostly unconscious) w ay of d ealing w ith d istressing w ishes/ d rives/ thou ghts. They can be subd ivid ed into im matu re, neu rotic, or mature d efenses. The low er functioning d efense mechanisms are as follow s: Denial (B) is w hen an ind ivid ual refuses to accept reality because it is too d istressing; splitting (I) is also an immature d efense w here the person sees others or other situ ations as “all good or all bad ” and is unable to tolerate positive and negative aspects to be present w ithin one p erson; d isplacement (C) is the channeling of one’s unacceptable w ishes or feelings into less anxiety prod ucing ones—for instance, jerking the d og’s leash harshly w hen one w as really u pset at having been w oken up by a spouse (truly being angry w ith the spouse); in projection (E) one’s u nacceptable id eas or thoughts are seen as com ing from another (for instance, a cheating husband accuses his w ife of being unfaithfu l); projective id entification (F) can be thou ght of as a self-fulfilling prophecy, w herein the patient’s unacceptable feelings are projected to another, but the other (e.g., therapist) acts in such a w ay that they become true—for instance, a patient view s the w orld as fu ll of u nloving people (she hates herself), then acts to push her therapist to the breaking point w here he terminates her care, therefore confirm ing her belief that the w orld is full of unloving people. Examples of neu rotic d efenses are as follow s: rationalization (H) is the process of “making excu ses” or cognitively reframing a situ ation to make it less anxiety or d istress provoking; reaction formation (G) d escribes the process of turning unacceptable d rives and d esires into their opposite. The follow ing are m atu re d efense mechanisms: altru ism (A) involves acceptable actions that serve others and bring pleasure to the ind ivid u al; hu mor (D ) involves expressing unpleasant thoughts or feelings in a w ay that brings enjoyment and pleasure to others.

CHAPTER 3

S o matic Tre atme nt and Ps yc ho pharmac o lo g y Que s tions

D IRECTION S (Questions 1 through 73): For each of the multiple-choice questions in this section select the lettered answ er that is the one best response in each case. Questions 1 and 2 A 32-ye r-o n is itte to gener hos it fter ingesting n u nknow n qu ntity of hene zine in su ici e tte t. After g stric v ge n inistr tion of ch rco s u rry, he is tr nsferre to the e ic intensive c re u nit for onitoring. Tw entyfou r hours ter, he begins to see horses running in the h n u s out his intr venous (IV) ines. 1. Which of the fo ow ing tre t ents w ou the ost i ort nt t this ti e? (A) (B) (C) (D) (E)

be

Ch or ro zine Cy rohe t ine Lor ze Me eri ine Phenytoin

2. Tw e ve ys fter his su ici e tte t, he receives ven f xine (Effexor) to tre t his e ression. One hour fter ingestion of the ven f xine, he beco es t chyc r ic n i horetic n eve o s yoc onic jerks. Which of the fo ow ing con itions h s he ost ike y eve o e ? (A) Acute ystoni (B) Ak thisi (C) N eu ro e tic

ign nt syn ro e

(D) O isthotonos (E) Serotonin syn ro e Questions 3 and 4 A 24-ye r-o w o n resents to her ri ry c re hysici n for regu r checku . During the ex in tion, her octor notes 15- b w eight g in. The tient co ins th t she w ys fee s tire uring the y es ite s ee ing fro 8 pm to 9 a m i y. On further questioning, the hysici n e rns th t the tient fee s s n e ty, often thinks bout e th, c nnot concentr te t w ork, n cks the energy to c re for her tw o chi ren. H ow ever, her oo icks u w hen her hysici n questions her bout her chi ren. 3. Which of the fo ow ing e ic tions w ou the ost ro ri te to tre t her i ness? (A) (B) (C) (D) (E)

be

F u oxetine N ortri ty ine P roxetine Phene zine Tr zo one

4. H er hysici n initi tes tre t ent w ith n ntie ress nt t ther eu tic ose. Which of the fo ow ing ti es w ou be consi ere iniu equ te tri for the e ic tion? (A) (B) (C) (D) (E)

1 w eek 2 w eeks 6 w eeks 3 onths 6 onths

81

82

3: S oma tic Tre a tme nt a nd P syc hop ha rma c ology

Questions 5 and 6 Whi e on c in gener hos it , you re c e t 4:30 a m to ev u te tient w ho w nts to be isch rge . U on rriving on the f oor, the nurse te s you he ju st u e ou t his thir IV of the y n st rte to sw ing the IV o e in the ir w hi e ye ing rof nities. H e te s nu rse’s i e he u st c tu re the tiger th t is oose in the rking ot. You qu ick y ook t his ch rt n see th t he is 55-ye r-o n w ith history of coho u se isor er, itte 36 hou rs e r ier for b o in in, n u se , n vo iting. H is tr ns in ses w ere e ev te on ission, bu t his he titis rofi e is sti en ing. H e is sche u e for u er en osco y in the orning. 5. Which of the fo ow ing shou be the ost ro ri te e ic tion given t this ti e? (A) (B) (C) (D) (E)

Ch or i ze oxi e Ch or ro zine Disu fir H o eri o Ox ze

6. You s e k brief y w ith this tient n re b e to sett e hi ow n. H is te er tu re is 102.1°F, u se is 130 be ts/ in, n b oo ressu re is 220/ 120 H g. Which of the fo ow ing is the ost ro ri te e ic tion n rou te of inistr tion to give? (A) (B) (C) (D) (E)

H o eri o intr u scu r y (IM) H o eri o intr venou s y (IV) L bet o IV Lor ze IM Lor ze IV

7. A 32-ye r-o sing e w o n resents to the e ergency e rt ent escorte by her f i y fter she reve e to the her su ici e n to t ke fu bott e of cet ino hen n rink bott e of w ine. She st tes th t she c nnot h n e w ork ny onger bec u se her boss is trying to h ve her fire n her cow orkers re he ing hi fin f u t in her w ork. She c i s they h ve even t e her hone n re h ving her fo ow e . She re orts ifficu ty f ing s ee n oor concentr tion t w ork. She h s ost 10 b over the st 2 onths, in rt

bec u se she is w orrie th t her foo y be oisone . U on interview, she is ys oor eye cont ct, sycho otor ret r tion, n te rfu n constricte ffect. Which of the fo ow ing w ou be the ost ro ri te e ic tion regi en for this tient? (A) (B) (C) (D) (E)

C oz ine F u oxetine n ris eri one F u oxetine n c on ze F u oxetine onother y Ris eri one onother y

8. A 40-ye r-o w o n w ith bi o r isor er resents to the e ergency roo 2 w eeks fter st rting new e ic tion. She re orts th t she w s oing “w e ” u nti she got vir g stroenteritis n w s u n b e to e t for sever ys. Whi e the vir sy to s reso ve , she now co ins of n u se n vo iting, t xi , n tre or. Which of the fo ow ing stu ies shou be or ere first? (A) (B) (C) (D) (E)

C rb ze ine eve De kote eve H e CT Lithiu eve Urine toxico ogy screen

9. A 25-ye r-o n resents to the e ergency e rt ent cco nie by his f i y w ho re ort th t the tient h s not eft his be roo in 3 w eeks. They re ort th t he s en s his y re ing science fiction nove s th t they rovi e for hi . On ex in tion, you fin hi to be o orou s n isheve e . H e is gu r e w hen you s e k w ith hi . H e oes not ke eye cont ct n he voi s ooking t the te evision in the w iting roo bec u se, he s ys, he w nts to revent intru ers fro contro ing his thoughts. After co eting the ex in tion, you it hi to the in tient sychi tric u nit n begin ri i r zo e. After 1 w eek, he is no onger fr i to ook t the te evision bu t he s en s uch of the y cing the f oor, st ting th t he fee s rest ess. After re u cing the ri i r zo e to the ow est ose you be ieve is ro ri te, he sti ces the h s. Which of the fo ow ing is the ost ro ri te next ste ?

Que s tions : 5–13

(A) (B) (C) (D) (E)

A benztro ine A i henhy r ine A or ze A ro r no o Discontinu e ri i r zo e n try nother nti sychotic

10. A 35-ye r-o w o n resents to the office st ting th t she is e resse . She w s i off fro her w ork 5 w eeks go, n is h ving trou b e fin ing new job. She escribes fee ing very ow, w ith fee ings of ho e essness bout the fu tu re, n ifficu ty getting herse f u n resse in the orning. She h s ost 5 b n is s ee ing oor y. She re orts one revious e iso e of e ression. A ition y, she escribes erio bou t ye r go in w hich she st ye u for sever ys in row nning big rty for her hu sb n ’s birth y, c e ning the hou se n st rting nu erou s other rojects; t th t ti e, others co ente th t she w s t king qu ick y n cting “ ike the energizer bu nny.” Which of the fo ow ing e ic tions w ou be the ost ro ri te tre t ent for her e ression? (A) (B) (C) (D) (E)

Bu ro ion F u oxetine L otrigine V roic ci Ven f xine

Questions 11 and 12 A 33-ye r-o inte ectu y is b e e resi ent of grou ho e resents to the e ergency e rtent escorte by st ff e bers w ho be ieve he h s beco e confu se n isoriente over the st few ys. H e h s been resi ent there for sever ye rs w ithou t inci ent, bu t st w eek his thiori zine w s ch nge to h o eri o bec use of concerns bout the ong-ter u se of high oses of thiori zine. On ex in tion, you fin hi to be isoriente to ce n ti e; he is i horetic w ith te er tu re of 105.8°F, h s he rt r te of 130 be ts/ in, n res ir tory r te of 20 bre ths/ in. H is extre ities re stiff. Rou tine bor tories reve w hite b oo ce (WBC) cou nt of 15,000/ L, b oo u re nitrogen (BUN ) 75 g/ L, cre tinine 1.2 g/ L, n cre tinine hos hokin se 2,300 Iµ/ L.

83

11. Which of the fo owing is the ost ike y serious co ic tion this tient y ex erience? (A) (B) (C) (D) (E)

Diffu se intr v scu r co gu tion Myoc r i inf rction Pu on ry e bo is Res ir tory f i u re Rh b o yo ysis

12. Which of the fo ow ing ong-ter si e effects of high- ose thiori zine ost ike y e his hysici n to sw itch to h o eri o ? (A) (B) (C) (D) (E)

Agr nu ocytosis H y er ro ctine i Pri is Retin ig ent tion T r ive yskinesi

Questions 13 and 14 A 45-ye r-o u ne oye invest ent b nker resents to you r office cco nie by his w ife. H e h s been ou t of w ork for 1 onth bec u se he is fr i to t ke the tr in into w ork. H e never h ifficu ty tr ve ing u nti 2 onths go, w hen on tr in uring f i y v c tion, he su ffere chest in, it tions, shortness of bre th, n use , ting ing in his extre ities, n fe r of ying. These sy to s ste bou t 15 inu tes. A w orku by c r io ogist w s neg tive. H ow ever, these sy to s h ve continu e , so cco nie by intense fe r n fee ing of being et che fro hi se f. H e h s h roxi te y tw o tt cks er w eek bu t so h s severe nxiety w hen just thinking bout returning to w ork s he is fr i he w i ie on the tr in. H e te s you th t he cou not h ve co e to see you w ithou t his w ife’s he . 13. Which of the fo ow ing e ic tions w ou be the ost ro ri te first choice for this tient? (A) (B) (C) (D) (E)

Bu ro ion Bu s irone F u oxetine Pro r no o Tr zo one

84

3: S oma tic Tre a tme nt a nd P syc hop ha rma c ology

14. The tient is nxiou s bou t w iting u nti the e ic tion t kes effect, n sks if you c n give hi so ething th t w i rovi e ore i e i te re ief. You eci e to rescribe benzo i ze ine in the interi . Which of the fo ow ing e ic tions w ou be the ost otent? (A) (B) (C) (D) (E)

A r zo Ch or i ze oxi e Te ze Di ze Ox ze

Questions 15 and 16 A 22-ye r-o sing e e co ege u sic stu ent re u ct nt y resents to you r office t the requ est of his rents. H is rents re w orrie bec u se he h s not been e ting, t ks const nt y, n never see s to s ee . The tient st tes he h s never fe t better, th t he’s been co osing sever songs y, n requ ires on y few hou rs of s ee to fu nction. H e be ieves he w i be the next gre t rock st r n w nts to finish the interview in or er to get b ck to w riting, bu t he c nnot st y focu se ong enou gh to nsw er ny of you r qu estions. H e oes not h ve ny rior or f i y sychi tric history n enies u sing ny coho or i icit ru gs. 15. You eci e to begin tri of ithiu . Before initi ting tre t ent, w hich of the fo ow ing w ou be the ini u bor tory infor tion th t shou be obt ine ? (A) (B) (C) (D) (E)

Co ete b oo cou nt (CBC) F sting g u cose n i i ne Liver fu nction tests Seru oni Seru cre tinine, BUN n e ectro ytes, thyroi stu ies

16. One w eek fter st rting the ithiu , the tient s r ins his nk e w hi e exercising. H is riry c re octor ex ines hi n te s hi to use over-the-counter ibu rofen, w hich he w i h ve to t ke for ore th n 1 w eek. Which of the fo owing wou be the ost ro ri te course of ction?

(A) Continu e ibu rofen t the rescribe ose. (B) Decre se the ose of ibu rofen. (C) Incre se the ose of ibu rofen. (D) Sto the ithiu . (E) Sw itch fro ibu rofen to s irin. Questions 17 and 18 A 9-ye r-o boy is brou ght to the hysici n bec use his rents h ve receive note fro his secon gr e te cher co ining th t he is isru tive in c ss. H is te cher be ieves he cou chieve better gr es if he cou sit sti n y ttention. H is rents re ort he h s w ys been n ctive chi bu t oes h ve ifficu ty getting ong w ith other chi ren in the neighborhoo . In the ex in tion roo , the boy see s un b e to sit sti n h s ifficu ty co eting t sk you h ve ssigne . You eci e to rescribe hi ethy heni te. 17. Which of the fo ow ing is the si e effect of this tre t ent? (A) (B) (C) (D) (E)

D yti e row siness Decre se in systo ic b oo Difficu ty f ing s ee Incre se in etite S ow e grow th

ost co

ressu re

18. Which of the fo ow ing sy to s w ou ike y be ex cerb te by the e ic tion? (A) (B) (C) (D) (E)

on

ost

Be w etting Myo i Poor i u se contro Re ing ifficu ties Tics

19. An 8-ye r-o boy is brou ght to you r office by his other bec u se of his “h bits,” w hich h ve w orsene over the st ye r. She first notice the st ye r w hen sever ti es y he w ou re e te y b ink his eyes n frow n w hi e c e ring his thro t. H e continu es to o this es ite trying not to, n , in ition, he often sticks ou t his tongu e n s e s his shirt w hi e s e king w ith c ss tes. These beh viors h ve resu te in his being

Que s tions : 14–24

te se n osing frien s t schoo . Which of the fo ow ing gents w ou be the ost effective to tre t his isor er? (A) (B) (C) (D) (E)

A r zo A itri ty ine C oni ine P roxetine Ris eri one

(A) (B) (C) (D) (E)

A ithiu . A thyroi hor one (T3). Sw itch to cit o r . Sw itch to nortri ty ine. Sw itch to ven f xine.

Questions 21 and 22 You h ve been tre ting 55-ye r-o n w ith schizo hreni for 20 ye rs w ith h o eri o n benztro ine. Gener y, he h s one w e n h s not requ ire jor e ic tion ch nges or hos it iz tions. Abou t 2 ye rs go, you notice so e i s cking n tongu e rotru sions th t i not bother the tient; how ever, now he so h s o , irregu r ove ents of his r s w hich ke it ifficu t for hi to e t. 21. Which of the fo ow ing con itions is ike y the c u se of these sy to s? (A) (B) (C) (D) (E)

Anticho inergic toxicity H u ntington ise se Meige syn ro e Sy enh chore T r ive yskinesi (TD)

22. Sw itching to w hich of the fo ow ing e ic tions w ou be the ost ike y to i rove these sy to s? (A) (B) (C) (D) (E)

20. A 45-ye r-o n w ith history of hy ertension, yoc r i inf rction, n chronic inso ni is referre to you by his ri ry c re hysici n for ev u tion of his jor e ressive e iso e. H e h s been tre te for 3 onths w ith f u oxetine 60 g i y, w ith no i rove ent. H e h s no history of rior jor e ressive e iso es. Which of the fo ow ing w ou be the ost ro ri te next ste in his tre t ent?

ost

85

Ari i r zo e C oz ine H o eri o O nz ine Ris eri one

23. A 24-ye r-o rrie secret ry h s co ine of izziness, it tions, n sw e ty s for the st 10 onths. She so occ sion y h s extre e u sc e tension in her neck th t occu rs both t w ork n t ho e. She h s ifficu ty f ing s ee n fee s “e gy” ost of the y. In ition, she h s h ifficu ty concentr ting t w ork resu ting in her king ist kes. She h s gone to oc e ergency roo three ti es bec u se of the it tions bu t “nothing w s foun .” H er f i y r ctitioner ce her on hy og yceic iet n referre her to neu ro ogist w ho fou n no foc bnor ities. She te s you she h s ny w orries bout her f i y but oes not fee rticu r y e resse . Which of the fo ow ing e ic tions w ou rovi e the ost i e i te re ief of her sy to s? (A) (B) (C) (D) (E)

Bu ro ion Bu s irone C o i r ine F u oxetine Lor ze

24. A 22-ye r-o w o n is itte to sychitric u nit fter seriou s su ici e tte t. She h s h ny su ici e tte ts in the st w ith v rying severity. H er r s re sc rre fro rior tte ts t hu rting herse f. She h been goo stu ent u nti high schoo , w hen she took u w ith “f st” crow , beg n bu sing coho n riju n , n r n w y fro ho e sever ti es. She h s h sever intense, stor y re tionshi s w ith en. Ou t tient tre t ent h s ost y consiste of her co ints to her ther ist bou t her f i y. She u su y c s her ther ist iy bou t ifferent crises; how ever, her ther ist w s on v c tion u ring her ost recent crisis.

86

3: S oma tic Tre a tme nt a nd P syc hop ha rma c ology

She escribes fee ings of ho e essness n he essness consistent y for the st sever w eeks; in ition, she is s ee ing oor y, tire throu ghou t the y, istr cte , n h s ost so e w eight ue to ck of etite. Which of the fo ow ing e ic tions w ou be the ost he fu for this tient? (A) (B) (C) (D) (E)

C on ze F u oxetine H o eri o Lithiu Zi r si one

25. A 30-ye r-o w o n, 10 w eeks regn nt w ith her first chi , resents to you r office st ting th t she is e resse n is h ving oubts bou t beco ing other. She h s been ex eriencing i y e esis n is h ving trou b e fu nctioning t w ork. She re orts th t she w s e resse throu ghou t her 20s, but iscontinu e her nti e ress nts w hen she bec e regn nt bec u se of concerns bout h r ing her chi . She w ou ike you r he in ev u ting risks n benefits of rest rting sertr ine, w hich w orke w e for her in the st. Which of the fo ow ing risks w ou be the ost ike y to occu r? (A) (B) (C) (D) (E)

Ebstein no y Gest tion i betes Inf nt eve o ent e y N eur tu be efects Persistent u on ry hy ertension of the new born

26. A 19-ye r-o w o n is itte to n in tient sychi tric f ci ity for her e ting isorer. She w eighs 82 b n st n s 5 ft 6 in t . She beg n ieting in high schoo to ose n unw nte 5 b. Encour ge by co i ents on her figu re, she continu e to ose nother 10 b. H er e ting h bits re ritu ize : She cuts her foo into s ieces n oves it rou n on her te. She h s n intense fe r of being overw eight. Which of the fo ow ing e ic tions w ou be the ost usefu for this tient? (A) Cy rohe t (B) F uoxetine

ine

(C) N o e ic tion (D) O nz ine (E) To ir te 27. A 36-ye r-o n w ith his first e iso e of jor e ression h s been tre te for 8 onths w ith roxetine n is cu rrent y in re ission. Bec u se of sexu ysfu nction, he eci e to sto t king the e ic tion 3 ys go w ithou t first infor ing you . Yester y, he bec e cu te y irrit b e n even h crying s e . H e so co ins of bo y ches, chi s, n gener eth rgy. Which of the fo ow ing ste s w ou best evi te his sy to s? (A) (B) (C) (D) (E)

Rest St rt St rt St rt St rt

rt roxetine. cet ino hen. bu ro ion. v roic ci . ven f xine.

28. A 20-ye r-o w o n is st rte on otrigine for bi o r isor er, cu rrent y e resse . Tw o onths ter, she co es b ck re orting th t her sy to s re unch nge , es ite her being co i nt w ith the e ic tion. You review her e ic tion ist n iscover th t she w s recent y st rte on new e ic tion(s). Which of the fo ow ing e ic tions w ou ost ike y be res onsib e for the ck of effic cy? (A) (B) (C) (D) (E)

Dros irenone n ethiny estr Ibu rofen Lithiu V roic ci Zi r si one

io

Questions 29 and 30 A 68-ye r-o n resents to you r office on referr fro his ri ry c re hysici n for ev u tion of e ression. H e fee s e resse n ho e ess, n he h s ost his etite. H e no onger enjoys seeing his gr n chi ren. It t kes hi sever hours to f s ee t night, n he st ys s ee for on y bou t 3 hou rs. H e occ sion y oes not bu y foo bec u se he is concerne bou t going broke; how ever, his f i y infor s you th t he is fin nci y w e -off. In ition, the tient fee s signific nt gu i t bou t

Que s tions : 25–34

beco ing bu r en on his f i y. H is height is 6 ft w ith w eight of 155 b, ecre se fro his u su 175 b. 29. Which of the fo ow ing e ic tions w ou be the ost ro ri te for tre ting his inso ni ? (A) (B) (C) (D) (E)

Bu ro ion Di ze Di henhy r F uoxetine Tr zo one

ine

Anorg s i Gyneco sti I otence Pre tu re ej cu tion Pri is

Questions 31 and 32 A 57-ye r-o n w ith bi o r isor er, w ho you h ve tre te su ccessfu y for 20 ye rs w ith ithiu 300 g ti , is itte to the gener hos it w ith chest in. 31. Which of the fo ow ing fin ings w ou be ost ike y seen on his e ectroc r iogr (ECG)? (A) (B) (C) (D) (E)

As irin Decre se f u i int ke H y roch orothi zi e N itrog ycerin Pro r no o

Questions 33 and 34

30. Which of the fo ow ing si e effects of this e ic tion w ou be the ost i ort nt to e u c te the tient bou t? (A) (B) (C) (D) (E)

(A) (B) (C) (D) (E)

87

First- egree trioventricu r (AV) b ock PR interv ro ong tion QT interv ro ong tion Sino tri b ock T-w ve e ression

32. The tient is eter ine to h ve su ffere yoc r i inf rction (MI) n qu ick y u n ergoes ngio sty. H e is eventu y st bi ize n isch rge fro the hos it on ro r no o , to ic nitrog ycerin, hy roch orothi zi e, n s irin, in ition to his ithiu . Prior to his MI, his ithiu eve w s u su y 0.8 to 0.9 o / L. N ow it is 1.3 o / L on the s e ose. Which of the fo ow ing is the ost ike y c u se for this ch nge?

A 47-ye r-o e nove ist h s fe rs of cont in tion fro nything he be ieves is “ irty.” In rest u r nts, he u ses his ow n stic u tensi s th t he c rries w ith hi . E sew here, he w e rs g oves or u ses er tow e s to voi tou ching “ irty objects.” U on retu rning ho e, he then rocee s to w sh his h n s 10 ti es before going bou t his ctivities. If he ccient y tou ches nything rior to the co etion of w shing his h n s, he ex eriences nxiety n is then u n b e to erfor his u su night y t sks. As resu t of these sy to s he re ins u n b e to h ve e ningfu soci ife. 33. Which of the fo ow ing e ic tions w ou be the ost effective tre t ent for his con ition? (A) (B) (C) (D) (E)

C o i r ine C on ze N ortri ty ine O nz ine Phene zine

34. After 12 w eeks of ther eu tic ose of the e ic tion, the tient no onger nee s to w sh his h n s qu ite so often bu t sti h s signific nt istress w hen he touches “ irty things.” Which of the fo ow ing tre t ents w ou be the ost ro ri te next ste ? (A) (B) (C) (D) (E)

A bu s irone. A ithiu . A ris eri one. Ch nge to ris eri one. Refer for cingu oto y.

Questions 35 and 36 A 21-ye r-o co ege stu ent h s been ex eriencing u itory h u cin tions w ith co n s te ing hi to hurt hi se f for ore th n ye r. H e h s h tris w ith the fo ow ing gents since th t ti e: qu eti ine 400 g bi for 2 onths, then h o eri o

88

3: S oma tic Tre a tme nt a nd P syc hop ha rma c ology

10 g bi for 6 w eeks, then o nz ine 30 g i y for 2 onths. Des ite inor re u ction in the voices, he re ins iso te fro f i y n frien s. 35. Which of the fo ow ing gents w ou ost ro ri te to begin? (A) (B) (C) (D) (E)

be the

Ari i r zo e C oz ine Per hen zine Ris eri one Zi r si one

36. Which of the fo ow ing rece tors is ost ike y res onsib e for the bove e ic tion’s effic cy? (A) (B) (C) (D) (E)

D2 D3 D4 5-hy roxytry t 5-hy roxytry t

ine-1 (5-H T1) ine-3 (5-H T3)

37. A 65-ye r-o n w ith jor e ression n hy ertension resents to the e ergency e rt ent bec u se he fe to the grou n fter rising fro ch ir. H e h s no signific nt c ri c history n his ECG, e ectro ytes, n neu ro ogic ex in tion re u nre rk b e. H is f i y re orts th t his hysici n recent y st rte new nti e ress nt bu t they o not know its n e. Which of the fo ow ing nti eress nts w ou ost ike y be res onsib e for his resent tion? (A) (B) (C) (D) (E)

Bu ro ion F u oxetine I i r ine Mirt z ine N ortri ty ine

38. A 21-ye r-o w o n w ith history of bu ii nervos resents to the e ergency e rt ent w ith her rents. She returne fro co ege st w eek, n since th t ti e, h s been st ying u very te, c ing frien s roun the w or n “t king nonsense” bout n internet st rt-u th t she is nning. She sks you to hu rry u w ith you r interview, since she h s w ork to get b ck to. H er rents

re ort th t she f i e sever c sses bec u se she h s not been going to c ss or tu rning in ssign ents. She is not t king ny e ic tions n her toxico ogy screen is neg tive. On ent st tus ex in tion, her s eech is ressu re n her thou ght rocess is t ngenti . You eci e to it her to the hos it . Which of the fo ow ing e ic tions w ou be the ost ro ri te to begin? (A) (B) (C) (D) (E)

Cit o r L otrigine O nz ine To ir te V roic ci

39. A 28-ye r-o w o n w ho you re tre ting for bi o r isor er infor s you th t she is 3 w eeks regn nt n is nxiou s bou t the b by’s he th. She is cu rrent y rescribe ithiu 300 g bi , n she h s not h nic e iso e in 3 ye rs. Wh t w ou be best to vise her reg r ing the u se of ithiu uring regn ncy? (A) Lithiu is highest risk u ring the thir tri ester of regn ncy. (B) Lithiu incre ses the risk of neu r tu be efects. (C) Lithiu c u ses eve o ent e y in ex ose inf nts. (D) Lithiu incre ses the risk of c r i c for tion. (E) Lithiu incre ses the risk of c eft i n te. 40. You re c e to consu t on n 85-ye r-o surgic tient w ho h s beco e co b tive, ye ing, u nching st ff, n u ing ou t her IVs. She e n s to e ve bu t is too w e k to get ou t of her hos it be . Which of the fo ow ing w ou be the ost ro ri te intervention t this ti e? (A) (B) (C) (D) (E)

Di henhy r ine Done ezi Lor ze Orient tion to her su rrou n ings Ris eri one

Que s tions : 35–46

Questions 41 and 42 A 67-ye r-o w o n resents to your office for ev u tion of 20- b w eight oss over 4- onth erio n inso ni for the st 4 w eeks. H er ughter infor s you th t her other no onger enjoys ny of her hobbies n h s been s e king te y bou t ying. They h ve re y consu te n internist w ho erfor e n extensive e ic w orku , inc u ing che istries, b oo cou nt, thyroi stu ies, g strointestin i ging, n en osco y, of w hich w ere nor . 41. You w ou ike to begin n nti e ress nt e ic tion bu t w ou ike to ini ize verse si e effects w hi e “ex oiting” other si e effects. Which of the fo ow ing gents w ou be the ost ro ri te? (A) (B) (C) (D) (E)

Bu ro ion F u oxetine Mirt z ine N ortri ty ine Sertr ine

42. U on fu rther history, you e rn th t she h s h sever f i e tri s of nti e ress nts n w ou ike to st rt tricyc ic nti e ress nt. Which of the fo ow ing con itions w ou be the ost ike y to rec u e u sing su ch n gent? (A) (B) (C) (D) (E)

Estrogen re ce ent ther y L cu n r inf rcts Left bu n e br nch b ock on ECG Tre t ent w ith o i ine Urin ry retention

89

43. Which of the fo owing gents wou be the ost ro ri te to inister for his git tion? (A) (B) (C) (D) (E)

Ch or ro zine H o eri o Lor ze Ris eri one Thiori zine

44. Which of the fo ow ing gents w ou be the ost ro ri te to give in or er to ini ize the tient’s risk of f ing? (A) (B) (C) (D) (E)

H o eri o Per hen zine Qu eti ine Ris eri one Thiori zine

Questions 45 and 46 A 32-ye r-o n resents to your c inic fter osing his job bec use he w s intoxic te w hi e w orking. H e h s been rinking i y since he w s 16 ye rs o . H e w s b e to co ete co ege n w ent to w ork fu ti e right fter gr u tion, but h s ost sever jobs since. H is w ife h s thre tene to e ve hi if he oes not get he . 45. Which of the fo ow ing e ic tions w ou be the ost u sefu s beh vior o ifier to ecre se his coho use? (A) (B) (C) (D) (E)

Ac ros te Disu fir F u zeni N oxone N trexone

Questions 43 and 44 A 72-ye r-o n is itte to gener hos it ’s intensive c re u nit bec u se of tere ent st tu s. H is e ic w orku h s reve e neu oni n congestive he rt f i u re (CH F). On the secon hos it y, he is git te n u s out his IV ccess. H e so h s been note to s e k ou t ou w ith no one in the roo . H is eve of consciou sness see s to w x n w ne. H e oes not h ve sychitric history n is not ergic to ny e ic tions. Besi es his CH F n neu oni , he oes not h ve other co orbi con itions.

46. Further history reve s th t he oes not h ve oo isor er. He te s you th t often he si y c nnot contro his cr ving to h ve nother rink. Which of the fo owing gents wou be the ost ike y to ecre se his cr vings for coho ? (A) (B) (C) (D) (E)

Bu ro ion Disu fir F u zeni N oxone N trexone

90

3: S oma tic Tre a tme nt a nd P syc hop ha rma c ology

Questions 47 and 48 A 36-ye r-o w o n is referre to you by her riry c re hysici n for n ge ent of nxiety n fe r th t h s ersiste for sever onths. She re orts th t she w s r e n he host ge on bo t for sever ys by tw o r e en. Du ring this, she ex erience intense fe r for her ife. Since then, she h s h intense stress w henever she is ne r the w ter n h s frequ ent night res. She c nnot rec et i s of the or e bu t tries to voi riving her c r w ithin sight of the oce n, w hich h s been ifficu t bec u se of the oc tion of her ho e. She fee s et che fro her hu sb n n f iy n h s b n one ns to u rsue her c reer s inter. She h s ifficu ty f ing s ee n is e si y st rt e by hone c s. As resu t of her ex erience, she no onger goes to u b ic ces one. 47. Which of the fo ow ing e ic tions w ou be ost he fu in tre ting the sy to s she is ex eriencing? (A) (B) (C) (D) (E)

A r zo C rb ze ine F u oxetine N trexone Thiori zine

48. She is tre te w ith the ro ri te e ic tion n h s signific nt re u ction in ost of her sy to s. H ow ever, she continu es to ex eriences frequ ent, intense night res, resu ting in her h ving oor s ee n f tigu e the next y. Which of the fo ow ing gents w ou be the ost benefici in re u cing these sy to s? (A) (B) (C) (D) (E)

Lor ze N trexone O nz ine Per hen zine Pr zosin

49. You h ve been ske to ev u te n 80-ye ro e nu rsing ho e resi ent bec u se of ch nge in ent st tu s. The nu rsing st ff re orts th t he h s been tient there for bou t ye r w ithout ny rob e s, u nti

bou t 1 onth go w hen he beg n h ving ifficu ty s ee ing. The st ff hysici n rescribe or ze 2 g t be ti e; how ever, he w s sti u n b e to s ee . H e st rte w n ering u ring the y n h s since been restricte to his roo . Bec u se he re ins git te u ring the y, they h ve been giving hi or ze 2 g every 6 hou rs for git tion. H is other e ic tions inc u e hy roch orothi zi e 25 g/ , igoxin 0.125 g every other y, i ti ze 240 g su st ine re e se i y, n tr ns er nitrog ycerin 0.4 g/ h s nee e . H is e ectro ytes re nor exce t for ot ssiu eve of 3.4 o / L. A recent igoxin eve is not v i b e. Which of the fo ow ing interventions shou be consi ere first? (A) (B) (C) (D) (E)

Checking the hy roch orothi zi e eve . Monitoring orthost tic vit s. Re cing the ot ssiu . Sto ing the roo sec u sion. T ering the or ze .

Questions 50 and 51 A 64-ye r-o n, cco nie by his w ife, resents to you r office co ining of e ory oss. H e is retire stockbroker w ho c n no onger rec stock qu otes ike he i sever ye rs go. H is w ife h s beco e both concerne n nnoye bec u se he see s to i e i te y forget w h tever she te s hi . These e ory rob e s h ve s ow y rogresse over sever ye rs. Recent y, he h ifficu ty getting resse , n ore th n once, he u t his u n erw e r on over his nts. H is w ife h s so note th t he s e ks u ch ess u ring inner th n he once i . H e oes not h ve ny other signific nt e ic i nesses. 50. Which of the fo ow ing e ic tions w ou ost ike y to i rove his con ition? (A) (B) (C) (D) (E)

A r zo Done ezi F u oxetine Methy heni Tr zo one

te

be

Que s tions : 47–56

51. U on fu rther questioning, his w ife re orts th t he oes not s ee w e t night n w n ers rou n the hou se i ess y. Su bsequ ent y, he s ee s for rge rt of the y. Which of the fo ow ing e ic tions w ou be the ost u sefu to contro this beh vior? (A) (B) (C) (D) (E)

H o eri o Lor ze Tr zo one Tri zo Qu eti ine

Questions 52 and 53 A 25-ye r-o n is brou ght into the e ergency e rt ent eth rgic n stu orou s. H e res on s on y to infu sti u i, w kes u brief y n ye s, then goes b ck to s ee . A bu nce ersonne re ort th t they fou n hi ne r hou se know n for ru g tr fficking. There is no evi ence of hysic inju ry. 52. Which of the fo ow ing receive first?

e ic tions shou

he

(A) Dextrose n f u zeni (B) Dextrose, f u zeni , n n oxone (C) Dextrose, f u zeni , n oxone, n thi ine (D) Dextrose n n oxone (E) Dextrose, n oxone, n thi ine 53. The tient is su bsequent y tre te n u rine toxico ogy screen is ositive for the resence of o ioi s. When he is ore ert, he infor s you th t he h s been using intr venous (IV) heroin i y for sever w eeks. H is st use w s bout 8 hours go. H is heroin u se esc te fro snorting to IV use fter he no onger fe t goo “high” fro snorting. H e h s never etoxifie before. H e h s no co orbi e ic con itions n he oes not use coho . Which of the fo ow ing e ic tions w ou be the ost ro ri te to etoxify this tient? (A) (B) (C) (D) (E)

Bu renor hine Ch or i ze oxi e C oni ine N oxone Pro r no o

91

54. A 48-ye r-o w o n resents to you r r ctice w ith co ints of e ression. H er other sse w y 6 onths go fro u ng c ncer n her f ther ie 4 ye rs go fro the s e ise se. She h s seen ther ists to he w ith her grief but she continu es to h ve ongoing s ness, ter in inso ni , ow energy, oor etite, n ifficu ty concentr ting. She h s ssive thou ghts of su ici bu t no n or intent. She enies rior su ici e tte ts. In ition, es ite sever tte ts to qu it s oking, she sti s okes tw o cks of cig rettes y n h s h w orsening sth s resu t. She h s no ition e ic robe s. Which of the fo ow ing e ic tions w ou be the ost ro ri te to u se in this tient? (A) (B) (C) (D) (E)

Bu ro ion Bu s irone I i r ine Phene zine Tr zo one

55. A 52-ye r-o n w ho h s oor y contro e i betes is referre to you for ev u tion of e ic tion nonco i nce. You e rn th t he c nnot f s ee t night n h s no energy u ring the y. H is etite is gone n he oes not enjoy ctivities s he once i . Overhe fee s e resse . A ition y, he h s been ex eriencing chronic in in his feet, ee e to be u e to neu ro thic in, w hich is not re ieve by n gesics. Which of the fo ow ing e ic tions w ou be the ost u sefu in tre ting this tient’s sy to s? (A) (B) (C) (D) (E)

A A Bu Cit Du

r zo itri ty ine ro ion o r oxetine

56. A 47-ye r-o w o n resents to your office for sychi tric c re bec u se she h s ju st re oc te to you r tow n. She h s history of bi or isor er, bu t her rior sychi trist recent y sto e her ithiu bec u se she eve o e thyroi uto ntibo ies. She co ins of being irrit b e n not b e to s ee for sever weeks.

92

3: S oma tic Tre a tme nt a nd P syc hop ha rma c ology

She h s “ ots” of energy bu t is king ist kes t w ork s she is istr cte . She te s you th t her thou ghts re r cing n you h ve ifficu t ti e re irecting her nsw ers. H ow ever, she be ieves her biggest rob e is her f ci in, w hich w s i gnose s trige in neur gi n h s not i rove w ith n gesics. Which of the fo ow ing e ic tions w ou be the ost ro ri te in tre ting this tient’s sy to s? (A) (B) (C) (D) (E)

A itri ty ine C rb ze ine F u oxetine G b entin Lithiu

57. A 33-ye r-o w o n w ith history of schizohreni h s recent y st rte tre t ent w ith o nz ine. She h s to er te the e ic tion w e n is iving in grou ho e, ntici ting oving into her ow n rt ent. She no onger he rs voices n no onger h s esire to hurt herse f. Which of the fo ow ing si e effects w ou be ost ike y to interfere w ith her co i nce? (A) (B) (C) (D) (E)

Agr nu ocytosis Deve o ent of c t r cts G ctorrhe Incre se excit bi ity Weight g in

58. A 32-ye r-o e w ith schizo hreni is being tre te for n cu te ex cerb tion of his u itory h u cin tions. As is his ttern, he res on s very f vor b y to e ic tions w hi e hos it ize , bu t w hen he is isch rge forgets to t ke the regu r y. This resu ts in cyc e of subsequent w orsening of his sychosis n rehos it iz tion. H e is current y t king ris eri one 5 g/ or y, n he is to er ting it w e w ith goo effic cy. Which of the fo ow ing w ou be the ost ro ri te next ste in his tre t ent? (A) Continu e the cu rrent regi en. (B) Sw itch to c oz ine. (C) Sw itch to h o eri o .

(D) Sw itch to o nz ine. (E) Sw itch to ris eri one ong- cting injection. Questions 59 and 60 A 50-ye r-o n w ho h s fe t e resse for sever onths, resents to you r office for ev u tion. He h s h oor s ee , ecre se etite w ith w eight oss, no interest in his hobbies, n c nnot concentr te u ring you r cognitive ex in tion. H e h s ssing thou ghts of ying bu t no ctive n or intent. H e re u ct nt y its to frequ ent y he ring voices te ing hi th t he is going to ie soon. You su bsequ ent y eci e to st rt n nti e ress nt n ris eri one t 0.5 g bi . 59. Which of the fo ow ing si e effects of ris erione w ou be the ost ike y? (A) (B) (C) (D) (E)

Agr nu ocytosis Anticho inergic si e effects Leu kocytosis Orthost tic hy otension Weight oss

60. As his ose of ris eri one is incre se , the risk of w hich of the fo ow ing si e effects w i so incre se? (A) (B) (C) (D) (E)

Agr nu ocytosis Anticho inergic effects Extr yr i sy to s Leu kocytosis Weight oss

61. A 33-ye r-o rrie fe e w ith history of bi o r isor er, w ith eri rtu onset, h s been st b e since being int ine on riseri one for the st sever ye rs. H er son is oing w e , n the tient n hu sb n w ish to h ve nother chi . For the st ye r she h s been tte ting to conceive w ithou t su ccess. She is on no other e ic tions besi es the nti sychotic, n she h s not been u sing birth contro . She h s no other e ic robe s, n she is current y sy to tic, w ithou t e ression or sychotic sy to s. She rinks 1 to 2 g sses of w ine er w eek, enies i icit rugs, n oes not use tob cco ro ucts.

Que s tions : 57–66

Invo ve ent of w hich of the fo ow ing br in thw ys is ost ike y res onsib e for her inferti ity? (A) (B) (C) (D) (E)

Locus ceru eu s Mesocortic tr ct Meso i bic tr ct N igrostri t tr ct Tu beroinfu n ibu r tr ct

Questions 62 and 63 A 19-ye r-o co ege stu ent resents to the office t the u rging of her frien s, w ho re concerne bout her w e -being. For the st sever onths, she h s been incre sing y reoccu ie w ith her w eight n bo y i ge, n h s been regu r y c nce ing ns w ith frien s bec u se she oes not w nt to e t w ith other eo e. She st tes th t she is “f t” n w ou ike to ose 10 b. When qu erie fu rther, she its th t she is bingeing n throw ing u ost every y. She enies ny x tive use, n she h s not been restricting her e ting. She is 5 ft 4 in n w eighs 110 b. H er ot ssiu is 3.4; other e ectro ytes re w ithin nor i its. You re ete her ot ssiu n refer her to sychi trist for fu rther ev u tion. 62. Which of the fo ow ing w ou be the effective e ic tion for her con ition? (A) (B) (C) (D) (E)

ost

Bu ro ion F u oxetine L otrigine Lithiu Mirt z ine

63. You ter e rn th t she w s tre te in high schoo for the s e sy to s w ith sertr ine n cit o r , both t high oses for over 3 onths e ch, bu t w ith itt e i rove ent in her sy to s. Which of the fo ow ing e ic tions w ou be the ost ro ri te next ste ? (A) (B) (C) (D) (E)

Bu ro ion Lithiu P roxetine To ir te V roic ci

93

64. A 45-ye r-o fe e execu tive w ho tr ve s often to Eu ro e fro the Unite St tes resents to you r office co ining of signific nt inso ni n yti e f tigu e fter tr ns t ntic f ights. Which of the fo ow ing e ic tions w ou be the ost ro ri te tre t ent for her co ints? (A) (B) (C) (D) (E)

A itri ty ine Di henhy r ine Me tonin Qu eti ine Te ze

Questions 65 and 66 A 60-ye r-o n resents to you r office co ining of the su en onset of it tions, sw e ting, sh king, shortness of bre th, choking, n fee ing th t he is “going cr zy.” These sy to s st 5 to 10 inu tes n h ve occu rre sever ti es er w eek for the st 3 onths. H e enies ny rticu r stressor or event w hich rovokes the tt cks, thou gh he is “const nt y” fr i of h ving futu re e iso es. H e h s recent y been seen by his c r io ogist w ho h s ru e ou t c r i c etio ogy. 65. Which of the fo ow ing w ou be the ost ro ri te e ic tion to tre t this tient’s isor er? (A) (B) (C) (D) (E)

Bu ro ion I i r ine Phene zine Qu eti ine Sertr ine

66. If the tient’s sy to s w ere u e to c ri c etio ogy n he su bsequ ent y bec e e resse , w hich e ic tion w ou ose the highest risk? (A) (B) (C) (D) (E)

A itri ty ine Bu ro ion Methy heni te Sertr ine Ven f xine

94

3: S oma tic Tre a tme nt a nd P syc hop ha rma c ology

67. A 33-ye r-o w o n w ith i gnosis of schizo hreni h s been int ine on h oeri o since she w s i gnose t ge 24. In you r office, she s ys she h s not h her enstru erio in sever onths, h s i inishe sex rive, h s been isch rging i k fro her bre sts, n h s in u ring sexu intercou rse. Which of the fo ow ing w ou ost ike y be incre se in this tient? (A) (B) (C) (D) (E)

A renocorticotro ic hor one (ACTH ) Do ine N ore ine hrine Pro ctin Serotonin

68. A 24-ye r-o n is hos it ize bec use he h s been he ring voices for 6 onths te ing hi to ki hi se f n now fe rs th t he y ct on the . H e ost his job 2 onths go n on ent st tu s ex in tion e rs u nke t n isorg nize , w ith f t ffect. H e is begu n on ri i r zo e, n his sy to s eventu y ecre se enou gh for hi to be isch rge . On fo ow -u visit to you r office, he re orts th t his h ucin tions h ve i rove bu t th t he “c n’t sit sti ” n fee s ike he nee s to be const nt y in otion. The tient inter ittent y st n s u n w ks rou n you r office s you interview hi . Which of the fo ow ing con itions ccount for his new co ints? (A) (B) (C) (D) (E)

Acu te ystonic re ction Ak thisi M nic e iso e N eu ro e tic ign nt syn ro e T r ive yskinesi

69. A 55-ye r-o n w ith ong history of jor e ression resents to your office fter u tie f i e nti e ress nt tri s. Since the ge of 20, he h s h equ te tri s of nortri ty ine, hene zine, f uoxetine, sertr ine, bu ro ion, irt z ine, ven f xine, n u oxetine, w ith ini res onse. Other sychi trists h ve suggeste ECT in the st, bu t he is concerne bout e ory oss. H e w ou ike to try tr nscr ni gnetic sti u tion (rTMS), n w nts to know ore bout si e effects. Which

of the fo ow ing si e effects o you infor is the ost ike y? (A) (B) (C) (D) (E)

hi

He che H e ring oss Me ory oss Seizures Tooth in

70. You h ve been tre ting 33-ye r-o w o n w ith bi o r isor er for 4 ye rs. In re r tion for nne regn ncy, you h ve gree to t er her e ic tions but see her frequ ent y in c se of nic e iso e. Six w eeks into her regn ncy, she resents to you r office w ith ressu re s eech n the be ief th t her b by is Jesu s Christ. H er hu sb n s ys he is concerne bec u se she h s sto e t king her ren t vit ins n is cting biz rre y. Which of the fo ow ing tre t ents w ou be the s fest to tre t her isor er? (A) (B) (C) (D) (E)

C rb ze ine Di ze Div roex so iu E ectroconvu sive ther Lithiu c rbon te

y (ECT)

71. You re the consu t- i ison sychi trist on c for the tr u su rgery te . You re ske to ev u te 39-ye r o n w ho w s hit by c r n severe y inju re . H e is un b e to give history, bu t n cqu int nce w ho cco nie the tient to the hos it re orts th t he is i y, he vy ru g u ser. On ex in tion, the tient is note to h ve tr ck rks on his r s. Which of the fo ow ing c sses of ru gs o you te the surgeons is the ost ngerou s if bru t y w ith r w n? (A) (B) (C) (D) (E)

C nn binoi s H u cinogens O i tes Se tive-hy notics Sti u nts

Questions 72 and 73 A 44-ye r-o coho u se

n w ith history of chronic, severe isor er is brou ght to the e ergency

Que s tions : 67–81

roo . You get re ort fro f i y e ber th t he “h s been iving off booze n ru gs n nothing e se for w eeks.” The e ergency e rt ent nu rse is bou t to go in to r w b oo fro the tient n offer hi foo . 72. Which of the fo ow ing is the ost ro ri te next ste in the tre t ent of this tient? (A) (B) (C) (D) (E)

A A Obt Obt Obt

inister or ze . inister thi ine. in co ete b oo cou nt. in iver fu nction tests. in vit in B12 eve .

Ac ros te C oni ine Disu fir Lor ze N trexone

Questions 74 through 81

(A) (B) (C) (D) (E) (F) (G) (H ) (I)

76. An 11-ye r-o gir being tre te for jor e ressive isor er re orts frequ ent su ici i e tion.

78. The WBC count of 27-ye r-o n being tre te for the tre t ent refr ctory schizohreni ro s to 3,000/ L.

80. A 36-ye r-o wo n being tre te for bi o r isor er fin s herse f urin ting very frequent y. 81. Sever onths fter st rting on new ntisychotic, 32-ye r-o tient’s tot cho estero incre ses fro 200 to 250 g/ L. Questions 82 and 83

ost ike y

C rb ze ine C oz ine Div roex so iu H o eri o Lithiu P roxetine Per hen zine Pi ozi e Zi r si one

74. A 14-ye r-o boy being tre te ogic y for Tou rette isor er is

75. A 51-ye r-o w o n being tre te for schizohreni eve o s i betes.

79. A 42-ye r-o n sto s his nti e ress nt. Tw o ys ter, he ex eriences fee ing of ho e essness, h s ifficu ty s ee ing, n fee s qu ite nxious.

D IRECTION S (Questions 74 through 83): The follow ing group of numbered items are preceded by a list of lettered options. For each vignette, select the one lettered option that is most closely associated w ith it. Each lettered option may be used once, multiple times, or not at all.

For e ch vignette, choose the e ic tion to be ssoci te w ith the si e effect.

st rte on cit o r n h s n incre se of 11 sec in his QTc v u e on ECG.

77. A 34-ye r-o w o n being tre te for bi or I isor er eve o s cu te in in her b oen n is i gnose w ith ncre titis.

73. After he is itte to the e icine service, w hich of the fo ow ing e ic tions w ou be the ost ro ri te to initi te? (A) (B) (C) (D) (E)

95

h r coition y

For e ch vignette, choose the ro ri te to rescribe. (A) (B) (C) (D) (E) (F) (G) (H ) (I) (J) (K)

A r zo Cit o r Div roex so iu F u oxetine H o eri o Lithiu C oz ine P roxetine Ris eri one Sertr ine Tr zo one

e ic tion

ost

96

3: S oma tic Tre a tme nt a nd P syc hop ha rma c ology

82. A 34-ye r-o n w ith schizo ffective isorer h s been nonco i nt w ith e ic tions in the st. You w nt to be b e to give hi e ot injection of n nti sychotic.

(C) C o i r ine (D) F u oxetine (E) F u vox ine Questions 86 and 87

83. You re tre ting 45-ye r-o veter n w ith history of coho use isor er for osttr u tic stress isor er (PTSD), n he sks you for so ething to he hi s ee . D IRECTION S (Questions 84 through 105): For each of the multiple-choice questions in this section, select the lettered answ er that is the one best response in each case. Questions 84 and 85 A 28-ye r-o sing e w o n resents w ith 8 w eeks of e ression, inso ni , ow energy n f tigu e, istr ctibi ity, n oor etite. She h s no rior sychi tric history n no f i y sychi tric history. After further history n bor tory ev u tions re co ete , she is begu n on sertr ine, w hich is eventu y incre se to 100 g/ . After 2 onths on th t ose, the tient re orts th t she fee s “b ck to nor ” n sks w hen she c n sto the e ic tion. 84. H ow ong shou you reco continu e the e ic tion? (A) (B) (C) (D) (E)

en

her to

1 onth 2 onths 4 onths 10 onths 2 ye rs

85. She returns for fo ow-u visit 1 onth ter. Her sy to s h ve continue to re in in re ission, though she escribes recent conf ict with her boyfrien ue to h ving “ itt e sex rive.” The tient st tes th t it h s gotten to the oint where she h s eci e to sto the e ic tion un ess so ething c n be one. She is wi ing to switch to nother e ic tion es ite the risk of re se. Which of the fo owing gents wou be the ost ro ri te choice? (A) Bu ro ion (B) Cit o r

You re tre ting 28-ye r-o n w ith 6-ye r history of schizo hreni w ho ives w ith his rents. The tient h s h nu ber of ebi it ting si e effects, su ch s severe extr yr i sy to s, fro tr ition nti sychotics inc u ing h o erio . H is f i y is w orrie bou t w eight g in, w hich the tient ex erience on o nz ine. Des ite the nu erous e ic tions, his sychotic sy to s continu e. You iscu ss c oz ine w ith the tient n his f i y n they gree to try the e ic tion. You h ve initi te tre t ent s ow y n he h s show n signific nt i rove ent; how ever, he h s been ro u cing rge ou nt of s iv n roo so king his shirts. 86. Which of the fo ow ing is the next ste ? (A) (B) (C) (D) (E)

ost

ro ri te

A nt ine. A ro r no o . A su b ingu tro ine. Discontinu e c oz ine. Incre se the ose of c oz ine.

87. In ition to the bove, the tient co ins of it tions. An ECG reve s sinu s t chyc r i , w hich is consistent w ith rior ECGs erfor e over the st onth. Which of the fo ow ing is the ost ro ri te next ste ? (A) (B) (C) (D) (E)

A benztro ine. A bet o . A or ze . A ro r no o . Discontinu e c oz ine.

88. A 21-ye r-o sing e fe e co ege stu ent is brou ght to the u niversity he th service by her roo tes. She kes it c e r to you th t she oes not think there’s nything w rong w ith her, bu t her roo tes te you th t they “c n’t e w ith her ny ore” bec u se she never s ee s n “never shu ts u .” The tient insists this is bec u se she h s iscovere n

Que s tions : 82–93

i ort nt the tic roof th t she u st finish w riting. She’s convince th t the university w i i e i te y gr nt her tenure s soon s they see the roof, thou gh she h s troub e st ying on to ic w hen you sk qu estions bout it. She h s not seen sychi trist before, n u rine toxico ogy screen, w hich she consents to “to rove I’ not cr zy,” is neg tive. Which of the fo ow ing is the ost ro ri te gent for initi n inten nce tre t ent of this tient? (A) (B) (C) (D) (E)

H o eri o L otrigine Lithiu Lor ze O nz ine

89. A 34-ye r-o e ectric engineer co ins to his octor th t he h s been fee ing e resse for sever onths fter the bre ku of his rri ge. Since th t ti e, he h s h signific nt inso ni , ifficu ty concentr ting, h s ost 25 b in the st 3 onths bec u se his etite isn’t the s e, fee s f tigu e , h s signific nt fee ings of gu i t reg r ing the se r tion, fee s ho e ess n he ess, n h s fe t su ici w ithou t s ecific n. A i gnosis of jor e ressive isor er is e n he is begu n on n nti e ress nt. Which of the fo ow ing sy to s w ou be ex ecte to t ke the ost ti e to i rove? (A) (B) (C) (D) (E)

Decre se etite Decre se concentr tion Decre se energy Inso ni Su ici ity

90. A 58-ye r-o n w ith history of cirrhosis, he titis C, n coho is w ishes to qu it rinking bu t w nts to etox first. Which of the fo ow ing e ic tions w ou be the ost ro ri te to rescribe in this tient? (A) (B) (C) (D) (E)

A r zo Ch or i ze oxi e C on ze Di ze Lor ze

97

91. The tient is 59-ye r-o fe e w ith schizohreni , s w e s oor y contro e i betes e itu s, obesity, hy er i i e i , n hy ertension. She h s been tre te for over 35 ye rs w ith v riou s first-gener tion nti sychotics, n thou gh she is on n equ te ose, she sti h s resi u sychotic sy to s n h s now begu n to eve o invo u nt ry b inking n tongu e-ro ing ove ents. She is greeb e to sw itching to n ty ic / secon -gener tion nti sychotic. Which of the fo ow ing e ic tions w ou be the ost ro ri te for this tient? (A) (B) (C) (D) (E)

Ari i r zo e C oz ine O nz ine Qu eti ine Ris eri one

92. You ev u te 34-ye r-o ivorce , e, w ith history of coho use isor er w ho resents w ith the chief co int of “I c n’t sto rinking, oc.” He h s h sever reh bi it tion issions but is un b e to int in sobriety. He is current y rinking one-fifth of vo k ost i y. Whi e he is very iscour ge n gui ty bout his coho use, he is ho efu bout getting better, n he enies erv sive e ressive sy to s or suici i e tion. H e h s rior history of signific nt coho w ith r w , inc u ing one seizure n e iriu tre ens, both requiring n ICU ission. Which of the fo ow ing e ic tions w ou be ost rori te to rescribe for this tient? (A) (B) (C) (D) (E)

Disu fir Meth one N trexone Sertr ine V roic ci

93. A 35-ye r-o sing e n resents to you r office co ining of inso ni . H e re orts th t he is b e to f s ee t night, bu t st ys s ee for on y 2 or 3 hou rs, then w kes u frequent y throu ghout the night. H e is exh uste n w ou ike so ething to he hi s ee . Other th n fee ing fru str te n f tigue u ring the y, he enies other e ressive

98

3: S oma tic Tre a tme nt a nd P syc hop ha rma c ology

sy to s or ny st sychi tric history. Which of the fo ow ing e ic tions w ou be the ost ro ri te for this tient? (A) (B) (C) (D) (E)

A itri ty ine R e teon Tr zo one Z e on Zo i e

94. A 40-ye r-o rrie , regn nt w o n w ith history of hy ertension, yoc r i inf rction, egener tive isc ise se, n jor e ressive isor er w ith sychotic fe tu res is being consi ere for e ectroconvu sive ther y (ECT) fter f i ing u ti e nti eress nt n nti sychotic tri s. Which of the fo ow ing con itions w ou be re tive contr in ic tion to ECT? (A) (B) (C) (D) (E)

Degener tive isc ise se H y ertension Pregn ncy Psychotic e ression Recent yoc r i inf rction

95. You re c e to consu t on 70-ye r-o w o n, hos it ize in the ICU fter yoc r i inf rction. The tient is ye ing t st ff, u ing ou t her IV n trying to get out of be . She refuses to t ke ny e ic tion by outh. H er ECG show s QTc of 480. Which of the fo ow ing e ic tions w ou be best to he her git tion? (A) (B) (C) (D) (E)

Ari i r zo e IM Di ze IV H o eri o IV Lor ze IV Ris eri one Const IM

96. A 30-ye r-o w o n w ith no reviou s sychi tric history resents to you r office sking for he in n ging erfor nce nxiety. She oves to ct n sing in her co unity the tre, but she beco es terrifie w hen getting on st ge in front of crow s. As resu t, her voice sh kes n she h s been un b e to get “goo rts.” Which of the fo ow ing

e ic tions w ou rescribe? (A) (B) (C) (D) (E)

be the

ost

ro ri te to

Bu s irone F u oxetine F u vox ine Lor ze Pro r no o

97. A 30-ye r-o e resents w ith 4 w eeks of ow energy, oor s ee , oor etite, ecre se interest in e su r b e ctivities, n ow oo . H e re orts being hos it ize s you ng u t for 2-w eek erio of gr n iosity, hy ersexu ity, ecre se nee for s ee , n eng ging in risky beh viors. H e w s st rte on ithiu bu t i not to er te the e ic tion u e to eve o ent of i betes insi i u s n h s not t ken e ic tion since. H e w orks s n ccou nt nt, his BMI is 35, n his st A1c w s 5.2. Which of the fo ow ing e ic tions w ou be the ost ro ri te to rescribe for this tient? (A) (B) (C) (D) (E)

Lithiu Lu r si one Qu eti ine Sertr ine V roic ci

98. A 29-ye r-o w o n w ith history of seizu res is i gnose w ith bi o r isor er, ost recent e iso e e resse , n st rte on otrigine. Abou t w hich of the fo ow ing si e effects w ou be the ost i ort nt to e u c te the tient? (A) (B) (C) (D) (E)

Acu te ystoni Ak thisi A stic ne i R sh Ren f i u re

99. A 19-ye r-o n w ith bi o r isor er is brought in to the e ergency roo by the o ice. H e is gr n iose n e usion , s w e s git te n co b tive. H e is given n intr uscu r injection of h o eri o . After 30 inutes, the tient is fou n rching forw r . On hysic ex in tion, stiffness of the

Que s tions : 94–104

neck n b ck usc es re note . Which of the fo ow ing ter s best escribes this si e effect? (A) (B) (C) (D) (E)

L ryngos s Ocu ogyric crisis O isthotonos P eu rothotonos Tortico is

100. The tient is 59-ye r-o w o n w ith st e ic history of hy ercho estero e i , i betes, n i betic neu ro thy, w ho resents to her ri ry c re octor w ith the chief co int of “he ches.” U on further interview, she st tes her sy to s beg n fter the e th of her hu sb n 5 onths go. Since th t ti e, she h s fe t “e ty,” w ith oor s ee , f tigu e, ecre se concentr tion, n w ishes th t she “w ou ie so I c n join hi .” H er review of syste s is re rk b e for ongoing “sh r ins n nee es” in her extre ities. H er hysic ex in tion is u nre rk b e, n her b oo su g rs h ve been re tive y w e contro e . Which of the fo ow ing w ou be the ost ro ri te tre t ent for her sy to s? (A) (B) (C) (D) (E)

Bu ro ion Du oxetine F u oxetine Mirt z ine Re ssu r nce

101. A 44-ye r-o rrie e tient w ith history of recurrent jor e ressive isor er resents to sychi trist fter sto ing his sertr ine 1 onth go. A though it h s been effective in the tre t ent of his e ression, he h s h incre sing ifficu ty int ining n erection, n this h s c use friction n conf icts w ith his w ife. H e is concerne , how ever, th t he w i h ve re se of his e ressive sy to s if not e ic te . Which of the fo ow ing w ou be the ost ro ri te h r co ogic tre t ent for this in ivi u ? (A) (B) (C) (D) (E)

Begin irt z ine. Begin roxetine. Begin ven f xine. N o e ic tion. Rest rt sertr ine.

99

102. A 58-ye r-o n w ith tri fibri tion, hy ercho estero e i , n hy ertension is sent to you by his ri ry c re hysici n for he in n ging his nxiety. H e re orts th t for ny ye rs he h s w orrie “ bout everything,” n th t he occ sion y h s to st y ho e fro w ork bec use he beco es so fe rfu . H e often h s ifficu ty f ing s ee n not u nco on y gets tension he ches w hen rticu r y stresse . H is e ic tions inc u e w rf rin, torv st tin, n hy roch orothi zi e. H e oes not rink coho or c ffeine. Which of the fo ow ing e ic tions w ou be the ost ro ri te to tre t this tient’s sy to s? (A) (B) (C) (D) (E)

Bu ro ion Bu s irone Cit o r C on ze Ven f xine

103. A 40-ye r-o n is st rte on f u oxetine for jor e ressive isor er. H e notes i roveent in his oo , bu t h s ifficu ty s ee ing. Tr zo one is su bsequ ent y e to his regien. Three ys ter, he c s co ining of infu erection th t h s not su bsi e fter sever hou rs. Which of the fo ow ing best escribes this tient’s con ition? (A) (B) (C) (D) (E)

Erecti e ysfu nction Ny ho ni P r r xis Pri is S tyri sis

104. A 35-ye r-o rrie fe e is being fo ow e by sychi trist for her first e iso e of jor e ression. She is rescribe cit or 40 g i y n h s been in re ission for 3 onths. She now co ins of the new onset of ecre se ibi o w hich is c u sing her o er te ou nt of istress. Which of the fo ow ing w ou be the ost ro ri te to to her cu rrent regi en? (A) Bu ro ion (B) Desi r ine (C) Lithiu

100

3: S oma tic Tre a tme nt a nd P syc hop ha rma c ology

(D) Tr ny cy ro ine (E) Ven f xine 105. A 41-ye r-o w o n resents to you r office sking for he in n ging her binge e ting. She e ts rge ou nts of ju nk foo three or fou r ti es w eek, hi es foo , n su bsequ ent y fee s gu i ty bou t her ctions. H er BMI is 31. After erfor ing thorou gh history n hysic ex in tion, you eci e to rescribe to ir te. Which of the fo ow ing si e effects o you te her is ost ike y? (A) (B) (C) (D) (E)

I rove ttention P ncre titis Po yu ri Ren stones Weight g in

D IRECTION S (Questions 106 through 109): The follow ing group of questions are preceded by a list of lettered options. For each question, select the one lettered option that is most closely associated w ith it. Each lettered option may be used once, multiple times, or not at all. Questions 106 through 109 For the fo ow ing vignettes, choose the correct ter fro the ist be ow . (A) (B) (C) (D) (E) (F) (G) (H ) (I) (J)

A otiv tion C nn bis For ic tion H u cinogen isor er Inh nts Intoxic tion Psycho ogic Su bst nce u se To er nce With r w

syn ro e

ersisting erce tion

e en ence isor er

106. A 45-ye r-o n w ith chronic coc ine u se isor er resents to the e ergency e rtent w ith co ints of bugs cr w ing on his skin.

107. A 16-ye r-o boy is brou ght to you by his other. She st tes th t, es ite h ving high IQ, he oesn’t co ete his ho ew ork, h s been f i ing in schoo , n h s sto e going out w ith frien s. H e its to s oking tw o to three joints of riju n i y. 108. A 21-ye r-o n is brou ght into the e ergency e rt ent by the o ice fter getting rreste for ste ing g so ine. The tient fee s isconnecte fro re ity n frightene . On hysic ex in tion, he is note to h ve r sh rou n his ou th. 109. A 54-ye r-o n w ith o i te u se isor er re se on heroin fter sever onths of sobriety. Whi e he beg n u sing $10 to $20 er y t first, he qu ick y incre se his u se to $50 to $60 er y bec u se “I on’t get high now u n ess I u se ore.” D IRECTION S (Question 110): Select the lettered answ er that is the one best response in this case. 110. A 27-ye r-o n w ith schizo hreni notices the eve o ent of bre sts, ecre se ibi o, n not ej cu ting w hen he h s n org s . Which of the fo ow ing e ic tions is ost ike y res onsib e for these si e effects? (A) (B) (C) (D) (E)

Ari i r zo e C oz ine Qu eti ine Ris eri one Zi r si one

D IRECTION S (Questions 111 through 121): The follow ing group of questions are preceded by a list of lettered options. For each question, select the one lettered option that is most closely associated w ith it. Each lettered option may be used once, multiple times, or not at all. Questions 111 through 121 For the fo ow ing vignettes, choose the correct si e effect fro the ist be ow . (A) Agr nu ocytosis (B) Ak thisi

Que s tions : 105–122

(C) (D) (E) (F) (G) (H ) (I) (J) (K) (L) (M) (N ) (O) (P)

B e h ros s Consti tion G ctorrhe Gyneco sti N eu ro e tic ign nt syn ro e Obstru ctive j u n ice O isthotonos Orthost tic hy otension Pig ente retino thy Pseu o rkinsonis R bbit syn ro e Retrogr e ej cu tion T r ive yskinesi Tortico is

111. A 35-ye r-o n w ith chronic schizo hreni t king thiori zine notices th t, if he urin tes fter sturb ting, his u rine e rs c ou y. 112. A 46-ye r-o C uc si n w o n w ith schizo ffective isor er, bi o r ty e h s been nge for ny ye rs on ris eri one 3 g tw ice i y or y. She enies ny co ints but on ent st tus ex in tion is note to h ve re etitive, r i ove ents of her ou th (bu t not tongue) w ith s cking of her i s. 113. A 22-ye r-o Afric n-A eric n n on c oz ine for 3 onths re orts sore thro t n fever. A CBC is obt ine w ith WBC cou nt of 3 w ith 1% gr nu ocytes. 1,000/ 114. A 43-ye r-o n w ith schizo hreni is on h o eri o . H e is note to h ve f exe osture, festin ting g it, resting tre or, n br ykinesi . 115. The s e 43-ye r-o n w s continu e on h o eri o es ite his si e effects n fter sever ye rs eve o e choreo thetoi ove ents of the trunk n i bs, ong w ith i s cking n tongu e ove ents. 116. A 21-ye r-o n w ith sychosis w s st rte on h o eri o t high oses. Eight ys into his tre t ent, he eve o e fever,

gener ize rigi ity, i ent st tu s.

horesis, n

101

tere

117. A 36-ye r-o n w ith schizo ffective isor er, e resse ty e h s been receiving f u hen zine ec no te 50 g intr u scu r y every 2 w eeks. Whi e his sy to s re w e contro e , he h s recent y notice w hite isch rge fro his ni es, bi ter y. 118. A 26-ye r-o n being tre te w ith highose thiori zine is fou n on o hth o ogic ex in tion to h ve e ery ig ent tion of the retin . Des ite iscontinu tion of the e ic tion, the ig ent tion continu es n the tient eventu y suffers tot b in ness. 119. A 22-ye r-o n co ins of severe restessness n git tion fter st rting on ri i r zo e. H e co ins th t he c nnot sit sti n ces const nt y. 120. A 26-ye r-o tient st rte on ch or ro co ins of f u - ike obt ine n is nor the tient eve o s skin n sc er .

w ith schizo hreni is zine. After 1 w eek, he sy to s. A CBC is . After nother w eek, ye ow ish co or to his

121. A 45-ye r-o tient w ith schizo hreni is st rte on high ose of ch or ro zine. The next y, he co ins of being ight-he e every ti e he st n s. On ex in tion, he is note to h ve signific nt ecre se in b oo ressu re n n incre se in he rt r te. D IRECTION S (Questions 122 and 123): For each of the multiple-choice questions in this section, select the lettered answ er that is the one best response in each case. 122. You re c e in the i e of the night by the su rgic service to see 52-ye r-o n w ho w s itte to the hos it for n e ergency en ecto y. Review of the ch rt ocu ents nu erou s st hos it iz tions for coho intoxic tion n w ith r w . The su rgic te is concerne bout his w ith r w ing

102

3: S oma tic Tre a tme nt a nd P syc hop ha rma c ology

w hi e in recovery. H ow ong fter this tient’s st rink w ou you ost ike y ex ect to see serious w ith r w sy to s eve o ? (A) (B) (C) (D) (E)

6 hou rs 12 hou rs 1 y 3 ys 7 ys

123. A 33-ye r-o n w ith history of bi o r I isor er is itte to the in tient u nit w ith cu te ni . Bec u se he h s f i e reviou s

tri s of ithiu , v roic ci , n o nz ine, he is st rte on oxc rb ze ine. Three ys ter, he co ins of n u se n w e kness; he e rs confu se bou t w hy he is in the hos it . Which of the fo ow ing bor tory stu ies w ou be the ost i gnostic in this c se? (A) (B) (C) (D) (E)

BUN n cre tinine CBC E ectro ytes Liver fu nction tests Oxc rb ze ine eve

Ans we rs a nd Expla na tions

1. (C) Phene zine is ono ine oxi se inhibitor, use (r re y now but frequent y in the st) for the tre t ent of severe e ression. Phene zine, tr ny cy ro ine, n isoc rbox zi re irreversib e b ockers of MAO-A ( ono ine oxi se-A) n MAO-B ctivity; se egi ine, given or y or in the for of skin tch, is n MAO-B inhibitor th t w s eve o e for the tre t ent of P rkinson ise se; t higher oses, it is nonse ective MAO inhibitor n is use in the tre t ent of e ression. These e ic tions re extre e y ngerous in overose, n , fter brief sy to tic erio of 12 to 24 hours, y ro uce hy er yrexi n utono ic excit bi ity sufficient enough to c use rh b o yo ysis. Su ortive c re shou be institute . If e iriu eve o s, s oses of IV benzo i ze ines shou be use . Lor zeis referre bec use of its short e i in tion h f- ife. N euro e tics, es eci y short- cting gents such s ch or ro zine, shou be voi e bec use of their ten ency to contribute to hy otension. If ventricu r rrhyth i s eve o , they c n be tre te w ith i oc ine. Cy rohe t ine is 5-HT2 nt gonist th t is so eti es use in the tre t ent of the ost severe c ses of serotonin syn ro e. Me eriine shou be voi e bec use it y contribute to the renergic crisis. Phenytoin y be use if seizures eve o . Other e ic tions th t re contr in ic te w ith MAOIs inc u e sti u nts, econgest nts, ine recursors such s l - o n l -try to h n, n the ntihy ertensives ethy o , gu nethi ine, n reser ine. 2. (E) B ocking reu t ke of c techo ines n in o ines in tients re y u sing n

MAOI c n resu t in otenti y ife-thre tening ru g inter ction know n s serotonin synro e. Me ic tions th t b ock su ch reu t ke inc u e the SSRIs, TCAs, bu s irone, n other nti e ress nts su ch s ven f xine. Fe tu res of i serotonin syn ro e inc u e tri of ent st tus ch nges, utono ic hy erctivity, n neu ro u scu r bnor ities. A ition sy to s inc u e t chyc r i , f ushing, fever, hy ertension, ocu r osci tions, n yoc onic jerks. Severe serotonin syn ro e y resu t in seriou s hy erther i , co , u tono ic inst bi ity, convu sions, n e th; therefore, one u st w it t e st 14 ys fter iscontinu ing n MAOI before st rting serotonergic gent. Acu te ystoni (inc u ing o isthotonos) occurs w ith the u se of nti sychotic e ic tions, not MAOIs. Ak thisi , or the inner fee ing of rest essness, is nother extr yr i si e effect of neu ro e tic rugs. N eu ro e tic ign nt syn ro e is n i iosyncr tic re ction to neu ro e tic ru gs rese b ing ign nt hy erther i . 3. (A) This tient is ike y su ffering fro jor e ressive e iso e. Bec u se of her w eight g in, hy erso ni , n oo re ctivity, her e ression h s ty ic fe tu res. A thou gh ty ic fe tu res h ve tr ition y been thou ght to res on best to MAOIs su ch s hene zine, these e ic tions re consi ere thir - ine gents bec u se of the otenti risk of serotonin syn ro e n hy ertensive crisis w hen e ting cert in tyr ine-cont ining foo s. SSRIs such s f uoxetine n roxetine y be s effic cious s MAOIs in tre ting ty ic e ression; how ever, roxetine shou be voi e given this tient’s hy erso ni 103

104

3: S oma tic Tre a tme nt a nd P syc hop ha rma c ology

n w eight g in. Bec u se of their f vor b e s fety rofi e, SSRIs h ve beco e first- ine tre t ent in jor e ression. Tricyc ic ntie ress nts su ch s nortri ty ine re ess effective th n MAOIs n SSRIs for ty ic e ression. Tr zo one is n extre e y se ting nti e ress nt th t is gener y u se for ju nctive tre t ent of inso ni . 4. (C) Most tients w ho re going to h ve res onse to n nti e ress nt o so w ithin the first 6 w eeks. H ow ever, it c n t ke u to 12 w eeks to chieve fu res onse. Few er th n 50% of eo e w ith jor e ressive isor er chieve fu re ission fro their first nti eress nt tri . 5. (E) A coho w ith r w shou be strong y consi ere in this tient. Benzo i ze ines re the ru g of choice for contro of coho w ith r w sy to s s w e s for ro hyxis g inst w ith r w seizu res n the otenti y ife-thre tening e iriu tre ens. Long- cting benzo i ze ines su ch s ch ori ze oxi e n i ze re ro ri te; how ever, both re extensive y et bo ize by the iver. Bec u se this tient h s e ev te tr ns in ses, ox ze or or ze , w hich on y u n ergo g u cu roni tion ( n re therefore not e en ent on iver fu nctioning) rior to e i in tion, re refer b e. The ne onic “LOT” (Lor ze , Ox ze , Te ze ) c n be he fu in re e bering w hich benzo i ze ines re et bo ize in this w y. A high- otency nti sychotic gent su ch s h o eri o y be necess ry to he contro this tient’s git tion, how ever, n nti sychotic w i not revent coho w ith r w , n ow otency nti sychotics, su ch s ch or ro zine, shou be voi e bec u se they y ctu y ow er the seizure thresho . Disu fir is u se to tre t coho u se isor er, not cu te coho w ith r w . 6. (E) This tient h s e iriu tre ens, w hich is e ic e ergency. H e shou be tr nsferre to n intensive c re setting w here IV benzo i ze ines c n be inistere w ith gre ter s fety. The IV rou te is referre in this situ tion to ensu re equ te n r i

bsor tion. H o eri o c n be u se for its se tive effect bu t it shou not t ke the ce of benzo i ze ine. Si i r y, IV bet o y be u sefu in the intensive c re unit bu t it w i not n ge the u n er ying w ith r w n shou not re ce benzo i ze ine. 7. (B) This tient is su ffering fro jor e ression w ith sychotic fe tu res. Best r ctice reco en s co bin tion of n nti e ress nt n n nti sychotic e ic tion, such s f uoxetine n ris eri one. E ectroconvu sive ther y, not iste ong the nsw er choices, is nother effective tre t ent for sychotic e ression. N either nti e ress nt onother y nor nti sychotic onother y is s effective s the co bin tion. C oz ine, secon -gener tion nti sychotic, is reserve for the tre t ent of tients w ith schizo hreni w ho h ve f i e tw o or ore nti sychotic tri s. C on ze , benzo i ze ine, y rovi e so e re ief fro nxiety, bu t w i not tre t the u n er ying con ition. 8. (D ) This tient is ex eriencing sy to s of ithiu toxicity, the severity of w hich is eterine by checking ithiu eve . A ther eu tic ithiu eve is 0.8 to 1.2. At eve s rou n 1.2, tients ex erience tre or, n u se , i rrhe , n t xi ; this is fo ow e by seizu res t eve s 1.5 to 2.0, then cu te ren f i u re (requ iring i ysis) t eve s gre ter th n 2.0, w ith su bsequ ent co n e th t eve s bove 2.5. Bec u se ithiu h s su ch n rrow ther eutic in ex, ehy r tion c n e to toxicity. Whi e c rb ze ine n e kote re both u se to tre t bi o r isor er, they h ve u ch w i er ther eutic in exes. A he CT is not in ic te in this c se, given th t the ost ike y c u se of the tient’s sy to s is ithiu toxicity. Si i r y, there is nothing in the vignette to su ggest subst nce intoxic tion s c u se of her sy to s. 9. (D ) Ak thisi , ove ent isor er th t is frequ ent si e effect of nti sychotic e ic tion, c n be ifficu t to istingu ish fro nxiety in sychotic in ivi u ; how ever, new -onset rest essness in tient recent y st rte on nti sychotic e ic tion suggests

Answe rs : 4–15

this i gnosis. A thou gh ore co on w ith first-gener tion nti sychotics, it c n be seen w ith secon -gener tion nti sychotics, s w e , t r tes s high s 20%. Onset u su y begins betw een 5 n 60 ys fter initi tion of tre t ent bu t c n occu r fter just one ose. First, the nti sychotic (neu ro e tic) ose shou be re u ce s u ch s ossib e. If sy to s of k thisi continu e, in the bsence of other extr yr i si e effects, ny c inici ns w ou then st rt tre t ent w ith bet b ocker su ch s ro r no o . In gener , nticho inergic gents su ch s benztro ine re u se to tre t extr yr i sy to s su ch s ystoni or rkinsonis . Di henhy r ine is he fu in cu te ystoni s bu t not in k thisi . Lor ze , benzo i ze ine, is n ro ri te secon - ine tre t ent for k thisi if sy to s re sti not n ge or the tient c nnot to er te bet b ocker. Given the tient’s i rove ent of his sychotic sy to s, it is refer b e to continu e tre t ent w ith ri i r zo e us the bove junctive e ic tions r ther th n sw itching to nother nti sychotic. 10. (C) This tient is su ffering fro bi o r II e ression, w ith history ch r cterize by current jor e ressive e iso e n history of hy o ni . The tre t ent of bi o r e ression is controversi . Most tre t ent gu i e ines reco en ithiu , otrigine, u r si one, or qu eti ine s first- ine o tions. V roic ci y be re son b e secon - ine choice. Monother y with nti e ress nts (e.g., bu ro ion, f uoxetine, ven f xine) is not reco en e ue to the risk of in ucing ni , n the use of nti e ress nts in conjunction with oo st bi izers re ins eb t b e. 11. (E) This tient is ex eriencing neu ro e tic ign nt syn ro e (N MS), r re bu t seriou s si e effect of nti sychotic e ic tions. Rh b o yo ysis is the ost co on ngerou s co ic tion of N MS, occurring in u to 25% of tients in one series. Di ysis y be requ ire to rotect tients fro ren f i u re. The w hite b oo ce count n cre tinine hos hokin se re often e ev te in N MS bu t there re no s ecific bor tory fin ings. A

105

of the other iste co ic tions h ve been re orte in N MS bu t w ith ess frequ ency. 12. (D ) Retin ig ent tion is know n to occu r w hen thiori zine is u se in high oses (>1,000 g/ ). It y not re it w hen thiori zine is iscontinu e n c n eventu y e to b in ness. The other choices c n occu r w ith nti sychotics (neu ro e tics) n re not s ecific to thiori zine. 13. (C) This tient is suffering fro nic isorer n gor hobi . There re ny e ic tions th t h ve e onstr te effic cy in nic isor er, inc u ing TCAs, SSRIs, MAOIs, n high- otency benzo i ze ines. Of the gents iste , on y f u oxetine, n SSRI, is one of the . Bu ro ion, n nti e ress nt w ith o ine ctivity, n bu s irone, rti 5-H T1 nt gonist w ith nti nxiety ro erties, re not effective in nic isor er. Pro r no o , bet - renergic b ocker, y he to evi te the hysic sy to s of nic su ch s t chyc r i , but it oes not revent the tt cks. Tr zo one, n nti e ress nt w ith ixe serotonergic effects, h s so shown conf icting resu ts. 14. (A) A r zo is the ost otent benzo ize ine iste . Using 1 g of or ze for the equ iv ent ose in i igr s, the otency re tionshi of so e co on benzo i zeines is s fo ow s: r zo 0.5 g, i ze 10 g, ox ze 20 g, te ze 20 g, n ch or i ze oxi e 25 g. 15. (E) Bec u se ithiu is grou IA onov ent ion, the ki ney h n es it u ch s it oes so iu . N inety-five ercent of ithiu is excrete u nch nge throu gh the ki neys. Therefore, e ectro ytes, cre tinine, n BUN re requ ire to check b se ine ki ney function. Thyroi stu ies re so requ ire bec u se ithiu inhibits the synthesis of thyroi horone n its re e se fro the thyroi . A CBC is o tion , thou gh ithiu y c u se benign e ev tion in the WBC. F sting g u cose n i i s shou be checke rior to initi ting tre t ent w ith nti sychotic gents. Liver fu nction tests shou be erfor e rior to

106

3: S oma tic Tre a tme nt a nd P syc hop ha rma c ology

initi ting tre t ent w ith iv roex so iu ; r re c ses of f t he totoxicity h ve been re orte w ith iv roex so iu . In ition, iv roex so iu y so e ev te seru oni eve s in r re c ses. 16. (E) In so e in ivi u s, nonsteroi ntiinf tory gents (NSAIDs) c use n incre se in ithiu eve s. Therefore, these gents shou be use w ith c ution in tients t king ithiu , n voi e if ossib e. As irin n su in c, how ever, o not ffect ithiu eve s n re therefore s fe for use w ith ithiu . Given his bi o r isor er, the tient’s oo st bi izer shou not be iscontinue , n the other choices re not ro ri te in this c se.

20. (C) Few er th n h f of tients chieve fu re ission fro e ression fter tre t ent w ith n SSRI. Best r ctice w ou be to sw itch to nother SSRI (cit o r ) or n SN RI (venf xine); how ever, bec u se of his history of hy ertension, ven f xine, w hich c rries w ith it 5% to 7% r te of hy ertension, is not the best choice. N ortri ty ine, tricyc ic nti eress nt, is otenti y ngerou s choice in tient w ith history of yoc r i inf rction. Au g ent tion w ith ithiu n thyroi hor one re re son b e choices for tient w ith partial res onse, bu t re not s rori te w hen tient h s no res onse to tre t ent.

18. (E) Sti u nts h ve been frequ ent y re orte to ex cerb te the tics ssoci te w ith Tou rette isor er, n so e stu ies h ve w rne th t they shou not be given to chi ren w ith tics or f i y history of Tourette. H ow ever, this is co ic te by the f ct th t so e stu ies h ve i entifie frequ ent co orbi ity of ttentioneficit hy er ctivity isor er (ADHD) n Tourette isor er. The other sy to s re not incre se by or ssoci te w ith sti u nt use.

21. (E) The ost ike y syn ro e is TD, oveent isor er th t y occu r fter ong-ter tre t ent w ith nti sychotic e ic tions (es eci y first gener tion) su ch s h o erio . In tients on first-gener tion nti sychotics, TD occu rs t n ver ge r te of bou t 5% er ye r, w ith bou t 20% eve o ing TD w ithin 5 ye rs, n 40% eve o ing it w ithin 20 ye rs. TD consists of nu ber of bnorn invo unt ry ove ents su ch s i s cking, f ci gri cing, n choreo thetoi ove ents of the i bs n trunk. Anticho inergic toxicity, w hich u su y resents s e iriu , is not resent. In H u ntington ise se, n u toso o in nt genetic isor er, the invo u nt ry ove ents re cco nie by rogressive e enti . Meige syn ro e is n or -f ci ystoni invo ving b inking n chin thru sting, i u rsing or tongu e oveents, n occ sion y shou er ove ent. Sy enh chore , ssoci te w ith rheu tic fever, occu rs in chi ren.

19. (E) This tient is su ffering fro Tou rette isor er. Ph r co ogic tre t ent is strict y sy to tic n not cu r tive. Do ine (D 2) rece tor nt gonis w ith nti sychotic e ic tion such s ris eri one rovi es the gre test sy to re ief in chi ren su ffering fro Tou rette isor er. C oni ine, n h -2gonist, is refer b e in the tre t ent of i Tou rette’s s it oes not h ve the s e ongter si e effects s nti sychotics. The other e ic tions iste re not u se for Tourette’s.

22. (B) Whi e t r ive yskinesi y be rogressive, ost tients st bi ize n so e y ctu y i rove, even if they continue t king the nti sychotic. Whi e nti sychotics c use TD, secon -gener tion nti sychotics re ess ike y th n first-gener tion nti sychotics to c u se TD. H ow ever, c oz ine, secon -/ thir - ine nti sychotic th t requ ires regu r w hite b oo ce cou nt onitoring, is the e st ike y to c u se TD n y, in f ct, i rove sy to s.

17. (C) Difficu ty f ing s ee n ecre se etite re the tw o ost co on si e effects of ethy heni te. Sti u nts h ve been re orte to s ow grow th; how ever, it is be ieve to be ess co on w ith ethy heni te n ose re te . In ition, w hen rug ho i ys re given (e.g., su erti e), grow th reboun is seen. Methy heni te y so c u se n incre se in systo ic b oo ressu re.

Answe rs : 16–27

23. (E) This tient ike y h s gener ize nxiety isor er (GAD). Benzo i ze ines re the ost effective e ic tions for qu ick y re u cing sy to s of GAD. A benzo i ze ines re effective n the choice shou be b se on otency, h f- ife, n si e effects. Bu roion is n nti e ress nt w ith o inergic n nor renergic ro erties, bu t is not u sefu in the tre t ent of GAD. Bu s irone is effective for re u cing sy to s of GAD bu t requ ires sever w eeks for signific nt i rove ent. The tricyc ic gent c o i r ine n SSRIs su ch s f u oxetine h ve nxio ytic ro erties bu t so o not w ork s qu ick y s benzo i ze ines. 24. (B) This tient ike y h s the i gnosis of borer ine erson ity isor er (BPD) with co orbi jor e ressive isor er. Bor er ine erson ity isor er is ri ri y tre te by sychother y, rticu r y i ectic beh vior ther y. However, e ic tion c n rovi e usefu junctive tre t ent. Given her co orbi e ressive sy to s, f uoxetine wou be the ost ro ri te tre t ent. However, recent et - n ysis of h r cother y of BPD showe th t oo st bi izers (e.g., v roic ci , otrigine, to ir te) n secon -gener tion nti sychotics (e.g., ri i r zo e, o nz ine, zi r si one) y be the ost effective e ic tions for this con ition (es eci y ffective regu tion n i u sivity) in the bsence of co orbi e ression. 25. (E) Decisions bou t w hether to continu e ntie ress nts u ring regn ncy re high y in ivi u ize , b se on w eighing risks n benefits for e ch in ivi u w o n. In one stu y of regn nt w o en w ith histories of jor e ression, 68% of those w ho iscontinu e e ic tion re se , versu s 26% of those w ho continu e e ic tion. One of the ost seriou s otenti risks of SSRI ex osure u ring regn ncy is ersistent u on ry hy ertension of the new born (PPH N ), r re con ition (1–2 er 1,000 ive births) in w hich new borns eve o res ir tory f i ure u e to ostn t ersistence of e ev te u on ry v scu r resist nce. Whi e it is not c e r, there is be ieve to be n incre se risk (2–3 ti es)

107

of PPH N in w o en ex ose to SSRIs te in regn ncy. Ebstein no y, for tion of the tricu s i v ve, is seen w ith ithiu ex osu re. Gest tion i betes is not ssocite w ith nti e ress nt u se. Inf nt eve o ent e y c n be seen w ith ex osu re to v roic ci . N eu r tube efects re seen w ith ex osu re to oo st bi izers su ch s c rb ze ine n v roic ci . 26. (C) Un ess there is co orbi sychi tric isor er su ch s e ression or obsessiveco u sive isor er, h r cother y gener y h s i ite ro e in the tre t ent of norexi nervos . Re orts on the usefu ness of cy rohe t ine to sti u te etite h ve been ixe . Gener y, these tients re y h ve goo etite, bu t w ork h r not to give in to their ow n hu nger. F u oxetine, n SSRI, n to ir te, n nticonvu s nt, h ve both show n to be effective in the tre t ent of bu i i nervos , bu t not in the tre t ent of norexi . The secon -gener tion nti sychotic o nz ine c n in u ce w eight g in, but bec u se of this, ost tients w ith norexi w i not gree to t ke it. 27. (A) This tient ike y h s SSRI iscontinu tion syn ro e, w hich y occu r w ithin 1 to 3 ys of bru t y sto ing n SSRI. The ost co on hysic sy to s re izziness, n use n vo iting, f tigu e, eth rgy, n f u - ike sy to s. Psycho ogic sy to s of nxiety, irrit bi ity, n crying s e s re so not u nu su . P roxetine y be ore ike y to c u se this bec u se of its short h fife n ck of n ctive et bo ite. F u oxetine h s ong h f- ife (2–4 ys) s w e s n ctive et bo ite w ith very ong h f- ife (7–15 ys), n therefore, oes not require t ering. The iscontinu tion syn ro e is not know n to be eth ; how ever, rest rting the roxetine n then t ering it is re son b e choice. Acet ino hen is un ike y to signific nt y he . Bu ro ion is n ntie ress nt w ithou t sexu si e effects, n w ou be re son b e choice for the tre tent of e ression, thou gh w ou not he w ith the iscontinu tion sy to s. De kote is tre t ent for bi o r isor er, w hich

108

3: S oma tic Tre a tme nt a nd P syc hop ha rma c ology

this tient oes not h ve. Ven f xine is serotonin-nore ine hrine reu t ke inhibitor w ith si i r w ith r w si e effects. 28. (A) Concu rrent u se of otrigine n or contr ce tives c n ffect eve s of both e ic tions. The c e r nce of otrigine is signific nt y gre ter in w o en ex ose to or contr ce tives; ow er eve s of the e ic tion w ou ccou nt for the tient’s ck of i rove ent. A ition y, otrigine y in u ce the et bo is of or contr ce tives, otenti y ren ering the ess effective. Ibu rofen, ithiu , n zi r si one o not ffect otrigine et bo is . H ow ever, v roic ci decreases the c e r nce of otrigine, necessit ting ow er otrigine oses w ith concurrent u se. 29. (E) Tr zo one is n nti e ress nt e ic tion w hich ffects the serotonin syste by w e k reu t ke inhibition n nt gonist ctivity t 5-H T1 , 5-H T1c, n 5-H T2 rece tors. It h s se tive effect, ike y ro uce by both h - renergic n hist ine b ock e. It is extre e y se ting t nti e ress nt oses (300–500 g), bu t is often u se t s er oses (50–200 g) s n ju nct to other ntie ress nts to he w ith s ee . Both i ze ( benzo i ze ine) n i henhy r ine ( n ntihist ine) re u se s se tives, bu t neither is n nti e ress nt, n both c n c u se e iriu in the e er y. Both f uoxetine n bu ro ion re ore ctiv ting nti e ress nts n y initi y w orsen inso ni . 30. (E) Pri is , infu , ro onge erection, is r re si e effect of tr zo one. The nuf ctu rer h s re orte the inci ence of ny bnorerecti e fu nction to be bout 1 in 6,000 en. This si e effect u su y occu rs w ithin the first onth of tre t ent but c n occu r t ny ti e. Any bnor erecti e fu nction shou ro ti e i te iscontinu tion of the e ic tion. The other sy to s re not co on or ngerous enou gh to w rr nt infor ing the tient. 31. (E) Lithiu c uses ECG ch nges in bou t 20% to 30% of tients, ost co on y T-w ve

e ression or inversion, bu t the ch nges re u su y not c inic y signific nt. First- egree AV b ock n sinu s no e ysfu nction re r re. Lithiu toxicity c n r re y c u se sino tri b ock, AV b ock, AV issoci tion, br y rrhyth i s, ventricu r t chyc r i , n ventricu r fibri tion, bu t these re usu y u e to u n er ying c r i c ise se. 32. (C) Thi zi e iu retics su ch s hy roch orothi zi e c n incre se ithiu eve s ow ing to ecre se ithiu c e r nce. Other e ic tions th t c n incre se ithiu eve s re the iu retics eth crynic ci , s irono ctone, n tri terene; N SAIDs (exce t s irin n su in c); n the ntibiotics etroni zo e n tetr cyc ine. Nitrog ycerin n ro r no o o not interfere w ith ithiu c e r nce. 33. (A) C o i r ine, ixe serotonin n nore ine hrine reu t ke inhibitor (bu t c ssifie s tricyc ic nti e ress nt) w s rove for the tre t ent of obsessive-co u sive isor er (OCD) in the Unite St tes in 1989. Tris co ring SSRIs to c o i r ine re ort equ effectiveness; how ever, the si e effect rofi e f vors the SSRIs, w hich re the ost co on first- ine tre t ent in c inic r ctice. Other tricyc ic gents (su ch s nortri ty ine) re not s effective s c o i r ine bec u se they re not s serotonergic. C on zey be he fu for the nxiety ssoci te w ith OCD, bu t it is not effective in tre ting the u n er ying isor er (i.e., obsessions or co u sions). O nz ine is secon -gener tion nti sychotic w hich y be u se for u g ent tion in severe c ses of OCD, bu t it is not reco en e s onother y. Phene zine is ono ine oxi se inhibitor (MAOI), use very r re y for e ression, bu t not u se in OCD. 34. (C) Just ike in the tre t ent of refr ctory e ression, tre t ent of refr ctory OCD is rge y b se on c se re orts n c inic references. There re very few contro e b in e tri s in refr ctory OCD. Pr ctice gu i e ines reco en u g enting SSRIs w ith secon -gener tion nti sychotic (su ch s ris eri one) for tients w ith o er te

Answe rs : 28–39

res onse to the SSRI. For tients w ith no res onse, one shou consi er sw itching to ifferent SSRI or to c o i r ine. In c ses, sychother y—in rticu r, ex osure thery w ith res onse revention—shou be reco en e . Bus irone n ithiu y h ve so e u ti ity s u g enting gents, bu t their evi ence b se is ess th n for nti sychotics. Anti sychotics re not gener y u se s onother y in OCD. Cingu oto y, surgic roce u re, shou be reserve for severe, refr ctory c ses. 35. (B) This tient is su ffering fro co n u itory h u cin tions, w hich h ve not su fficient y res on e to equ te tri s of one first-gener tion nti sychotic e ic tion (h o eri o ) or tw o secon -gener tion nti sychotics (qu eti ine n o nz ine). Therefore, tri of c oz ine is w rr nte . C oz ine w s the first secon -gener tion nti sychotic n h s been fou n to be su erior to other nti sychotics in re u cing both ositive n neg tive sy to s. It y so i rove cognitive eficits in tients w ith schizo hreni . It is not u se s first- ine tre tent bec u se of 1% risk of gr nu ocytosis. Ari i r zo e, ris eri one, n zi r si one re secon -gener tion nti sychotics th t re not s effective s c oz ine for tre t entrefr ctory schizo hreni . Per hen zine is first-gener tion nti sychotic ike h o eri o . 36. (C) C oz ine cts t ny rece tors, inc u ing D 1, D 2, n D 4; hist ine-1; u sc rinic; h 1- renergic; n serotonin ty es 5-H T2, 5-H T2c, n 5-H T3. Efforts to i entify the ex ct ech nis of the nti sychotic ction of c oz ine h ve reve e t e st tw o ossibi ities. Un ike first-gener tion nti sychotics, c oz ine h s uch ore otent nt gonis t the D 4 rece tor co re to the D 2 rece tor, es eci y in the i bic syste . This h s e to s ecu tion th t the D 4 rece tor y e i te sychotic sy to s. C oz ine so h s ctivity t the 5-H T2 rece tor, ctivity th t first-gener tion nti sychotics ck. 37. (C) Tricyc ic nti e ress nts, inc u ing i i r ine n nortri ty ine, b ock v rious rece tors,

109

inc u ing usc rinic, h -1- n h -2renergic, o inergic, n hist inergic, to v rying egrees. As c ss, their b ock e of h -1-rece tors is gener y be ieve to be res onsib e for ny orthost tic hy otension th t y occur, es eci y in the e er y. I i r ine is terti ry ine which b ocks u ti e rece tors n therefore h s ny si e effects; nortri ty ine, secon ry ine, h s re tive y ess h -1-b ocking otency n ten s to be better to er te in gener . Bu ro ion, f uoxetine, n irt z ine re not ssoci te with orthost tic hy otension. 38. (E) This tient is resenting w ith bi o r isor er, current y nic. First- ine tre t ent for ni inc u es v roic ci or ithiu , w ith or w ithou t concu rrent nti sychotic gent. Anti e ress nts su ch s cit o r shou not be u se in the setting of cu te ni , s they y w orsen the con ition. L otrigine is n nticonvu s nt, u se for bi o r e ression n inten nce; it is not s n effective nti nic gent. O nz ine is re son b e tern tive to v roic ci , bu t ue to the risk of w eight g in, y not be the o ti choice for this tient w ith n e ting isorer. To ir te is n nticonvu s nt use in the tre t ent of bu i i nervos ; how ever it is not consi ere effective for the tre t ent of bi o r isor er. 39. (D) A of the co on y use oo st bi izing e ic tions c rry so e risk of fet for tions or otenti e eterious effect on ter cognitive eve o ent. Use of ithiu uring the first tri ester incre ses the risk of Ebstein no y, for tion of the tricusi v ve, fro 1:20,000 to 1:1,000 tients. If fetus h s been ex ose to ithiu , fet echoc r iogr shou be one between weeks 16 n 18 of regn ncy. Lithiu y be use in the secon n thir tri esters with c refu ttention to equ te hy r tion. C rb zeine c n c use cr niof ci efects (11%), fingern i hy o si (26%), neur tube efects, n eve o ent e y (20%). Div roex so iu is ssoci te with neur tube efect r tes of roxi te y 2%, n y c use intr uterine growth ret r tion. Benzo i ze ines such s

110

3: S oma tic Tre a tme nt a nd P sychop ha rma c ology

i ze y be ssoci te with c eft i n te, though this re ins controversi . 40. (E) It is not u nu su for e iriou s tients to beco e hosti e or co b tive, osing risk to the se ves or to hos it st ff. Low - ose ty ic n ti sych otics, su ch s ris eri one, re very effective in re u cin g git tion in e iriou s tients. Di henhy r ine shou be voi e s the nticho inergic effects y w orsen the e iriu n confu sion. Done ezi is n n ticho inester se inhibitor u se for jor neu rocognitive isor ers ( e enti s). A thou gh benzo i ze ines ike or ze c n be u se for git tion in e iriu , th ey shou be voi e in e er y tien ts u e to the risk of w orsening e iriu n oten ti isinhibition. Orient tion to su rrou n ings is often ition y he fu in e iriu , bu t it w i not i e i te y c the tient. 41. (C) Mirt z ine is n nti e ress nt w hose h r co ogic rofi e is ifferent fro other v i b e gents. It is centr h -2- renergic nt gonist n n nt gonist of 5-H T2 n 5-H T3 rece tors, s w e s H 1 rece tors. B ock e of h -2-rece tor e s to enh nce serotonin re e se; how ever b ock e of 5-H T2 n 5-H T3 e s to re tive enh nce ent of 5-H T1 ctivity giving irt z ine ifferent si e effect rofi e th n SSRIs. For ex e, irt z ine ten s to incre se etite n c use w eight g in co re to cebo. In ition, its H 1 nt gonis c u ses se tion. H ow ever, t higher oses, h -2- renergic b ock e so e s to incre se nore ine hrine re e se, w hich y cou nter ct the H 1- e i te se tion. In this tient, u se of irt z ine (to ex oit its otenti incre se etite n se tion si e effects) shou be consi ere . Bu ro ion is n nti e ress nt w hose si e effect rofi e inc u es inso ni n w eight oss, neither of w hich w ou be benefici in this tient. F u oxetine n sertr ine re SSRI nti e ress nts w hich, w hi e effective, y initi y c u se ctiv tion. N ortri ty ine is tricyc ic nti e ress nt th t is eth in over ose n w hose nticho inergic ro erties c n e to e iriu in the e er y.

42. (C) Tricyc ic nti e ress nts (TCAs) ro u ce sever c r iov scu r si e effects, the ost signific nt being qu ini ine- ike effect s ow ing c r i c con uction. So e c inici ns voi TCAs if there re ny ECG ch nges. Cert in y, if there re ch nges in con uction, such s ro onge QT interv , w i ening of the QRS co ex, or AV con uction bnor ities, TCAs shou be voi e . In over ose, they c n w i en the QRS co ex, c use bun e br nch b ock, n c use t chy rrhyth i s; how ever, even t ther eu tic concentr tions, they y h ve verse effects on c r i c con uction. Estrogen re ce ent ther y n o i ine re not contr in ic tions. TCAs c n be u se in tients w ith rior cerebr inf rctions. Whi e TCAs y ex cerb te urin ry retention fro their nticho inergic effects, this is not contr in ic tion to their use. 43. (B) This tient is ike y e iriou s n ro t i entific tion n tre t ent of the u n er ying c use is in ic te . To he contro the git tion th t y cco ny e iriu , ow - ose h o eri o is frequ ent y u se . H o eri o oes not tre t the e iriu , how ever. It is u se ost frequent y bec u se it is the ost otent of the ty ic nti sychotics, therefore requ iring ow er oses, w ith few er nticho inergic or orthost tic si e effects. It so h s the v nt ge of being v i b e in or , IM, or IV for . Low - otency gents (su ch s ch or ro zine or thiori zine) re not on y ssoci te w ith orthost tic hy otension n nticho inergic si e effects, bu t so w ith ro ong tion of the QT interv . Lor ze y he se te the tient bu t it w i not he his sychosis n cou c use isinhibition n w orsening of his e iriu . Ris eri one is not v i b e in renter for , n is therefore not s usefu in n git te tient w ho requ ires IV or IM e ic tion. 44. (A) Of the first-gener tion nti sychotics iste (h o eri o , er hen zine, n thiori zine), h o eri o is the ost otent n h s the e st ctivity t h -1 rece tors. Therefore, it is the e st ike y to c u se orthost tic hy otension. The secon -gener tion gents iste (ris eri one n queti ine)

Answe rs : 40–50

h ve ctivity t h -1 rece tors n re both ssoci te w ith orthost tic hy otension. 45. (B) The et bo is of ethy coho invo ves tw o-ste enzy tic rocess. The first enzy e, coho ehy rogen se, et bo izes eth no to cet ehy e, w hich is qu ick y et bo ize by ehy e ehy rogen se. Disu fir inhibits ehy e ehy rogen se, resu ting in n ccu u tion of cet ehy e. Acet ehy e c u ses f ci f u shing, t chyc r i , hy otension, n u se n vo iting, n hysic isco fort. Therefore, tient on isu fir h s n incentive to re in bstinent. Obviou s y, isu fir w orks on y if tients continu e to t ke it. Ac ros te, GABA-ergic gonist, n n trexone, n o ioi nt gonist, re both u se to re u ce cr vings for coho . F u zeni is benzoi ze ine nt gonist u se in benzo i ze ine over ose. N oxone is n o ioi nt gonist u se in e ergency n ge ent of o i te intoxic tion n over ose. 46. (E) Mu ti e neu rotr ns itter syste s h ve been investig te in tte ts to contro coho cr vings. One syste th t ike y ys ro e in the rew r thw y is the o ioi syste . N trexone is n o ioi nt gonist th t h s been show n to ecre se the nu ber of ys erson w ith coho use isor er rinks n to incre se the ti e before re se of he vy rinking. Tre t ent of e ression w ith n gent su ch s bu ro ion y he contro rinking if co orbi e ression exists, bu t u se of nti e ress nts in the bsence of oo isor er h s not been effective in re u cing cr vings. Disu fir is n coho -sensitizing gent w hich eters tients fro rinking. F u zeni is benzo i ze ine nt gonist u se in benzo i ze ine over ose. N oxone is n o i te nt gonist th t is u se to cu te y reverse the effects of o i te intoxic tion. 47. (C) This tient is ike y su ffering fro osttr u tic stress isor er (PTSD). First- ine h r co ogic tre t ent for PTSD is the se ective serotonin reu t ke inhibitors (e.g., f uoxetine, roxetine, sertr ine), w hich sever r n o ize contro e tri s h ve show n

111

re u sefu in re u cing the core sy to s of PTSD (reex eriencing, voi nce, nu bing, n hy er rou s ). Benzo i ze ines such s r zo re not benefici in tre ting these sy to s. Anticonvu s nts re not rou tine y u se ; o en- be stu ies of iv roex, c rb ze ine, n to ir te h ve show e ixe resu ts. N trexone, n o ioi nt gonist, h s not been extensive y stu ie in PTSD. Anti sychotics su ch s thiori zine h ve gener y not been u sefu , though the secon -gener tion nti sychotics y be u sefu s ju nctive e ic tions, es eci y if the tient h s co orbi sychotic sy to s. 48. (E) Pr zosin is n h -1- renergic rece tor b ocker, tr ition y use to tre t hy ertension n benign rost tic hy ertro hy, w hich h s e onstr te effic cy in tre ting night res n other sy to s in tients w ith PTSD. Se tive-hy notic gents, such s or ze , re not rticu r y he fu in tre ting PTSD n h ve rger otenti for hysio ogic n sycho ogic e en ence. N trexone h s not been extensive y stu ie in PTSD. O nz ine, secon -gener tion nti sychotic, n er hen zine, first-gener tion nti sychotic, y be usefu in severe PTSD w ith sychotic sy to s, but they c rry risk of w eight g in n et bo ic si e effects, s w e s t r ive yskinesi over the ong ter . 49. (E) This tient is ike y ex eriencing e iriu , s evi ence by his cu te ch nge in ent st tu s, w ith confu sion, isorient tion, n git tion. E er y tients c n be rticu r y sensitive to the verse effects of benzo i ze ines. This tient is t king 8 g/ of or ze , w hich is rge ose. S ow y t ering the or ze , w hi e investig ting other otenti c u ses of e iriu , is re sonb e first ste . H y roch orothi zi e eve s re not e sure , n the other choices w i not ress or he to n ge his e iriu or the ike y u n er ying c u se. 50. (B) This tient is ike y su ffering fro jor neu rocognitive isor er ( e enti ), ue to A zhei er ise se. H e h s c e r cognitive ec ine, n , thou gh reversib e c uses of

112

3: S oma tic Tre a tme nt a nd P syc hop ha rma c ology

cognitive i ir ent shou be investig te , there is no in ic tion th t ny reversib e c use is resent. There re five e ic tions cu rrent y v i b e to tre t A zhei er ise se: the nticho inester se inhibitors one ezi , g nt ine, riv stig ine n t crine, n the N - ethy -d - s rt te (N MDA) rece tor nt gonist, e ntine. Of these e ic tions, t crine is r re y u se u e to the risk of he tic f i ure. A r zo is short- cting benzo ize ine w hich y w orsen confu sion. There oes not e r to be n in ic tion for f u oxetine in this tient (su ch s e ressive isor er). Tr zo one is serotonergic nti eress nt often u se to tre t inso ni ; it w i not i rove cognitive bi ity. Methy henite is sti u nt th t oes not e r to be in ic te in this c se. 51. (C) Tr zo one is serotonergic gent frequent y use in tients to he with s ee , n in this tient y he w ith his w n ering beh vior. It c n c u se orthost tic hy otension n shou therefore be u se w ith c u tion in those w ith risk f ctors for f s. H o n qu eti ine re nti sychotic e ic tions th t shou both be u se c refu y in the e er y. H o eri o is high- otency firstgener tion nti sychotic th t is so eti es u se to n ge severe git tion, bu t it y c u se extr yr i sy to s. Qu eti ine is secon -gener tion nti sychotic; how ever, given the b ck box w rning bou t the risk of su en e th in e er y tients w ith e enti -re te sychosis, nti sychotics shou be u se s ring y in the geri tric o u tion. Benzo i ze ines su ch s or ze n tri zo shou so be u se w ith c u tion in the e er y. They y ctu y w orsen the beh vior n c u se e iriu . If necess ry, st rting oses shou be u ch ow er th n st n r osing n s ow y t ere u . 52. (E) P tients w ho resent w ith severe y tere eve s of consciou sness nee to be e ic y ev u te n qu ick y tre te for sever reversib e c u ses. These inc u e hy og ycei , o ioi over ose, n coho intoxic tion. Airw y rotection n onitoring of ir exch nge n c r iov scu r st tu s re

requ ire . Sever tre t ents th t shou be i e i te y consi ere inc u e IV extrose, u su y D 50, to tre t hy og yce i ; thi ine to gu r g inst the eve o ent of Wernicke ence h o thy in n coho ic tient w ith thi ine eficiency; n n oxone, n o ioi nt gonist, to reverse the effects of o ioi intoxic tion. F u zeni is benzo ize ine nt gonist th t shou not be u se before obt ining fu rther history bec u se it y cu te y ow er the seizu re thresho . 53. (A) Bu renor hine is one of the ost effective e ic tions for o ioi etoxific tion. Given in sub ingu for , it is ixe o ioi gonist– nt gonist w hich su resses o ioi w ithr w n b ocks the effects of other o ioi s. Benzo i ze ines such s ch or i ze oxi e y be he fu s juncts to contro nxiety, but they re not sufficient for o i te w ith r w . C oni ine, centr h -2- gonist, su resses the sy thetic res onse to the heroin w ith r w n he s to contro git tion n utono ic inst bi ity, such s e ev te b oo ressure n he rt r te; how ever, it w i not ress the heroin cr ving n is ess effective in n ging w ith r w th n bu renor hine. N oxone wou not be ro ri te beyon the initi resuscit tion efforts. Pro r no o is not use for heroin etoxific tion. 54. (A) Bu ro ion ( rkete s Zyb n) is n ntie ress nt th t h s so been show n to be effective s rt of s oking cess tion rogr . The ech nis of ction is u nc e r, bu t it is be ieve to h ve n effect on o inergic tr ns ission. Giving u cig rettes y be extre e y ifficu t for tient w ho is so suffering fro jor e ression; how ever, st rting bu ro ion u ring the e ression y he her qu it s oking in the fu tu re. The other nti e ress nts iste h ve no i ct on s oking cess tion. 55. (E) Du oxetine is serotonin-nore ine hrine reu t ke inhibitor (SN RI) th t is use in the tre t ent of both jor e ression n in, inc u ing neuro thic in n fibro y gi . A r zo is benzo i ze ine not rori te for tre ting either jor e ressive

Answe rs : 51–61

isor er or in. Whi e bu ro ion is usefu nti e ress nt, it so w i not he w ith the tre t ent of in. Sever tricyc ic ntie ress nts, inc u ing itri ty ine n nortri ty ine, re so u se to tre t chronic in. H ow ever, tricyc ic nti e ress nts re ore eth in over ose th n new er e ic tions ike se ective serotonin reu t ke inhibitors (SSRIs) n SN RIs, n they gener y h ve ore si e effects, rticu r y nticho inergic n c r iov scu r ones. SSRIs su ch s cit o r y y ro e in chronic in n ge ent, bu t the t re not s robust s w ith u oxetine. 56. (B) C rb ze ine is n nticonvu s nt th t is u se to tre t both bi o r ni n trige in neu r gi . A itri ty ine n f u oxetine re both nti e ress nts, w hich re not ro ri te in this tient w ho is in the i e of nic e iso e. G b entin is n nticonvu s nt th t h s e onstr te effic cy in the tre t ent of trige in neu r gi ; how ever, it is not consi ere effective s n ntinic gent. Lithiu shou not be rest rte ue to concerns bou t its rior effect on her thyroi fu nctioning, n it so oes not tre t trige in neur gi . 57. (E) As n nti sychotic w ith oo -st bi izing ro erties, o nz ine is usefu for both sychotic isor ers, ike schizo hreni , s w e s bi o r isor er. Weight g in is si e effect of ost nti sychotic e ic tions, both first n secon gener tion; how ever, it is rticu r y concerning w ith o nz ine. In one stu y of nti sychotic u se in eo e w ith schizo hreni , over 45% of tients on o nz ine g ine ore th n 7% of their bo y w eight; o nz ine ten s to in uce ore w eight g in th n other nti sychotic gents. Agr nu ocytosis is r re bu t otenti y f t si e effect of c oz ine. O nz ine is not ssoci te w ith the eve o ent of c t r cts or incre se excit bi ity. O nz ine is ike y to c use se tion n is u su y inistere in the evening. E ev te ro ctin n otenti g ctorrhe re si e effects of first-gener tion nti sychotics n ris eri one, but ess co on w ith o nz ine. 58. (E) Given th t the tient to er tes n res on s to or ris eri one bu t su ffers ex c-

113

erb tions u e to oor co i nce, he is goo c n i te for ong- cting e ic tion. Ris er Const (Ris eri one) is ong- cting for of ris eri one given in bi onth y injections. Continu ing his cu rrent regi en or sw itching to nother or e ic tion su ch s o nz ine w i ike y on y e to continu tion of the viciou s cyc e of oor co i nce n re se. C oz ine is ro ri te for tients w ho h ve f i e tw o or ore nti sychotic e ic tions, w hich is not the c se in this tient, n the regu r b oo onitoring so requ ires co i nt tient. Whi e h o eri o is so v i b e in e ot for , it is not s goo choice for this tient, given his rior res onse to ris eri one. 59. (D ) Ris eri one is n ty ic or secon -gener tion nti sychotic gent th t h s otent 5-H T2 nt gonist ro erties s w e s b ocking t D 2 n h -1-rece tors. Bec u se of the h -1-b ocking ctivity, it y c use orthost tic hy otension. Agr nu ocytosis is concern w ith c oz ine, not ris eri one. A benign eu kocytosis c n occu r w ith ithiu u se, bu t not w ith ris eri one. Ris eri one y c use w eight g in r ther th n w eight oss, n it oes not c u se reci b e nticho inergic effects. 60. (C) Secon -gener tion nti sychotic gents, ike c oz ine n ris eri one, re consi ere ty ic bec use of their ecre se ro ensity for c using extr yr i si e effects (EPS) n resu b y re uce (but not zero) risk of t rive yskinesi . Unfortun te y, on y c oz ine e rs to tru y obey this “ru e.” The other secon -gener tion nti sychotic gents e r to h ve so e egree of EPS, with ris eri one s the e ing cu rit; t oses higher th n 6 g/ , ris eri one c uses EPS t r tes co r b e to h o eri o . Agr nu ocytosis is concern with c oz ine, not ris eri one. A benign eukocytosis c n occur with ithiu use, but not with ris eri one. Ris eri one oes not c use reci b e nticho inergic effects, n usu y c uses weight g in r ther th n weight oss. 61. (E) Anti sychotic rece tors in sever B ock e of o

gents b ock o ine thw ys in the br in. ine rece tors in the

114

3: S oma tic Tre a tme nt a nd P sychop ha rma c ology

tu beroinfu n ibu r tr ct resu ts in incre se ro ctin re e se ( o ine cts s n inhibitor of ro ctin re e se), w hich y c use inferti ity, enorrhe , g ctorrhe , n gyneco sti . Ris eri one c u ses ore roctin incre se th n other nti sychotic e ic tions—in f ct, it y r ise ro ctin eve s u to 100-fo . Do ine nt gonis in the esocortic n eso i bic tr cts is res onsib e (in rt) for the nti sychotic effect. B ocking o ine rece tors in the nigrostri t syste c uses the extr yr i sy to s. The ocu s ceru eu s, oc te in the br in ste , is the ost i ort nt nor renergic nuc eu s in the br in. 62. (B) This tient is su ffering fro bu i i nervos . First- ine h r co ogic tre t ent is n SSRI su ch s f u oxetine, w hich ecre ses sy to s of bingeing n u rging in e enent of ny oo isor er. Bu ro ion is contr in ic te in tients w ith e ting isor ers ue to n incre se risk of seizure. L otrigine is n nticonvu s nt u se in the tre tent of bi o r e ression n inten nce. Lithiu is so u se to tre t bi o r isor er. Mirt z ine is n nti e ress nt w hich frequ ent y c u ses w eight g in, n is therefore oor choice for tient w ith bo y-i ge issu es. 63. (D ) To ir te is n nticonvu s nt u se in the tre t ent of bu i i n binge-e ting isor er. In sever contro e tri s, to ir te ecre se the frequency of binge e iso es n e to w eight oss. Lithiu n v roic ci re both use to tre t bi o r isor er n not ro ri te for bu i i . Bu ro ion is n ntie ress nt but is contr in ic te in tients w ith e ting isor ers u e to n incre se risk of seizure. P roxetine is n SSRI th t c n be u se in the tre t ent of bu i i ; how ever, this tient h s re y f i e tw o SSRI tri s, n is u n ike y to benefit fro thir ; in ition, it ten s to incre se w eight g in w hich w ou ke it oor choice in this tient. 64. (C) Me tonin, hor one invo ve in circ i n rhyth , h s been show n to be effective in the tre t ent of jet g n circ i n

rhyth s ee –w ke isor ers, e ye s ee h se ty e. Me tonin is gener y benign intervention w ithou t risks of bu se, e enence, or res ir tory su ression. A itri ty ine n qu eti ine re se ting but h ve seriou s si e effects n shou not be u se for ri ry inso ni . Di henhy r ine is u se for inso ni bu t c n e ve tients fee ing row sy the next y. Short- cting benzo i ze ines su ch s te ze c n be u sefu in the tre t ent of inso ni ; how ever, these e ic tions c rry w ith the risk of e en ence n rebou n inso ni . 65. (E) This tient ike y h s nic isor er. Sertr ine is se ective serotonin reu t ke inhibitor (SSRI) rove for the tre t ent of nic isor er. A thou gh other e ic tions re effective, SSRIs re the s fest in over ose n h ve few er si e effects. Bu ro ion, n nti e ress nt w ith o inergic ech nis , is not effective in the tre t ent of nic isor er. I i r ine is tricyc ic nti e ress nt n hene zine is ono ine oxi se inhibitor; w hi e both re effective in tre ting nic isor er, SSRIs re f r better to er te n s fer. Qu eti ine is n nti sychotic e ic tion th t w ou not be ro ri te for this tient. 66. (A) The tricyc ic nti e ress nts, such s itri ty ine, h ve been show n to incre se ort ity in c r i c tients bec u se of their qu ini ine- ike effects n ten ency to both incre se he rt r te n ecre se b oo ressure. Bu ro ion is n nti e ress nt th t is s fe in c r i c tients. As c ss, hetines, su ch s ethy heni te, h ve been fou n to be s fe in tients w ith c r i c ise se, thou gh re not ro ri te for tre ting jor e ression in this tient. As there y be inor ch nges in b oo ressu re n u se, tients shou be onitore c ose y. Sertr ine is so s fe for tients w ith c r i c ise se n ossesses few ru g– rug interctions. Ven f xine c n incre se the b oo ressu re n ver ge of 8 to 10 H g bu t this effect is ose re te . At oses ess th n 150 g/ , c inic y signific nt ch nges re re tive y r re.

Answe rs : 62–71

115

67. (D ) The sy to s the tient is ex eriencing re co on effects of e ev te ro ctin eve s. H o eri o ’s ech nis of ction is o ine rece tor b ock e (D 2). Do ine nor y inhibits ro ctin secretion in the tuberoinfun ibu r thw y, but bec use h o eri o b ocks o ine, ro ctin secretion is u no ose . H o eri o oes not signific nt y incre se ACTH , o ine, nore ine hrine, or serotonin.

(ECT), w hich requires gener nesthesi , rTMS c n be erfor e in the office, w ithou t ny nesthesi . The ost co on si e effect is he che t the site of the ic tion. The tre t ent is very ou n tients re ske to w e r e r u gs; how ever, he ring oss h s not been re orte . A so, u n ike w ith ECT, there is no e ory oss ssoci te w ith the roce u re. Seizu res n tooth in re r re co ic tions.

68. (B) This tient ost ike y h s k thisi , w hich is n nti sychotic(neu ro e tic)-in uce si e effect. P tients gener y ex erience su bjective fee ings of rest essness n c n be seen sw inging their egs, rocking b ck n forth w hi e sitting, cing, n r i y tern ting betw een sitting n st n ing. Acute ystoni s re ch rcterize by infu contr ction of usc es resu ting in bnor ove ents or ostu res, such s s s s of the j w , bnor ositioning of the he , or ifficu ty sw ow ing. The onset of cu te ystoni u su y eve o s w ithin the first w eek of initi ting or incre sing neuro e tic e ic tion. Ak thisi c n so eti es be ist ken for nxiety or ni , but in this c se there re no other signs (such s e usions of gr n eur, ecre se nee for s ee , incre se energy, or hy ersexu ity) to in ic te nic e iso e. N euro e tic ign nt syn ro e is ife-thre tening co ic tion invo ving u sc e rigi ity n ystoni , s w e s u tono ic sy to s su ch s e ev te te er ture, incre se he rt r te, n incre se b oo ressu re. T r ive yskinesi is ongter consequence of nti sychotic use, n it is ch r cterize by invo u nt ry, choreo thetoi ove ents of the bo y, es eci y in the extre ities. It is not infu , thou gh y be so ew h t is b ing, n is often irreversib e.

70. (D ) Decisions bout e ic tions u ring regn ncy re high y in ivi u ize , b se on w eighing risks n benefits for rticu r in ivi u . ECT h s been u se in regn ncy for ore th n 50 ye rs n its s fety n effic cy is w e ocu ente . V roic ci n ithiu (in ition to secon -gener tion ntisychotics) re in ic te s first- ine gents for the tre t ent of nic e iso es, w ith c rb ze ine s goo tern tive; how ever, three re ssoci te w ith n incre se risk of congenit bnor ities w hen t ken uring the first tri ester. Benzo i ze ine such s i ze re not first- ine tre t ent for ni , n y be ssoci te w ith n incre se risk of or c efts w ith first-tri ester ex osu re.

69. (A) Tr nscr ni gnetic sti u tion is new tre t ent for e ression, rove in 2008 by the FDA for tre t ent of jor e ression th t h s f i e t e st one nti e ress nt tri . The tre t ent invo ves ying series of e ectro gnetic u ses to the eft orbit refront cortex; fu tre t ent consists of t e st 20 i y sessions, e ch sting bou t 40 inu tes. Un ike e ectroconvu sive ther y

71. (D ) Cess tion or re u ction in se tive, hy notic, or nxio ytic e ic tions th t h ve been u se he vi y or for ro onge erio of ti e y resu t in w ith r w syn ro e ch r cterize by sy to s th t eve o w ithin hou rs to few ys fter cess tion or re u ction. Au tono ic hy er ctivity, orthost tic hy otension, usc e w e kness, tre or, inso ni , n u se , vo iting, u itory/ visu / t cti e h ucin tions, git tion, or nxiety y occur. By f r, the ost serious seque e re gr n seizu res or e iriu . De en ing on the otency, ou nt, frequ ency, n ength of u se, u to 75% of tients c n ex erience gr n seizu res on the secon or thir y of w ith r w , n ny h ve ore th n one seizu re. De iriu y eve o betw een the thir n eighth y of w ith r w . Minor sy to s y ersist for u to 4 w eeks. With r w fro other e ic tions iste is u n e s nt, bu t is not ife thre tening.

116

3: S oma tic Tre a tme nt a nd P sychop ha rma c ology

72. (B) Chronic coho ics often t ke in c ories fro itt e e se besi es coho n re thu s t risk for thi ine eficiency. If thi ineeficient tient is given foo (g u cose), he or she c n eve o Wernicke ence h o thy ( e iriu ) fro the bo y’s tte ts to et bo ize g u cose in the bsence of thi ine yro hos h te. A though the other choices re i ort nt concerns, none re of the s e cu ity s thi ine eficiency. 73. (D ) This n wi rob b y su ffer fro coho w ith r w . Benzo i ze ines n coho h ve si i r o es of ction in their o u tion of GABA rece tors in the br in. This si i rity kes benzo i ze ine su ch s or ze sensib e n co on choice for the tre t ent of coho w ith r w . Ac ros te, isu fir , n n trexone re use for re se revention in tients w ith coho u se isor er, bu t not for n ging coho w ith r w . C oni ine, n h -2- gonist th t o u tes utono ic inst bi ity, is u se to n ge sy to s of o i te w ith r w , n ctu y y b ock ( sk) the u tono ic w rning signs of coho w ith r w . 74–81. [74 (H), 75 (B), 76 (F), 77 (C), 78 (B), 79 (F), 80 (E), 81 (B)] Psychi tric ru gs c n h ve seriou s si e effects, so e of w hich re the subjects of “b ck box” w rnings on rescribing infortion ckets. Pi ozi e, first-gener tion nti sychotic, inter cts w ith nu ber of e ic tions, inc u ing cit o r ; the co bin tion of these tw o e ic tions c n ro ong the QTc. A thou gh c oz ine, h o eri o , er hen zine, i ozi e, n zi r si one re nti sychotics u se to tre t schizo hreni , c oz ine is the ost ike y to c u se hy erg yce i n e to the eve o ent of i betes. After controversi set of he rings on the subject of chi ren n nti e ress nts in the Unite King o n Unite St tes in 2003 n 2004, regu tors eci e there w s enou gh evi ence to show th t so e of the SSRIs, inc u ing roxetine, y incre se the risk of su ici thoughts in chi ren. This w rning is now exten e to you ng u ts, u to ge 24. A thou gh c rb ze ine, iv roex so iu , n ithiu re use to tre t

bi o r isor er, on y v roic ci c rries w rning of n incre se risk of ncre titis. A thou gh c rb ze ine n c oz ine c n both c u se gr nu ocytosis, c oz ine is ore ike y to be u se ( n is FDA rove ) for the tre t ent of refr ctory schizo hreni . P roxetine is the on y nti e ress nt iste n h s very short h f- ife su ch th t it rou ces signific nt w ith r w syn ro e fter ju st 2 ys. Whi e c rb ze ine, iv roex so iu , n ithiu ( s w e s nti sychotics) y be u se to tre t bi o r isor er, on y ithiu c uses both o yu ri ( benign si e effect) n ne hrogenic i betes insi iu s, resu ting in frequ ent u rin tion. Whi e nti sychotics h ve the risk of incre sing choestero , of those iste , c oz ine h s the ost signific nt effect. 82. (I) H o eri o , c oz ine, n ris eri one re u sefu in the tre t ent of schizo hreni , bu t on y h o eri o n ris eri one re v i b e in e ot injection. Ris eri one w ou be refer b e bec u se it is secon gener tion nti sychotic w ith better si e effect rofi e. 83. (K) Tr zo one, serotonergic nti e ress nt, is n effective s ee ing i th t is not h bit for ing. Whi e benzo i ze ines such s r zo y he w ith inso ni , it is w ise to voi rescribing the in tient w ith history of coho use isor er bec u se of their cross-re ctivity w ith coho . 84. (D ) The reco en e ength of tre t ent of first e iso e of u ni o r e ression is t e st 6 onths n u su y on the sc e of 9 to 12 onths, n ossib y onger, e en ing on tient f ctors such s f i y history of oo isor er, severity n u r tion of the e ressive e iso e, n co orbi sychi tric sy to s such s nxiety n subst nce buse. Discontinu tion w ithin the first 16 w eeks of tre t ent is ssoci te w ith high risk of re se. 85. (A) Bu ro ion is the one nti e ress nt iste th t oes not c use sexu ysfu nction, n is so eti es e to SSRIs in or er to he

Answe rs : 72–91

w ith sexu ysfu nction. Cit o r , f u oxetine, n f u vox ine re SSRIs th t h ve been re orte to c u se v rying egrees of sexu ysfunction. C o i r ine is tricyc ic nti e ress nt th t h s gre ter inci ence of sexu ysfunction th n other TCAs, ike y ue to its otent serotonin reu t ke inhibiting ro erties. 86. (C) Si orrhe is co on si e effect of tre t ent w ith c oz ine (u to 30% of tients). It c n be extre e y botherso e to the tien t n oth ers bu t oes n ot u su y requ ire iscontin u tion of tre t en t. Often , beh vior e su res re su fficient, su ch s the u se of ozenges or cin g tow e on the tien t’s i ow t n ight. Anticho in ergic gents su ch s tro ine ro s often he to re u ce the vo u e of s iv . A n t in e y be h e fu in th e tre t ent of rkin soni n sy to s n ro r n o o is u sefu in the tre t ent of k thisi . Incre sing the ose w ou not n ro ri te ro ch in this situ tion . 87. (D ) P tients on c oz ine often eve o ersistent sinus t chyc r i th t oes not requ ire cess tion of tre t ent. Often, the t chyc r i reso ves w ithou t necessit ting fu rther intervention; how ever, shou it ersist, it y be tre te w ith the bet - nt gonist ro r noo . Benztro ine is n nticho inergic gent th t w i not he the t chyc r i , n y, in f ct, w orsen it. C oz ine h s signific nt h -rece tor b ock e, w hich often c u ses orthost tic hy otension, requ iring gr u titr tion of the ose; bec use of this, bet o w ou not be ro ri te s it y ex cerb te this si e effect. Lor ze is benzo i ze ine th t yc the tient, bu t w i not irect y ffect he rt r te. Sto ing the c oz ine w ou not be necess ry or ro ri te in this c se. 88. (C) This tient resents w ith signs n sy to s su ggestive of first nic e iso e. Lithiu n iv roex so iu re in the inst y of initi n inten nce tre t ent of bi o r I isor er. Anti sychotics, such s h o eri o n o nz ine y be u se

117

in conju nction w ith ithiu or iv roex so iu initi y; how ever, u e to their si e effects n risk of t r ive yskinesi , they re not rou tine y u se s inten nce tre t ent. L otrigine is n nticonvu s nt th t is u se for the tre t ent of bi o r e ression n inten nce tre t ent; it is not u sefu u ring n cu te nic e iso e. Lor ze c n be u se initi y for se tion n s ee , bu t it is not effective for tre ting or int ining tients w ith bi o r isor er. 89. (E) The neu roveget tive sy to s (ch nges in etite, oor concentr tion, nergi , n s ee isturb nces) ten to res on first w ith nti e ress nt tre t ent, often rior to signific nt oo ch nges or re u ction in su ici ity. This is w hy it is co on w is o in sychi try th t tients w ho begin to recover fro e ression y be t gre ter risk for suici e th n w hen they re t the height of e ression. As they st rt to function better hysio ogic y (w ith i rove s ee , energy, n otiv tion), they y be ore c b e of c rrying out suici i u se. 90. (E) In c ses of su s ecte iver i ir ent, it is vis b e to u se benzo i ze ine ini y et bo ize by the iver (e.g., or ze n ox ze ). These benzo i ze ines o not go through oxi tion, bu t r ther on y throu gh g u cu roni tion. The other benzo i ze ines iste re et bo ize ost y by the iver n cou qu ick y bu i to toxic eve s in cirrhotic tient. 91. (A) The tient h s ike y eve o e t r ive yskinesi (TD). A of the secon -gener tion nti sychotics h ve ow er r tes of TD co re to the first-gener tion nti sychotics. A thou gh they h ve w rnings reg r ing the incre se risk of i betes n hy er i i e i ( et bo ic syn ro e), they c u se it t ifferent r tes, so this is often f ctor in choosing rticu r nti sychotic e ic tion. The e st ike y to c u se et bo ic syn ro e re ri i r zo e n zi r si one, fo ow e by ris eri one n queti ine. C oz ine n o nz ine h ve the highest r tes of weight g in, hy er i i e i , n insu in resist nce.

118

3: S oma tic Tre a tme nt a nd P sychop ha rma c ology

92. (C) N trexone, u-o ioi rece tor b ocker, exerts its h r co ogic effect through o u ting coho ’s reinforcing effects. It h s been show n to signific nt y re u ce the risk of he vy rinking in tients. Another v nt ge is th t it c n be converte to onth y intr u scu r injection, w hich c n incre se co i nce. Whi e isu fir , n versive gent th t inhibits ehy e ehy rogen se, c n so be use , ore recent stu ies h ve re eg te it to secon - ine e ic tion. N one of the other e ic tions iste re effective for coho use isor er: Meth one is ongcting o ioi gonist u se in o i te u se isorer. N either sertr ine ( n nti e ress nt) nor v roic ci ( oo st bi izer) tre ts coho u se isor er in the bsence of n u n er ying oo isor er.

rob e s reso ve, so e nite y.

y

ersist in efi-

95. (A) This tient h s e iriu , co on co ic tion in the ICU. Low - ose nti sychotics su ch s ri i r zo e c n be very he fu if u se ju icious y. H o eri o is often use in c inic r ctice; how ever, IV h o eri o c n engthen the QTc n therefore shou not be u se for this rticu r tient. Benzo i ze ines su ch s i ze n or ze c n w orsen e iriu in the e er y n shou be voi e . Ris eri one (Ris er Const ) is the ong- cting for of ris eri one, n w ou not be ro ri te in this setting.

93. (D ) A of the e ic tions iste re s ee i s; how ever, on y z e on, nonbenzo i ze ine hy notic w ith h f- ife of on y 4 hou rs, is ro ri te for i e inso ni (f ing s ee bu t frequ ent y w kening u ring the night). A of the tern tives iste re ore u sefu for eo e w ho h ve initi inso ni ( ifficu ty f ing s ee ) n requ ire e ic tion t be ti e. A itri ty ine n tr zo one re both se ting nti e ress nts. R e teon is e tonin-rece tor gonist. Zo i e is nother nonbenzo i ze ine hy notic w ith onger h f- ife th n z e on.

96. (E) The tient ike y suffers fro erfor nce nxiety, for of soci nxiety isor er (soci hobi ). Pro r no o , bet b ocker, is very he fu in n ging erfor nce nxiety. T ken i e i te y before the fe re event, it ecre ses he rt r te, b oo ressu re, n tre or, n consequent y ecre ses nxiety. Bu s irone is n nxio ytic gent th t is he fu in the tre t ent of gener ize nxiety isor er. F uoxetine n f u vox ine re nti e ress nts u se in the tre t ent of gener ize nxiety isor er n nic isorer. Lor ze is benzo i ze ine th t c n be he fu in n ging cu te nxiety, bu t its cco nying se tion y be rob e tic in erfor nce context.

94. (E) ECT h s re tive y few con tr in ic tions n in so e c ses is referre for its r i onset of ction. H ow ever, bec u se of the c r iov scu r effects of ECT, history of recen t yoc r i inf rction (w ithin the st 6 onths) is re tive con tr in ic tion. Another re tive contr in ic tion is the resence of c inic y sign ific nt in tr cr ni s ce-occu ying esion u e to the risk of br in ste herni tion. ECT y be erfor e in tients w ith egener tive isc ise se or hy ertension, n u ring regn ncy. ECT is s fe n effective tre t ent for sychotic e ression. The ost co on co ints tients h ve fo ow ing ECT re i ir en ts in both nterogr e n retrogr e e ory. A th ou gh ost e ory

97. (B) The tient resents w ith history of bi or isor er n is cu rrent y e resse . Tre tent for bi o r e ression is co ic te ; how ever, the cu rrent best r ctice ro ches inc u e u r si one, qu eti ine, n ithiu s onother ies. The tient h s re y f i e ithiu n w ou be better serve by ifferent tre t ent. Lu r si one is w eight neu tr ty ic nti sychotic th t h s been show n to be effective for the tre t ent of bi o r e ression. Queti ine is so n effective tre t ent, how ever it is ssoci te w ith w eight g in n the tient is re y overw eight. Sertr ine n other SSRIs h ve ixe evi ence for bi or e ression, n they shou never be rescribe s onother y s they incre se the r te of fu tu re nic e iso es. V roic ci

Answe rs : 92–109

119

is secon - ine tre t ent for bi o r e ression; it is ore effective for cu te ni th n cute e ression.

his jor e ression. Rest rting sertr ine w ou be ike y to c u se the s e rob e s n e to non herence.

98. (D ) A thou gh there re v riou s si e effects of otrigine, the ost ngerou s is th t of severe r sh, w hich c n eve o into Stevens–Johnson syn ro e, otenti y ifethre tening co ic tion. Beginning t ow ose n s ow y incre sing re u ces the risk signific nt y. Acu te ystoni n k thisi re both si e effects of nti sychotic e ic tions. A stic ne i is r re si e effect of c rb ze ine. Ren f i u re c n occur w ith ithiu .

102. (D ) This tient h s sy to s of gener ize nxiety isor er. A of the e ic tions iste , exce t c on ze , benzo i ze ine, c n inter ct neg tive y w ith w rf rin. Bu ro ion is so ike y to w orsen nxiety. Bu s irone n cit o r re re son b e choices for his nxiety, bu t his PT/ PTT n IN R w ou h ve to be onitore c refu y. Ven f xine y so e ev te b oo ressu re.

99. (C) A of these re ty es of ystonic re ctions th t re in u ce by the u se of nti sychotics (neu ro e tics). O isthotonos, so know n s arc de cercle, is s s of the neck n b ck th t c uses the tient to rch forw r . The ost r ing of these ystoni s is ryngos s , w hich is the s s of the u sc es contro ing the tongue n the thro t; this c n e to res ir tory istress. Ocu ogyric crisis is s s of the extr ocu r u sc es, often resu ting in the tient ooking u n u n b e to ook ow n. P eu rothotonos, so know n s Pis syn ro e, is e ning ostu re in u ce by the s s of the torso u sc es. Tortico is is s s of the neck u sc es th t u su y brings the neck to one si e or nother bu t y so u forw r or b ckw r . 100. (B) A thou gh ny of the iste nti e ress nts w i tre t her e ression equ te y, on y u oxetine, co bine serotoninnore ine hrine reu t ke inhibitor, h s been rove in the tre t ent of both e ression n neu ro thic in. Given her ike y jor e ressive isor er, re ssur nce w ou not be ro ri te or effic ciou s. 101. (A) With the exce tion of irt z ine, bu roion, n nef zo one (now off the rket ue to risk of he tic f i ure), nti e ress nts c n c u se signific nt sexu ysfu nction, inc u ing ecre se ibi o, erecti e ysfu nction, n norg s i . Given the recurrent n tu re of his i ness, e ving hi w ithou t e ic tion w ou ike y e to re se of

103. (D ) Pri is , infu , ro onge erection, is r re bu t seriou s si e effect of tr zo one. Erecti e ysfu nction refers to ifficu ty obt ining or int ining n erection, co on si e effect of SSRIs. N y ho ni is ins ti b e sexu esire in w o n w hi e s tyri sis is its cou nter rt in en. A r r xis, or s i of the tongu e, is so know n s “Freu i n s i .” 104. (A) Sexu ysfunction (e.g., ecre se ibi o, erecti e ysfunction, norg s i ) is not unco on y seen w ith SSRIs n other nti e ress nts (such s those iste ). Bu ro ion is one of the few nti e ress nts w hich c uses itt e to no sexu ysfunction, n , in f ct, y reverse the si e effect w hen e to tre t ent. In ition, ny other sychotro ic e ic tions inc u ing ithiu n nti sychotics c n so c use sexu ysfunction. 105. (D ) To ir te is n nticonvu s nt u se in the tre t ent of bu i i n binge e ting isor er. It c rries w ith tw o to fou r ti es incre se risk of ren stones. To ir te often c u ses cognitive i ir ent, rticu r y w or -fin ing ifficu ty, shortter e ory rob e s, n rob e s w ith ttention. P ncre titis is r re si e effect of v roic ci , n o yu ri is seen w ith ithiu u se. To ir te c u ses w eight oss, not w eight g in. 106–109. [106 (C), 107 (A), 108 (E), 109 (I)] For ic tion is the fee ing of bu gs cr w ing on the skin, u su y ssoci te w ith coho w ith r w ,

120

3: S oma tic Tre a tme nt a nd P syc hop ha rma c ology

bu t so w ith ru g intoxic tion, rticu r y w ith coc ine. A otiv tion syn ro e is ck of rive or otiv tion u su y ssocite w ith chronic ru g u se, es eci y c nn bis. Inh nts, inc u ing g so ine, re vo ti e subst nces th t h ve r i -onset intoxic ting effects. To er nce is the nee for higher oses of ru g in or er to chieve the s e effect. M riju n is c ssifie s c nn bis. H u cinogen ersisting erce tion isor er is so know n s f shb ck. It is ch r cterize by istressing retu rn of erce tu ch nges w ithou t the ingestion of h u cinogens, su ch s LSD. Intoxic tion is re resente by beh vior , cognitive, or erce tu ch nges t ken in re tionshi to ru g ingestion. Psycho ogic e en ence is cr ving or nee to continu e t king rug, frequent in those tients w ith su bst nce u se isor ers. Su bst nce u se isor er enco sses revious i gnoses of buse n e en ence n w ys inc u es continu e u se of su bst nce (e.g., coho or i icit ru gs) in s ite of verse consequ ences re te to the su bst nce (fu nction i irent). With r w is set of signs n sy to s, hysic n sycho ogic , th t re ssoci te w ith coho or ru g cess tion. 110. (D ) Anti sychotic gents, in ition to b ocking o ine rece tors in the nigrostri t thw y (c u sing extr yr i sy to s) n the eso i bic thw y ( ecre sing h u cin tions), so b ock o ine in the tu beroinfu n ibu r syste . Do ine nor y inhibits ro ctin re e se in this thw y, n therefore o ine b ock e incre ses ro ctin eve s, c u sing sexu ysfu nction, gyneco sti in en, n g ctorrhe in w o en. The ost ike y e ic tions to c u se these si e effects re first-gener tion nti sychotics n ris eri one. 111–121. [111 (N ), 112 (M), 113 (A), 114 (L), 115 (O), 116 (G), 117 (E), 118 (K), 119 (B), 120 (H), 121 (J)] Anti sychotics, so know n s neu ro e tics, c n c u se w i e v riety of si e effects. These si e effects c n be broken ow n into so e of the fo ow ing c tegories for n e sier ro ch. The o inergic si e effects inc u e those th t re u e to the b ock e of

the n tu r o ine inhibition of ro ctin re e se fro the nterior itu it ry c u sing g ctorrhe (E) n gyneco sti (F). The si e effects th t rise fro o ine b ocke in the b s g ng i c u se v riou s ystoni s, inc u ing b e h ros s (s s of the eye i s) (C), o isthotonos (s s of the neck n b ck c u sing n rche ostu re) (I), n tortico is (s s of the sternoc ei ostoi ) (P). The b ock e of D 2 rece tors in the nigrostri t thw y c n so c u se other extr yr i syn ro es su ch s k thisi (su bjective fee ing of rest essness) (B), seu o rkinsonis (L) w hich rese b es P rkinson ise se, ch r cterize by u sc e rigi ity n short festin ting g it, r bbit syn ro e (M) ( r re si e effect of ong-ter nti sychotic u se ch r cterize by invo u nt ry, fine, rhyth ic otions of the ou th n i s), n t r ive yskinesi (O), isor er of bnorinvo u nt ry ove ents c use by roonge u se of nti sychotics. An i iosyncr tic bu t r re co ic tion of nti sychotics (neu ro e tics) is neu ro e tic ign nt syn ro e (G). This is ch r cterize by hy erther i , ch nge in ent st tu s, n incre se u sc e tone. It c n e to rise in u sc e bre kow n ro u cts (cre tine kin se eve s re ty ic y e ev te ), ren f i u re fo ow ing ehy r tion, u on ry co ic tions, n e th. C oz ine c n c u se gr nu ocytosis (A), ngerou s si e effect th t so eti es c n resent w ith sore thro t n fever; if this occu rs, the ru g u st be sto e i ei te y. Consti tion (D ) is co ic ting si e effect of so e nti sychotics, rticu r y those w ith nticho inergic effects. This c n often be re ieve w ith u se of x tive. Obstru ctive j u n ice (H) is r re n occu rs in y w ith ch or ro zine. Sy to s inc u e fever, n u se , ise, n ru ritu s. Orthost tic hy otension (J), su en ro in b oo ressu re u on st n ing, is u e to h 1- renergic b ock e. It is es eci y trou b eso e w ith ch or ro zine n c oz ine n best h n e by r ising oses s ow y. Pig ente retino thy (K) is observe w ith high oses of thiori zine n best etecte w ith goo o hth o ogic ex in tion. An ition si e effect of so e nti sychotic

Answe rs : 110–123

121

e ic tions th t y ecre se co i nce is retrogr e ej cu tion (N ); org s y be chieve bu t ej cu tion is bnor , w ith retrogr e ro u sion of the se en, resu ting in c ou y e r nce of the u rine.

zu res re r re y foc n u su y occu r w ithin 48 hou rs fter coho consu tion ce ses. De iriu tre ens ost often occu rs w ithin 72 hou rs of cess tion of rinking, thou gh y e k for sever ys fu rther.

122. (D ) A coho w ith r w c n t ke the for of inor w ith r w sy to s su ch s sw e ting, f u shing, n tre u ou sness, or ore serious consequ ences inc u ing w ith r w seizu res or w ith r w e iriu , so know n s e iriu tre ens. Minor w ith r w sy to s c n st for sever ys. With r w sei-

123. (C) This tient is exhibiting sy to s of hy on tre i , re tive y co on si e effect of oxc rb ze ine, so obt ining seru e ectroytes w ou be critic . The other tests w ou not he in i gnosing hy on tre i , n oxc rb ze ine eve s re not rou tine y checke (un ike eve s of c rb ze ine).

This page intentionally left blank

CHAPTER 4

Ps yc ho lo g ic al Tre atme nt and Manag e me nt Que s tions

DIRECTIONS (Questions 1 through 42): For each of the multiple-choice questions in this section, select the lettered answ er that is the one best response in each case. Questions 1 and 2 A 38-year-old m ale thinks that he is a “failu re and w ill never find a p artner” w henever he feels rejected . H e often find s him self getting d epressed and isolating him self w hen he thinks like this. H is therap ist thinks that it w ould be a good id ea for him to m ake a log of the situ ations, his im m ed iate thou ghts, his im m ed iate feelings, alternate resp onses to those thoughts and feelings, and re-rate his feelings after com pleting the exercise so they can be exam ined d uring their sessions. 1. Which of the follow ing types of therapeu tic ap proaches is the therapist m ost likely u sing? (A) (B) (C) (D) (E)

Cognitive–behavioral Exp ressive H yp notherap y Psychoanalysis Psychod ynam ic

2. Which of the follow ing term s refers to the im m ed iate thou ghts (e.g., “I am a failu re”) that the p atient exp eriences and record s in this type of therapy? (A) Au tom atic thou ghts (B) Core beliefs

(C) Ego strengths (D) Projections (E) Unconsciou s thou ghts 3. A 43-year-old m an is chastised at w ork. When he com es hom e, his friend asks him how his d ay w ent. H e respond s angrily saying that a “real friend w ou ld n’t be so nosy.” Which of the follow ing d efense m echanism s m ost likely characterizes this reaction? (A) (B) (C) (D) (E)

Denial Disp lacem ent H u m or Intellectu alization Isolation of affect

4. A 20-year-old w om an d iagnosed w ith bord erline p ersonality d isord er thinks of her therap ist as “the best person I’ve ever know n.” The next w eek, the therapist announces that he w ill be going on vacation. The patient becomes enraged and states that he is “the cru elest d octor in the w orld !” Which of the follow ing d efense mechanism s best characterizes this reaction? (A) (B) (C) (D) (E)

Devalu ation Id ealization Intellectu alization Repression Sp litting

123

124

4: P syc holog ica l Tre a tme nt a nd Ma nag e me nt

5. A 16-year-old boy has just lost his mother and father in a car accident. In your office, he talks philosophically about death and its implications. When asked how he feels about his parents’ death, he responds by saying that “it is the nature of things to pass away.” Which of the following defense mechanisms best d emonstrates this patient’s inability to talk directly about his emotional experience concerning personal loss? (A) (B) (C) (D) (E)

Denial Intellectu alization Projection Su ppression Und oing

Questions 6 and 7 A 44-year-old w om an w ith schizop hrenia is stru ggling w ith p aranoia, au d itory hallu cinations, and d elu sions. She lives w ith her m other but has a poor relationship w ith her. She tells you that everyone w ants her to sp end a lot of m oney and bu y d ru gs. 6. Which of the follow ing resp onses w ou ld be the m ost usefu l? (A) “Well, d ecid ing for you rself is best.” (B) “If you bu y d ru gs, I’ll call the p olice.” (C) “Perhap s w e shou ld look at w hat you r m other w ou ld think abou t that.” (D) “Why d o you think everyone w ants you to d o that?” (E) “N o, they d on’t.” 7. You d ecid e that su p p ortive therap y w ou ld be helpfu l to this patient. Which of the follow ing w ou ld be the m ost ap p rop riate goal of su p p ortive therap y for this p atient? (A) Correcting fau lty id eas (B) Exp loring the feeling of m eaninglessness in life (C) Investigating the freed om of ind ivid u als (D) Personality change (E) Strengthening of d efenses 8.

A 32-year-old p sychiatry resid ent is in p sychoanalysis 4 d ays p er w eek. H e is encouraged to lie on the cou ch and say “w hatever com es to

m ind .” Which of the follow ing analytic techniques d oes this best rep resent? (A) (B) (C) (D) (E)

Free association H yp nosis Rep ression Thou ght record s Transference

9. A psychotherapist has been m eeting regu larly w ith a 10-year-old boy for 18 months. During one of the therapy sessions, she find s herself feeling very angry at the patient after he reports “beating up” his little brother. Which of the follow ing phenomena is the psychotherapist most likely exhibiting? (A) (B) (C) (D) (E)

Countertransference Extinction Interp retation Resistance Transference

10. In an acu te inp atient p sychiatric u nit, the resid ent p sychiatrist is in charge of lead ing a grou p consisting of p atients new ly ad m itted to the w ard . Du ring group d iscu ssion she asks tw o participants to act out a scenario, d escribe their feelings abou t the situ ation, and then exp lore the ind ivid u al conflicts w hich arose. Which of the follow ing techniques of p sychotherapy d oes this m ost represent? (A) (B) (C) (D) (E)

Feed back Free association Go-arou nd Psychod ram a Resistance

Questions 11 and 12 A 28-year-old fem ale p atient w ho is ad d icted to opioid analgesics states that she has been up set w ith fam ily for not continu ing to su p p ort her financially and allow ing her to stay w ith them . She has been u nem ployed for m any years since leaving college. They recently told her that she shou ld be m ore ind ep end ent; they w ant her to start w orking and find a p lace to live. In her next therap y session, she tells her therapist that she u nd erstand s w hy they w ou ld

Que s tions : 5–16

say that, bu t then goes on to d escribe a fight w ith her boyfriend , stating she w ants to break u p w ith him becau se he is not su p p ortive of her. H er therap ist asks if it is p ossible she is u pset at her parents for w anting her to be m ore ind ep end ent bu t exp ressing this anger tow ard her boyfriend instead . 11. Which of the follow ing therapeu tic techniqu es best d escribes the therap ist’s resp onse? (A) (B) (C) (D) (E)

Cou ntertransference Em p athy Interp retation Parad oxical intervention Working throu gh

12. The p atient’s fam ily m em bers and close friend s w ork w ith the therap ist and join the p atient in a session. Du ring the m eeting, they tell her about her m alad aptive behaviors and how they have affected her and her fam ily negatively. They then give her an u ltim atu m , w hereby if she d oes not get treatm ent for her ad d iction they w ill not continu e to su pp ort her. Which of the follow ing techniqu es best d escribes this scenario? (A) (B) (C) (D) (E)

Behavioral therap y Fam ily therap y Grou p therap y Intervention Relap se p revention

13. A 24-year-old female patient with a history of generalized anxiety disorder presents for therapy. In a previous session she reluctantly discussed how much she did not like her new step-mother and didn’t approve of her father marrying her. This session she reports that her step-mother is in the hospital and she has been spending most of the past few days at the hospital caring for her, buying many get-well gifts and inserting herself in her step-mother ’s care. Which of the following concepts best describes this behavior? (A) (B) (C) (D) (E)

Abreaction Dep end ency Projection Reaction form ation Sp litting

125

14. A 34-year-old male is referred by his job because of ongoing interpersonal conflicts. During the interview, he appears very focused on his health, his attractiveness to others, and his success at work. When confronted with his difficulties, he becomes defensive, blaming others, and accusing them of being “jealous of me.” He describes himself as “better” than his friends and colleagues and admits to taking advantage of others in order to “get what I deserve.” Which of the following treatment modalities would be the most appropriate for this patient? (A) Anger m anagem ent (B) Com bined ind ivid u al and grou p therapy (C) Psychoanalysis (D) Psychopharm acotherap y (E) Social skills training 15. A 29-year-old w om an has been d epressed for 2 months prior to seeking med ical attention. She believes that nobod y likes her even though she is alw ays cord ial, and that there is nothing she can d o to change the w ay other people perceive her. Therefore, w hen she is around peop le she feels tense and u ncomfortable and has started avoid ing leaving the hou se. Which of the follow ing cognitive–behavioral therapy (CBT) interventions w ould most d irectly target the physical feelings she has around people? (A) (B) (C) (D) (E)

Cognitive restru cturing Exp osu res Keep ing a m ood d iary Keep ing a thou ght record Progressive m u scle relaxation

16. After several weeks, a 40-year-old female patient who is in psychodynamic psychotherapy begins to show up late and miss appointments; each time this happens she has a reason for doing so. However, the therapist begins to feel that her behavior is interfering with her treatment. The therapist is not sure if the patient understands that her behavior is inappropriate and potentially damaging to her therapy. Which of the following topics would be the most important to review with the patient before making an interpretation of her behavior?

126

4: P syc holog ica l Tre a tme nt a nd Ma nag e me nt

(A) Bou nd aries and ru les d efining the w ay in w hich therap y is cond u cted (B) Focu s on the therapeu tic goals (C) Method u sed to change the p atient’s m alad aptive thou ghts (D) Patient’s need for attention (E) Reasons the p atient is late to the ap pointm ents 17. A 45-year-old m an p resents to you r ou tp atient clinic hand icap ped by a fear of parking lots and field s. The fear started 4 m onths p rior to this visit. At the beginning of cognitive therap y, he tells you that his behavior is constantly being scru tinized and criticized by other p eop le. H e claim s that he cannot change his behavior, becau se if he d oes, others w ill think he is a fool. Which of the follow ing w ou ld be the m ost appropriate response? (A) “That’s silly. People w ill not think you are a fool.” (B) “Yes, I can see that.” (C) “What m akes you think th at others are con stantly scru tinizing you r beh avior?” (D) “H ow can you possibly think other p eople care enou gh abou t you to constantly scru tinize your behavior?” (E) “Well, m aybe you are a fool. H ave you ever thou ght of that?” 18. A 32-year-old fem ale in therap y d escribes how she feels that she should have a life p artner; and since she is not currently d ating, she is u nlikely to ever get m arried and is therefore u nlovable. H er therapist points ou t to her that she has had a few m eaningful long-term relationships in the past and that she has d ated as recently as a cou p le of m onths ago. The therap ist also explains that m any p eople m ay not d ate, bu t this d oes not m ean they are u nlovable or w ill never get m arried . Which of the follow ing term s best d escribes the therap eu tic intervention d escribed above? (A) Clarification (B) Confrontation (C) Em p athy

(D) Interp retation (E) Refram ing Questions 19 and 20 An 18-year-old , single m an p resents to you r office com plaining that he cannot pass a m ovie theater w ithou t stop p ing, going insid e, and bu ying cand y. This behavior is trou blesom e to him and interferes w ith his d aily activities, resu lting in his being constantly late to other app ointm ents. You d ecid e to help the p atient w ith behavior therap y. 19. Which of the follow ing w ould be the m ost ap p rop riate focu s of the therapy? (A) Analyzing the p atient’s relationship s (B) Decreasing the m alad ap tive behavior of stopping by m ovie theaters (C) Exam ining the p atient’s negative eating habits (D) Exp loring the p atient’s child hood trau m as (E) Working on resolving the p atient’s u nconsciou s conflicts 20. Which of the follow ing behavioral interventions w ould be the m ost help fu l in this case? (A) (B) (C) (D) (E)

Aversion therap y Environm ental m od ification Exp osu re and resp onse training Mod eling Relaxation training

21. A 39-year-old m an p resents to you r office com plaining of chronic “stress,” anxiety, and p oor sleep. H e also ad m its to interm ittent head aches, w hich you d iagnose as tension head aches. You recom m end behavioral therap y for red u ction of his head aches. Which of the follow ing behavioral techniqu es w ould be m ost appropriate for this patient? (A) (B) (C) (D) (E)

Aversive stim u li Biofeed back N egative reinforcem ent Stim u lu s control System atic d esensitization

Que s tions : 17–26

Questions 22 and 23 A 32-year-old m ale com p lains of ongoing d epression for the past m onth w hich he attribu tes to the breakup w ith his fiancée. H e has been having insom nia, d ecreased appetite, and low energy, althou gh he d enies anhed onia or problem s concentrating. H e also d enies any su icid al id eation. H e has ju st started to d ate som eone else w ithin the last w eek. After d iscussion w ith the patient, it is d ecid ed to u tilize both m ed ication and interp ersonal psychotherap y for his d ep ression. 22. Which of the follow ing goals w ou ld be the m ost likely focu s of his p sychotherapy? (A) (B) (C) (D) (E)

Character change Clarifying com m u nication p atterns Strengthening d efenses Interpreting transference Pointing ou t resistance

23. Which of the follow ing techniqu es w ou ld be m ost likely u tilized d u ring this p atient’s therapy? (A) (B) (C) (D) (E)

Dream interp retation Defense analysis Free association H ypnosis Role p laying

Questions 24 and 25 A 22-year-old college stud ent presents com plaining of having “no goals” in his life, not d oing as w ell as he could in school, and d esiring but having no serious long-term romantic relationships. H e has not chosen a m ajor and states that his ind ecision is paralyzing him from “m oving on w ith the rest of m y life.” After d iscussing various form s of psychotherapy w ith the patient, you recomm end psychoanalysis fou r times per w eek.

127

(C) Com p lete hom ew ork frequ ently to better learn abou t their issu es. (D) Prevent conflicts from arising betw een the therapist and p atient. (E) Requ ire m ore intensive treatm ent d u e to severity of m ental illness. 25. Which of the follow ing w ou ld be the m ost im p ortant tool in his p sychoanalysis? (A) (B) (C) (D) (E)

Altering cognitive d istortions Altering m alad ap tive behaviors Interp retation of transference Interp reting d ream s Solving interp ersonal p roblem s

26. A m ale th erap ist h as been w orkin g w ith a fem ale p atient for over 3 years, enabling her to im p rove her self-esteem and confid ence, and to eventu ally d ivorce her em otion ally abu sive h u sban d 1 year ago. The p atient continu ally thanks the therap ist and exp lains that she is sexu ally attracted to the therap ist. H e is initially taken aback and then som ew hat flattered by her sexu al interest. H ow ever, he also realizes that he feels a strong attraction to her and feels the n eed to care for her. Which of the follow ing w ou ld be the m ost ap p rop riate n ext step for the th erap ist to take? (A) Act cold , show less em p athy, and set m ore rigid bou nd aries d u ring sessions. (B) Exp lore the p atient’s feeling in ord er to increase her erotic transference. (C) Ignore the p atient’s d isclosu re of her sexu al attraction. (D) Seek su p ervision w ith a m entor or colleague. (E) Term inate or fire the p atient from his care. Questions 27 and 28

24. Which of the follow ing is the m ost im p ortant reason for the frequ ency of p sychoanalysis? (A) Allow the p atient to get enou gh su p p ort d u ring the course of their treatm ent. (B) Allow transference to bu ild betw een p atient and therapist.

You are seeing a 25-year-old man in cognitive–behavioral therapy w ho suffers from generalized anxiety d isorder, as w ell as epilepsy. H e often talks about the many dreams he has w hile in his sessions, and he is having a hard time w ith the restructuring of

128

4: P syc holog ica l Tre a tme nt a nd Ma nag e me nt

malad aptive thoughts. You d ecid e that hypnosis may be helpful as an adjunctive therapy for this patient, as he has been hypnotized before w ith good results and you have a skillful hypnotist in your clinic. The patient is hopeful that hypnosis w ill be beneficial for him again. 27. Which of the follow ing characteristics is the best p red ictor of resp onse to hyp nosis in this p atient? (A) (B) (C) (D) (E)

Patient has a lot of d ream s. Patient is resp onsive to su ggestion. Patient’s d iagnosis. Patient w ith low seizu re threshold . Trained hyp notist w ill be d oing his treatm ent.

28. The p atient is started in hyp nosis, bu t there is a concern as to w hether he is in a trance d u ring the sessions. Which of the follow ing ind icators w ou ld be m ost likely increased in a trance state? (A) (B) (C) (D) (E)

Am nesia Pain p ercep tion Pu lse Reflexes Resp irations

29. You are treating an 11-year-old boy for op p ositional d efiant d isord er and have heard from the p arents that his d isord er has had a significant im p act on the fam ily d ynam ics. They have tw o old er child ren w ho often feel left ou t and feel d istanced from the fam ily u nit. You recom m end a trial of fam ily therap y. In this case, w hich of the follow ing w ou ld be the m ost ap p rop riate focu s of the fam ily therap y? (A) (B) (C) (D) (E)

Assigning roles in a hou sehold Early child hood exp eriences Im p u lsive behavior Relationship p atterns Unconsciou s conflicts

30. A 32-year-old man presents for psychotherapy. He denies pervasive depression or anxiety symptoms, but he has had a string of failed

relationships. He blames the problem on “his sabotaging things” when the women “start to get serious.” He is clearly ambivalent about his desire to “settle down” with someone. According to Erik Erikson, which of the following stages of development is the patient most likely in? (A) (B) (C) (D) (E)

Tru st versu s m istru st Au tonom y versu s sham e Initiative versu s gu ilt Intim acy versu s isolation Id entity versu s role confu sion

31. A 9-year-old boy w ith attention-d eficit/ hyperactivity disorder (ADHD), predominantly hyp eractive/ im p u lsive presentation, presents to you r office accom panied by his m other. H e is cu rrently on m ethylp henid ate (Ritalin) and they both w ond er if there is som ething else that can be d one to help the patient. Despite som e benefit and tolerability, the child continu es to have d ifficulty w ith com pleting hom ew ork and follow ing instru ctions at school and at hom e. H e has also recently gotten into a m inor p hysical fight w ith a p eer. Parent m anagem ent training is recom m end ed for the parents. Which of the follow ing aim s best illu strates how p arent m anagem ent training w ill help the fam ily? (A) Focu sing on m alad ap tive com m unication in the fam ily (B) Focu sing on the fam ily’s interp ersonal relationship s (C) Increasing p arental u se of p ositive reinforcem ent to increase d esired behaviors (D) Interp reting p arent’s negative cognitions abou t the p atient (E) Teaching parents to p ositively refram e a negative exp erience 32. A 14-year-old boy w ith attention-d eficit/ hyperactivity d isorder (ADH D) is frequently late for school and forgets to d o his chores around the house d ue to his d isorganization. H is parents are fed up w ith him and ask you w hat they can d o to help change his behavior. Which of the follow ing represents the most helpful behavioral tool for patients w ith ADH D?

Que s tions : 27–37

(A) A chart of behaviors to be rew ard ed by p arents (B) Aversion therapy (C) Interp retation of m alad ap tive behaviors (D) Positively refram ing a negative exp erience (E) Pu nishm ent Questions 33 and 34 A 25-year-old u nm arried m an in his fou rth year of m ed ical school presents to you r office com p laining of not being able to rem ain in a relationship w ith a girlfriend for longer than 3 m onths. H e says that he w ould like an insight-oriented therap y, and you agree that this form of therapy w ou ld help him . You decide to treat this patient with brief psychod ynamic p sychotherap y. 33. Which of the following factors would best predict a positive outcome with brief psychodynamic psychotherapy in this particular patient? (A) (B) (C) (D) (E)

Age Gend er Marital statu s Motivation for change Socioeconom ic statu s

34. Which of the follow ing asp ects best d istingu ishes his sp ecific typ e of p sychotherap y from other form s of therap y? (A) (B) (C) (D) (E)

Correcting cognitive errors Id entification of a focal conflict Interp reting d ream s Interp reting resistance Mod ifying m alad ap tive behaviors

35. A 20-year-old m an is cu rrently in college and has com p leted the first sem ester of his sop hom ore year. H alfw ay into the second sem ester, his grad es d rop from As to Fs over a 3-m onth p eriod . H e becom es increasingly isolated and p aranoid , believing that the governm ent is after him becau se he has solved “all theological p roblem s” throu gh d irect com m u nication w ith God . H e is started on olanzap ine 10 m g by m ou th (PO) d aily and is tentatively

129

d iagnosed w ith schizop hreniform d isord er. H e recovers after 4 m onths bu t has another relap se 6 m onths later in the absence of an elevated , irritable, or d ep ressed m ood . H e is then given a d iagnosis of schizop hrenia and referred for m ed ication m anagem ent and therap y. After a few w eeks of therap y, the p atient exp eriences a w orsening of his p sychotic sym p tom s. Which typ e of therap y w as he m ost likely referred to? (A) (B) (C) (D) (E)

Behavioral therap y Fam ily therap y Grou p p sychotherap y Psychoanalytic p sychotherap y Su p p ortive p sychotherap y

36. A 50-year-old m an is in p sychoanalysis becau se he feels stagnant in his p ersonal life. H e notes he is not having any p roblem s w ith his w ork or relationship w ith his w ife, althou gh he feels that he has never had m ore than a p latonic relationship w ith her. While he feels cap able and confid ent in m ost sp heres of his life, inclu d ing his relationship s, he feels like he is at a “road block” in therap y and d oes not know how to p roceed . H e has, how ever, m entioned a reoccu rring d ream about his w ife tu rning into his m other and taking care of him . The therapist thinks that interpreting his d ream m ay be a good tool to u se in therap y. Which of the follow ing u ses of his d ream w ou ld be the m ost ap p rop riate in this case? (A) Interp reted by the analyst to su blim inally influ ence the m ind of the p atient. (B) Provid e inform ation abou t p sychic conflicts. (C) Rep resent the consciou s fram ew ork for behavior. (D) Used to alter cognitive errors. (E) Used to change behavior. 37. You have been assigned to lead a d ialectical behavioral therap y grou p for p atients w ith bord erline p ersonality d isord er. Du ring the grou p one of the p atients relates a story abou t getting into a fight w ith her m other. Afterw ard she w as so d espond ent she had thou ghts

130

4: P syc holog ica l Tre a tme nt a nd Ma nag e me nt

abou t cu tting herself. Instead of resp ond ing, she w atched her favorite com ed y m ovie. Which of the follow ing skills d id the p atient m ost likely em p loy? (A) (B) (C) (D) (E)

Cognitive restructu ring Distress tolerance Em otion regu lation Interpersonal effectiveness Mind fu lness

Questions 38 and 39 A 33-year-old w om an p resents to the em ergency d epartm ent saying that she w ants to kill herself. She has felt increasingly d epressed for m any m onths, w ith late insom nia, p oor ap p etite, w eight loss, d istractibility, and fatigu e. She cu rrently feels hopeless and help less, and has a p lan to overd ose on p ills that she has stockpiled at hom e. 38. Other than her feelings of helplessness and hopelessness, w hich of the follow ing cognitive d ifficulties w ou ld she m ost likely also exhibit? (A) (B) (C) (D) (E)

Begging the question Circu lar reasoning Personalization Rigid , black-or-w hite thinking Selective abstraction

39. After hospitalization, w hich of the follow ing form s of p sychotherap y w ou ld be the m ost ap p rop riate to help m inim ize this patient’s su icid ality? (A) (B) (C) (D) (E)

Behavioral therapy Cognitive therap y H yp nosis Interpersonal p sychotherap y Parad oxical therap y

40. In the context of a w om en’s grou p , one m em ber has been frequ ently sid e talking. As th e facilitator, you bring this u p d u ring the grou p . Th e m em ber states she only d oes this becau se you d on’t give enou gh tim e for each m em ber to talk. The other grou p m em bers agree w ith h er.

Which of the follow ing stages of grou p form ation is this grou p m ost likely in at this tim e? (A) (B) (C) (D) (E)

Form ing Storm ing N orm ing Perform ing Ad jou rning

41. A 38-year-old law yer is referred for behavioral therapy because of extrem e social anxiety w hich is interfering in his ability to litigate. In his early therap y sessions, he is told to m ake a hierarchy of situ ations that m ake him anxiou s. The therap ist then begins w orking w ith him by first exposing him to the least anxietyp rovoking item s on the list and then grad u ally increasing the severity. Which of the follow ing therapeu tic techniques is the therapist m ost likely u tilizing? (A) (B) (C) (D) (E)

Aversion Flood ing Mod eling Su ggestion System atic d esensitization

42. A 47-year-old , d ivorced m ale has been attend ing p sychotherap y for 8 m onths d u e to ongoing “stress” and conflicts at w ork. Du ring m ost of his therap y sessions, the p sychiatrist resp ond s to his com m ents w ith em p athic resp onses. Which typ e of p sychotherap y is the p sychiatrist m ost likely u tilizing in this case? (A) (B) (C) (D) (E)

Behavioral Cognitive Dynam ic Existential Su p p ortive

D IRECTION S (Questions 43 through 47): The follow ing group of questions is preceded by a list of lettered options. For each patient scenario, select the one lettered option that is most closely associated w ith it. Each lettered option may be used once, multiple times, or not at all.

Que s tions : 38–50

(A) (B) (C) (D) (E) (F) (G) (H ) (I) (J) (K) (L) (M) (N ) (O)

Biofeed back Cou ntertransference Flood ing H ypnosis N egative reinforcem ent Positive reinforcem ent Projection Projective id entification Punishm ent Real relationship Schem a Sp litting Therap eu tic alliance Transference Und oing

43. You are seeing a 26-year-old w om an w ho com plains of m igraines. She is otherw ise healthy. You exp lain to her that you w ou ld like to try a techniqu e in w hich you give her inform ation on m u scle tension and tem peratu re. 44. You are seeing a 45-year-old m an in p sychoanalysis fou r tim es p er w eek. Du ring one session he appears less com fortable talking. H e eventu ally tells you that you r new glasses rem ind him of his au nt w ho cared for him as a child bu t w as cru el to him . 45. A p atient w ho is afraid of clow ns is taken to the circus in the beginning of her therapy in ord er to get over her fear.

131

46. To train a 4-year-old boy to say “thank you,” his m other gives him a sw eet w henever he d oes. 47. To train a 4-year-old boy not to bite his classm ates, his m other p uts soap in his m ou th w henever he d oes. D IRECTION S (Questions 48 through 50): The follow ing group of questions is preceded by a list of lettered options. For each question, select the one lettered option that is most closely associated w ith it. (A) (B) (C) (D) (E) (F)

Josef Breu er Jean-Martin Charcot Erik Erikson Sigm u nd Freu d H einz Kohu t Franz Anton Mesm er

48. Mad e m any contributions to p sychiatry and neu rology w ith a sp ecial interest in d escribing hysteria. 49. Know n as the inventor of the “cathartic” treatm ent or talking therap y. 50.

Know n as the d evelop er of self-p sychology.

Ans we rs a nd Expla na tions

1. (A) Cognitive–behavioral therap y refers to a variety of techniqu es that focus on the construction and reconstruction of people’s cognitions, emotions, and behaviors. Cognitive–behavioral therapists use a range of m od alities to help p atients assess, recognize, and d eal w ith problem atic and d ysfu nctional w ays of thinking, em oting, and behaving. In this case, a thou ght record is being u sed to exam ine the p atient’s feelings and beliefs. Exp ressive therap y u ses d ifferent form s of exp ression (e.g, art, m ovem ent, m usic, or w riting) to have a patient exp ress his or her feelings and w ork throu gh them . H ypnotherapy uses hypnosis to u ncover u nd erlying conflicts and hypnotic su ggestion in ord er to change behaviors. Psychoanalysis encou rages patients to verbalize all their thoughts, includ ing free associations, fantasies, and d ream s. The analyst u ses these to u nd erstand the u nconsciou s conflicts causing the patients sym ptom s or character problem s. Psychod ynam ic therap y attem pts to d iscover the unconscious content of a patient’s psyche and then w ork on the issu es in therap y; it has its root in p sychoanalysis. 2. (A) Au tom atic thou ghts are the im m ed iate thoughts that com e to m ind in any situation that are based on underlying core beliefs; these are d eep ly held id eas regard ing the w orld and oneself that d rive a patient’s em otional resp onses and behavior. Ego strength is the ability of the ego to cope w ith com p eting conflicts betw een the id , ego, and su perego w hich help one m anage stress and m aintain stability. Projections are d efense m echanism s, w hereby an ind ivid u al u nconsciou sly attribu tes u nd esirable asp ects of oneself onto 132

others. Unconscious thoughts are instinctual thoughts, d rives, and need s that are ou t of one’s aw areness. 3. (B) Disp lacem ent is a d efense that transfers a feeling abou t, or a resp onse to, one object (p erson) onto another object (p erson). Denial is a d efense that keep s ou t of consciou s aw areness an asp ect of external reality or subjective d istress that is too u ncom fortable for the p erson to accept. H u m or is consid ered a m atu re d efense m echanism that em p hasizes the am u sing or ironic asp ects of a stressor. Intellectu alization is a d efense in w hich the ind ivid u al favors abstract thinking over d ealing w ith the d istu rbing feelings of an id ea or exp erience. Isolation of affect involves d etachm ent of feelings from a p articu lar id ea or exp erience. 4. (E) Splitting occu rs w hen an ind ivid u al is u nable to see others m od erately, w ith both p ositive and negative qualities (i.e., people’s actions are either all good or all bad ). Devalu ation is a d efense that attribu tes excessive negative qu alities to another. Id ealization attribu tes excessive p ositive qu alities to another. Intellectu alization is a d efense that u tilizes excessive abstract thinking or generalizations to m anage threatening em otions. Rep ression expels d isturbing w ishes, thoughts, or exp eriences from consciou s aw areness. 5. (B) This is an exam p le of intellectu alization, a d efense that u tilizes excessive abstract thinking or generalizations to m anage threatening em otions. Denial is a d efense that keep s out of consciou s aw areness an aspect of external reality or subjective d istress that is too

Answe rs : 1–10

u ncom fortable for the person to accep t. Projection is a d efense w hereby a thou ght, feeling, or id ea that is u naccep table to the p erson is falsely attribu ted to another. Su p p ression is w here one intentionally avoid s thinking of d istressing thou ghts, feelings, id eas, or experiences. Und oing is a ritu alistic act com p leted in ord er to red u ce gu ilt abou t an event that occurred in the past. 6. (D ) The best strategy in any com m u nication is to try to gain clarity. Asking this patient w hy she believes w hat she d oes m ay lead now here bu t it m ay help to u nd erstand the p atient’s concerns. Choice (A) m ay be helpful in som e situ ations and certainly has a positive ring to it. H ow ever, this patient’s com m ent is an im p licit qu estion. She is asking for help and genu inely d oes not know w hat to d o. H id ing behind p ithy m axim s is not u su ally help fu l. Choices (B) and (C) are coercive and block com m u nication. Giving supp ortive guid ance is ap p rop riate in this situ ation, bu t coercion is never ap p rop riate in a therap eu tic relationship. Choice (E) is not coercive but also blocks com m u nication. While the p atient m ay requ ire reality testing, the resp onse m ay sacrifice a m ore insightfu l, em p ow ering rep ly. 7. (E) The goal of su p p ortive therap y is the strengthening of current d efense m echanism s. Throu gh su p p ortive care, the p atient can learn to cope w ith d ifficu lt problem s u sing alread y established abilities. Correcting fau lty id eas is the exp ressed goal of cognitive therap y. Exp loring the feeling of m eaninglessness in life and investigating the freed om of ind ivid u als are goals of existential p sychotherap y. Personality change is a goal of psychod ynam ic p sychotherap y, w here d efenses are exam ined grad u ally in ord er to achieve a m ore op tim al level of fu nctioning. 8. (A) This is an exam p le of free association. Freu d believed that all m ental activity is cau sally related , m eaning that free associations are not truly rand om . They follow logically, bu t the m issing connection is being held d ow n or rep ressed u nconsciou sly by d efenses. H yp nosis is w here a p atient is placed in a trance and

133

their unconscious is explored . Rep ression is a d efense m echanism that exp els d isturbing w ishes, thou ghts, or exp eriences from conscious aw areness. Thou ght record s refer to a p atient record ing situ ations, feelings, au tom atic thou ghts and alternative responses then re-rating the initial feelings; this techniqu e is com m only u sed in cognitive–behavioral therapy. Transference, in strict term s, is the re-exp eriencing of p ast exp eriences w ith the analyst in the setting of p sychoanalytic psychotherapy. Cou ntertransference is the analyst’s resp onse to this. H ow ever, these term s have com e to m ean the transferring of em otions and feelings that one has from one’s past to the physician or care provid er in the case of transference and the p hysician tow ard the p atient in countertransference. 9. (A) Cou ntertransference is the consciou s or u nconsciou s em otional reaction of the therap ist to the p atient. Transference is the conscious or u nconsciou s em otional reaction of the p atient to the therapist based on a past relationship the p atient had w ith a significant other person (e.g., a p arent). Extinction is a term frequ ently u sed in learning theory, w hich refers to the red u ction in frequ ency of a learned resp onse as a resu lt of the cessation of reinforcem ent. Interp retation is insight offered by the therap ist regard ing p atterns of thou ght or behavior. Resistance is the patient’s u nconscious opposition to full d isclosure of feelings or id eas. 10. (D ) Psychod ram a is a techniqu e in w hich grou p m em bers are u sed as the au d ience and the cast in reenactm ents of scenarios and conflict. The content of the d ram a is u sed to exp lore ind ivid u al p roblem s in a grou p setting. Feed back, a card inal featu re of grou p p sychotherapy, com es from peers as w ell as from the lead er. Feed back from p eers in the grou p can be a p ow erfu l stim u lu s for insight and change. Free association is u sed in p sychoanalytically oriented grou ps. These grou ps are largely ou tpatient grou ps w hose m em bers can tolerate and effectively u se this techniqu e. Mem bers are encou raged to be sp ontaneou s w ith their thou ghts and feelings, and the therap ist

134

4: P syc holog ica l Tre a tme nt a nd Ma nag e me nt

assists in p ointing ou t com m on them es and p ossible connections. Go-arou nd is a com m on techniqu e w here each grou p m em ber is asked to introd u ce them selves, state how they are feeling, or otherw ise specifically respond ; this gives each ind ivid u al a chance to p articip ate. Resistance is the u nconsciou s op position to fu ll d isclosu re of thou ghts or feelings. 11. (C) Interp retation is u sing a hyp othesis to exp lain the p sychological m eaning of a behavior or sym p tom s. Cou ntertransference is w hen the therap ist’s feelings and em otions are d irected tow ard a p atient. Em p athy is w hen a therap ist id entifies and u nd erstand s a p atient’s situ ation or feelings. Parad oxical intervention is a therap eu tic techniqu e w here a therap ist instru cts a p atient to hold onto a sym p tom in ord er to get the p atient to u nd erstand that the sym p tom is not help fu l. Working throu gh refers to thinking abou t and d iscu ssing a p roblem in ord er to lessen its intensity or im p act. 12. (D ) Intervention is an orchestrated attem p t by one or m any to get an ind ivid u al to accep t p rofessional help or treatm ent, u su ally for an ad d iction or crisis. Behavioral therap y focu ses on changing a patient’s m alad aptive behaviors or actions; its pu rpose is not to get a patient to com m it to treatm ent. Fam ily therap y involves an entire fam ily in therapy together w orking to change the d ynam ics, d evelop m ent, and system s of interaction; fam ily therap y d oes not usu ally focu s on the problem s of one m em ber. Grou p therapy is w hen a group of ind ivid uals w ork on issu es in conju nction w ith each other; it can be help fu l becau se the d ifferent backgrou nd s and problem s can be looked at together in ord er to help each p erson w ith their ow n d ifficu lties. Relapse prevention is u sed in ad d ictions treatm ent, w here ind ivid u als are tau ght self-control and how to m aintain changes in behavior, as w ell as how to anticipate and cope w ith relap se. 13. (D ) This p atient is d em onstrating the concep t of reaction form ation, w hich is w hen u nacceptable or anxiety-provoking em otions are d ealt w ith by acting in the opposite m anner.

Abreaction is w hen a p atient relives an em otional or traum atic experience to get rid of or p u rge negative em otions and m ove forw ard . Dep end ency is w here one need s another p erson or ad d ictive su bstance in ord er to m aintain a level of em otional or p hysical fu nctioning. Projection is a prim itive d efense consisting of attribu ting one’s ow n und esirable thou ghts, d esires, or em otions onto som eone or som ething else. Sp litting is a d efense m echanism w here one view s them selves and / or others as “all good ” or “all bad .” It is com m only seen in p atients w ith bord erline personality d isord er. 14. (B) This ind ivid u al has featu res of narcissistic p ersonality d isord er. Althou gh p atients w ith narcissistic p ersonality d isord er have d ifficulty w ith confrontation, it is necessary to help them w ork tow ard change; therefore, group therapy is a good setting for this to take p lace. In ad d ition, in ind ivid u al therap y p atients can process these situ ations and exam ine how p ossible insu lts to the ego can be d ealt w ith, as w ell as ad d ressing the other ind ivid u al issu es. Anger m anagem ent as w ell as social skills training can be helpfu l, but are not enough to exam ine the d eep u nd erlying issu es and d efenses that patients w ith narcissistic p ersonality d isord er have. Sim ilarly, p sychoanalysis alone w ou ld not be enou gh to help patients in w hich m any of their relationship s and social interactions are affected by their p ersonality d isord er. Pharm acotherapy is often not help fu l w ith personality d isord ers unless severe im p u lsivity or p sychosis is p resent, or u nless a p atient has a com orbid p sychiatric d isord er (e.g., m ajor d ep ressive d isord er). 15. (E) Progressive m u scle relaxation is a relaxation technique u sed in CBT to control and m inim ize m u scle tension. Of the op tions, it is the only one d irectly targeted at the p atient’s u ncom fortable feelings w hile arou nd other p eop le. Cognitive restru ctu ring and keep ing a thou ght record are techniqu es w hich m ay im p rove the sym p tom s by id entifying and m inim izing au tom atic thou ghts. Exp osu re therapy is a techniqu e used to d ecrease the avoid ance behaviors by exp osing p atients to the feared stim u li in ord er to show they can

Answe rs : 11–19

tolerate it. It is com m only u sed in treating anxiety d isord ers. A m ood d iary is a techniqu e u sed for m onitoring one’s m ood to d eterm ine triggers, au tom atic thou ghts, and treatm ent p rogression. 16. (A) In this case, it w ou ld be im p ortant for the therap ist to review the “fram e” for therap y. In p sychod ynam ic p sychotherap y the “fram e” refers to the bou nd aries and ru les d efining the w ay the therap ist and p atient interact (e.g., the m eeting tim e, p lace, length, other d etails d ow n to w here the therap ist and p atient u su ally sit, how fees for service are collected , w hat hap p ens w hen a p atient starts m issing ap p ointm ents). The fram e is im p ortant as it gives p sychotherap y a stru ctu re and sets u p a sp ecific d ynam ic betw een the p atient and therap ist. Therefore, if bou nd aries or ru les are challenged the therap ist can u se this to better u nd erstand the reasons as w ell as d eterm ine any transference issu es at p lay. In this case the therap ist su sp ects the p atient m ay not u nd erstand the fram e u sed for her therap y sessions and w hy it is im p ortant. Review ing the therap eu tic goals and the typ e of therap y p rovid ed m ay both be reasonable top ics for the therap ist to d iscu ss; how ever, it w ou ld not ad d ress her reasons for being late and not show ing u p for ap p ointm ents. Interp reting the p atient’s behavior as a need for attention m ay not only be incorrect, bu t it m ay be exp erienced by the p atient as an attack or hu rtfu l, p articu larly if she d oes not u nd erstand the therap eu tic fram e correctly. Sim ilarly, looking at the ind ivid u al reasons w hy she is late avoid s the su bject of w hy this p attern of behavior is p resent. 17. (C) One reason this is the best resp onse is that it is a qu estion. Patients w ith cognitive d istortions often take their view of the w orld as a given. They d o not qu estion their p ersp ectives. Asking a qu estion invites p atients to exam ine their perspectives and thereby becom e open to the p ossibility that their thoughts m ay be inaccu rate. Again, gaining clarity is alw ays a good first step in com m u nication. Choices (D ) and (E) are also qu estions bu t are au thoritarian and p u nitive, not to m ention insu lting; to

135

have the best ou tcom e in any form of therap y, there m u st be a good relationship or alliance betw een the p atient and therap ist. Choice (A) m ay also be harm fu l as a p atient shou ld not be called “silly.” Choice (B) is am biguou s—to w hich p art of the qu estion are you resp ond ing? The p atient m ay believe that you are insu lting him rather than attem p ting to be sup portive. 18. (E) Refram ing is a techniqu e com m only u sed in cognitive–behavioral therap y. First, a therap ist w ill exam ine how p atient’s thou ghts or beliefs abou t them selves or the w orld m ay be skew ed . Then the thou ghts or beliefs are refram ed by com p aring them to a m ore norm alized p ersp ective. Clarification is w hen a therap ist asks for fu rther d etails abou t a p atient’s exp erience or situ ation they are d escribing. Confrontation is w hen a therap ist d irectly asks p atients abou t their m alad ap tive behaviors or sym p tom s. Em p athy is w hen a therap ist show s u nd erstand ing or agrees w ith p atients, show ing them that they are able to “p u t them selves in the p atient’s shoes” or see their p ersp ective. Interp retation is w hen a therap ist p resents the p atient w ith a hyp othesis regard ing the reason(s) (often u nconsciou s) a p atient has certain behaviors or sym p tom s. 19. (B) Behavior therap y is focu sed on behaviors, not on p ossible und erlying factors that cause the behaviors. In this case, ad d ressing the negative or m alad ap tive behavior (e.g., stop p ing by m ovie theaters to bu y cand y) w ould be ad d ressed . Behavior therap y is a hereand -now ap p roach to m alad ap tive behaviors. It m ay u tilize inform ation su rrou nd ing the em ergence of a p articu lar m alad ap tive behavior in ord er to constru ct a behavior m od ification p lan. Althou gh behavior therapy can and d oes change neu robiology, this is not its exp ressed goal. It d oes not focu s on the p atient’s relationship s or seeking the sou rces for the p atient’s negative eating habits. This is also w hy child hood issu es su ch as trau m a are not typ ically ad d ressed in behavioral therapy. Resolving und erlying conflicts in an attem p t to change behavior is a d escrip tion of

136

4: P syc holog ica l Tre a tme nt a nd Ma nag e me nt

the goals of psychod ynam ic p sychotherapy or p sychoanalysis. 20. (C) In this case, exp osu re and resp onse train ing w ou ld be the m ost help fu l in d ecreasing the p atient’s m alad ap tive behavior of stop p ing at m ovie th eaters. It w ou ld teach m ore ad ap tive behaviors throu gh exp osin g him to situ ations w h ere h e cou ld p ass by a m ovie th eater an d en gage in an alternative behavior. Aversion therap y is w hen a negative (p ainfu l) stim u lu s is p aired w ith a m alad ap tive behavior in ord er to extin gu ish the behavior; it is not com m only u sed and often the beh avior retu rn s over tim e. In this case, en vironm en tal m od ification w ou ld be less help fu l as the p atient w ill likely encou n ter situ ation s w here he m ay p ass m ovie theaters in th e fu tu re; in ad d ition , changing th e en vironm en t d oes n ot ad d ress the behavior. Mod eling is w here ad ap tive behaviors are tau ght by d em onstration, w hich w ou ld be d ifficu lt to teach in this p articu lar case. Relaxation train ing is u sed to help a p atient to han d le an xiety-p rovoking situ ations, not chan ge an u nh ealthy beh avior. 21. (B) Biofeed back and stress red u ction are u sed to red u ce the frequency and severity of tension head aches. They are equ ally effective in accom plishing this goal. Su bstance w ithd raw al synd rom es are exam ples of aversive stim uli, and the resu ltant negative reinforcem ent theoretically lead s to a d ecrease in su bstance u se. Stim u lus control is u sed in the treatm ent of insom nia: The negative association betw een m u ltip le failed attem p ts at falling asleep and the specific environm ent w here they occurred is elim inated by red u cing d istractions and creating an environm ent m ore cond ucive to sleep (e.g., taking televisions and com pu ters ou t of the bed room ), as w ell as teaching good sleep hygiene. System atic d esensitization is a behavioral techniqu e w hich can be u sed in asthma, where the patient is exposed to provoking stimuli in a progressive manner. 22. (B) The m ajor focus of interpersonal psychotherapy is comm unication analysis. Ind irect, unclear interpersonal commu nication patterns

are id entified in the cou rse of therapy. These com m u nication p atterns are then altered , for instance, by role playing or asking the p atient to try ou t d ifferent form s of com m u nication d u ring the session. Character change, interp reting transference, and p ointing ou t resistance are m ore the focu s of p sychod ynam ically oriented p sychotherap y. Strengthening of d efenses is the goal of su p p ortive p sychotherap y. 23. (E) Interp ersonal p sychotherap y focu ses on the patient’s role and com m unication style w ithin im p ortant relationship s and how his or her position m ay be cau sing or com p licating the illness (e.g., d epression). If one or tw o specific w ays of com m u nicating can be altered so that a relationship im p roves, the d ep ressive illness also im proves. This therap y also has the ad d ed benefit of d u rability; once a com m unication pattern is im p roved and the p atient better u nd erstand s how his or her role in a relationship m ay create d istress, the d epressive illness is not as likely to recu r. Variou s techniqu es u sed in interpersonal p sychotherap y are role p laying, su p portive listening, clarification, and com m u nication analysis. Interpreting d ream s, analyzing p sychic d efenses, and free association are used in m ore p sychod ynam ically based therap ies, such as psychoanalysis. H ypnosis is a sleeplike state ind u ced by a therap ist, w hich has been of som e u se in behavioral m od ification such as sm oking cessation. It is not a goal or com ponent of interpersonal p sychotherapy. 24. (B) Psychoanalysis tend s to be am ong the longest of therap ies and involves the p rocess of free association, in w hich the patient lies on a couch, facing aw ay from the therap ist, and says w hatever com es to m ind . It takes four to five sessions a w eek to su ccessfully com p lete an analysis becau se it is d ifficu lt for patients to bu ild , su stain, and w ork through their transference and d efensive stru ctu re; less frequ ent sessions d ecrease the form ation of a “transference neurosis” and increase d efenses. Psychoanalysis is based on an exp ressive or exp loratory m od el of therap y, as op p osed to a su p portive m od el aim ed at stabilizing crisis

Answe rs : 20–30

and reestablishing a baseline level of fu nction. H om ew ork is a m ethod u sed on cognitivebehavioral therap y and not p sychoanalysis. In p sychoanalysis, conflicts betw een therap ist and patient are analyzed and d iscussed as they often represent a recreation of child hood conflicts and relationship s. There is no correlation betw een severity of m ental illness and length of therapy. Patients w ho are in psychoanalysis are not m ore likely to be severely ill and w ill need a certain level of insight and com petence to gain the benefits of p sychoanalysis w ithout encou ntering severe p roblem s. 25. (C) The m ajor tool of p sychoanalysis is the carefu l interp retations of the transference neu rosis. Altering cognitive d istortions is the m ain goal of cognitive therap y. While psychoanalysis m ay accom p lish m any tasks, inclu d ing changing m alad aptive thou ghts and behaviors and “solving” interp ersonal problem s, these are the consequ ences of analysis, not the instru m ents of change. Althou gh interpreting d ream s can lead to insights into a patient’s u nconsciou s thou ghts and feelings, it is an ad ju nctive tool. 26. (D ) In this case, the therap ist is having an inap p rop riate attraction to his p atient and shou ld seek su pervision in ord er to u nd erstand the reasons for his (and the p atient’s) sexu al feelings. In ad d ition, a su p ervisor can help to m ake sure that the transference or countertransference is not significantly interfering in the therapy. N ot ad d ressing the situ ation, or acting less em pathic or cold w ould be evasive and confu sing to the p atient, m aking it d ifficult for the patient to m ove forw ard in therap y. The therap ist shou ld not try to elicit m ore behaviors or continu e to d eep en any erotic transference that cou ld lead to a greater pow er d ifferential or m ore sed uctive behavior; this cou ld p ut the relationship in jeopard y or cross bou nd aries inap p rop riately. Althou gh the therapist can fire or term inate the patient, the situ ation m ay not be seriou s enou gh to w arrant su ch severe actions; it m ay also send the m essage to the p atient that her sexu al feelings are d angerous and that the therap y w as not a safe place to d iscuss them .

137

27. (B) While a skilled hyp notist is very im portant, far and aw ay the m ost im portant factor in hyp nosis is a p atient w ho is highly resp onsive to suggestion, as ind icated by the patient’s prior p ositive exp erience w ith hyp nosis and belief that it w ill be help fu l. Som e ind ivid u als are able to alter their su bjective experience in resp onse to su ggestion m ore than others; som e patients can be tau ght self-hypnosis. N one of the other characteristics is as im p ortant in the su ccess of hypnosis. 28. (A) There are nu m erou s ind icators of a trance state, su ch as am nesia, anesthesia, and red u ced p ulse, reflexes, and respiration. Other changes inclu d e changed voice qu ality, eye closu re, lack of bod y m ovem ent, p u p illary changes, red u ced blinking, and tim e d istortion. 29. (D ) Fam ily therap y focu ses on the relationships betw een m em bers and how to m ake them m ore ad aptive to the situations. In this case, fam ily therapy w ill help the p arents and child ren u nd erstand w hy there are conflicts betw een them and w ork w ith the fam ily as a u nit; the p u rp ose is neither to blam e nor change the brother’s behavior. Fam ily therapy d oes not stress assigning roles to each m em ber. Unlike p sychoanalysis or p sychod ynam ic therapy, fam ily therapy d oes not focu s on any ind ivid u al’s early child hood exp eriences or traum a, u nconsciou s conflicts, or im pu lses that led to the fam ily m em bers’ pathology. 30. (D ) This p atient is in the Erikson stage of intim acy versus isolation w hich occu rs in you ng ad u lts from the ap p roxim ate age of 20 to 40 years. This is the p eriod w hen ad u lts are struggling w ith becom ing em otionally close and physically intim ate w ith another versu s feelings of loneliness. Tru st versu s m istru st occu rs from birth to 1 year w hen infants learn w hether or not p rim ary caregivers can satisfy basic need s. The au tonom y versu s sham e/ d ou bt stage occu rs in tod d lers from ages 1 to 3 years w hen child ren attem pt to m aster tasks on their ow n (as w ell as their ow n im pu lses), and , if su p p orted (as op p osed to p u nished ), grow confid ent and secu re in their ability to su rvive. Initiative versus gu ilt occu rs from age

138

4: P syc holog ica l Tre a tme nt a nd Ma nag e me nt

3 to 5 years w here p reschool child ren gain a sense of w ell-being from d isplaying initiative w ithou t feeling gu ilty. The id entity versu s role confu sion stage occu rs d u ring ad olescence from ages 11 to 19 years w hen teenagers attem p t to d eterm ine w ho they are and w hat they w ish to accom plish in life. 31. (C) Parent m anagem ent training is a type of therapy that focu ses on giving parents tools to m anage p roblem behaviors in child ren w ith d isrup tive behaviors. The goal is to increase the u se of positive reinforcem ent to increase d esired behaviors at hom e and school. This is typically d one w ith behavior charts and earning p rivileges. The other choices apply m ore to other types of fam ily therapy. 32. (A) A behavioral tool th at is u sed qu ite effectively is a beh avior ch art. It can serve as a p ositive incen tive to “good ” beh avior w h en th e rep ort is p ositive and is then rew ard ed . Th e rew ard s can be ad ju sted for th e d egree of p ositive beh avior obtain ed . N either aversion therap y nor p u n ish m en t is help fu l for teachin g p atien ts w ith ADH D n ew , m ore constru ctive behaviors. In terp retive strategies su ch as a w ell-tim ed in terp retation of m alad ap tive beh aviors or p ositively refram in g a n egative exp erien ce are n ot beh avioral tools. 33. (D ) Motivation for change goes beyond a d esire for rem oval of sym ptom s and inclu d es a w illingness to tolerate d iscom fort and to take risks in a search for u nd erstand ing. The other d em ographics d o not play as significant a role. 34. (B) In brief p sychod ynam ic therap y a single, focal area of conflict is id entified , and d u ring the therapy the tim e is sp ent actively interpreting transferences as they p ertain to this id entified focal conflict. Many types of therapy correct cognitive errors and mod ify malad aptive behaviors, although cognitive–behavioral therapies are specifically concerned w ith these ou tcom es. Interpretations of d ream s and resistance m ay certainly take place in brief psychod ynam ic therapy; how ever, they are only

interp reted in so far as they relate to the id entified focal conflict. 35. (D ) Psychoanalytic p sychotherap y or psychoanalysis is not generally ind icated in p atients w ho have d ifficu lty w ith reality testing, such as those w ith schizophrenia. The inevitable regression, introsp ection, and self-exam ination required in these intense, insight-orientated p sychotherap ists are u su ally overw helm ing to p sychotic p atients and actu ally m ay cau se m ore harm than good , w ith a resu ltant w orsening of p sychosis. All of the other types of psychotherapies can be helpfu l in the treatm ent of schizophrenia or other psychotic d isord ers. 36. (B) Dream s p rovid e inform ation abou t p sychic conflicts. In this case, d ream interpretation m ay help the p atient learn m ore abou t his cu rrent u nhap p iness and conflict in his m arried life. Dream s are u sed to u ncover p sychic conflicts, w hich, w hen interp reted app ropriately and consciou sly (not su blim inally), can lead to a p rod u ctive change in behavior. Accord ing to p sychoanalytic theory, d ream s rep resent the u nconsciou s w orkings of the m ind , not the conscious ones, and may be symbolic in content. Interpreting d reams may eventually lead to an alteration in a patient’s cond uct, but reducing cognitive errors or changing behavior is not its primary purpose. 37. (B) Dialectical behavioral therap y (DBT) w as d evelop ed to treat bord erline personality d isord er. It is often p erform ed in both grou p and ind ivid u al settings to m inim ize the intense transferences often d eveloped by patients w ith bord erline p ersonality d isord er in ind ivid ual therapy. The focu s is on teaching skills to m inimize m ood d ysregulation and selfharm. Distress tolerance focuses on d istracting from and tolerating negative em otions. Cognitive restru cturing is a concept from cognitive therap y that is som etim es em ployed in DBT. It refers to recognizing and changing m alad ap tive thou ghts. Em otion regu lation focu ses on recognizing and regulating m ood changes. Interp ersonal effectiveness relates to interp ersonal skills and assertiveness training.

Answe rs : 31–43

Mind fulness is based on trad itional Bu d d hist p ractice of being “in the m om ent” w ithou t ju d gem ents. 38. (D ) Patients w ho are su icid al characteristically have severe hopelessness and think abou t events in rigid , black-and -w hite w ays. They see very few op tions and becom e overw helm ed w ith their feelings of hop elessness w hich are exacerbated by seeing “no w ay ou t.” Begging the qu estion and circu lar reasoning are both logical fallacies. They occur w hen an id ea is “p roven” to be tru e w hen assu m ed im p licitly or exp licitly in the p rem ise (e.g., only w eak p eop le p u t them selves in m y position, therefore I am a w eak p erson). Personalization is one’s tend ency to relate events to oneself w ithou t any reason for d oing so. Selective abstraction is term from cognitive therapy w here d etails of a scenario are taken ou t of context and believed w hile everything else contrary abou t the scenario is ignored . 39. (B) Cognitive therap y is the p sychotherap y that w ould be m ost ap prop riate to help this p articu lar p atient solve problem s m ore p rod uctively. Behavioral therapy is focu sed on behaviors, not on the p ossible u nd erlying factors that cause behaviors. H ypnosis is a sleep-like state ind uced by a therapist, w hich has been of som e u se in u ncovering psychod ynam ic processes and in behavioral m od ification such as stopp ing sm oking. Interp ersonal therapy is aim ed at und erstand ing com m unication patterns and roles in relationship s. In p arad oxical therap y, d evelop ed by Bateson, the therapist su ggests that the patient engage in the behavior w ith negative connotations (e.g., a phobia or com pulsion). 40. (B) Storm ing is the second stage of Tu ckm an stages of group d evelopm ent w hich includ es p ow er stru ggle and bou nd ary testing; grou p m em bers m ay have internal conflict w ith each other or rebel against the facilitator. Form ing is the first stage in w hich grou p m em bers are oriented to the grou p and roles are form ed . N orm ing is the third stage in w hich grou p m em bers get to know each other better and resolve their d ifferences. Perform ing

139

is the next stage w here m em bers w ork hard to achieve goals. Ad journing is the final stage w here the w ork is com p lete and / or the grou p end s. 41. (E) System atic d esensitization is the process of exposing a patient to feared objects or situ ations and teaching them to d ecrease their anxiety and fear in ord er to overcom e them . System atic d esensitization is som etim es called grad u ated exp osu re therap y and can be u sed w ith m any typ es of anxiety d isord ers, inclu d ing social anxiety d isord er (p hobia) and sp ecific phobias. Aversion therapy is w hen a stim ulu s is paired w ith som ething negative or d iscom forting in ord er to get a patient to avoid or stop a certain m alad aptive behavior. Flood ing is w hen a p atient is exp osed to the m ost feared situ ation all at once. Mod eling is w hen observed behaviors are im itated . Su ggestion is w hen an id ea or behavioral change is consciou sly su ggested to a p atient in ord er to change the behavior. 42. (E) Su pportive psychotherap y attem p ts to fortify p sychological d efenses by provid ing em p athic reassu rance as op p osed to p robing into a p atient’s p sychological conflicts. Behavioral therap ies u tilize op erant cond itioning, relaxation training, and exposure techniqu es (am ong others) in ord er to change d ysfunctional behavior, m ostly in the treatm ent of anxiety d isord ers. Cognitive therapy is often com bined w ith behavioral therap y; it w as d evelop ed by Aaron Beck and is u sed for d ep ressive and anxiety d isord ers. It is based on the belief that the w ay that p atients think and act are d ue to cognitive schem as w hich d erive from child hood exp eriences and tem p eram ental factors. Dynam ic psychotherapy d elves into cu rrent and p ast conflicts in ord er to bring unconscious m aterial into consciousness. Existential psychotherap y is based on existential p hilosop hy and em p hasizes p ersonal feeling and exp erience over rational thinking. 43. (A) Biofeed back is the p rocess w hereby a p atient receives inform ation on physiologic variables in ord er to alter those variables and

140

4: P sycholog ica l Tre a tme nt a nd Ma nag e me nt

u ltim ately the corresp ond ing behavior. For exam p le, p atients w ith m igraine head aches obtain inform ation on m u scle tension or tem p erature. They w ill then use this inform ation to d ecrease the intensity of the head aches by d irectly trying to relax and alter the know n p aram eters. 44. (N ) Transference (and transference neu rosis) can be und erstood as a general phenom enon in w hich the p atient d isp laces onto others (inclu d ing the therapist) the feelings, thoughts, and conflicts from the patient’s past (p articu larly early child hood ). 45. (C) Flood ing is a techniqu e u sed in behavior therapy w here a patient is exp osed to a feared or anxiety-provoking stim u lus all at once to help him or her overcom e the fear or anxiety. It is u sed m ost frequ ently in sp ecific p hobias as in this case. 46. (F) Positive reinforcement is a behavioral technique w here a rew ard (a sw eet) is given after a specific behavior (saying “thank you”) in ord er to increase the frequency of that behavior. 47. (I) Punishm ent is presenting an aversive stim ulu s (soap in the m ou th) after a behavior (biting) in ord er to d ecrease the frequency of that behavior. Countertransference (B) is the com pilation of a therapist’s feelings tow ard a patient. H ypnosis (D ) is a sleep -like state ind u ced by a therap ist, w hich has been of som e u se in u ncovering u nconsciou s processes and in behavioral m od ification su ch as sm oking cessation. N egative reinforcem ent (E) is an operant cond itioning techniqu e w here an aversive stim u lu s is rem oved in ord er to increase the frequ ency of a d esired behavior. For exam p le, p u tting on you r seat belt so the car w arning stops beeping. Projection (G) is a d efense m echanism w here u nw anted feelings, attitu d es, and thou ghts are u nconsciou sly attribu ted to others. Projective id entification (H) is sim ilar to p rojection, bu t w ith the ad d i-

tion that the other ind ivid ual is ind u ced to behave in su ch a w ay as to confirm the p rojections. The real relationship (J) is consid ered to be the relationship betw een a p sychoanalyst and p atient w hich is not influ enced by transference. A schem a (K) refers to an organized p attern of thou ghts or behaviors relating to oneself, others, and the environm ent arou nd . Splitting (L) is another d efense m echanism characterized by view ing feelings, others, and situ ations in “all good or all bad ” term s. Therap eu tic alliance (M) refers to the healthy relationship betw een a p hysician/ therap ist and p atient. Und oing (O) is a d efense m echanism w here a p atient attem pts to “u nd o” an u nw anted thou ght or action (e.g., an obsession) by engaging in contrary behavior (e.g., a com pu lsion). 48. (B) Jean-Martin Charcot is known for his contributions to neurology, including the description of many medical diseases including Charcot– Marie tooth disease, amyotrophic lateral sclerosis (ALS), Charcot joint, Charcot aneurysms, and Charcot triad. He was also known for his descriptive work on hysteria and other neuroses. 49. (A) Josef Breu er is know n for his u se of the cathartic treatm ent or talking therap y in the fam ous case of Anna O, w hich w as su bsequ ently follow ed closely by Sigm u nd Freu d . 50. (E) H einz Kohu t is m ost w ell-know n for his d evelop m ent of self-psychology. It w as a very influ ential school of thou ght w ithin p sychoanalysis and help ed d irect m od ern ap p roaches to therap y, p articu larly concerning the treatm ent of narcissism . Erik Erikson (C) w as a p sychoanalyst w ho p osited eight stages of d evelopm ent, from infancy to old age. Sigm u nd Freu d (D ) w as the fou nd er of p sychoanalysis, best know n for his theories of the u nconsciou s m ind . Franz Anton Mesm er (F) is consid ered the father of m od ern hyp nosis; the w ord “m esm erism ” is d erived from his nam e.

CHAPTER 5

Le g al and Ethic al Is s ue s in Ps yc hiatry and Me dic ine Que s tions

DIRECTIONS (Questions 1 through 51): For each of the multiple-choice questions in this section, select the lettered answ er that is the one best response in each case. Questions 1 and 2 The case m anager for a 35-year-old m an that you are treating contacts you to d iscuss concerns regard ing a change in your p atient’s behaviors. You learn from the case m anager that the patient has been m aking p rovocative sexu al com m ents tow ard her over the past m onth. Du ring your session, the p atient exp resses concern that his case m anager is som ehow conspiring against him and he plans to d o som ething abou t it. On fu rther qu estioning, he becom es increasingly irritable and abru p tly storm s ou t of you r office. You hear him in the hallw ay exclaim ing, “That w om an, I’m going to stab her and she w on’t bother m e anym ore!” 1. You r first cou rse of action shou ld be w hich of the follow ing? (A) Attem p t to contact the p atient over the next several hou rs to d iscu ss the intent of his threat tow ard his case m anager. (B) Contact the p atient’s fam ily and let them know abou t his threats tow ard the case m anager. (C) Inform the p olice of the content of you r p reviou s sessions and his recent threat tow ard the case m anager. (D) N otify the case m anager of the p otential d anger.

(E) Resp ect the p atient’s confid entiality and w ait u ntil you r next sched u led ap p ointm ent w ith him to d iscuss his feelings. 2. The legal p reced ent that gu id es the ap p rop riate cou rse of action in this case is w hich of the follow ing? (A) Durham v. United States (B) O’Connor v. Donaldson (C) Rogers v. Commissioner of the Department of M ental Health (D) Rouse v. Cameron (E) Tarasoff v. Regents of University of California Questions 3 through 5 Tw o years ago, you w ere the anesthesiologist involved in a cesarean d elivery of a baby born w ith cerebral palsy. You had heard from your obstetrical colleagu e that the fam ily w as planning to sue, and tod ay a process server d elivers papers notifying you that the fam ily has brought an action against the p hysicians involved in the su rgery, inclu d ing you. 3. In m ed ical m alp ractice cases, the p laintiff m ust establish w hich of the follow ing? (A) Burd en of p roof beyond a reasonable d ou bt (B) Clear and convincing evid ence of w rongd oing (C) Crim inal intent (D) Crim inal m ischief (E) H arm or d am age 141

142

5: Le ga l a nd Ethica l Is s ue s in P sychia try a nd Me d ic ine

4. Which of the follow ing behaviors best p rotects a p hysician against a m alp ractice su it? (A) H aving the patient sign a “no harm ” contract. (B) H iring a m alp ractice attorney to exam ine the p hysician’s treatm ent p ractices. (C) Maintaining a good therap eu tic alliance w ith the p atient. (D) Refusing to prescribe m ed ications that m ay have ad verse effects. (E) Seeing the p atient in clinic on a w eekly basis. 5. When d eterm ining m ed ical m alpractice, the term “stand ard of care” refers to the use of m ed ical and p sychiatric treatm ents that are: (A) End orsed by the Am erican Med ical Association (B) Evid ence based (C) Safe and free from any p otential negative sid e effects (D) The m ost cu rrent treatm ents available (E) Used by average, reasonable p ractitioners in sim ilar circu m stances Questions 6 and 7 An 80-year-old w id ow w ith a h istory sign ificant for schizop hrenia w as recently d iagnosed w ith end -stage hep atic cancer. She is concerned abou t the d isp osition of her estate and d oes not w ant her fam ily to receive any of h er m on ey. Sh e p lans to d onate her entire estate to the local hu m ane society. 6. Which of the follow ing is m ost im p ortant in establishing a legally valid w ill? (A) (B) (C) (D) (E)

Conservatorship of estate Inform ed consent Lack of m ental illness Testam entary cap acity Witnesses at the signing of a w ill

7. The p atient’s fam ily hires an attorney to challenge the valid ity of her w ill. Which of the follow ing item s, if p resent, w ou ld u nd erm ine this patient’s com petence to m ake a w ill? (A) Inability to read and w rite. (B) Paranoid d elu sions regard ing the p atient’s fam ily. (C) Patient’s nond elu sional exp lanation of w hy she w ants to d onate her estate to the local hum ane society. (D) Presence of a d iagnosable p sychiatric d isord er. (E) Refu sal to u nd ergo treatm ent recom m end ed by her p rim ary care p hysician. Questions 8 and 9 A 23-year-old m an w ith no p rior p sychiatric history is charged w ith m u rd ering his next-d oor neighbor. H is friend s note that he becam e increasingly isolative and su sp iciou s of others in the w eeks p rior to the crim e. The exam ining psychiatrist reported that the m an harbored paranoid d elu sions regard ing his neighbor and noted that his thou ght p rocess w as too d isorganized to com p lete the exam ination. 8. Which of the follow ing legal stand ard s is the cu rrent basis for establishing an insanity d efense? (A) Am erican Law Institu te test (Mod el Penal Cod e) (B) Du rham ru le (C) Irresistible im p u lse ru le (D) M’N aghten ru le (E) Prod u ct ru le 9. Crim inal resp onsibility requ ires d em onstration of the crim inal act along w ith w hich of the follow ing elem ents? (A) (B) (C) (D) (E)

Dim inished m ental cap acity Mens rea Mod u s op erand i Psychiatric d isord er Witness to the act

Que s tions : 4–13

Questions 10 and 11 An obese 54-year-old w oman presents via ambulance to the emergency d epartment complaining of severe substernal chest pain lasting 40 minutes, profuse sw eating, and nausea. Her vital signs are blood pressure 195/ 96 mm H g, heart rate 63 beats/ min, respiratory rate 18 breaths/ min, temperature 98.8°F. An electrocardiogram reveals 3-mm ST-segment elevations in lead s V4, V5, and V6. Cardiac enzymes and laboratory w orkup are pend ing. You suspect a lateral w all myocardial infarction and recommend immed iate thrombolytic therapy. The patient exclaims, “You’re crazy if you think I’m going let some intern care for me! My family w ill d rive me across tow n to the private hospital!” The patient then jumps from the gurney and begins w alking tow ard the exit. 10. Which of the follow ing is the m ost ap prop riate next step? (A) Ad m it the patient to the card iology service on a physician em ergency certificate. (B) Allow the p atient to leave against m ed ical ad vice. (C) Detain the p atient u ntil the resu lts of the card iac enzym es are available. (D) Detain the p atient u ntil you can assess her ability to p rovid e inform ed consent. (E) Sed ate the p atient and begin throm bolytic therapy. 11. On further evalu ation, the patient d em onstrates a thorou gh u nd erstand ing of the inform ation you have given to her and ap p reciates the consequences of not being treated im m ed iately. The ap propriate next step is w hich of the follow ing? (A) Ad m it the p atient to the card iology service on a physician em ergency certificate. (B) Allow the p atient to leave against m ed ical ad vice. (C) Detain the p atient u ntil the resu lts of the card iac enzym es are available. (D) Detain the p atient so that her fam ily can m ake d ecisions on her behalf. (E) Sed ate the p atient and begin throm bolytic therap y.

143

12. A 19-year-old m an w ith a history significant for bip olar I d isord er is charged w ith assau lting a p olice officer. This is the m an’s fifth arrest in the p ast 4 m onths for aggression tow ard an au thority figu re. The assigned attorney has inform ed the forensic p sychiatrist involved in the case that the p atient has been noncooperative and belligerent w hile attem p ting to prepare for his d efense. The attorney also questions w hether the patient has an und erstand ing of the severity of his crim e given his acu te m ental state and lack of form al ed ucation. Which of the follow ing item s w ou ld be consistent w ith the assertion that the patient is incom p etent to stand trial? (A) Failu re to com p rehend the crim inal charges. (B) H istory of p rior assau lt charges. (C) Inability to p rovid e inform ed consent. (D) Inability to read and w rite. (E) Presence of a d iagnosable m ental illness. 13. A 35-year-old m an is brou ght to the em ergency room by his fam ily for threatening to shoot his m other in the chest. Urine toxicology is negative for cocaine, p hencyclid ine (PCP), or op ioid s, and his blood alcohol level is 0.04. You learn from the p atient’s fam ily that for several d ays he has been threatening to kill his m other and has even w ritten notes about how he w ou ld execu te his p lan. On m ental statu s exam ination (MSE), the p atient’s sp eech is lou d , p ressu red , and threatening tow ard the em ergency d ep artm ent staff. Fam ily history is significant for a bip olar sp ectru m d isord er. The patient states that he has not d one anything w rong and d em and s to be released im m ed iately. Which of the follow ing next steps w ou ld be the m ost appropriate? (A) Arrange for a p olice hold given the p atient’s p otential for violence. (B) Involu ntarily ad m it the p atient to an inp atient p sychiatric w ard . (C) Med icate the p atient w ith a m ood stabilizer and arrange for outpatient follow -u p .

144

5: Le ga l a nd Ethica l Is s ue s in P sychia try a nd Me d ic ine

(D) N otify the police abou t the patient’s hom icid al threats. (E) Warn the patient’s m other about the p atient’s threats. Questions 14 and 15 You are the psychiatrist on d uty w hen a man, arrested by the police for assaulting his w ife, is brought into the emergency room for an evaluation. The man’s behavior is w ild and unpredictable. H e refuses to answ er your questions and accuses you of having an affair w ith his w ife. At one point he picks up a chair and looks menacingly at a nurse before being subd ued by police officers. Detectives tell you that the man has been arrested in the past for aggression tow ard family members, and that the only chance of his avoid ing jail time that night is to be ad mitted to the psychiatric unit. H ow ever, the man makes it clear that he d oes not w ant to stay in the hospital. 14. Which of the follow ing shou ld ju stify the p hysician’s d ecision to involu ntarily ad m it the patient? (A) H istory of prior p sychiatric hosp italization. (B) Im m inent threat to others. (C) Med ication noncom p liance. (D) Presence of a d iagnosable m ental d isord er. (E) Refu sal to follow -u p w ith p sychiatric ou tpatient services. After being ad m itted to the psychiatric unit, the patient becom es increasingly aggressive tow ard staff. H e ad am antly refu ses all m ed ications offered to help calm his agitated behaviors. The patient at one point accu ses a staff m em ber of stealing m oney from his w allet and attem p ts to p u nch him . You place a stat ord er for a H ald ol 5 m g intram u scu lar (IM) injection to sed ate the p atient and p revent fu rther escalation of his com bative behaviors. 15. Ad m inistering the H ald ol w ithou t the p atient’s consent d irectly challenges w hich of the follow ing ethical principles? (A) Au tonom y (B) Beneficence

(C) Confid entiality (D) Ju stice (E) N onm aleficence 16. A 36-year-old woman with delusional disord er is brou ght to the em ergency room by p olice after she w as fou nd tresp assing on the Governor ’s p rivate estate. She claim s she has been having an affair w ith the Governor and d em and s that he acknow led ge he is the father of her 5-year-old son. This is the third tim e the p atient has been caught stalking the Governor. The patient is su bsequ ently ad m itted to the inp atient unit and d ischarged several d ays later to her p arents w ith w hom she lives. The Governor u ltim ately d ecid es to pu rsue charges against the w om an. The w om an’s attorney em p loys a forensic p sychiatrist to evalu ate the w om an and rend er an op inion abou t her m ental status at the tim e of the crim e. Which of the follow ing statem ents regard ing her incarceration is m ost accu rate if fou nd not guilty by reason of insanity? (A) She w ill not serve any tim e and w ill be released w ith close p sychiatric follow -u p . (B) She w ill serve half her crim inal sentence incarcerated and the other half in a locked p sychiatric facility. (C) She w ill serve her original crim inal sentence once she is d eem ed sane. (D) She w ill serve less tim e than if fou nd gu ilty. (E) She w ill serve m ore tim e in a locked p sychiatric u nit than she w ou ld in p rison if she w ere fou nd gu ilty. Questions 17 and 18 A 9-year-old girl is brou ght to the p ed iatric em ergency d ep artm ent by her p arents for evalu ation of a p ersistent cough. She is w ithd raw n and com plains of a “scratchy throat.” Vital signs are stable and the p atient is afebrile. On exam ination, the lungs are clear to auscultation bilaterally and the posterior oropharynx is clear. There are m ultiple bruises on her bu ttocks and back. A chest x-ray d em onstrates several rib fractu res in d ifferent stages of healing. H er p arents rep ort that their d au ghter is qu ite active

Que s tions : 14–21

w ith her you nger siblings and often gets into fights w ith them . The p atient agrees w ith her p arents. 17. Which of the follow ing is the m ost likely d iagnosis? (A) (B) (C) (D) (E)

Age-app ropriate “rou gh p lay” Major d ep ressive d isord er Physical abu se Sexu al abu se Som atic sym p tom d isord er

18. Which of the follow ing is the m ost ap p rop riate next step ? (A) Confront the siblings abou t their behavior. (B) Discharge the p atient w ith follow -u p in 1 w eek to reevalu ate her bru ises. (C) N otify Child Protective Services. (D) Refer for fam ily therapy to ad d ress the issu e of rou gh p lay. (E) Treat the p atient w ith p enicillin. Questions 19 and 20 A 17-year-old girl p resents to you r office com plaining of a bu rning vaginal d ischarge. She inform s you that she has been involved in a consensual sexu al relationship w ith her boyfriend and is w orried that she has contracted a sexu ally transm itted d isease (STD). You start the p atient on antibiotics. Three days later, you discover that the culture of her vaginal d ischarge grew N eisseria gonorrhoeae. 19. A few d ays later, you receive a call from the p atient’s p arents d em and ing to know w hy their d au ghter w as seen in you r office. Which of the follow ing is the m ost appropriate next step? (A) Alert the parents that their d au ghter is at risk for hu m an im m u nod eficiency virus (H IV) and shou ld be tested . (B) Ask the p arents abou t their d au ghter’s sexu al history. (C) Exp lain to the p arents that their d au ghter has contracted an STD and requ ires im m ed iate antibiotic treatm ent.

145

(D) Find ou t the boyfriend ’s nam e and telephone num ber to confirm the history. (E) Maintain confidentiality by disclosing no information, but encourage the patient to d iscuss this issue w ith her parents. 20. At the p atient’s next follow -u p visit, she requ ests to be tested for H IV. She rep orts that her boyfriend recently tested negative, bu t given her ad m itted p rom iscu ity, she w ond ers if she has contracted H IV. Which of the follow ing shou ld be d one p rior to ad m inistering the H IV test? (A) Directly inform the p atient’s boyfriend that he is at risk for H IV. (B) Exp lain to the p atient how the test is p erform ed and interp reted , along w ith inform ation on confid entiality and how you w ill p roceed if the resu lt is p ositive. (C) Inform the d ep artm ent of p u blic health of you r intent to ad m inister an H IV test. (D) N otify her p arents abou t the patient’s requ est for an H IV test. (E) Perform a p regnancy test. 21. A 44-year-old m an w ith insu lin-d ep end ent d iabetes m ellitu s and end -stage renal failu re has been on d ialysis for several years w hile aw aiting a kid ney transp lant. H e feels as thou gh he has “w aited long enough” and d oes not w ant to continu e “living tied to a m achine.” After several fam ily m eetings and consultations w ith other p hysicians, he inform s you that he no longer w ishes to be d ialyzed . You obtain a p sychiatric consu lt that conclu d es no evid ence of a m ood or thou ght d isord er. The fam ily is u p set w ith the p atient’s d ecision and d em and s that you continue to ad minister d ialysis u ntil a transplant is available. Which of the follow ing is the m ost ap p rop riate next step ? (A) Coerce the p atient into continu ing treatm ent. (B) Continu e d ialysis u ntil you convince the ethics com m ittee to su p p ort the fam ily’s d ecision. (C) Encou rage the fam ily to d issu ad e the p atient from w ithhold ing treatm ent.

146

5: Le ga l a nd Ethica l Is s ue s in P sychia try a nd Me d ic ine

(D) Resp ect the patient’s w ishes and d iscontinu e d ialysis. (E) Tell the patient that a transplant w ill arrive soon and encou rage him to rem ain in treatm ent. 22. A 40-year-old su rgical attend ing is ad m itted to the m ed ical u nit after d eveloping severe right flank pain. Fu rther w orkup confirm s a d iagnosis of nep hrolithiasis. One of the su rgical resid ents asks you abou t the attend ing’s cond ition in ord er that ad equ ate coverage can be arranged . Which of the follow ing is the m ost ap prop riate next step? (A) Arrange a conference betw een you r m ed ical attend ing and the su rgical hou se staff. (B) Inform the su rgical resid ent of the attend ing’s cond ition and length of stay. (C) Reveal only the estim ated length of stay. (D) Tell the su rgical resid ent that you w ill not say anything, bu t she cou ld take a look in the attend ing’s chart. (E) Tell the su rgical resid ent to ad d ress all concerns d irectly to the attend ing. 23. A 43-year-old m an is referred to you for continued treatm ent of d epression after his release from jail. The cou rt m and ated p sychiatric treatm ent w hile he is on p robation. The p atient’s probation officer calls you for inform ation regard ing his cond ition, p rogress, and treatm ent com pliance. Which of the follow ing is the m ost ap p rop riate resp onse? (A) Corresp ond w ith the p robation officer only through w ritten d ocum ents. (B) Discuss the case with the probation officer because the treatment is court mandated. (C) Ignore the requ est altogether becau se p sychiatric treatm ent bears no relation to law enforcem ent. (D) Lim it you r d iscussion w ith the p robation officer to only treatm ent com pliance because the rest of the inform ation is confid ential. (E) Obtain a confid entiality w aiver from the p atient before speaking to the probation officer.

24. A 23-year-old w om an w ith a know n history of heroin u se is ad m itted for intravenou s (IV) antibiotic treatm ent for infective end ocard itis. The nu rse inform s you that the p atient w as accid entally given the w rong antibiotic bu t has su ffered no ad verse reaction. Which of the follow ing is the m ost ap p rop riate next step ? (A) Encourage the p atient to seek legal action becau se a critical m istake has occu rred . (B) Enforce the “no harm , no d isclosu re” ru le. (C) First rep ort the m istake to the hosp ital ad visory com m ittee. (D) Inform the p atient that she w as given the w rong m ed ication. (E) Notify the patient’s family of the mistake. 25. A 38-year-old w om an is ad m itted to the oncology u nit w ith severe aplastic anem ia. She ap p ears p ale and w eak. Vital signs ind icate a blood p ressu re of 110/ 75 m m H g and a p u lse of 110/ m in. H em atocrit is 18%. On MSE, cognition is intact and there is no evid ence of a m ood or p sychotic d istu rbance. The p atient states that she is a Jehovah’s Witness and refu ses any blood transfu sion on the basis of her religiou s beliefs. Which of the follow ing is the most appropriate next step in the treatment of this patient? (A) Ad m inister p acked red blood cells. (B) Exp lain the im p lications of no treatm ent bu t resp ect the p atient’s refu sal for treatm ent. (C) Persu ad e the p atient that she m u st accep t the transfu sion. (D) Refer the case to the hosp ital’s ethics com m ittee. (E) Refer the p atient for involu ntary p sychiatric treatm ent based on her life-threatening d ecision. 26. The p arents of a new born w ith Dow n synd rom e find their d au ghter to be lethargic and m inim ally resp onsive. Med ical evalu ation is significant for the follow ing cerebrospinal flu id find ings: opening pressu re of 100 m m H g, w hite blood cell cou nt of 5,000/ µL

Que s tions : 22–29

(p red om inantly neu trop hils), protein m ore than 40 m g/ d L, glu cose content m ore than 40 m g/ d L, and Gram stain p ositive for bacteria. You suspect grou p B streptococcal m eningitis and recom m end IV antibiotic therap y. The p arents feel as thou gh their child w ill u ltim ately have a p oor quality of life and requ est that treatm ent be w ithheld . Which of the follow ing is the m ost app rop riate next step ? (A) Refer the case to the ethics com m ittee for review at their next sched u led m eeting. (B) Rep eat the lum bar punctu re to verify the d iagnosis. (C) Resp ect the p arents’ w ishes becau se they are the prim ary d ecision m akers. (D) Start intravenous antibiotics against the p arents’ w ishes. (E) Threaten to rep ort the p arents to child p rotective services unless they change their d ecision. 27. You are a p sychiatrist w ho hosts a m orning rad io show d ed icated to ed u cating the general p u blic abou t m ental illness. Du ring a qu estion and answ er segm ent, a caller p hones in to ask abou t you r op inion regard ing a prom inent politician w hose recent erratic behaviors have gained significant m ed ia attention. The caller asks you d irectly w hether you believe the p olitician has a bipolar sp ectru m d isord er. Which of the follow ing resp onses is the m ost ap prop riate? (A) Com m ent in w ritten form only. (B) Offering a p sychiatric d iagnosis in su ch instances is u nethical. (C) Provid e an “off-the-record ” or anonym ou s opinion. (D) State you r d iagnosis bu t ind icate that other problem s m ay account for the sym p tom s. (E) Stating you r op inion p u blicly is legitim ate as long as the p olitician is not you r p atient. 28. An 84-year-old w id ow ed m an w ith severe m ajor neu rocognitive d isord er (d em entia), for w hom you have been the p rim ary care

147

p hysician for years, is d iagnosed w ith hepatocellu lar carcinom a. The patient d oes not have an ad vance d irective and never d esignated a p ow er of attorney. Psychiatric consultation conclud es that the patient is unable to m ake an inform ed d ecision regard ing treatm ent options, and you tu rn to his fam ily for gu id ance. The p atient’s old est son is ad am ant that his father receive chem otherapy, w hile the tw o you nger d au ghters feel that he shou ld not su ffer the ad verse effects of chem otherapy “especially becau se he’s so d em ented .” Which of the follow ing is the next appropriate step? (A) Abid e by the d aughters’ w ishes becau se the patient’s quality of life is already poor. (B) Abid e by the son’s w ishes becau se he is the old est. (C) Ask the p atient w hich fam ily m em ber he w ou ld like to d esignate p ow er of attorney. (D) Consu lt the hosp ital’s ethics com m ittee. (E) Let the p atient d ecid e w hether or not to p roceed w ith treatm ent. 29. A 27-year-old w om an w ho is 3 m onths postp artu m is brou ght to the em ergency room by her hu sband and m other w ith concerns that the patient “is not acting like herself.” The hu sband inform s the consu lting p sychiatrist that for the past few w eeks the p atient has been increasingly irritable, w ithd raw n, and crying alm ost d aily. H e d oesn’t u nd erstand w hy the p atient is behaving like this given that the p regnancy w as planned and the p atient w as looking forw ard to having a fam ily. The p atient tearfu lly ad m its that this is her first child and that she is overw helm ed w ith the responsibilities of being a new m other. On fu rther qu estioning, she hesitantly confesses to intru sive thou ghts of su ffocating her child and som etim es w orries abou t being alone w ith the baby. She d enies su icid al id eation or aud itory/ visu al hallu cinations. The p atient is w illing to seek ou tp atient treatm ent bu t ad am antly refu ses volu ntary ad m ission. H er m other states that the p atient is “ju st exhau sted ” and that “everything w ill be fine after she gets som e rest.” Which of the follow ing is the m ost ap p rop riate next step ?

148

5: Le ga l a nd Ethica l Is s ue s in P sychia try a nd Me d ic ine

(A) Ad m inister a stat d ose of lorazep am 2 m g IM and reevalu ate w hen the p atient is m ore calm . (B) Ad m it the patient involuntarily. (C) Call Child Protective Services becau se the infant is at risk of harm . (D) Discharge the p atient and inform the hu sband and m other to bring the patient back to the hosp ital if her sym ptom s d o not im prove in the next few w eeks. (E) Give the p atient a 2-w eek sup ply of flu oxetine to treat her d ep ression and then arrange ou tpatient follow -up. 30. A physician is at a comm unity fair w ith her spouse w hen a patient approaches the p sychiatrist to say hello. The spou se d oes not recognize the patient. The patient d oes not introd u ce him self to the sp ou se, nor d oes the physician acquaint the tw o. After a brief conversation, the patient politely excu ses him self and leaves. On the w ay hom e, the spouse asks, “Who w as that man you w ere talking to earlier?” The physician should d o w hich of the follow ing? (A) Answ er the sp ou se’s qu estion tru thfu lly. (B) Ask the sp ou se to gu ess the id entity of the person. (C) Inform the sp ou se that he m u st first p rom ise not to reveal the id entity of the p atient before answ ering the qu estion. (D) Inform the sp ou se that revealing su ch inform ation w ou ld com prom ise confid entiality. (E) Lie to the sp ou se to p rotect the id entity of the patient. Questions 31 and 32 You have been treating a fem ale patient w ho has been seeing you for p sychod ynam ic p sychotherap y for app roxim ately 6 m onths. N ear the end of the su ggested cou rse of treatm ent, the p atient reports that she feels m arked ly better abou t her progress and attributes her im p rovem ent to you r exp ertise. Prior to the last session, she confesses that she has alw ays found you attractive and that she w ou ld like begin an intim ate relationship w ith you . You feel

flattered by the p atient’s sexu al interest and are su rp rised by you r ow n interest in the patient. 31. Which of the follow ing is the most appropriate cou rse of action? (A) Decline p articip ation in the relationship becau se sex w ith a form er p sychiatric p atient is u nethical. (B) Engage in sexu al relations becau se sex w ith a cu rrent or form er p sychiatric p atient is ethical. (C) Engage in sexu al relations becau se sex w ith a form er p sychiatric p atient is p erm issible only if you d o not exp loit you r p ast p osition of au thority. (D) Engage in sexu al relations becau se there is no established p rofessional cod e of ethics regard ing sex w ith p sychiatric p atients. (E) Inform the p atient that the p rofessional cod e of ethics requ ires that you w ait 1 year after term ination before you can ethically engage in sexu al relations. 32. Which of the follow ing w ou ld be the m ost ap p rop riate cou rse of action if the above p atient w ere you r m ed ical or su rgical p atient? (A) Decline particip ation in the relationship becau se sex w ith a cu rrent p atient is u nethical. (B) Engage in sexu al relations becau se sex w ith a cu rrent nonp sychiatric p atient is ethical. (C) Engage in sexu al relations becau se sex w ith a cu rrent nonp sychiatric p atient is p erm issible only if you d o not exp loit you r p ast p osition of au thority. (D) Engage in sexu al relations becau se there is no established p rofessional cod e of ethics regard ing sex w ith nonp sychiatric p atients. (E) Inform the p atient that the p rofessional cod e of ethics requires that you w ait 1 year after term ination before you can ethically engage in sexu al relations. 33. A 52-year-old man, for w hom you have been the primary care physician for the last 20 years,

Que s tions : 30–36

w as recently diagnosed w ith amyotrophic lateral sclerosis. The disease has rapid ly progressed and he has experienced multiple respiratory complications that likely w ill require a tracheotomy. Severe muscle w eakness and atrophy are apparent in all limbs. The patient states that there is no meaning in continuing w ithout his physical capacities. H e asks for your help in ending his life in a humane and d ignified manner. H is MSE is unremarkable and there is no evidence of any psychiatric d isorder. You d iscuss the patient’s request w ith his family and they unanimously support his d esire to “end the suffering.” Which of the follow ing w ould be the most appropriate course of action? (A) Ignore the p atient’s requ est. (B) Provid e the p atient w ith inform ation regard ing how to effectively end his life. (C) Provid e the p atient w ith enou gh m ed ication refills to p rovid e a lethal d ose. (D) Refu se to p articip ate in assisting the p atient w ith su icid e and focu s on resp ond ing to the p atient’s end -of-life issu es. (E) Resp ect the p atient’s w ishes by help ing him end his life in a p ainless and resp ectfu l m anner. 34. You are an inp atient p sychiatrist treating a p atient w ith bip olar I d isord er. The p atient has a long history of m ed ication noncom p liance resu lting in severe, persecu tory d elu sions d u ring his m anic ep isod es. After 1 w eek of treatm ent, you receive a p hone call from hospital ad m inistration inform ing you that the p atient’s insu rance w ill not cover the cost of an ad d itional inpatient stay. You are encou raged to d ischarge the p atient so that the hosp ital w ill not have to incu r these costs. You feel, how ever, that the p atient requ ires m ore tim e on the inpatient u nit because of safety concerns. Which of the follow ing actions w ou ld be the m ost ap p rop riate? (A) Contact the insu rance com pany w ithou t the patient’s perm ission and request coverage for ad d itional d ays.

149

(B) Ensu re that the p atient is established w ith a casew orker to su p ervise the p atient in the com m unity before d ischarge. (C) Establish follow -u p at the p atient’s outpatient com m u nity m ental health facility p rior to d ischarge. (D) Keep the p atient on the u nit as long as it is m ed ically necessary. (E) Sp eak w ith the inp atient social w orker to d eterm ine w hether the patient is eligible for a loan. Questions 35 and 36 You receive a subp oena from an attorney rep resenting a p arty that has filed a law su it against one of you r p atients. The su bp oena p ertains to releasing the m ed ical record s of you r patient. 35. Which of the follow ing is the m ost ap p rop riate next step ? (A) Contact the attorney w ho obtained the su bpoena to d iscu ss the p rocess of releasing the m ed ical inform ation. (B) Do not release the inform ation and contact your patient regard ing the su bpoena. (C) Release the m ed ical record s d irectly to the presid ing jud ge. (D) Release the m ed ical record s u p on receiving the su bp oena. (E) Requ est that the p atient sign a release of inform ation form and tu rn over the record s. 36. A cou rt hearing has been organized by you r p atient’s attorney to qu ash the su bpoena you have been issu ed . At the hearing, the ju d ge rules that you should release the entire m ed ical record even thou gh your patient has not consented to the release of information. Which of the follow ing w ould be the most appropriate next step? (A) Ap p eal to the state’s Sup rem e Cou rt to block the release of the m ed ical record . (B) Refu se to d isclose the p atient’s m ed ical record regard less of the cou rt ord ers.

150

5: Le ga l a nd Ethica l Is s ue s in P sychia try a nd Me d ic ine

(C) Release only inform ation that w ill not incrim inate you r p atient. (D) Release the com p lete m ed ical record to the jud ge. (E) Work ou t a p lan of legal action w ith you r p atient. 37. A cou rt-ap pointed forensic psychiatrist is evalu ating a m an charged w ith arm ed robbery to d eterm ine w hether he is com petent to stand trial. Du ring the interview, the d efend ant confesses to m u rd ering a w om an 3 years ago and hid ing her bod y in an u nd isclosed area. Which of the follow ing actions w ou ld be the m ost appropriate? (A) Avoid d etails of the d efend ant’s prior crim inal history in the report. (B) Encou rage the d efend ant to sp eak w ith his attorney about the m u rd er. (C) Im m ed iately notify au thorities of the d efend ant’s confession. (D) Inclu d e this d etail in the final rep ort. (E) Withd raw from the case. 38. A 28-year-old w om an w ith m ajor d ep ressive d isord er has been seeing you for w eekly psychotherap y and has failed to p ay her bill for 2 m onths. Which of the follow ing is the m ost ap p rop riate next step ? (A) Contact the patient’s family to d etermine if the patient is financially stable. (B) Contract w ith a billing collector to d em and im m ed iate p aym ent. (C) Inform the p atient that you w ill not see her if she d oesn’t pay for her treatm ent. (D) Inqu ire as to the reasons she has been avoid ing p aym ent at the p atient’s next visit. (E) Term inate the treatm ent. 39. A wealthy 46-year-old male banker is in psychotherapy with you for treatment of a single episode of major depression. After significant improvement in his symptoms, he offers you the opportunity to take part in one of his financial ventures. The investment appears to be sound and fairly lucrative. Which of the following is the most appropriate response to the banker?

(A) “Becau se you r d ep ression is im p roved , it w ou ld be ap p rop riate for u s to be bu siness p artners.” (B) “I can invest w ith you only w hen ou r treatm ent is nearing its end .” (C) “I have to d ecline; it p otentially m ay interfere w ith ou r treatm ent relationship .” (D) “It is p robably a bad id ea; I’m alread y com m itted in other investm ents.” (E) “Thank you for thinking abou t m e. I’d be honored to invest w ith you .” Questions 40 and 41 A 23-year-old Cau casian Catholic w om an w ith a history of m ajor d epressive d isord er and m u ltiple m ed ical illnesses p resents to your office after argu ing w ith her hu sband . She is a stay-at-hom e m other of fou r child ren and ad m its that she is u nable to w ork becau se of her m ed ical lim itations. She rep orts that her hu sband d oes not allow her to visit w ith friend s or fam ily w ithou t his consent and generally d oes not help w ith child care or other household d u ties and obligations. As a result, she is becom ing increasingly resentfu l, d ep ressed , and isolative. Du ring the session, she ad m its to thou ghts of killing herself. Of note, she has a p ast history of an overd ose attem pt w ith aspirin. 40. Which of the follow ing is her strongest risk factor for su icid e? (A) (B) (C) (D) (E)

Age Gend er Marital statu s Past history of su icid e attem p t Term inal m ed ical illness

41. Which one of the follow ing, if d ocu m ented , w ould mostly likely legally protect a physician in the event of a patient suicid e? (A) A w ritten “no self-harm ” contract signed by the p atient. (B) An assessm ent of su icid e risk and p rotective factors. (C) The p atient’s m issed ap p ointm ents. (D) The p atient’s refu sal to consid er p harm acological treatm ents.

Que s tions : 37–45

(E) The patient’s verbal p rom ise to seek m ed ical attention if feeling su icid al. 42. A 36-year-old m an w ith a history of bip olar I d isord er is brought to the em ergency d epartm ent by p olice after stabbing a patron in a bar room braw l. H is blood alcohol level w as 0.320 u pon arrival, and the p atient requ ired intram u scu lar (IM) H ald ol for agitation. The p atient has no recollection of the event, and the victim d ied 2 hours later. The p atient has a history of assau lt w hile being m anic. H e cu rrently p articip ates in w eekly p sychotherap y sessions and attend s his m ed ication m anagem ent ap pointm ents every m onth. H is law yer has chosen to assert an insanity p lea in d efense of the patient. Which of the follow ing factors is m ost likely to u nd erm ine his assertion of the insanity d efense? (A) (B) (C) (D) (E)

Inability to recall the event. Mental d isease or d efect. Prior history of assau lt. Voluntary intoxication. Violent nature of the crim e.

43. You are a forensic p sychiatrist hired as an exp ert w itness by the d efense attorney of a m entally ill p atient charged w ith crim inal m iscond u ct. The attorney is seeking you r help in convincing the ju ry that the patient w as m entally ill at the tim e of the crim e. Which of the follow ing is your prim ary resp onsibility as an exp ert w itness? (A) Cou ntering evid ence of crim inal resp onsibility. (B) Establishing reasonable d ou bt. (C) Evalu ation, d iagnosis, and initiation of treatm ent of the accu sed . (D) Obtaining a not guilty by reason of insanity verd ict. (E) Rend ering an opinion based on reasonable m ed ical certainty. Questions 44 and 45 A 48-year-old m an is involu ntarily ad m itted to the hosp ital after an acute m anic episod e. The patient is hyperactive, d em and ing, and increasingly talk-

151

ative on the u nit. H e becom es intru sive, not able to be red irected , and d em and s im m ed iate release. You exp lain to him that you feel he is gravely d isabled and u nable to care for him self. H e d isagrees w ith you and d em and s “d u e p rocess.” 44. On w hich of the follow ing legal princip les is the patient’s request for a hearing based ? (A) (B) (C) (D) (E)

Actu s reu s H abeas corp us Mens rea Parens p atriae Rights u nd er the Fou rth Am end m ent

45. The cou rt agrees that the p atient is severely d isabled and in need of acu te m ed ical m anagem ent. You initiate treatm ent w ith qu etiapine and his sym ptom s of m ania m arked ly d im inish. The p atient requ ests im m ed iate d ischarge and agrees to follow u p w ith a p artial hospitalization program . You feel that he w ou ld benefit from fu rther inp atient treatm ent bu t is no longer gravely d isabled or a threat for selfharm . Which of the follow ing is the next m ost ap p rop riate step ? (A) Ap pease the patient by increasing sm oking privileges. (B) File for another cou rt hearing to d etain the patient fu rther. (C) Ignore the p atient’s requ est becau se he has been com m itted by the court. (D) Persu ad e the patient to stay for a few m ore d ays. (E) Release the patient to the partial hosp italization p rogram . Questions 46 and 47 You are consu lted by the m ed ical team to evalu ate a w om an on the m ed ical u nit w ho su ffers from a m ajor neurocognitive d isord er (d em entia). The internist believes that she requ ires a central line for flu id s and m ed ication, bu t is u nsu re if she is able to fu lly com p rehend the risks and benefits of the p roced ure. The team is requ esting help in d eterm ining her capacity to give inform ed consent.

152

5: Le ga l a nd Ethica l Is s ue s in P sychia try a nd Me d ic ine

46. Which of the follow ing com ponents is the m ost im portant in obtaining inform ed consent in this case? (A) Ability to read and w rite. (B) Absence of m ental illness. (C) Involving fam ily m em bers in this d iscu ssion. (D) Petitioning a court to establish the p atient’s com petence. (E) Raising alternative treatm ent op tions. 47. The fam ily is su bsequ ently contacted abou t her cond ition, and they requ est a m eeting w ith the treatm ent team . Du ring the m eeting, the patient’s eld est son notifies you that the p atient d id create a living w ill ap proxim ately 1 year ago, but they are unsure whether it would be useful. Which of the follow ing w ould you tell them best d escribes the pu rpose of a living w ill? (A) Absolves personal resp onsibility. (B) Arranges for fu neral services and d istribu ting her estate. (C) Establishes p ersonal p references regard ing end -of-life issu es. (D) Prevents the p atient from changing her m ind about life su pport if she becom es term inally ill. (E) Requ ests p hysician-assisted su icid e if she becom es term inally ill. 48. A 28-year-old m an involved in a m otor vehicle accid ent brings a law su it against the d river. Em ergency d ep artm ent record s d o not show any p hysical inju ries, but the p atient is claim ing to su ffer from p osttrau m atic stress d isord er (PTSD). You are asked to evalu ate the p atient’s sym p tom s. H e com p lains of d istressing d ream s of the accid ent, having flashbacks w hile aw ake, and avoid ing the intersection w here the accid ent occu rred . H is sym p tom s have lasted for 3 m onths. Past psychiatric history is significant for m ajor d epressive d isord er, im p u lsivity, and violent behavior. H e has been incarcerated nu m erou s tim es, show ing a blatant d isregard for the law. On MSE, the p atient is likable and coop erative. H is m ood is rep orted as being d ep ressed and he cu rrently

d enies any hallu cinations or d elu sions. Which of the following items from your evaluation of the patient most raises your suspicion regarding a d iagnosis of m alingering? (A) (B) (C) (D) (E)

Du ration of sym p tom s. Flashbacks w hile aw ake. H istory of m ajor d ep ressive d isord er. Lack of physical inju ries. Prior incarcerations.

49. One of you r p atients consistently m isses ap p ointm ents w ithou t giving you ad vance notice. After num erou s failed attem p ts at resolving this issu e w ith the patient, you realize that the p atient’s behavior is not changing and you d ecid e to d ischarge the p atient from you r care. Which of the follow ing m ost ap p rop riately d escribes w hat you shou ld d o? (A) Contact the patient’s fam ily/ friend s to requ est their help in im p roving the p atient’s attend ance. (B) N otify the p atient’s insu rance com p any that she is d ischarged from you r care. (C) N otify you r staff that the p atient is not to be given fu rther appointm ents. (D) Write a letter to the p atient notifying her that she has been d ischarged effective im m ed iately d u e to her failu re to com ply w ith treatm ent. (E) Write a letter to the p atient stating that she w ill be d ischarged in 60 d ays d ue to her failu re to com p ly w ith treatm ent. 50. You are asked by the cou rt to p rovid e a forensic evaluation for a child cu stod y case. The child had been living w ith his biological m other and stepfather since birth. Recently, the child ’s stepfather passed aw ay from lu ng cancer leaving the u nem ployed m other alone to raise the child . The child ’s biological father, w ho is a renow ned orthop ed ic su rgeon in the com m unity, is requ esting full cu stod y becau se he asserts that he is better able to financially su pport the child . The child ’s m other refu ses to relinquish cu stod y d u e to concerns that leaving her hou sehold w ill em otionally im p act the child . She also im p lies that the father has “anger p roblem s” and voices su sp icion as to

Que s tions : 46–51

w hether he is, in fact, the biological father. Which of the follow ing factors w ou ld be the m ost im p ortant in d eterm ining w hich p arent shou ld get custod y? (A) Who can p rovid e for the best interests of the child ? (B) Who has the highest level of ed u cation? (C) Who is in the best m ental and p hysical health? (D) Who is the m ost financially stable? (E) Who is the biological p arent? 51. You are an internal m ed icine resid ent w ho has been assigned to rotate throu gh the inp atient m ed ical service for the next 2 m onths. Shortly after you begin, you notice that one of you r colleagu es consistently com es to w ork late and sm ells of alcohol. Som etim e later, you notice

153

that your colleagu e has been m aking increasingly careless m istakes and m issing im p ortant m eetings and d iscu ssions. Occasionally, you observe that his hand s shake w hen he is d ocu m enting his notes or hold ing his charts. You are concerned that he m ay have an alcohol use d isord er. Which of the follow ing is the m ost ap p rop riate cou rse of action? (A) Confront the colleagu e and d em and that he seek ad d iction cou nseling services. (B) Contact the Dru g Enforcem ent Ad m inistration to rescind his license. (C) Do nothing so as to avoid p ersonal liability. (D) N otify the hosp ital’s com m ittee for im p aired p hysicians. (E) N otify the local p olice.

Ans we rs a nd Expla na tions

1. (D ) A physician or therap ist has a d uty to p rotect id entifiable victim s from im m inent d anger. This obligation m ay be fu lfilled by d irectly contacting the p arty at risk, notifying the p olice, or taking other ap p rop riate m easu res to p rotect the victim . Contacting the p atient’s fam ily fails to protect the intend ed victim . Although notification of the police is an accep table intervention in this case, d isclosing u nrelated confid ential inform ation abou t the p atient is inap prop riate. When notifying third p arties, care should be taken to release only inform ation necessary to p rotect the p otential victim . The rem aining choices fail to protect the potential victim .

w arn.” The California Su p rem e Cou rt revised its p reviou s hold ing in Tarasoff II (1982) w ith a “d u ty to p rotect.” Durham v. United States held that an ind ivid u al “is not crim inally resp onsible if the u nlaw fu l act w as the p rod u ct of m ental d isease or m ental d efect.” The d ecision resu lted in the p rod u ct ru le of crim inal responsibility (insanity d efense) and u ltim ately w as rep laced by the Am erican Law Institu te stand ard . In Rogers v. Commissioner of the Department of M ental Health, the court exp ou nd ed op inions regard ing the treatm ent of involu ntarily com m itted patients. O’Connor v. Donaldson and Rouse v. Cameron relate to involu ntary hosp italization.

2. (E) Tarasoff v. Regents of University of California is the land m ark legal p reced ent establishing liability to third p arties. It ru led that m ental health p rofessionals have the d uty to p rotect id entifiable, end angered third p arties from im m inent threats of seriou s harm m ad e by their ou tpatients. Therefore, the physician m ay ethically break confid entiality in cases involving im m inent harm to others. In the Tarasoff case, a stud ent at the University of California (Prosenjit Pod d ar) exp ressed intentions to kill a fellow stu d ent (Tatiana Tarasoff) to his therapist. Pod d ar tragically follow ed throu gh on his threat and m u rd ered the you ng w om an at her hom e. Tarasoff v. Regents of University of California hold s legal preced ent only in the state of California; how ever, the case has been ad op ted by p hysicians/ therapists as the stand ard of care and has led to nationw id e legislative changes. There have been tw o ru lings regard ing the Tarasoff case. The original d ecision, Tarasoff I (1976), established a “d u ty to

3. (E) The fou r Ds of establishing m ed ical m alp ractice cases involve proving that a deviation of du ty directly cau sed a dam age. The plaintiff m ust establish the presence of a fid u ciary or d octor–patient relationship (a d u ty), negligence (d eviation from stand ard of care), and d am ages d irectly cau sed by that negligence. Typ ically, the bu rd en of p roof in m ed ical m alpractice cases (civil cases) is by a p rep ond erance of evid ence (greater than 50%). Proof beyond a reasonable d ou bt refers to crim inal cases. Clear and convincing evid ence is reserved for sp ecial cases d ecid ed by the cou rt. N either crim inal intent nor crim inal m ischief is requ ired to p rove m ed ical m alp ractice.

154

4. (C) As a broad generalization, a good p atient– d octor relationship is key for a p hysician to potentially avoid or d eflect a law su it. It is the physician’s d u ty as a health care p rovid er to facilitate op en com m u nication regard ing the patient’s illness, treatm ent recom m end ations,

Answe rs : 1–8

risks versu s benefits of su ch treatm ents, and to acknow led ge any errors that the physician m ay have m ad e. Requ iring that a p atient sign a “no harm ” contract is not reasonable nor d oes it stand u p as a legal d ocum ent in a court of law . H iring a m alpractice attorney, seeing the p atient on a m ore frequ ent interval, or avoid ing prescribing m ed ications w ith p otentially ad verse sid e effects d o not d ecrease a p hysician’s risk for m alp ractice su its.

155

5. (E) The standard of care for determining medical malpractice is based on how a similarly qualified practitioner w ould have performed und er the same or similar situation. Usually the testimony of an expert w itness (a physician qualified by evidence of his/ her expertise, training, and special know ledge to provid e an opinion about the case) is necessary to d emonstrate the stand ard of care in med ical malpractice suits. Treatments that are endorsed by the American Med ical Association, evid ence based , free from ad verse effects, or the most current are not sufficient to establish the stand ard of care.

7. (B) Com p etence is d eterm ined by an ind ivid u al’s ability to m ake d ecisions in accord ance w ith one’s ow n goals, concerns, and valu es. Althou gh p hysicians often rend er opinions on an ind ivid ual’s com petence, only a ju d ge’s ru ling d eterm ines w hether an ind ivid u al is com petent or incom petent. The p resence of p aranoid d elu sions regard ing the p atient’s fam ily qu estions her ability to m ake rational d ecisions, and significantly u nd erm ines her assertion that she w as com p etent to m ake a w ill. The inability to read and w rite, the p resence of a d iagnosable psychiatric d isord er, and the refu sal to u nd ergo treatm ent recom m end ed by her prim ary care physician d o not au tom atically u nd erm ine a p atient’s com p etence to m ake a w ill as long as the criteria for testam entary cap acity are m et. An exp lanation of w hy the p atient w ants to d onate her estate to the local hu m ane society d oes not u nd erm ine the p atient’s com p etence to m ake a w ill u nless su ch an exp lanation d em onstrates that the p atient is u nable to m eet any of the sp ecified criteria.

6. (D ) Testam entary capacity refers to ind ivid u al’s com p etence to m ake a legally valid w ill. Case law and statu tes d iffer across states. H ow ever, the central elem ents of testam entary cap acity inclu d e: (1) com p rehension of the act of w riting or signing a w ill, (2) know led ge of p otential heirs, and (3) u nd erstand ing the extent of one’s assets or p rop erty. Conservatorship of estate refers to an ind ivid u al’s ability to m anage his or her finances. Inform ed consent is the p rocess of obtaining p erm ission for m ed ical treatm ent. Im p ortant elem ents inclu d e the ability to m ake a volu ntary d ecision regard ing treatm ent, exp lanation of risks and benefits, and cap acity to m ake an inform ed d ecision. The p resence or absence of a m ental illness d oes not necessarily im p air an ind ivid u al’s com p etency to m ake a w ill. Althou gh w itnesses are necessary at the signing of a w ill in ord er for the w ill to be enforceable in a cou rt of law , the w itness m u st first ensu re that the ind ivid u al has testam entary cap acity in the event the w ill is challenged and the w itness is asked to attest to the ind ivid u al’s com p etence.

8. (A) The Am erican Law Institu te test is the m od ern stand ard for the insanity d efense. It hold s that a person w ith a m ental d isease or d efect shou ld not be held crim inally resp onsible for an act if the p erson (1) cou ld not app reciate the w rongfu lness of the cond uct and (2) cou ld not conform the cond uct to the requirem ents of the law . The Am erican Law Institu te test expand ed u p on the 1843 M’N aghten rule, w hich held that an ind ivid u al w as not gu ilty by reason of insanity if, as a resu lt of a m ental d efect, the p erson w as u naw are of the natu re, qu ality, and consequence of his or her act or d id not know that the act w as w rong (“rightw rong test”). The Du rham ru le arose from a 1954 District of Colu m bia case in w hich the cou rt held that “an accu sed is not crim inally resp onsible if his u nlaw fu l act w as the p rod u ct of a m ental d isease or m ental d efect.” This is also know n as the p rod u ct ru le of crim inal resp onsibility. The irresistible im p u lse ru le is an antiqu ated stand ard for p roving an insanity d efense. Established in England in 1922, the irresistible im p ulse rule states that an ind ivid u al is not resp onsible for a crim inal act if

156

5: Le ga l a nd Ethica l Is s ue s in P sychia try a nd Me d ic ine

he w as unable to resist that act d u e to a m ental illness. 9. (B) Crim inal resp onsibility requires (1) crim inal intent (m ens rea) and (2) a crim inal act (actu s reu s). H aving either d im inished m ental capacity or a know n psychiatric d isord er can serve to avoid crim inal responsibility, bu t are not required to p rove crim inal resp onsibility. Mod u s operand i is the pattern in w hich an action or crim e is execu ted by a p erson. A w itness to the crim e also is not requ ired to p rove crim inal responsibility. 10. (D ) Before discharging or transferring the patient, it is important to determine whether she can make an informed decision regarding her care. Informed consent requires that the patient (1) comprehend the issues and/ or choices provided by the physician, (2) appreciate the consequences of making a certain decision, and (3) arrive at a voluntary decision after weighing the facts. At this point in the vignette, there is not enough information to establish the patient’s ability to make an informed decision regard ing her care. Exceptions to informed consent include incompetence, emergency situations (e.g., comatose or unresponsive patient), a competent patient’s waiver of informed consent, and therapeutic privilege. The other choices are incorrect because this patient’s ability to make an informed decision has not been assessed. 11. (B) A com p etent patient has the right to refu se treatm ent, even if the consequ ences are life threatening. In the extension of this vignette, the patient has ad equately d em onstrated the elem ents of an inform ed d ecision. The other choices are incorrect becau se they w ould p revent a com petent patient from m aking an inform ed d ecision in her treatm ent. 12. (A) In Dusky v. United States (1960), the Su p rem e Cou rt ru led that com petency to stand trial requ ires the crim inal d efend ant to be able to (1) rationally consu lt w ith one’s attorney and (2) rationally and factu ally u nd erstand the p roceed ings against him . A history of p rior assau lt charges or illiteracy d oes not constitu te grou nd s for incom p etency. Inform ed consent

is im p ortant in m aking d ecisions related to m ed ical treatm ents bu t has little bearing on one’s com petency to stand trial. A person can suffer from a m ental illness and be com petent to stand trial. 13. (B) In volu n tary h osp italization is u sed w h en a p h ysician believes th at an in d ivid u al is an im m in en t th reat to th em selves or oth ers, or is oth erw ise u n able to ap p rop riately care for th em selves. Arran gin g a p olice h old is in correct as p olice h old s are issu ed w h en an in d ivid u al w h o is arrested requ ires a p sych iatric evalu ation . If th e arrested in d ivid u al d id n ot requ ire in p atien t p sych iatric care, h e w ou ld be released back in to th e cu stod y of th e p olice. Arran gin g for ou tp atien t treatm en t w h en th e p atien t is actively h om icid al is an im p rop er cou rse of action . N otification of th e p olice or w arn in g th e p atien t’s m oth er is n ot im m ed iately n ecessary if th e p atien t is w ell con tain ed in a closed p sych iatric u n it. 14. (B) Involu ntary ad m ission to the hosp ital is ju stified w hen the p atient is d eterm ined to be (1) an im m inent d anger to self or others or (2) gravely d isabled or unable to care for self. The criteria for involu ntary hosp italization is d erived from the d octrine of parens patriae w hich allow s the state to intervene and act as a su rrogate parent for those w ho are unable to care for them selves or m ay harm them selves. State law s d iffer on the length of tim e a person can be com m itted to the hospital on the basis of either criterion. A history of prior psychiatric hosp italization, m ed ication noncom p liance, presence of a p sychiatric d isord er, or refu sal to follow -u p w ith ou tp atient services are all im p ortant factors w hen d eterm ining w hether an ind ivid u al shou ld be hosp italized , bu t are not requ isite elem ents for involu ntary hosp italization. 15. (A) Forcing treatm ent on a patient d irectly challenges his au tonom y. In general, an ad u lt w ith d ecisional cap acity has the legal right to d eterm ine w hat m ay be d one to his bod y. H ow ever, a physician m ay m ed icate a patient against his w ill and w ithou t a cou rt hearing

Answe rs : 9–20

if it is d one to p revent a p atient from cau sing im m inent harm to him self or others. In the vignette, the patient is grossly im paired and p oses an im m ed iate threat to others. Beneficence refers to a physician’s d uty to d o good for the p atient. Confid entiality, or physician– p atient privilege, is a legal concep t that p rotects com m u nications betw een a p atient and his or her d octor from being d isclosed to a third p arty. Justice refers to the concep t of rew ard and pu nishm ent and the im p artial d istribu tion of social benefits (e.g., w hether resou rces shou ld be d istributed equ ally to those in need vs. w hether they shou ld go w here they w ou ld have the greatest im pact on the ind ivid u al or society). N onm aleficence refers to a physician’s intention to not harm or bring harm to the patient. 16. (E) N ot gu ilty by reason of insanity (or gu ilty but m entally ill in som e state) is a verd ict rend ered in a crim inal case w hich find s that the d efend ant w as insane at the tim e of com m itting the crim e. This is d eterm ined by the application of the test for insanity used in the particular jurisd iction. Contrary to popular belief, the insanity d efense (1) is seld om asserted , (2) is u su ally u nsu ccessfu l, and (3) typically lead s to a greater num ber of years that the d efend ant spend s in a m ental hospital. Ind ivid uals found not gu ilty by reason of insanity are usually com m itted to a m ental facility rather than serving tim e incarcerated . Typ ically, the com m itm ent is not for a set am ount of tim e bu t rather u ntil the ind ivid u al is d eem ed not to be a threat to society. Releasing an ind ivid ual found not guilty by reason of insanity w ithou t ap p rop riate psychiatric treatm ent is u nethical and fails to p rotect society. 17. (C) Typical signs of physical abuse inclu d e bu rns w ith p ecu liar p atterns (e.g., cigarette m arks, geom etric d esigns, bilateral im m ersion patterns), bruises in low -trau m a areas (e.g., bu ttocks, genital areas, or back), retinal hem orrhages, and m ultiple fractu res at d ifferent stages of healing. Child abu se can occur in all socioeconom ic levels, and physically abu sed child ren can d isplay a range of p sychop athology inclu d ing d ep ression,

157

PTSD, ADH D, and op p ositional d efiant d isord er. Age-appropriate “rou gh play” is not a likely cau se for this child ’s inju ries. The history and physical exam ination d o not provid e sufficient evid ence to d iagnose either a m ajor d ep ressive d isord er or som atic sym ptom d isord er. There is not enou gh evid ence to su ggest sexu al abu se in this p articu lar case. 18. (C) Every state requ ires m and atory rep orting of abu se or neglect to Child Protective Services. In cases in w hich there is a reasonable suspicion of abu se, a report should be filed even if the p atient and fam ily d eny the allegations. In cases of su sp ected abu se, step s to ensu re the safety of the m inor shou ld be taken im m ed iately. Referring the fam ily to therap y or d ischarging the patient w ith follow -u p to reevaluate her bruises d isregard s the child ’s safety and p otentially subjects the minor to continu ed abuse. Consid ering the extent of the physical injuries, it is u nlikely that the patient’s younger siblings inflicted them ; consequently, neither confronting the siblings about their behavior nor treatm ent w ith penicillin serves any pu rp ose. 19. (E) Accord ing to the Am erican Med ical Association (AMA) Cod e of Med ical Ethics, confid entiality in the treatm ent of m inors shou ld be m aintained u nless sp ecifically p rohibited by state law (e.g., in cases of abortion) or w hen the parents’ involvem ent is necessary to m ake complicated or life-threatening treatment d ecisions. The other choices all constitute violations of confid entiality to your patient. 20. (B) Inform ed consent for H IV testing involves a thorou gh exp lanation of the test’s interp retation, inform ation regard ing confid entiality, and fu rther evalu ation need ed if the test is p ositive. The p atient shou ld be aw are of the risk of false positives and negatives, and the need for confirm atory testing if the test resu lt is p ositive. Choices (A) and (D ) are incorrect becau se they violate p atient confid entiality. When a patient tests positive for H IV, state law s m ay requ ire notification of the sexual partner (this is u sually d one w ithou t revealing the p atient’s id entity) in ad d ition to

158

5: Le ga l a nd Ethica l Is s ue s in P sychia try a nd Me d ic ine

the d epartm ent of p ublic health. There is no reason to notify the state’s health d ep artm ent or perform a p regnancy test before ad m inistering an H IV test. 21. (D ) The American Medical Association (AMA) Cod e of Ethics states that the principle of patient autonomy requires that physicians respect the d ecision to forego life-sustaining treatment of a patient w ho possesses d ecision-making capacity. In this case, the patient’s d esire to stop d ialysis should be respected because adults w ith d ecisional capacity are able to determine w hich treatment they should accept or deny. The remaining choices und ermine the patient’s right to make an informed decision. 22. (E) Physician–patient confidentiality should be maintained except in certain cases (e.g., d uty to protect others, m and atory rep orting, or em ergencies). Thus, the physician should not reveal confid ential information w ithout the express consent of the patient. The remaining choices violate physician–patient confid entiality. 23. (E) Confid entiality betw een patient and physician should be respected even in cases of cou rt-mand ated treatm ent. Obtaining a confid entiality w aiver from the patient is essential before releasing any inform ation to the probation officer. Ignoring the probation officer’s request is inappropriate in such circumstances. The other choices are incorrect because they violate physician–patient confid entiality. 24. (D ) The patient shou ld im m ed iately be inform ed that she w as accid entally given the im p rop er m ed ication. Evalu ating the p atient for any ad verse reaction and exp laining the m istake is the ap p rop riate first step . Encou raging the p atient to seek legal action is both alarm ist and inap p ropriate in su ch a case. The fact that the patient d id not suffer harm is not a reason to conceal the m ed ication error. Althou gh m any hosp itals requ ire d ocu m entation or reporting of su ch to a p articu lar p erson or com m ittee, this shou ld not be the first course of action. N otifying the patient’s fam ily of the m istake is a breach of confid entiality and inap propriate.

25. (B) Exp laining the im p lications of no treatm ent bu t resp ecting the p atient’s refu sal for treatm ent is the m ost ap p rop riate cou rse of action. This allow s the p atient to m ake an inform ed d ecision w ith regard to her treatm ent. The elem ents integral to inform ed consent inclu d e (1) cap acity to m ake a d ecision; (2) an ad equ ate exp lanation of the risks, benefits, rationale for treatm ent, and alternative treatm ents (inclu d ing no treatm ent at all); and (3) a volu ntary d ecision. In this vignette, there is no evid ence that the p atient su ffers from a cond ition that m ight im p air her d ecisional cap acity. Fu rther, her stated religiou s belief is consistent w ith a Jehovah’s Witness’s refu sal of hu m an blood p rod u cts. Ad m inistering p acked red blood cells infringes on the p atient’s right to refu se a blood transfu sion and consequ ently is incorrect. Strongly p ersu ad ing the p atient to arrive at a d ecision for treatm ent is coercive and cou ld p revent the p atient from m aking a volu ntary d ecision. It is inap p rop riate to refer the case to the hosp ital’s ethics com m ittee if the p atient has d ecisional cap acity and d ecid es to refu se treatm ent. It is also inap p rop riate to involu ntarily hosp italize this p atient if she has arrived at her d ecision in com p liance w ith the elem ents of inform ed consent. 26. (D ) Accord ing to the AMA Cod e of Med ical Ethics, treatm ent d ecisions m u st be m ad e in the best interest of the child or neonate regard less of the d esires of the p arents. Treatm ent can be w ithheld or w ithd raw n only in cases in w hich the risks far ou tw eigh the benefits, there is low p otential for su ccess, or w hen treatm ent extend s the child ’s su ffering w ithou t p otential for a m eaningfu l existence. In this p articu lar case, the neonate su ffers from a seriou s, bu t treatable, central nervou s system infection and treatm ent shou ld not be w ithheld . The lu m bar p u nctu re resu lts u nequ ivocally d em onstrate bacterial m eningitis. Withhold ing treatm ent u ntil the next regu larly sched u led ethics m eeting w ou ld likely resu lt in the neonate’s d em ise. Threatening to rep ort the p arents to child p rotective cu stod y u nless they revoke their d ecision is coercive and inap p rop riate.

Answe rs : 21–32

27. (B) It is u nethical to offer sp ecific p sychiatric d iagnoses for ind ivid u als that you have not p ersonally exam ined . Moreover, com m enting on an ind ivid u al that you have evalu ated w ou ld be a breach of confid entiality. 28. (D ) Consu ltation w ith the ethics com m ittee shou ld be sought w hen the p atient’s prior w ishes are u nclear or if fam ily m em bers are u nable to agree on treatm ent options. Com p etent ad u lts m ay form ulate p references (ad vance d irectives) regard ing their m ed ical treatm ent in the event that an inju ry or illness cau ses severe im p airm ent or loss of d ecisional cap acity. Ad vance d irectives can be established by d ocu m enting the ad u lt’s m ed ical preferences and goals in a living w ill or by d esignating a health care proxy (d u rable p ow er of attorney for health care) to m ake treatm ent d ecisions on the patient’s behalf. Fam ily and / or close associates shou ld be consu lted to aid in reaching treatm ent d ecisions if there is no ad vance d irective or d esignated p ow er of attorney. Ethics com m ittee consultations are also u seful w hen the pow er of attorney’s d ecision is unreasonable or contrary to the patient’s prior w ishes. It w ou ld be inapp ropriate to allow a severely incapacitated p atient to d esignate a pow er of attorney or m ake treatm ent d ecisions. Choosing sid es in a fam ily conflict w ou ld be arbitrary; consu ltation w ith the hosp ital ethics com m ittee w ou ld help clarify issu es in a com p licated case such as this. 29. (B) The most appropriate course of action is to involuntarily ad mit the patient. Major d epressive d isord er with peripartum onset is a form of clinical d epression that d evelops in 3% to 6% of w omen during pregnancy or the weeks following d elivery. Approximately 50% of postpartum d epressive episod es actually begin prior to delivery, thus episod es are collectively referred to as peripartum episod es. Postpartum depression usually refers to episod es that start w ithin 4 w eeks after d elivery. Symptoms can last from several months to a year. It is imperative that examining physicians inquire about the mother’s thoughts tow ard her child when concerns for peripartum d epression are present.

159

Peripartum mood episod es can present either w ith or w ithout psychotic features. Infanticide is most often associated with psychotic episod es (e.g., command hallucinations to harm the infant, d elusions that the infant is evil, etc.). Although the patient in the vignette denied overt psychotic symptoms, she d emonstrated a significant clinical d ecline and acknow ledged that she had thoughts of harming her infant. Discharging the patient und er any circumstances is inappropriate because the infant remains at risk of harm. Child Protective Services should be notified in cases of suspected or known child abuse or neglect. Administering lorazepam and reassessing the patient do not ad d ress the issue of the baby’s safety. 30. (D ) Most physicians w ill somed ay find them selves in situations w here they encounter their patients in the com mu nity. Patient–physician confid entiality d oes not cease once a physician leaves the hospital and m ust be maintained regard less of w ho is making the inquiry. Confidentiality can only be broken when the physician has a d uty to protect others, in cases of m and atory reporting, or in med ical em ergencies. The remaining choices violate physician–patient confid entiality. 31. (A) Sexu al or rom antic relationship s w ith cu rrent or form er p sychiatric p atients (regard less of the tim e elap sed since treatm ent) are alw ays consid ered u nethical, m aking the other choices incorrect. Ad d itionally, p sychiatrists shou ld avoid situ ations in w hich their interactions w ith cu rrent or form er p atients cou ld be m isinterp reted as an intim ate relationship . 32. (A) Sexu al relations w ith cu rrent patients are u nethical and constitute sexual m iscond u ct. The AMA Cod e of Med ical Ethics states that sexu al and / or rom antic relations w ith form er p atients are u nethical if the physician u ses or exp loits tru st, know led ge, em otions, or influ ence d erived from the p reviou s p rofessional relationship . At the very least, the p hysician has an ethical d u ty to term inate the physician– p atient relationship prior to initiating any rom antic or sexu al relationship s.

160

5: Le ga l a nd Ethica l Is s ue s in P sychia try a nd Me d ic ine

33. (D ) Accord ing to the AMA Cod e of Med ical Ethics, “p hysician-assisted su icid e is fu nd am entally incom patible w ith the physician’s role as healer,” and the p hysician shou ld aggressively respond to a p atient’s end -oflife need s rather than assist in su icid e. Physician-assisted su icid e is d efined as the act of facilitating a p atient’s d eath by “p rovid ing the necessary m eans and / or inform ation to enable the p atient to perform the life-end ing act.” Sim p ly ignoring the p atient’s requ est is inap p rop riate becau se it fails to recognize and ad d ress p otential end -of-life issu es. End -oflife issu es that shou ld be ad d ressed w ith the p atient includ e ad equ ate pain control, em otional/ fam ily su pport, com fort care, respect of patient autonom y, and appropriate m u ltid isciplinary referrals (hospice, cou nseling, religiou s issu es, etc.). The rem aining choices are consistent w ith the d efinition of p hysicianassisted su icid e and are therefore u nethical. 34. (D ) The treating p hysician is u ltim ately legally resp onsible for d eterm ining w hether a p atient is ap p rop riate for d ischarge. Discharging a m ed ically unstable or u nsafe patient is consid ered negligence and can potentially lead to a law suit if there is an ad verse outcom e. A p sychiatric patient can be consid ered for d ischarged as long as he/ she d oes not m eet the criteria for involu ntary ad m ission (i.e., im m inent risk to self or others or grossly u nable to care for self). The patient in the vignette is not consid ered safe to leave the hospital, w hich m akes the other answ er choices incorrect. Contacting the insu rance com p any w ithou t p erm ission violates patient confid entiality. Regard less of w hether the p atient has insu rance or qualifies for a loan, the p hysician shou ld continu e to treat the patient as long as it is m ed ically necessary. 35. (B) When a physician is served a subpoena to release m ed ical information, the appropriate first step is to contact the patient to find out if he or she consents to the release. Discussing the implications of the law suit w ith your patient and clarifying w hether the patient has the proper legal cou nsel for the law su it are also important. Contacting the attorney of

the party requesting the subpoena and releasing the med ical record s w ithout consent from the patient constitute unethical breaches of physician–patient confidentiality. Releasing the m ed ical information d irectly to the jud ge at this stage makes no sense and w ould likew ise be a breach of confid entiality. Requesting that you r patient sign a release of information is inappropriate w ithout first d iscussing w hether the patient consents to releasing the record s. 36. (D ) Turning over m ed ical record s to com ply w ith a d irect cou rt ord er is ethical and legal. To d efy a ju d ge’s ord er can p otentially p lace a p hysician in contem p t of court. When the d isclosu re of confid ential inform ation is required by law or cou rt ord er, p hysicians shou ld d isclose the m inim al inform ation requ ired by law . H ow ever, in this case releasing p artial record s violates the cou rt ord er. Working ou t a p lan of legal action or ap p ealing to the state’s Sup rem e Cou rt is not the role of the physician, bu t rather the p atient’s attorney. 37. (A) In the United States crim inal ju stice system , a com petency evaluation is an assessm ent of the d efend ant’s ability to u nd erstand and rationally p articip ate in a cou rt process (Dusky v. United States, 1960). The p sychiatrist’s goal in assessing for com petency to stand trial involves evaluating w hether the d efend ant has a rational und erstand ing of the crim inal cou rt proceed ings, the capacity to u nd erstand the d ifferent potential outcom es of the cou rt proceed ings (e.g., plead ing gu ilty vs. not guilty), and has a su fficient ability to com m u nicate w ith and assist the attorney in the d efense. The exam iner should d ocu m ent that he or she has fu lly inform ed the d efend ant of the purpose and nature of the evaluation proced u re and the extent of nonconfid entiality. In situations such as this w here confid entiality is lim ited the p sychiatrist m u st m ake every effort to m aintain confid entiality w ith regard to any inform ation that d oes not bear d irectly u p on the legal purpose of the evaluation. There is no need for the exam iner to w ithd raw from the case in the event of a crim inal confession, nor it is app rop riate for the exam iner to p ersu ad e the d efend ant to act in any particular m anner.

Answe rs : 33–43

38. (D ) Inquiring abou t possible reasons the patient m ay be avoid ing or d eferring payment m ay help u ncover important treatment issues. Such inquiry often lead s to how the patient feels about the therapy and / or the therapist. These feelings can be u tilized to ad d ress factors related to her symptoms. Ad d itionally, concerns may arise regard ing the financial situation of the patient that otherw ise w ou ld not have been d iscussed in the sessions. Exploring such issues should be d one in a sensitive m anner to avoid alienating the p atient. Contacting the patient’s family violates physician–patient confid entiality. Requesting that a billing collector d em and s im med iate payment and term inating treatment w ould be far too aggressive at this point. Inform ing the patient that you w ill not see her if she d oes not im med iately pay her bill may be perceived as confrontational and threatens the therapeutic alliance. 39. (C) Becom ing involved in the business ventu re of a psychotherapy patient could have significant ad verse effects on the patient’s transference (feelings and m em ories exp erienced by the patient that are aroused by the therap ist) as w ell as on the p hysician’s cou ntertransference (feelings arou sed in the p hysician w hile w orking w ith the p atient). Therefore, it shou ld be avoid ed . Bu siness relationship s w ith form er p atients are often p erceived as p roblem atic, and business relationships w ith current patients are consid ered u nethical. Anger, d istru st, and gu ilt are som e of the feelings that m ay occu r if a bu siness ventu re su ch as the one d escribed w ere to fail. Fu rtherm ore, a physician’s bu siness involvem ent w ith a p atient m ight affect the neu tral stance essential to effective psychotherapy. The central concern that involvem ent in a business ventu re m ight interfere w ith treatm ent shou ld be exp lained to the p atient honestly. 40. (D ) The m ost significant pred ictor of su icid e is a p ast history of su icid e attem p t. Other su icid e risk factors includ e: ad vancing age (>45), gend er (M > F), m arital status (sep arated or d ivorced > m arried ), alcohol u se d isord er (su icid e rate is 50% higher in those w ith alcohol u se d isord er vs. those w ith no sub-

161

stance use), em ploym ent status (unem ployed or retired > em ployed ), su pport system (living alone > living w ith others), and health statu s (seriou s m ed ical cond ition or term inal illness > good health). 41. (B) Suicidal behavior is among the most stressful emergencies encountered by mental health professionals. There is no algorithm or scoring tool that is able to consistently identify a patient’s risk for suicide. If a malpractice claim is brought up against a psychiatrist, adequate documentation of the patient’s suicide risk and protective factors assist the court in determining the scope and complexities of the patient’s treatment. Failure to document these assessments and interventions potentially gives the court a reason to believe they were not done. “No self-harm” contracts do not hold up as legal documents in a court of law. The remaining choices are not sufficient enough to legally protect a psychiatrist in the event of a patient suicide. 42. (D ) An ind ivid u al w ho com m its a crim e w hile voluntarily intoxicated cannot assert an insanity d efense becau se the basis of su ch behavior stem s from a rational d ecision to d rink. Involu ntary intoxication m ay be u sed effectively as an insanity d efense if the ind ivid u al had no choice in becom ing intoxicated (e.g., the person w as unknow ingly d rugged ). A m ental d isease or d efect or inability to recall the event m ight be used to enhance an insanity d efense, not u nd erm ine it. Prior history of assau lt or the natu re of the crim e shou ld have no bearing on an insanity d efense. H ow ever, if p rior violent behavior has been associated w ith a m ental illness and the inability to appreciate the crim inality of one’s cond uct or conform to the law , this cou ld be u sed to strengthen an insanity d efense. 43. (E) Med ical expert w itnesses are called to p resent their professional opinions based on reasonable m ed ical certainty. An expert w itness shou ld have cu rrent and su bstantial experience and know led ge in the area in w hich they testify, and m ay be called to testify in both civil and crim inal trials. A p hysician w ho p rovid es exp ert testim ony is exp ected to

162

5: Le ga l a nd Ethica l Is s ue s in P sychia try a nd Me d ic ine

p erform an im p artial evaluation and p rovid e an ind epend ent op inion. Exp ert testim ony shou ld be based upon current scientific d ata and a stand ard of care that is accep ted am ong those w ho p ractice in the respective field . It is not the responsibility of the exp ert to ensu re a p articu lar ou tcom e for either sid e. Unlike the trad itional p hysician–patient relationship , the exp ert w itness d oes not initiate treatm ent of p atients and has lim its on confid entiality. 44. (B) H abeas corp u s refers to the U.S. Constitu tion’s clau se that the state shall not “d eprive any person of life, liberty, or prop erty, w ithou t d ue process of law .” Und er habeas corp us, citizens have the right to p etition the cou rt w hen d etained to d ecid e w hether they are being held law fu lly. Establishing crim inal resp onsibility requ ires the p resence of both actu s reu s and m ens rea. Actu s reu s refers to the volu ntary act of com m itting a crim e, and m ens rea refers to crim inal intent. Both choices are incorrect as they have nothing to d o w ith the patient’s right to d u e p rocess. Parens p atriae refers to the state’s fu nction as parent for those unable to care for them selves. This is often u sed for ju stification of involu ntary com m itm ent. The Fou rth Am end m ent p rotects citizens from u nreasonable searches and seizu res. 45. (E) It w ou ld be u nethical to involuntarily d etain a patient in treatment if he or she is no longer d eemed to be gravely d isabled or a threat to self or others. In this case, the patient shou ld be p rom ptly d ischarged to the partial hospital program . Appeasing the p atient by increasing sm oking privileges fails to ad d ress the key issue in this question. Filing for another cou rt hearing is approp riate only if the patient is still gravely d isabled or a d anger to self or others. Althou gh the patient w as initially committed by the court, it w ould be unlaw ful to ignore the patient’s request for d ischarge if he no longer meets criteria for involuntary hospitalization. Persu ad ing the patient to stay a few more d ays even thou gh he w ishes to leave the hospital and no longer m eets criteria for involuntary com mitment is inapp ropriate. 46. (E) The key elem ents to inform ed consent inclu d e the cap acity to m ake a d ecision, a

volu ntary choice w ithout coercion or d u ress, and an ad equ ate u nd erstand ing of the rationale, risks, benefits, and alternative treatm ent op tions. The rem aining choices are not requ ired for inform ed consent. 47. (C) Living w ills, also know n as ad vance d irectives, preserve patient autonom y by establishing p ersonal p references regard ing end -of-life issu es. If p atients lose their d ecisional cap acity in the fu tu re, ad vance d irectives ou tline w hat p roced u res they d o or d o not d esire w ithou t relying on p hysician or fam ily m em ber op inions. Patients are able to change their living w ills if d esired . Physician-assisted su icid e is not consid ered ethical. The rem aining choices are not characteristics of a living w ill. 48. (E) Malingering is characterized by intentional feigning of sym ptom s for second ary gain (i.e., financial com p ensation, d isability benefits, avoid ing crim inal p rosecu tion or m ilitary d uty, etc.). A d iagnosis of m alingering shou ld be su sp ected w hen there is the p resence of one or m ore of the follow ing factors: (1) litigation (esp ecially w hen the p atient is referred by a law yer), (2) d iscrep ancy betw een objective find ings and the p atient’s com p laints, (3) a d iagnosis of antisocial p ersonality d isord er (w hich his history su p p orts), and (4) a failu re to com ply w ith the evaluation and recom m end ed treatm ents. The d u ration of sym ptom s in this case has no bearing on a d iagnosis of m alingering. The lack of physical inju ries or the presence of flashbacks in a claim of PTSD is not su fficient to raise su sp icions of m alingering. PTSD occu rs in ind ivid u als exp osed to actu al or threatened d eath or inju ry and consequently m ay not involve p hysical injuries. A p ast history of m ajor d epression d oes not raise a su sp icion of m alingering. 49. (E) Accord ing to the AMA Cod e of Med ical Ethics, p hysicians are obligated to p rovid e continu ity of care for their patients. When d ischarging a patient from you r care, ad vance notice of the p end ing d ischarge m u st be given to allow the patient su fficient tim e to obtain another p hysician. Contacting the p atient’s fam ily breaches confid entiality and is not the

Answe rs : 44–51

ap propriate proced ure for d ischarging the p atient. N otifying the insu rance com p any fails to inform the patient of her d ischarge and d oes not provid e her tim e to obtain a new p hysician. The rem aining choices fail to provid e the p atient w ith su fficient notice to obtain prop er m ed ical follow -u p . 50. (A) The prevailing concep t in d eterm ining child custod y is based on ind ivid u al(s) w ho can provid e for the best interests of the child . Factors that can be used to determine the best interests of the child includ e assessing the ind ivid ual’s biologic relation, financial stability, level of ed u cation, and health. H ow ever, it is the aggregate rather than any single factor w hich d eterm ines w hat is in the best interest of the child .

163

51. (D ) Physicians are ethically obliged to rep ort colleagues w ho are im paired , incom petent, or engaging in unethical cond uct. The d u ty to report stem s from physicians’ obligation to protect patients against harm . Reporting p roced u res vary from state to state; how ever, the AMA Cod e of Med ical Ethics suggests initial reporting to a hosp ital’s im p aired physician p rogram . If no su ch program exists, the chief of staff or clinical service d irector shou ld be notified . For p hysicians w ithou t hosp ital p rivileges, an im p aired physicians p rogram ru n by a m ed ical society or the state board licensing com m ittee shou ld be m ad e aw are of the im paired physician. The other choices d o not m eet the AMA gu id elines and fail to initiate proper treatm ent and sup ervision of the im p aired p hysician.

This page intentionally left blank

CHAPTER 6

Diffe re ntial Diag no s is and Manag e me nt Que s tions

D IRECTION S (Questions 1 through 60): For each of the multiple-choice questions in this section, select the lettered answ er that is the one best response in each case. Questions 1 and 2 A 43-ye r-old w o n p resents to you r office telling you th t recently she h s been exp eriencing n incre se in the volu e of voice th t she h s been he ring for ye rs. It const ntly criticizes her beh viors reg rd less of her ood . She lso notes th t for ost of the p st ye r, her ood h s been very low . She is no longer ble to get ny ple su re fro w tching television. H er sleep is p oor nd her energy is low . In d d ition, she d escribes 20-lb w eight loss in the p st ye r bec u se she no longer feels the need to e t. She d enies the u se of ny d ru gs or lcohol. 1. Which of the follow ing d i gnoses best ccou nts for this p tient’s sy p to s? (A) (B) (C) (D) (E)

Bip ol r II d isord er M jor d epression w ith psychotic fe tures Schizo ffective d isord er Schizop hreni Schizoid p erson lity d isord er

2. Which of the follow ing ed ic tion co bin tions w ou ld best tre t this p tient’s sy pto s? (A) Div lp roex sod iu nd lor zep (Ativ n) (B) Flu oxetine (Proz c) nd d i zep (V liu )

(C) H lop erid ol (H ld ol) nd p erp hen zine (Tril fon) (D) Mirt z p ine (Re eron) nd cit lop r (Celex ) (E) Zip r sid one (Geod on) nd sertr line (Zoloft) Questions 3 and 4 A 29-ye r-old w o n w ho ju st d elivered 3 w eeks go is referred bec u se her obstetrici n noticed th t she ppe red to be d isheveled . Upon initi l interview , the p tient tells you th t she h s been feeling d ow n since d elivering her son. She tells you th t w hile she continu es to c re for her son, she is grow ing incre singly d ep ressed bec u se she d oes not get ny p le sure fro t king c re of hi . She is u n ble to sleep nd h s not been e ting uch either. She feels tired ll the ti e. She lso tells you th t t ti es, she w ill he r b by in the b ckground crying, bu t w hen she checks on her son, he is sleep ing sound ly. She d enies ny thoughts of w nting to hu rt her son or herself. 3. Which of the follow ing d i gnoses is the likely?

ost

(A) Ad ju st ent d isord er (B) Brief p sychotic d isord er (C) M jor d ep ressive d isord er w ith p erip rtu onset nd p sychotic fe tu res (D) Postp rtu obsessive-co p u lsive d isord er (E) Schizo ffective d isord er, d ep ressed typ e

165

166

6: Diffe re ntia l Diag nos is a nd Ma na g e me nt

4. Which of the follow ing is the te first step in her tre t ent?

ost pp rop ri-

(A) Flu oxetine (Proz c) nd qu eti p ine (Seroqu el) (B) H lop erid ol (H ld ol) (C) H osp it lize the p tient (D) Lithiu (E) Su p p ortive ther p y Questions 5 and 6 A 64-ye r-old w o n is brou ght to the e ergency d ep rt ent by her neighbor, w ho s ys “ y friend isn’t cting right.” The p tient requ ires the su p p ort of nurse w hile w lking to n ex in tion t ble. Ex in tion reve ls th t she c nnot correctly id entify the se son or the tow n she is in. She d oes not recognize her neighbor. She is in ttentive nd see ingly p thetic to the ctivity rou nd her. She d ozes off repe ted ly d uring the interview , bu t e ch ti e is rou s ble nd resu es nsw ering qu estions. H er nsw ers re illogic l nd inconsistent. Vit l signs re w ithin nor l li its nd she is neither tre u lou s nor d i p horetic. N eu rologic ex in tion find s bil ter l sixth nerve p lsy nd horizont l nyst g u s. Urine toxicology screen nd blood lcohol level re neg tive. 5. Which of the follow ing is the d i gnosis? (A) (B) (C) (D) (E)

ost likely

Acu te su bd u r l he to Alcohol w ithd r w l Folic cid d eficiency N or l p ressu re hyd rocep h lu s (N PH ) Wernicke encep h lop thy

6. Which of the follow ing is the ost i p ort nt first step in n ging this p tient? (A) (B) (C) (D)

Ad inistr tion of benzod i zep ine Ad inistr tion of folic cid Ad inistr tion of thi ine Co p u ted to ogr p hy (CT) sc n of the he d (E) Intr venou s (IV) flu id s nd observ tion

Questions 7 and 8 A 62-ye r-old w o n p resents to you r office long w ith her d u ghter. H er d u ghter exp l ins to you th t l tely, her other h s been extre ely concerned bout her bod y od or. Despite ultiple re ssu r nces fro her d ughter th t she d oes not s ell, she continues to be concerned th t other people find her bod y od or extre ely offensive. The p tient tells you th t she know s th t she s ells bec u se she c n s ell her ow n bod y od or ll the ti e d espite t king u ltip le show ers throu ghou t the d y. Despite being concerned bout her od or, she continues to w ork fro ho e s she h s d one for the p st 30 ye rs nd she continu es to sp e k w ith friend s on the telep hone. She p ys her onthly bills w ithou t the help of her f ily. H er thinking otherw ise see s logic l nd pp rop ri te. 7. Which of the follow ing is the d i gnosis?

ost likely

(A) Delu sion l d isord er (B) M jor d ep ressive d isord er w ith p sychotic fe tu res (C) Obsessive-co p u lsive d isord er (D) P r noid p erson lity d isord er (E) Schizop hreni 8. Which of the follow ing w ou ld be the p p rop ri te tre t ent? (A) (B) (C) (D) (E)

ost

C rb zep ine (Tegretol) Electroconvu lsive ther p y (ECT) Flu oxetine (Proz c) Lithiu (Esk lith) Ol nz p ine (Zyp rex )

Questions 9 and 10 A 29-ye r-old n w ith history of chronic schizop hreni co es to the e ergency d ep rt ent w ith te p er tu re of 102.9°F, l bile blood p ressu re rising to 210/ 110 H g, p u lse of 110/ in, nd resp ir tory r te of 22 bre ths/ in. This p tient’s ed ic tions includ e h lop erid ol, benztrop ine (Cogentin), nd clon zep (Klonop in). H e c nnot correctly id entify the d y, d te, or ye r, nd believes hi self to be in city fro w hich he oved 10 ye rs go. A f ily e ber ind ic tes th t 3 d ys go he w s

Que s tions : 4–13

he lthy nd co pletely oriented nd th t he h s no signific nt ed ic l or surgic l history. Physic l ex in tion reve ls th t he is in cute d istress w ith hypertonicity. L bor tory ex in tion reve ls cre tinine phosphokin se (CPK) of 45,000 IU/ L, w hite blood cell cou nt of 15,000/ µL nd no left shift, sod iu of 145 Eq/ L, nd cre tinine of 2.5 g/ d L. Lu b r p u nctu re prod u ces cle r fluid w ith slightly elev ted p rotein cou nt. 9. Which of the follow ing is the d i gnosis? (A) (B) (C) (D) (E)

ost likely

Anticholinergic synd ro e Centr l nervou s syste (CN S) infection M lign nt hyp erther i N eu rolep tic lign nt synd ro e (N MS) Prolonged i obiliz tion

10. With ppropri te tre t ent, the p tient recovers co pletely nd returns ho e. In onth’s ti e, he co es to the e ergency dep rt ent st ting th t the “voices in the w lls” re telling hi to kill hi self. He h s t ken no edic tions since he left the hospit l. His vit l signs re st ble nd edic l workup is neg tive. Which of the following ther pies should be initi ted first? (A) (B) (C) (D) (E)

ECT H lop erid ol d ep ot injections Ol nz p ine Physic l restr ints S fety onitoring only

Questions 11 and 12 A 39-ye r-old w o n co es to the e ergency d ep rt ent nd co pl ins th t since her boyfriend broke u p w ith her 3 onths go, she h s been sleep ing nd e ting p oorly, h s lost ll interest in her w ork, nd feels gu ilty th t she d rove her boyfriend w y. In the p st onth, she h s begu n to feel hop eless, help less, nd th t “life y not be w orth it.” In the p st 2 w eeks, she h s d eveloped belief th t r re d ise se is rotting her he rt, nd over the p st w eek, voice h s been telling her she is no good nd should t ke n overd ose of he rt ed ic tion she is prescribed . At first she w s ble to ignore the voice, how ever, she is now t the point th t she believes she shou ld ct on it.

11. Which of the follow ing shou ld be the i ed i te n ge ent?

167

ost

(A) Ad ission to ed ic l u nit (B) Ad ission to p sychi tric u nit (C) Med ic tion nd d isch rge to close f ily e ber (D) Referr l to n ou tp tient p sychi trist (E) Restr ints nd ed ic tion in the e ergency d ep rt ent 12. Which of the follow ing ed ic tion(s) w ou ld be the ost p p rop ri te? (A) Benzod i zep ine only (B) Lithiu nd selective serotonin reu p t ke inhibitor (SSRI) (C) Lithiu only (D) SSRI nd n ntip sychotic (E) Tricyclic ntid ep ress nt (TCA) only Questions 13 and 14 A 22-ye r-old n is brou ght to the e ergency d ep rt ent by p olice fter n ep isod e in w hich he r ns cked the office w here he w orks looking for “evid ence.” H e st rted this job 2 onths go fter gr d u ting fro college. H e lives w ith fou r roo tes, nd he believes they re je lous of hi bec u se of his job nd h ve therefore been p oisoning his food . H is f ily reve ls th t once before w hen he beg n college he w ent throu gh p eriod of “ cting cr zy” but got better w ithout tre t ent nd h s d one w ell since. In the e ergency d ep rtent, he is shou ting th t he h s been u p for w eek w riting “cl ssic” book bou t ccou nting bu t th t so eone t the office stole it fro hi . H e need s to be p hysic lly restr ined by e ergency d ep rtent secu rity. Physic l ex in tion nd co plete l bor tory w orku p nd toxicology screen p rove to be neg tive. 13. Which of the follow ing ed ic tion(s) w ould be the ost p p rop ri te to initi te? (A) (B) (C) (D) (E)

Antip sychotic nd benzod i zep ine Bu sp irone (Bu Sp r) C rb zep ine (Tegretol) Lithiu SSRI

168

6: Diffe re ntia l Diag nos is a nd Ma na g e me nt

14. Three onths l ter, the p tient sees his d octor for follow -u p . H e is t king lithiu nd h lop erid ol. H e is d oing w ell, except he co pl ins of p inful u scle cr p ing. H is lithiu level is 0.8 Eq/ L. Which of the follow ing w ou ld be the ost p p rop ri te next step in his n ge ent? (A) (B) (C) (D) (E)

Decre se the h lop erid ol d ose. Decre se the lithiu d ose. Incre se the h lop erid ol d ose. Incre se the lithiu d ose. St rt b clofen (Liores l).

Questions 15 and 16 A 29-ye r-old w o n tells her d octor th t bou t 3 w eeks go she w s c rin g for ch ild w ho r n into the street nd w s killed by bu s. Since then, she c nn ot get th e i ge of the ccid ent ou t of h er ind . Even in sleep , she d re s bou t it nd it p revents her fro sleep ing ore th n few hou rs t nigh t. She u sed to t ke bu s to w ork, bu t she n ow d rives bec u se she c nnot be r to be ne r bu ses s this c u ses her to think bou t the ccid ent. In the p st w eek, sh e h s begu n issin g w ork bec u se she is u n co fort ble le ving her h ou se. Sh e feels extre ely gu ilty, believing the ccid ent w s h er f u lt. 15. Which of the follow ing is the d i gnosis? (A) (B) (C) (D) (E)

ost likely

Acu te stress d isord er Ad ju st ent d isord er M jor d ep ressive d isord er (MDD) P nic d isord er nd gor p hobi Posttr u tic stress d isord er (PTSD)

16. The p tient d ecid es g inst ny ed ic tion bu t follow s u p w ith p sychother p y. A ye r l ter, lthou gh she is no longer h ving d istressfu l sy pto s rel ting to the ccid ent, she feels s d nd te rfu l ost of the ti e, is h ving trou ble e ting, h s lost interest in g rd ening, nd w kes u p t 4 a m every orning, u n ble to get b ck to sleep. She const ntly feels tired throughout the d y. Which of the follow ing is the ost likely d i gnosis?

(A) (B) (C) (D) (E)

Acu te stress d isord er Ad ju st ent d isord er MDD P nic d isord er PTSD

17. A p tient w ith history of bip ol r d isord er is d itted to p sychi tric hosp it l in n cute nic ep isod e. H er ed ic tions inclu d e n SSRI nd benzod i zepine, w hich re both d iscontinu ed on d ission. An ntipsychotic nd ood st bilizer re st rted . Tw o d ys fter d ission, she c lls the nu rsing st ff to her bed . She is extre ely frightened nd co p l ins excited ly th t she c nnot stop looking u p . On ex in tion, her eyes re noted to be d evi ted up w rd , bil ter lly. Which of the follow ing sid e effects is ost consistent w ith her p resent tion? (A) (B) (C) (D) (E)

N MS Ocu logyric crisis Retrocollis Torticollis Tris u s

Questions 18 and 19 A 59-ye r-old w o n w ith long history of generlized nxiety d isord er (GAD) tells her p ri ry c re d octor th t 2 d ys go w hile in crow d ed su p errket she felt d izzy, long w ith ssoci ted he rt p lp it tions, p ressu re on her chest, nd frightening sense of d oo . Shortly there fter, she fell u nconsciou s nd w oke u p inu tes l ter to crow d rou nd her. She felt so ew h t better nd rejected others’ d vice th t n bu l nce be c lled . She qu ickly d e her w y ho e. 18. Which of the follow ing w ou ld be the ost p p rop ri te next step in her n ge ent? (A) (B) (C) (D) (E)

Cognitive-beh vior l ther p y Electroc rd iogr (ECG) Re ssu r nce th t her cond ition is benign Short- cting benzod i zep ines SSRI

Que s tions : 14–24

19. Which of the follow ing d i gnoses is the i p ort nt to ru le ou t first? (A) (B) (C) (D) (E)

ost

Acu te stress d isord er C rd iov scu l r d ise se GAD Illness nxiety d isord er P nic tt ck

20. A 28-ye r-old le p resents to you r office bec u se he w nts to stop p rticu l rly d isturbing beh vior. H e tells you th t he often goes on p u blic tr ins w ith the intention of trying to ru b his genit ls on other p eop le w ithou t their consent. H e continu es to h ve f nt sies bou t this, bu t he w ishes to stop bec u se he is fr id of getting into trou ble. Which of the follow ing is his ost likely d i gnosis? (A) (B) (C) (D) (E)

Exhibitionistic d isord er Frotteu ristic d isord er Sexu l sochis d isord er Sexu l s d is d isord er Voyeu ristic d isord er

Questions 21 and 22 A 53-ye r-old n presents to your office co pl ining of w orsening d epression. H e st tes th t he no longer enjoys spend ing ti e w ith his f ily, he is only getting few hours of sleep t night, nd his ppetite is uch low er th n it used to be. H e feels tired ll d y long. You d ecid e to initi te n SSRI. 21. Which of the follow ing sid e effects is he ost likely to exp erience fter 3 onths of tre t ent? (A) (B) (C) (D) (E)

Di rrhe H e d ches N u se Sed tion Sexu l d ysfu nction

169

to feel d epressed , helpless, nd hopeless, nd he su bsequ ently overd oses on his ed ic tion. H is w ife find s hi u nconsciou s nd p roceed s to c ll 911. Which of the follow ing w ou ld be the ost likely c u se of d e th? (A) (B) (C) (D) (E)

C rd i c rrhyth i Resp ir tory f ilu re Seizu re Shock Stroke

Questions 23 and 24 A 19-ye r-old n w ith no p reviou s psychi tric history is noted by his college roo te to be cting biz rrely for the p st onth nd h lf, h ving convers tions w ith people w ho re not there, w lking rou nd the d or itory roo n ked , nd ccusing the roo te of c lling the Dep rt ent of H o el nd Secu rity to h ve hi onitored . The p tient’s vit l signs re ll w ithin nor l li its nd his neurologic ex in tion show s no d eficits or bnor lities. 23. Which of the follow ing tests w ou ld be ost usefu l in the initi l d i gnosis of this p tient? (A) (B) (C) (D) (E)

Co p lete blood cou nt (CBC) Erythrocyte sed i ent tion r te (ESR) Liver fu nction tests N oncontr st CT sc n of the br in Toxicology screen

24. If the bove test w ere neg tive, w hich of the follow ing w ou ld be the ost likely d i gnosis? (A) Delu sion l d isord er (B) M jor d ep ression w ith p sychotic fe tu res (C) Schizop hreni (D) Schizop hrenifor d isord er (E) Su bst nce-ind u ced p sychosis Questions 25 and 26

22. The p tient retu rns few ore ti es, nd d u ring e ch visit, you incre se the d os ge of the SSRI. H ow ever, he d oes not report ny d ecre se in his d epressive sy pto s. Bec use of his l ck of response to n SSRI, you d ecid e to initi te tricyclic ntid ep ress nt. H e continu es

You h ve been sked by the su rgery te to ev lu te 35-ye r-old n w ho h d surgery to rep ir his fr ctu red right w rist 24 hou rs go nd is now co pl ining of nxiety. The p tient h s been in the hospit l for 2 d ys. H is he rt r te is 120 be ts/ in, nd

170

6: Diffe re ntia l Dia g nos is a nd Ma na g e me nt

his blood pressure is 160/ 106 H g. H e is febrile nd reports th t he h s never su ffered nything like this before. H e is not in ny p in nd h s no previou s psychi tric history. H is ed ic tion inclu d es only cet inophen for p in control. You note th t he is d i phoretic, flu shed , nd tre u lou s. 25. Which of the follow ing is the ost p p rop ri te ed ic tion to tre t this p tient? (A) (B) (C) (D) (E)

C rb zep ine Clonid ine (C t p res) Lor zep Meth d one N ltrexone (ReVi )

26. Withou t the bove tre t ent, w hich of the follow ing co p lic tions is he ost t risk for d evelop ing? (A) (B) (C) (D) (E)

Abd o in l p in Cirrhosis F tty liver Mu scle cr p s Seizu res

Questions 27 and 28 You re w orking in the psychi tric e ergency d ep rt ent of l rge etropolit n hospit l. A 20-ye r-old n w ith unknow n p sychi tric history is brou ght in by the p olice fter being fou nd stu bling n ked rou nd loc l college c p u s. H e is rked ly git ted , p cing, nd pp e rs to be resp ond ing to intern l sti u li. On ex in tion, you note th t he is t chyc rd ic w ith he rt r te in the 110s, he h s tics or sp s s in his f ce, nd he h s vertic l nyst g u s. 27. Which of the follow ing tests w ou ld be helpfu l in the d i gnosis?

ost

(A) CBC (B) Electroencep h logr (EEG) (C) M gnetic reson nce i ge (MRI) of the br in (D) N oncontr st he d CT (E) Toxicology screen

28. Which of the follow ing w ou ld be the pp rop ri te tre t ent for this p tient? (A) (B) (C) (D) (E)

ost

A low -sti u lu s environ ent Benztrop ine Methylp henid te (Rit lin) Phenytoin (Dil ntin) Prop r nolol (Ind er l)

29. A 40-ye r-old d ivorced w o n is brought in to the e ergency d ep rt ent fter being fou nd sleep ing in p ile of le ves. Initi lly, she is d ifficu lt to rou se nd h s trou ble nsw ering you r qu estions d u e to u bling. She d oes not ppe r to be in respir tory d istress, bu t her p u p ils re noted to be p inp oint. After few hou rs, she beco es ore inter ctive nd is co p l ining of d iffu se, cr p y bd o in l p in nd sy p to s of “the flu .” H er pu p ils re now slightly d il ted , nd she is y w ning. H er vit l signs re te p er tu re of 99.3°F, he rt r te of 99 be ts/ in, nd blood pressu re of 142/ 90 H g. Cu rrently, w hich of the follow ing is the ost likely d i gnosis? (A) (B) (C) (D) (E)

C nn bis bu se Coc ine intoxic tion Coc ine w ithd r w l Op i te intoxic tion Op i te w ithd r w l

Questions 30 and 31 A 70-ye r-old w id ow is d itted for n ev lu tion of d epression nd nxiety. She tells you th t for the p st 15 ye rs her f ily d octor h s prescribed “so e p ills” th t h ve help ed her sleep nd feel less nervou s. She s ys th t she r n ou t of the yesterd y nd since th t ti e h s felt incre singly nxiou s nd jittery. She lso notes th t she’s now h ving tre ors in her h nd s th t h ve not been there before. 30. Which of the follow ing ed ic tions w ou ld be the ost d ngerou s to su d d enly d iscontinue? (A) (B) (C) (D) (E)

Flu p hen zine (Prolixin) I ip r ine (Tofr nil) N ortrip tyline (P elor) Thiorid zine (Mell ril) Tri zol (H lcion)

Que s tions : 25–35

31. Which of the follow ing w ould be the ost d ngerou s sid e effect of bru p tly stop p ing the bove ed ic tion? (A) (B) (C) (D) (E)

Au tono ic hyp er ctivity Seizu res H llu cin tions Worsening nxiety Vo iting

Questions 32 and 33 A 73-ye r-old n is d itted to the hosp it l for co u nity- cqu ired pneu oni nd d ehyd r tion. On d y 2 of his hosp it liz tion, you re sked to ev lu te the p tient for d ep ression; the st ff h s noted th t he see s very w ithd r w n. H e is not e ting or sleeping w ell. The nu rsing st ff reports th t l st night he w s ngry nd requ ested to le ve the hospit l. You t lk w ith the p tient’s f ily nd find th t the p tient h s no previou s psychi tric history. Prior to the onset of this illness 5 d ys go, he h d no d ep ressive sy pto s nd no d ifficu lties w ith cognition. On ex in tion, his vit l signs re te per tu re of 98.2°F, he rt r te of 87 be ts/ in, blood pressure of 130/ 86 H g, nd p erip her l oxygen s tur tion of 95% on roo ir. H e is d row sy nd oriented only to p erson. H is Mini-Ment l St te Ex in tion (MMSE) score is 23/ 30. 32. Which of the follow ing is the d i gnosis? (A) (B) (C) (D) (E)

ost likely

Anxiety d isord er Deliriu F ctitiou s d isord er M jor d ep ressive d isord er (MDD) M jor neu rocognitive d isord er (d e enti )

33. The ed ic l te sks for ed ic tion reco end tions if the p tient beco es git ted . Which of the follow ing w ou ld be the ost pp rop ri te ed ic tion to reco end ? (A) (B) (C) (D) (E)

Benztrop ine Dip henhyd r Lor zep H lop erid ol Thiorid zine

ine

171

Questions 34 and 35 A 34-ye r-old fe le p resents to you r office s she h s been feeling d epressed nd ngry for the p st cou ple of d ys bec u se she believes th t her boyfriend is going to le ve her. As resu lt, she h s been c lling hi every hour ju st to confir th t he is not le ving her. She tells you th t she h s h d ny rel tionships in the p st w hich h ve lw ys been very rocky, c u sing her e otions to const ntly go u p nd d ow n. When ny of these rel tionships end ed , she cu t on her r s sup erfici lly to “relieve stress.” At this ti e, she tells you th t she d oes h ve so e thou ghts of w nting to cut herself, bu t no thoughts of w nting to end her life. 34. Which of the follow ing is the d i gnosis? (A) (B) (C) (D) (E)

ost likely

Bip ol r d isord er Bord erline p erson lity d isord er (PD) H istrionic PD M jor d ep ressive d isord er N rcissistic p erson lity d isord er

35. Which of the follow ing is the ost pp rop rite tre t ent for this d i gnosis? (A) (B) (C) (D) (E)

Arip ip r zole (Abilify) Cit lop r (Celex ) Cognitive-beh vior l ther p y Di lectic l beh vior l ther p y Psycho n lysis

Questions 36 and 37 A 65-ye r-old w o n w ith p st ed ic l history of non–insu lin-d ep end ent d i betes ellitu s nd d epression is d itted w ith incre singly d epressed ood over the l st onth. She is u n ble to co plete her crossw ord pu zzles bec use of d ifficu lty concentr ting. She h s trouble f lling sleep nd lso w kes u p in the id d le of the night. She d enies su icid l id e tion, but d oes feel gu ilty th t she is d epressed . Prior to this episod e, she w s d oing w ell nd w s ctively eng ged in co u nity volunteer grou p s. In the l st onth, she h s lost 13 lb d ue to poor int ke. When sked w hy she is not e ting, she st tes she is w orried th t she w ill beco e infected w ith b cteri . She h s been to her p ri ry p hysici n for n ev lu tion, bu t she cl i s everything w s

172

6: Diffe re ntia l Dia g nos is a nd Ma na g e me nt

nor l. H er hu sb nd confir s th t the p tient h s been very w orried bou t “getting d ise se” to the p oint w here she w ill e t only food in se led cont iners. H e lso confid es in you th t she h s been w orried th t she ight h ve c ncer, nd , d esp ite re ssu r nces fro her p ri ry c re p hysici n, she continu es to voice her concerns to her hu sb nd th t “ y intestines re not w orking.” 36. Which of the follow ing is the d i gnosis?

ost likely

(A) M jor d ep ressive d isord er (MDD) w ithou t p sychotic fe tu res (B) MDD w ith p sychotic fe tu res (C) Obsessive-co p u lsive d isord er (OCD) (D) Persistent d ep ressive d isord er (d ysthy i d isord er) (E) So tic sy p to d isord er 37. Which of the follow ing ed ic tion(s) w ou ld be the ost p p rop ri te tre t ent for this p tient? (A) (B) (C) (D) (E)

Div lp roex sod iu Lithiu nd sertr line N ortrip tyline nd lor zep Sertr line Sertr line nd risp erid one

Questions 38 and 39 An 18-ye r-old n is brou ght to the p sychi tric e ergency d ep rt ent by his p rents for ev lu tion of his beh vior. Three onths go, the p tient st rted cl sses t the st te university loc ted in d ifferent city, lthough he w ou ld co e ho e e ch w eekend to visit. H is p rents h ve noticed th t over the p st 3 w eeks he h s beco e incre singly w ithd r w n nd d oes not see to be t king c re of hi self. The p rents w ere recently c lled by the p tient’s roo te w ho infor ed the th t the p tient h s not been going to cl sses for the l st w eek, h s not been e ting or b thing, nd h s been spe king bout how peop le in his cl sses re trying to kill hi . The p tient d enies ny d ru g u se nd is not on ny ed ic tion. On ex in tion, the p tient’s vit l signs re w ithin nor l p r eters. H is physic l (inclu d ing neu rologic ex in tion) is u nre rk ble. H e is p cing nd ppe rs to be resp ond ing to intern l sti uli. H e

looks frightened nd sks, “Wh t is h p p ening to e?” H is ood is “OK” nd his ffect is fl t. H e is fu lly oriented . H e s ys th t he he rs nu ber of p eople t lking to hi , “ ybe in y he d ,” s ying b d things bou t hi . 38. Which of the follow ing d i gnoses is likely? (A) (B) (C) (D) (E)

ost

Brief p sychotic d isord er Delu sion l d isord er Schizop hreni Schizop hrenifor d isord er Su bst nce-ind u ced p sychotic d isord er

39. Which of the follow ing l bor tory tests or proced u res w ould be the ost helpfu l in n rrow ing the bove d ifferenti l? (A) (B) (C) (D) (E)

CBC EEG Electrolytes N oncontr st he d CT Toxicology screen

Questions 40 and 41 A 35-ye r-old w o n w ith no previous person l or f ily psychi tric history is brought to the e ergency d ep rt ent by her husb nd, w ho reports th t his w ife w s tte pting to kill herself by cutting her w rists. H er husb nd tells you th t 6 onths go the p tient’s gr nd other d ied . Since th t ti e, her husb nd believes th t the p tient h s been beco ing ore d epressed . She h s d ifficulty f lling sleep nd h s lost 15 lb in 2 onths. She w s recently fired fro her job s p r leg l bec use she w s un ble to concentr te nd d e frequent ist kes. She feels guilty th t she is un ble to feel better, nd she end orses feelings of hopelessness nd w orthlessness. She believes th t her “only w y out of this” is to kill herself. 40. Which of the follow ing w ou ld be the p p rop ri te tre t ent? (A) (B) (C) (D) (E)

Flu zenil (Ro zicon) Flu oxetine Flu p hen zine Lor zep (Ativ n) Phenelzine (N rd il)

ost

Que s tions : 36–45

41. After being begu n on the p p rop ri te ed ic tion, w hich of the follow ing sid e effects y i ic w orsening sy pto of her illness? (A) (B) (C) (D) (E)

Ak thisi Constip tion Di rrhe Inso ni N u se

Questions 42 and 43 A 46-ye r-old n w ith no previou s p sychi tric history p resents w ith co pl ints of feeling d epressed since his w ife d ied 2 w eeks go fro p ncre tic c ncer. H e sh res th t lthough he know s she is d e d , he so eti es he rs her c lling his n e. This h s occu rred sever l ti es nd usu lly h pp ens ore often t night w hen he is f lling sleep. H e d enies ny visu l h llu cin tions, p r noi , or d elu sion l beliefs. H e expresses gu ilt bou t not sp end ing ore ti e w ith his w ife w hen she w s live bu t d enies ny thou ghts of w nting to end his life. H e is now sleeping only bou t 3 to 4 hou rs t night nd h s h d slightly d ecre sed pp etite since her d e th. Desp ite the bove, he h s still been ble to go to w ork nd focu s on his job. 42. Which of the follow ing is the d i gnosis?

ost likely

Ad ju st ent d isord er M jor d ep ressive ep isod e N or l grief Persistent d ep ressive d isord er (d ysthy ic d isord er) (E) Schizop hreni

43. Which of the follow ing w ou ld be the pp rop ri te tre t ent for this p tient? (A) (B) (C) (D) (E)

the d e n of her college is concerned bout her nd w orried th t she y be d ep ressed . The p tient tells you th t d esp ite being f r w y fro her ho e for the first ti e, she w s enjoying school nd her new friend s u ntil 2 onths go, w hen she le rned th t her p rents w ere getting d ivorced . Since th t ti e, her gr d es h ve gone fro As nd Bs, to Bs nd Cs bec u se she is concerned bou t her p rents nd her sister t ho e. She feels “bu ed ou t” ost of the ti e, nd her friend s note th t she see s unh ppy nd occ sion lly beco es te rfu l w hen t lking bou t her f ily. She d enies d ifficu lty sleep ing or ch nges in ppetite or w eight, nd she continu es to enjoy d ily trip s to the gy to exercise. She d enies ny suicid l id e tion. 44. Which of the follow ing is the d i gnosis?

ost likely

(A) Acu te stress d isord er (B) Ad ju st ent d isord er w ith d ep ressed ood (C) GAD (D) MDD (E) N or l grief 45. Which of the follow ing tre t ents w ou ld be ost p propri te? (A) (B) (C) (D) (E)

(A) (B) (C) (D)

173

Di lectic l beh vior l ther p y Flu oxetine (Proz c) N ortrip tyline Psychother p y Risp erid one (Risp erd l)

Questions 46 and 47 ost

A itrip tyline (El vil) Flu oxetine (Proz c) H lop erid ol (H ld ol) Lithiu N o intervention t this ti e

Questions 44 and 45 You re sked to see n 18-ye r-old w o n w ith no p reviou s psychi tric or ed ic l history bec u se

A 26-ye r-old w o n w ith no p reviou s psychi tric history is referred to you by her p ri ry c re p hysici n for ev lu tion of “ nxiety tt cks.” She tells you th t p p roxi tely 2 onths go she beg n h ving period s l sting 10 or 15 inu tes d u ring w hich, she s ys, “I feel like I’ going to d ie.” Du ring these episod es, her he rt r ces, she feels s thou gh she c nnot c tch her bre th, she is d izzy nd fr id she y p ss out, nd she h s tingling nd tre ors in her h nd s. She is concerned bec use she is now h ving d ifficu lty le ving her hou se d ue to w orry th t these ep isod es w ill occu r, king it i p ossible for her to get ho e. She is u n ble to id entify ny triggers le d ing

174

6: Diffe re ntia l Dia g nos is a nd Ma nag e me nt

to the episod es. She is not on ny h s no ed ic l proble s.

ed ic tion nd

46. Which of the follow ing is her d i gnosis? (A) (B) (C) (D) (E)

ost likely

GAD P nic d isord er nd gor p hobi Sep r tion nxiety d isord er Soci l nxiety d isord er (soci l p hobi ) Sp ecific p hobi

47. Which of the follow ing w ou ld be the p propri te tre t ent to initi te? (A) (B) (C) (D) (E)

ost

Alp r zol Lithiu Prop r nolol Sertr line (Zoloft) Tr nylcyp ro ine (P rn te)

Questions 48 and 49 A 36-ye r-old w o n w ith no for er p sychi tric history is referred to you by d er tologist for ev lu tion of her chronic lly ch pped h nd s. She s ys she h s been seeing her d er tologist for p proxi tely 5 ye rs for this proble nd tre ted w ith v riety of top ic l gents w ith li ited su ccess. Over the p st 3 w eeks, her h nd s h ve beco e w orse, to the p oint w here they re lw ys cr cked nd bleed ing. Relu ct ntly, she confid es in you th t she h s h d longst nd ing fe r of ger s, bu t since her colle gu e t w ork h s been sick, she h s been w shing her h nd s t le st 40 ti es p er d y bec u se she is fr id of contr cting the d ise se. She lso refu ses to tou ch nything th t ight infect her w ithou t u sing h nd kerchief. She d its to being very tid y t ho e, s w ell. She spend s bou t 2½ hou rs in the orning getting show ered . She re lizes her fe rs of cont in tion re irr tion l, but every ti e she tries to stop w shing her h nd s she beco es incre singly nxiou s. 48. Which of the follow ing d i gnoses w ou ld be ost likely? (A) GAD (B) Obsessive-co p u lsive d isord er (OCD)

(C) Obsessive-co p u lsive p erson lity d isord er (OCPD) (D) P nic d isord er (E) So tic sy p to d isord er 49. Which of the follow ing tre t ents w ould be ost ppropri te for this p tient? (A) (B) (C) (D) (E)

Cognitive-beh vior l ther p y ECT Lithiu Psychod yn ic p sychother p y Risp erid one

50. A 47-ye r-old secret ry co es to your office co pl ining, “I’ lw ys w orried .” She s ys th t she w orries bout her job, her kid s, her hou sew ork, nd her hu sb nd . She is seeking help now bec u se she h s been h ving n incre singly d ifficu lt ti e concentr ting t w ork nd h s been ore irrit ble w ith p eople rou nd her. H er sleep h s been “ok y,” bu t she d oes not feel rested w hen she gets u p in the orning. She h s been ore w re of these feelings over the l st 2 ye rs nd they occu r l ost every d y. She d enies ny d iscrete p nic tt cks. Which of the follow ing d i gnoses is the ost likely in this p tient? (A) (B) (C) (D) (E)

GAD OCD Soci l nxiety d isord er (soci l p hobi ) P nic d isord er Schizop hreni

51. A 28-ye r-old w o n w ith no p revious psychi tric or ed ic l history is d itted to the neu rology service for ev lu tion of cu te onset of nu bness nd w e kness of the right sid e of her f ce nd right r nd leg. Physic l ex in tion show s sy etric l 2/ 4 reflexes in ll d istribu tions, d ow ngoing p l nt r reflexes bil ter lly, nd 2/ 5 strength in the right u p per nd low er extre ity in ll u scle group s. N o trop hy or f scicul tions re noted . H er g it is t xic nd st ggering w ith extre e ex gger ted ove ents of her r s; how ever, she d oes not f ll w hen bu l ting w ithou t ssist nce. Given the severity of her d eficits, she

Que s tions : 46–55

see s u nconcerned by her level of d is bility. Which of the follow ing d i gnoses w ou ld be the ost p propri te? (A) (B) (C) (D) (E)

Conversion d isord er F ctitiou s d isord er M lingering So tic sy pto d isord er Und i gnosed neurologic d ise se

52. A 54-ye r-old w o n w ith p st ed ic l history of hypothyroid is is d itted w ith septic right knee. The surgery te sks you to ev lu te the p tient bec use they fou nd th t the flu id sp ir te fro the knee w s grow ing p thogen fou nd p ri rily in the hu n ou th. They su sp ect the p tient w s injecting s liv into her knee. You ev lu te the p tient nd find her to be p le s nt nd coop er tive. She tells you th t she h s h d very tough ti e l tely bec use her husb nd h s recently been sick. Fortu n tely, she is nu rse nd h s been ble to c re for hi t ho e. L tely, she d its to feeling overw hel ed nd not pp reci ted . She h s no id e w h t h s c used the proble w ith her knee. You t lk to the f ily nd they tell you the p tient is in no fin nci l d ifficu lty nd continu es to enjoy w ork s nu rse. After w orking closely w ith you for sever l w eeks, she eventu lly d its to injecting her knee lthou gh c nnot u nd erst nd w hy she d id it. Which of the follow ing is the ost p p rop ri te d i gnosis? (A) (B) (C) (D) (E)

Conversion d isord er F ctitiou s d isord er Illness nxiety d isord er M lingering So tic sy p to d isord er

Questions 53 and 54 An 18-ye r-old w o n in her first ye r of college co es to see you for ev lu tion of d epression, fter her roo tes encour ged her to seek help . She rep orts d ifficu lty f lling sleep nd e rly orning w kenings, poor concentr tion, f tigue, nd nxiety over the p st onth since rriving for the f ll se ester. She tells you th t her p rents re very strict nd she is w orried th t she w ill not “get str ight

175

As.” On ex in tion, she is c l , thin w o n d ressed in very b ggy jogging suit. You co ent on her thinness, nd she tells you she p rid es herself on her pp e r nce nd tries to st y sli by exercising bou t 4 hou rs d y long w ith good d iet. She d enies proble s w ith e ting too u ch or too little, nd cl i s th t only d iet nd exercise help her to control her w eight. Desp ite you r concerns, she d its th t she w ou ld like to lose few ore p ou nd s. Up on fu rther qu estioning, she infor s you she h s not h d regu l r enstru l period for over ye r. 53. Which of the follow ing w ou ld be the likely w orking d i gnosis? (A) (B) (C) (D) (E)

ost

Anorexi nervos Anxiety d isord er Bod y d ys orp hic d isord er Bu li i nervos MDD

54. Which of the following tests would be ost helpful in supporting your provision l di gnosis? (A) (B) (C) (D) (E)

ECG Seru yl se Seru gnesiu level Seru p ot ssiu level Weight nd height

55. A 5-ye r-old boy is referred to you by his ped itrici n for ev lu tion of ggressive beh vior. H is ed ic l nd extensive neu rologic w orku p w s neg tive. An interview w ith the p tient reve ls restless boy w ho is ble to eng ge in convers tion. H e tells you he gets ngry nd fru str ted in school s he “thinks it’s boring.” H is p rents rep ort th t he is cu rrently rep e ting kind erg rten d u e to p oor p erfor nce nd d ifficu lty soci lizing w ith other child ren. H is ggressive ou tbu rsts t school see to occu r t ti es w hen he d oes not und erst nd the schoolw ork. H is other tells you th t she still h s to help hi pick ou t his clothes for school nd get d ressed . H is f ily history is p ositive for tw o p tern l u ncles w ith le rning d is bilities. Which of the follow ing tests w ou ld be ost help fu l in the ev lu tion of this p tient?

176

6: Diffe re ntia l Dia g nos is a nd Ma na g e me nt

(A) Good enough-H rris Dr w -A-Person Test (B) Kohs Block Test (C) Minnesot Mu ltip h sic Person lity Inventory-2 (MMPI-2) (D) Pe bod y Voc bu l ry Test (E) Wechsler Intelligence Sc le for Child ren (WISC) Questions 56 and 57 An 8-ye r-old boy is brou gh t in to you r office by h is oth er bec u se sh e is con cern ed th t h e c nnot stop blinking his eyes nd shru ggin g his sh ou ld ers. Sh e tells you th t this occu rs l ost every d y n d h s been th is w y for the p st ye r nd h lf. Sh e tells h er son to stop d oin g this w h ile h e is in you r office, n d h e d oes stop . You notice, h ow ever, th t h e st rts b rkin g n d cou ghin g rep e ted ly. 56. Which of the follow ing ed ic tions w ou ld be the ost ppropri te in tre ting the p tient’s cond ition? (A) (B) (C) (D) (E)

Ato oxetine Clonid ine L otrigine Lor zep V lp roic cid

57. You begin the p p rop ri te ed ic tion nd 2 onths l ter the boy nd his other retu rn for follow -u p visit. She tells you th t w hile the eye blinking nd should er shrugging h ve gotten better, he is now f lling sleep in his cl sses t school. Wh t is the next ost p prop ri te step in the n ge ent? (A) Ad d nother ed ic tion to keep the boy w ke. (B) Ed u c te the other bou t p rop er sleep hygiene. (C) Discontinu e the ed ic tion. (D) Red u ce the d os ge. (E) Sw itch to nother ed ic tion. 58. A 17-ye r-old boy w ith h istory of tten tion-d eficit/ h yp er ctivity d isord er (ADH D)

p resents w ith od d beh vior, confu sion, blood p ressure of 128/ 85 H g, nd he rt r te of 68 be ts/ in. H is p rents rep ort th t t ti es they h ve observed hi repetitively tou ching his sto ch nd r pid ly blinking his eyes. On ex in tion, he p p e rs d zed nd u n ble to concentr te. Which of the follow ing is the ost likely d i gnosis? (A) (B) (C) (D) (E)

A p het ine toxicosis Ep ilep sy H yp oglyce i MDD So tic sy p to d isord er

59. A 50-ye r-old n w ith long history of IV d rug u se is brought to the hosp it l by p olice fter loc l ho eless shelter w orker noted hi to be confu sed nd “w lking funny.” While t the shelter, he bec e very su spiciou s of the w orkers nd ccu sed the of t king his belongings. On ex in tion, you observe th t he h s left p u p il th t cco od tes but d oes not re ct, d epressed d eep tend on reflexes in ll d istribu tions, nd loss of p osition sense t the gre t toes bil ter lly. Althou gh the p tient w s tre ted w ith ntip sychotic ed ic tions in the p st, he d enies cu rrently t king ny ed ic tion. Which of the follow ing d i gnoses best ccou nts for this p tient’s sy p to s? (A) (B) (C) (D) (E)

Antip sychotic-ind u ced d yskinesi Kors koff p sychosis N eu rolep tic lign nt synd ro e N eu rosyp hilis Wernicke encep h lop thy

60. A 57-ye r-old w o n w ith no p revious p sychi tric history co pl ins of incre sing nxiety over the l st 2 onths. Tod y she reports th t it bec e “very b d .” She lso notes th t w ith these p eriod s of nxiety she gets p ou nd ing he d che nd once f inted . She continu es to feel “sh ky.” When you check her vit l signs, her he rt r te is 170 be ts/ in nd her blood p ressu re is 230/ 130 H g. She is d i p horetic nd tre u lou s. Given this p tient’s sy p to s, w hich of the follow ing cond itions is ost likely?

Que s tions : 56–66

(A) (B) (C) (D) (E)

Acu te lcohol intoxic tion H yperc lce i H yp othyroid is Pheochro ocyto Posterior circu l tion stroke

D IRECTION S (Questions 61 through 66): For each patient vignette, select the one lettered option that is most closely associated w ith it. Each lettered option may be used once, multiple times, or not at all. (A) Conversion (fu nction l neu rologic l sy pto ) d isord er (B) H ep tic enceph lop thy (C) H u n i u nod eficiency viru s (H IV) (D) H yp erthyroid is (E) H yp oglyce i (F) Pheochro ocyto (G) So tic sy p to d isord er (H ) Syste ic lu p u s erythe tosu s (SLE) 61. A 55-ye r-old n is d itted to the hosp it l. H is w ife reports th t he’s ch nged over the l st ye r, h s beco e very forgetfu l, nd h s period s w hen he beco es very u p set. On ex in tion, he h s signific nt e ory i p ir ent, sterixis, p l r erythe , nd l rge ecchy otic re on his right sc p u l . H e w s initi lly cooper tive, bu t now is very git ted nd d e nd s to le ve. 62. A 52-ye r-old n w ith history of IV d ru g u se is brou ght to the e ergency d ep rt ent by his soci l w orker fro ho eless shelter. She h s know n the p tient for 10 ye rs bu t h s seen d r stic ch nge in hi over the l st ye r. While previou sly jovi l nd inter ctive, he is now d iseng ged nd subd u ed . H e h s been incre singly forgetfu l, tod y h ving d ifficu lty u sing his e ting u tensils. H e h s lost t le st 40 lb in the l st 6 onths nd co pl ined bou t feeling w e k nd losing his b l nce. H e scores 20/ 30 on the MMSE. 63. A 33-ye r-old w o n is brou ght to the e ergency d ep rt ent by her hu sb nd , w ho tells you th t over the l st few onths she h s been

177

incre singly te rfu l, irrit ble, restless, s w ell s h ving d ifficu lty p ying bills nd b l ncing her checkbook. She h s occ sion l “hot fl shes.” On ex in tion, she continuou sly ch nges position in her se t. She h s d ifficulty w ith si p le rith etic nd short-ter e ory. H er d eep tend on reflexes re brisk sy etric lly throu ghou t. She h s fine resting tre or of her h nd s nd d ifficu lty rising fro se ted p osition. 64. A 33-ye r-old w o n p resents w ith 1-ye r history of tension he d ches, recently w orsening nd now ssoci ted w ith blu rry vision. So f r, n extensive neu rologic ev lu tion h s been u nreve ling. She volu nteers th t she h s been sick ost of her life, beginning t ge 16 w hen she h d su rgery for p resu ed end oetriosis, w hich h s left her u n ble to h ve sexu l rel tions. Since the su rgery, she h s h d inter ittent bd o in l cr ping, blo ting, nd d i rrhe . Review of her ch rt show s th t 3 onths go, she s w rheu tologist for knee, b ck, nd eye p in. 65. A 30-ye r-old w o n is brou ght to the hosp it l by her fi ncé. H e s ys th t over the l st 2 w eeks, she h s been “ tot lly d ifferent p erson.” She h s not been e ting or sleeping, nd h s been irrit ble nd ngry. E rlier in the onth, she co pl ined of he d ches, p in in her h nd s nd feet, nd fever. On ex in tion, she looks tired ; she is fully oriented bu t h s d ifficu lty rel ting the events of the l st onth. Occ sion lly, she see s to beco e confused . 66. A 42-ye r-old w o n in the e ergency d ep rtent pp e rs confu sed nd w ill not llow blood to be d r w n. She is not oriented to ti e or pl ce. She t kes ed ic tions for “ cond ition,” bu t c nnot el bor te fu rther. The only vit l signs th t w ere t ken show he rt r te of 140 be ts/ in nd blood p ressure of 172/ 98 H g. Physic l ex in tion is re rkble for fine tre or of her h nd s bil terlly, nd d i phoresis. Five inu tes l ter, the p tient h s seizu re.

178

6: Diffe re ntia l Dia g nos is a nd Ma na g e me nt

DIRECTIONS (Questions 67 through 72): For each patient vignette, select the one lettered option that is most closely associated w ith it. Each lettered option may be used once, multiple times, or not at all. (A) Deliriu (B) MDD (p seu d od e enti ) (C) M jor neu rocognitive d isord er d u e to Alzhei er d ise se (D) M jor neu rocognitive d isord er d u e to frontote por l lob r d egener tion (Pick d ise se) (E) M jor neu rocognitive d isord er d u e to H u ntington d ise se (F) M jor neu rocognitive d isord er d ue to Lew y bod y d ise se (G) M jor neu rocognitive d isord er d u e to Prion (Creutzfeld t–J cob) d ise se (H ) M jor v scu l r neu rocognitive d isord er 67. A 71-ye r-old fe le is brou ght to the ER by her hu sb nd . Over the p st few ye rs, he h s noticed th t she h s been h ving e ory p roble s. She often forgets w here she p u t her keys or pu rse. H e no longer lets her d rive bec u se she gets lost e sily. On MMSE, she h s p rticul r p roble s w ith the d y of the w eek, d te, three-ite rec ll, nd n ing pencil. 68. A 68-ye r-old le w ith no p sychi tric history is brou ght by his d u ghter bec u se, d esp ite u ltiple re ssu r nces, he continu es to insist so eone else is in the house. The p tient st tes he sees s ll n every d y ju st insid e of the d oor. H is d ughter h s lso noticed th t he h s proble s w ith his e ory. On ex in tion, he h s shu ffling g it nd blu nted ffect, w hich the d ughter st tes beg n t the s e ti e s the visu l h llu cin tions. 69. A 55-ye r-old le is brou ght by his w ife bec u se she h s noticed ch nges in his beh vior. L tely, his food p references h ve ch nged nd he is now cu rsing lou d ly in pu blic. On ex in tion, he is e sily git ted . N eu rologic ex in tion d e onstr tes p ositive B binski resp onse nd snou t reflex. An MRI show s trophy in the front l nd te p or l lobes.

70. An 86-ye r-old fe le is brou gh t to th e ED fter the nu rsing ho e st ff noticed su d d en ch nge in h er beh vior over the l st sever l d ys. On history nd p hysic l ex in tion, it is d ifficu lt to elicit infor tion s the p tient keep s f lling sleep . H er vit l signs show te p er tu re of 101.2°F n d he rt r te of 101 be ts/ in. Urin lysis is p ositive for ketones. 71. A 59-ye r-old fe le is brou ght in by her son w ith w ho she lives. H e st tes th t over the p st cou p le of onths his other h s been isol tive, no longer sp end s her ti e re d ing w hich she u sed to enjoy. She w ill re in w ke l te t night nd e rly in the orning, nd her p p etite h s d ecre sed . H e is lso concerned bec use she w ill so eti es re e ber inor d et ils fro the d y before, bu t other ti es w ill not rec ll w h t she h d for bre kf st. Ment l st tu s ex in tion is signific nt for incre sed sp eech l tency, d ep ressed ood nd ffect, nd trou ble rec lling 2/ 3 ite s fter sever l inu tes. 72. A 39-ye r-old le w ith no p reviou s p sychitric history p resents w ith co p l ints of feeling “very d ep ressed l tely.” H is f ily notes th t he h s been u ch ore irrit ble recently, often yellin g w ith little p rovoc tion nd see ingly t lking to hi self. On MSE, he is noted to h ve qu ick, su d d en, bu t involu nt ry, jerking ove ents of his r s. D IRECTION S (Questions 73 through 98): For each of the multiple-choice questions in this section, select the lettered answ er that is the one best response in each case. 73. A 56-ye r-old n p resents to you r office t the requ est of his w ife, w ho s ys th t he d rinks too uch. Wh t w ould be the ost i port nt str tegy in ev lu ting this p tient for lcohol u se d isord er? (A) Ascert in how often he d rinks. (B) Ask hi how frequ ently he gets d ru nk. (C) Ask hi w h t his f ily nd friend s s y bou t his d rinking.

Que s tions : 67–78

(D) Perfor co plete l bor tory investig tion. (E) Qu ntify the ver ge ou nt he d rinks. 74. A 42-ye r-old d ivorced le p resents to you r office st ting th t for the p st 7 onths he h s been w orrying bout fin nci l nd w orkrel ted p roble s. H e st tes th t ore often th n not, he is w orrying bou t these issues throu ghou t the d y. H e h s d ifficu lt ti e st ying sleep, he feels tired ll d y long, nd he c n no longer concentr te d equ tely t w ork d u e to the w orrying. H e lso tells you th t he feels very tense in his shou ld er nd b ck. Which of the follow ing is the ost p propri te d i gnosis? (A) (B) (C) (D) (E)

Depend ent person lity d isord er Gener lized nxiety d isord er (GAD) M jor d ep ressive d isord er (MDD) P nic d isord er Soci l nxiety d isord er

Questions 75 and 76 You re interview ing 54-ye r-old rried w o n w ho h s been u rged to “see shrink” by her f ily. She d escribes sy p to s of feeling ineffectu l, believing th t the w orld is lw ys hostile to her, nd know ing th t things w ill never ch nge. 75. This tri d of sy p to s is ost ssoci ted w ith w hich of the follow ing d isord ers? (A) (B) (C) (D) (E)

Dep ressive d isord ers Dissoci tive d isord er GAD P nic d isord er Schizop hreni

76. Which of the follow ing tre t ents w ou ld ost likely t rget these sy p to s? (A) (B) (C) (D) (E)

Beh vior l ther p y Cognitive ther p y Cou p les ther p y Interp erson l ther p y P r d oxic l ther p y

179

Questions 77 and 78 A 55-ye r-old le w ith history of lcohol u se d isord er presents to you r office bec use he w ould like to stop d rinking lcohol. H e believes he d rinks bec u se he is d ep ressed nd w ishes to lso get tre tent for his d epression. After fu rther history is obt ined , it is reco end ed th t the first step is to bst in fro ll lcohol s it c n c u se d ep ression. H e grees to this pl n. 77. Wh t is the likelihood of his re d ep ressed if he is ble to refr in fro lcohol for 1 onth? (A) (B) (C) (D) (E)

ining u sing

5% 15% 25% 33% 50%

78. The bove p tient retu rns fter 6 w eeks of int ining sobriety fro lcohol. H e tells you th t he h s continu ed to feel d ep ressed . H e h s p roble s sleep ing t night, feels tired throughou t the d y, h s poor pp etite, nd he no longer d erives ple su re fro pl ying w ith his d ogs? Which of the follow ing ed ic tions w ou ld be the ost p p rop ri te to begin t this ti e? (A) (B) (C) (D) (E)

A itrip tyline (El vil) Bu sp irone (Bu Sp r) Lithiu (Esk lith) Lor zep (Ativ n) Sertr line (Zoloft)

Questions 79 and 80 A 52-ye r-old le w ith longst nd ing history of lcohol use d isord er is brought into the ED intoxic ted w ith lcohol. B sic l bs re d r w n nd he is found to be d ehyd r ted , so he is given IV fluid s long w ith glu cose. Abou t n hou r l ter, the physici n reev lu tes the p tient nd find s th t his speech is ore slurred , he is very confused , nd he now h s nyst g us.

180

6: Diffe re ntia l Diag nos is a nd Ma na g e me nt

79. Which of the follow ing is the next ost p propri te step in the n ge ent of this p tient? (A) (B) (C) (D) (E)

Ad inister Ad inister Ad inister Ad inister Ad inister ctiv tor.

ntibiotics. benzod i zep ine. hep rin. thi ine. tissu e p l s inogen

80. After d in istr tion of th e correct ed ic tion the p tien t i p roves signific ntly. H is f ily rrives th e n ext orn in g n d th ey tell you th t he h s recen tly been st rting to ke u p f cts bou t h is life. You ord er n MRI of his br in nd d iscover lesion s in h is ill ry bod ies. Wh ich of th e follow in g d i gn oses w ou ld best exp l in th ese sy p to s? (A) Alcohol-ind u ced jor neu rocognitive d isord er (Kors koff synd ro e) (B) Conversion d isord er (C) F ctitiou s d isord er (D) M jor neu rocognitive d isord er d u e to Alzhei er d ise se (E) Wernicke encep h lop thy Questions 81 and 82 A 27-ye r-old n co p l ins th t he h s felt “d ow n in the d u ps” for onths nd is feeling gu ilty bec u se he h s been h ving n extr rit l ff ir. In recent w eeks, he h s st rted to believe th t his w ife is p oisoning his food nd the rest of his f ily is involved in n el bor te plot to d rive hi fro the hou se. 81. Assu ing his thinking is d elu sion l, how w ou ld his d elu sions be best ch r cterized ? (A) (B) (C) (D) (E)

Biz rre Ego-syntonic Mood congru ent Mood incongru ent So tic

82. Which of the follow ing d i gnoses w ou ld ost likely be resp onsible for the bove d elu sions?

(A) (B) (C) (D) (E)

Ad ju st ent d isord er Bip ol r d isord er MDD Persistent d ep ressive d isord er Schizop hreni

83. A 45-ye r-old n is in the e ergency dep rtent bec use of di betic foot ulcer. In g thering history, the physici n le rns th t this n lives lone nd works nights s security gu rd. He s ys he h s no friends but th t this does not bother hi . He h s never been hospit lized or received ny psychi tric help. On MSE, his ffect is fl t. Although he nswers questions nd see s to trust the judg ent of the doctors, he h s little interest in the interview. He exhibits no signs or sy pto s of psychosis or depression. Wh t is the ost likely di gnosis? (A) (B) (C) (D) (E)

MDD P r noid p erson lity d isord er Schizoid p erson lity d isord er Schizop hreni Schizotyp l p erson lity d isord er

84. The id entified p tient is 30-ye r-old sep r ted fe le brou ght into the e ergency roo by her id entic l tw in sister. The p tient’s history is not ble for prior episod e of d epression 5 ye rs go su ccessfu lly tre ted w ith venl f xine. The p tient h s been st ying w ith her sister since her sep r tion 1 onth go. For the p st 2 w eeks, she h s been p cing rou nd the hou se, not sleep ing ore th n 2 to 3 hou rs per night. Desp ite her feeling “s d ” i ed i tely fter the sep r tion, the p tient now feels “w ond erfu l, like I c n cco p lish nything!” In f ct, she h s been tte p ting to re od el her sister ’s b throo , even thou gh she h s no tr ining or experience. H er sister h s been extre ely concerned bou t her, bu t she h s been u n ble to t lk to her bout it s, “I c n’t get w ord in ed gew ise, nd she d oesn’t lw ys ke sense.” The p tient is only t king or l contr ceptives nd o epr zole for cid reflux. H er sister is concerned th t she, herself, y eventu lly d evelop this illness. Wh t is her p p roxi te risk of d eveloping this d ise se?

Que s tions : 79–89

(A) (B) (C) (D) (E)

0% to 10% 10% to 20% 20% to 30% 50% to 70% 80% to 90%

85. A 33-ye r-old w o n w ith history of schizop h reni tells you she h s been h e ring voices for the p st 5 ye rs. Sh e h s b seline u d itory h llu cin tion th t she c nnot u nd erst n d ost of the ti e. Du ring p eriod s w here the voices beco e w orse, she typ ic lly he rs co nd u d itory h llu cin tions. Fro s fety st n d p oint, w hich of the follow ing is the ost i p ort nt infortion to obt in reg rd ing her u d itory h llu cin tions? (A) (B) (C) (D) (E)

H ow long she h s been he ring voices. H ow lou d the voices re. Wh t the voices re s ying. Whether she recognizes the voices. Whether the voices co e fro insid e or ou tsid e her he d .

86. A 38-ye r-old n co p l ins th t for the p st 2 ye rs he h s requ ired sever l n p s over the cou rse of the d y; he find s the n ps qu ite refreshing, but sees his d octor bec use l tely, s he is w king u p , he feels o ent rily “p r lyzed .” H e is cu rrently not on ny ed ic tions. H e d enies the u se of ny illicit su bst nces. Which of the follow ing is the ost likely d i gnosis? (A) Circ d i n rhyth sleep –w ke d isord er, d v nced sleep -ph se type (B) Circ d i n rhyth sleep –w ke d isord er, d el yed sleep -p h se typ e (C) H yp erso nolence d isord er (D) N rcolepsy (E) P r so ni 87. A 45-ye r-old le w ith history of op i te u se d isord er p resents to you r office co p l ining of p roble s w ith sleep . H e cl i s th t he h s been sober fro heroin for the p st 10 ye rs. H e sks to be st rted on ed ic tion for his inso ni . Which of the follow ing

181

ed ic tions w ou ld be the best choice for this p tient? (A) (B) (C) (D) (E)

Alp r zol (X n x) Cod eine (Brontex) Dip henhyd r ine (Ben d ryl) Qu eti p ine (Seroqu el) Zolp id e (A bien)

88. An 8-ye r-old girl p resents to her p ed i trici n w ith her p rents second ry to incre sed p roble s t school. She recently entered the third gr d e nd h s been h ving d ifficu lties w ith beh vior in cl ss. At ho e, her p rents d escribe her s ctive nd lw ys oving fro one ctivity to the next. At school, the child is u n ble to sit still in her ch ir nd frequ ently interru p ts the lessons. On ex in tion, you notice th t she is fid gety nd const ntly p l ying w ith her h nd s. When she tte p ts to co p lete t sk she is e sily d istr cted . Which of the follow ing w ou ld be the ost effective p h r cother p y for this p tient? (A) (B) (C) (D) (E)

Clonid ine I ip r ine Methylp henid te Risp erid one Sertr line

89. A 30-ye r-old le co es to n urgent c re f cility 8 hou rs fter the onset of u scle ches, ru nny nose, runny eyes, d ifficu lty sleeping, nd loose stools th t he ttribu tes to flu viru s. H e lives in d ifferent city nd is visiting for conference. H e rep orts th t he forgot his ed ic tions t ho e. H e t kes orphine IR every 6 hou rs for chronic low b ck p in, d ip henhyd r ine occ sion lly to help w ith sleep , venl f xine 150 g d ily for d ep ression nd Lisinopril for blood pressu re control. On ex in tion, he is sw e ting, y w ning, h s d il ted pu pils nd is ss ging his u scles. H is blood p ressu re is 160/ 95 H g, pu lse of 102 bp , nd he is febrile. When sked to cont ct his pri ry c re p hysici n he beco es incre singly git ted , st ting they w ou ld not contribute ny infor tion to this visit. L b v lues re ll w ith in nor l li its. Which

182

6: Diffe re ntia l Diag nos is a nd Ma na g e me nt

of the follow ing sy pto tology? (A) (B) (C) (D) (E)

ost likely exp l ins his

Anticholinergic toxicity H ypertensive crisis Influ enz Op i te w ithd r w l Serotonin synd ro e

92. Which of the follow ing is his d i gnosis?

Questions 90 and 91 A 28-ye r-old fe le w ith p st ed ic l history of hypothyroid is nd syste ic lu pu s erythe tosus (SLE) is hospit lized for lupu s fl re requ iring IV steroid s. You re consu lted for the p tient s she is co pl ining of ud itory h llu cin tions nd h s been beco ing incre singly git ted . H er ed ic tions inclu d e IV ethylp red nisone, u ltivit in, nd levothyroxine. On ex in tion, the p tient’s ood is l bile nd she end orse u d itory h llucin tions. H er vit l signs re st ble. H er TSH is 0.7 IU/ L nd her T4 is 7.2 u g/ d L. 90. Which of the follow ing is the d i gnosis? (A) (B) (C) (D) (E)

ost likely

Bipol r d isord er d u e to hyperthyroid is Brief psychotic d isord er MDD w ith p sychotic fe tu res Schizop hreni Steroid -ind u ced p sychotic d isord er

91. Which of the follow ing is the best next step in the n ge ent of this p tient? (A) (B) (C) (D) (E)

etfor in, u ltivit in, n OTC herb l su pp leent, tr d ol for chronic b ck p in, nd recent ntibiotic prescription for linezolid . H is vit l signs re: BP of 160/ 92, he rt r te of 110 bp , nd te p er tu re of 101.2°F. On ex in tion, he d i p horetic nd h s clonu s in his bil ter l low er extre ities.

Decre se steroid d os ge Incre se levothyroxine Initi te ECT Initi te lithiu Initi te sertr line

Questions 92 and 93 A 38-ye r-old n w ith p st ed ic l history of chronic low er b ck p in nd hypertension presents w ith friend to the e ergency roo co p l ining of git tion nd he d che for 2 d ys. Upon fu rther qu estioning, he d its to p st p sychi tric history of MDD w ith psychotic fe tures. Soon fter p resent tion he beco es confu sed nd nonresponsive. H is ed ic tion list inclu d es p henelzine, risp erid one,

(A) (B) (C) (D) (E)

ost likely

Bipol r d isord er H ypertensive crisis Migr ine w ith u r N eu rolep tic lign nt synd ro e (N MS) Serotonin synd ro e

93. Which of the follow ing is the best next step in the n ge ent of this p tient? (A) Continu e ll ho e ed ic tions. (B) Discontinu e linezolid , tr d ol, p henelzine. (C) Discontinu e etfor in. (D) Discontinu e over the cou nter herb l sup ple ent. (E) Incre se risp erid one. 94. A 58-ye r-old le w ith no p st ed ic l history is brou ght to the d octor ’s office by friend . The friend rep orts the p tient h s ccu u l ted too u ch “stu ff” in his one-story ho e nd h s not been ble to le ve his ho e. The p tient cl i s th t he y need these ite s t so e point in the futu re nd refu ses to throw nything w y. When sked if he h s ever tried to cle n out his hou se he gets d efensive, fid gety nd st tes he cou ld “ bsolutely never throw nything ou t!” H is friend brings in p ictu res of the p tient’s ho e show ing ound s of ite s in every roo . The p tient h s been u ne p loyed for 1 ye r, expl ining th t he need s the ti e to t ke c re of his ho e. Which of the follow ing is the ost likely d i gnosis? (A) (B) (C) (D)

GAD H o rd ing d isord er OCD Obsessive-co pu lsive person lity d isord er (OCPD) (E) P nic d isord er

Que s tions : 90–100

95. A 31-ye r-old fe le is fou nd in D ll s, Tex s sitting in coffee shop for n entire d y. When sked to le ve the coffee shop u p on closing, the p tient c nnot rec ll ny id entifying infortion or w here she is fro . The p olice re c lled , w ho escort her to n e ergency d ep rtent. When se rched , her ID c rd is fou nd in her p u rse, d ocu enting th t the p tient is fro Michig n. Fu rther investig tion reve ls her f ily rep orted her issing 2 w eeks go. Of note the p tient w s d op ted t you ng ge second ry to child bu se. Wh ich of the follow ing is the ost likely d i gnosis? (A) (B) (C) (D) (E)

Bord erline p erson lity d isord er Dep erson liz tion d isord er Dissoci tive nesi Dissoci tive id entity d isord er M jor neu rocognitive d isord er d u e to Alzhei er d ise se

Questions 96 and 97 A 26-ye r-old fe le p resents to her pri ry c re physici n w ith d ifficu lty sleep ing. She co pl ins of const ntly fid geting in bed w ith const nt urge to ove her legs. She gets u p nd w lks rou nd in ord er to help relieve it. This occu rs ost nights of the w eek nd h s gre tly interfered w ith her bility to sleep . Physic l ex in tion is neg tive for ny bnor lities. H er l b w ork show s low MCV, low ferritin, nd he oglobin of 9.0 g/ d L. 96. Which of the follow ing is the d i gnosis for this p tient? (A) (B) (C) (D) (E)

ost likely

GAD Inso ni d isord er N on-REM sleep rou s l d isord er Obstru ctive sleep p ne Restless legs synd ro e

97. Which of the follow ing is the ost p p rop rite step in the tre t ent of this p tient? (A) (B) (C) (D) (E)

Initi Initi Initi Initi Initi

te te te te te

hyd rocod one. iron rep l ce ent. lor zep . el tonin. risp erid one.

183

98. A 35-ye r-old le is u rged to go to psychi trist by cow orker bec u se of frequent nger ou tbu rsts. H e rep orts th t s long s he c n re e ber he h s h d “hot te p ter.” L tely, he h s been getting into to trou ble t w ork d u e to u ltip le verb l confront tions w ith cow orkers, w hich occ sion lly esc l te to the p tient throw ing v rious office objects cross the roo . H e feels very gu ilty fter, but he is u n ble to p revent his beh vior. H e h s no signific nt leg l proble s otherw ise. Which of the follow ing is the ost likely d i gnosis? (A) (B) (C) (D) (E)

Antisoci l p erson lity d isord er Bip ol r II d isord er Bord erline p erson lity d isord er Cond u ct d isord er Inter ittent exp losive d isord er

D IRECTION S (Questions 99 through 108): For each of the follow ing patients, choose the most appropriate diagnosis. Each lettered option may be used once, multiple times, or not at all. (A) (B) (C) (D) (E) (F) (G)

Antisoci l p erson lity d isord er Avoid nt p erson lity d isord er Bord erline p erson lity d isord er Dep end ent p erson lity d isord er H istrionic p erson lity d isord er N rcissistic p erson lity d isord er Obsessive-co p u lsive p erson lity d isord er (H ) P r noid p erson lity d isord er (I) Schizoid person lity d isord er (J) Schizotyp l person lity d isord er

99. A 45-ye r-old w o n recently is rele sed fro p rison for r ed robbery. She h s nu erous j il sentences for ste ling, bribery, nd rson. She end orses th t she s w p sychi trist s child for “beh vior l proble s,” getting into trou ble t school nd t ho e. Upon further questioning, she rec lls p eriod of ti e torturing her f ily’s p et c t s child . 100. A 57-ye r-old n presents to his pri ry p hysici n w ith co p l ints of bd o in l p in. H e is cco p nied by his other, w ith w ho

184

6: Diffe re ntia l Diag nos is a nd Ma na g e me nt

he lives. The p tient ppe rs to d efer to his other to nsw er ny of the qu estions d u ring the interview. The p tient t lks bou t how he c lls his other throughou t the w orkd y to sk her trivi l qu estions nd so she c n help hi ke ny, everyd y d ecisions. H e h s recently been ore nxiou s s he fe rs th t she is “old er” nd y d ie soon, “w hich w ou ld le ve e lone, help less, nd u n ble to cop e.” 101. A 30-ye r-old n w orks in lighthou se d u ring the overnight shift by hi self. H e h s no friend s, prefers to be lone in his sp re ti e, nd d oes not d esire to ke person l or inti te rel tionships. H e presents d ue to incre sed stress now th t he h s cow orker t his job. 102. A 34-ye r-old w o n h s just gone throu gh her second d ivorce nd is cu rrently in tu u ltu ous rel tionship w ith n she et 1 onth go. H er ex in tion reve ls sc rs on her w rist, w hich she expl ins re fro cutting herself “w hen I get ngry or up set.” She often feels e pty insid e bu t d its to intense ch nges of ood , qu ickly beco ing d espond ent or ngry. She reports it is ll her f u lt she feels this w y. 103. A 45-ye r-old w o n presents to her p ri ry c re ppoint ent p rovoc tively d ressed for rou tine ex in tion. She exu ber ntly exp l ins her cu rrent rel tionship w ith so eone she h s et only online nd not in person. She goes into gre t d et il reg rd ing his erits, cl i ing th t “he is the one.” H ow ever, t the end of the interview she sks the p hysici n if he is single. 104. A 32-ye r-old n p resents to the e ergency d ep rt ent fter suffering l cer tion to his left ind ex finger. When ed ic l stu d ent tte p ts to ev lu te hi he rebu kes hi , excl i ing “I only t lk to ttend ing p hysici ns.” H e is noted to be t lking lou d ly on his telephone bout how su ccessful he h s been in the fin nci l rket, how he is su re he w ill

see the Chief of the d ep rt ent, nd how he c nnot be “bothered ” by low -level e ployees. 105. A 46-ye r-old n is reco end ed to see e p loyee he lth s he continu ou sly believes his colle gu es t w ork re ou t to s bot ge his e p loy ent. As resu lt, he h s w ritten thre tening e ils to his peers nd su pervisors. When confronted w ith the f ct th t he h s been invited to t ke p rt in soci l ctivities w ith others, he st tes th t he is being “tricked into co p l cency” nd pl ns to sue the org niz tion. 106. A 21-ye r-old w o n w ho is in her third ye r of college p resents to ther p ist u p set th t she is u n ble to ke friend s. She exp resses th t she w ou ld like to join school clu b to eet new p eop le nd ke friend s bu t she is too shy. She d oesn’t go ou t to p rties d esp ite being sked , bu t “they w ou ld n’t like e nyw y.” 107. A 34-ye r-old w o n is new ly d i gnosed w ith hyp ertension. Upon interview, she d its th t she st ys l te t her job on regul r b sis bec use she spend s n excessive ount of ti e on e ch project, ensuring everything is in ex ct ord er nd properly for tted . She triple-checks not only her ow n but ll of her colle gues’ w ork. She d oesn’t believe th t others re “c rrying their lo d ” but believes she is the only one w ho excels t her job. She is d isppointed , how ever, bec use her fellow colle gues re ll ble to le ve t the ppropri te ti e. 108. A 32-ye r-old w o n p resents to her OB/ GYN for routine c re. She is noted to be d ressed w ith ulticolored sc rves, b ngles, nd l rge nu ber of cryst l jew elry. She st tes she is p l re d er s profession, believing she h s the tools to u nlock p erson’s fu tu re. She c e to her p point ent fter re d ing her strologic l forec st, bu t w s concerned bec u se of the “color of y ur .”

Ans we rs a nd Expla na tions

1. (C) The ost likely d i gnosis in this c se is schizo ffective d isord er. The p tient h s pro inent psychotic sy pto s, includ ing ud itory h llu cin tions, w hich re ru nning co ent ry of her beh viors, bu t lso concu rrent ood sy p to s, includ ing d epressed ood , d ecre sed sleep, nhed oni , nd poor ppetite. Also i p ort nt, the p tient’s h llu cin tions h ve occurred in the bsence of ood sy pto s, nd her ood sy pto s ppe r to h ve been present for jority of the tot l d ur tion of her illness. The presence of pro inent psychotic sy p to s in the bsence of ood sy pto s kes jor d epression w ith psychotic fe tures unlikely. A d i gnosis of schizophreni lone w ou ld not d equ tely ccou nt for this p tient’s ood sy pto s. Bipol r II d isord er is not v lid choice since there is no cle r history of hypo nic episod e. Schizoid person lity d isord er is unlikely bec u se th t d i gnosis w ould not ccount for the p tient’s overt psychotic nd ood sy pto s. 2. (E) In schizo ffective d isord er, it is i p ort nt to tre t both the ood nd psychotic sy pto s. Bec u se the p tient’s sy pto s inly consist of d ep ression nd p sychosis, co bin tion of n ntip sychotic nd ntid epress nt ed ic tion w ould be re son ble ppro ch. Mirt z pine, cit lopr , flu oxetine, nd sertr line re ntid epress nt ed ic tions, w hile zipr sid one, h lop erid ol, nd p erp hen zine re ntipsychotic ed ic tions. Lor zep nd d i zep re benzod i zepines. Div lp roex sod iu is ood st bilizer nd is u sed for bip ol r d isord er.

3. (C) The p tient ost likely h s d i gnosis of jor d ep ressive d isord er, w ith p erip rtu onset nd p sychotic fe tu res, since the d ep ression st rted w ithin 4 w eeks of giving birth to her son. H e ring child crying is co on p sychotic sy p to of p ostp rtu d ep ression. Althou gh the p tient h s been h ving p sychotic sy p to s, d i gnosing her w ith brief p sychotic d isord er w ou ld not fu lly ccou nt for her ood sy p to s. In ord er to eet the d i gnosis of schizo ffective d isord er, p tient u st h ve p sychotic sy p to s d u ring p eriod w hen there re no ood sy p to s p resent. Ad ju st ent d isord er w ou ld not ccou nt for her signific nt ood or p sychotic sy p to s. Postp rtu obsessive-co p u lsive d isord er w ou ld u su lly p resent s other w ith signific nt ou nt of nxiety d u e to ego d ystonic, obsessive thou ghts. 4. (A) The p tient w ou ld best be tre ted by inititing n ntid epress nt (su ch s n SSRI) nd n ntip sychotic (to d d ress her h llu cin tions). H osp it lizing this p tient w ou ld be pretu re bec use the other is still t king c re of her child nd is not h ving ny thou ghts of w nting to hu rt the child or herself. Lithiu is ind ic ted s ood st bilizer for so eone w ho h s d i gnosis of bip ol r d isord er. H loperid ol lone w ou ld help tre t her p sychosis, bu t w ou ld d o nothing to d d ress her d ep ression. Provid ing su pportive ther py could help the p tient, how ever, consid ering the severity of her illness, ed ic tions w ould need to be initi ted .

185

186

6: Diffe re ntia l Diag nos is a nd Ma na g e me nt

5. (E) This p tient presents with the sy pto s nd signs of Wernicke enceph lop thy, which results fro thi ine (vit in B1) deficiency. It is ch r cterized by bil ter l bducens nerve p lsy, horizont l nyst g us, t xi , nd glob l confusion cco p nied by p thy. E ch of the other choices is possible nd should be ctively ruled out. An cute subdur l he to would typic lly present fter tr u to the he d. Depending on the loc tion of the he to , p tients c n h ve proble s with speech, ltered ent l st tus, he iplegi , nd/ or nonre ctive pupil. Alcohol withdr w l usu lly presents with unst ble vit l signs, tre ulousness, nd git tion. Folic cid deficiency presents with di rrhe , cheilosis, nd glossitis; neurologic bnor lities re usu lly not seen. NPH is ssoci ted with the cl ssic tri d of de enti , incontinence, nd g it disturb nce. 6. (C) If Wernicke encep h lop thy is su sp ected , e ergent d inistr tion of IV thi ine is essenti l, bec u se ny sequ el e of thi ine d eficiency re reversible w ith this tre t ent; ist ken d inistr tion of thi ine is r rely h r ful. A CT sc n of the he d should be ord ered i ed i tely in this c se; opti lly, the IV thi ine w ould be st rted s the CT w s being rr nged. The re ining choices do not d dress the urgent need to replete thi ine. It should lso be noted th t giving this p tient ny glucose prior to d inistering thi ine c n result in w orsening of the Wernicke enceph lop thy (the et bolic conversion of glucose to pyruv te requires thi ine s cof ctor). 7. (A) Delu sion l d isord ers p resent w ith d elu sion (typic lly not biz rre) for t le st 1 onth. These p tients otherw ise h ve n org nized thou ght p rocess nd function w ell in other spects of their life. This p tient d oes not h ve ny ch r cteristics of bip ol r d isord er or d ep ression nd therefore w ould not be given d i gnosis of schizo ffective d isord er. P tients w ith obsessive-co p u lsive d isord er h ve nxiety over p rticu l r thou ghts th t they know re not r tion l (obsessions), nd they p erfor ritu lized beh viors to relieve their nxiety (co p u lsions). P r noid p erson lity d isord er cou ld be d i gnosed if the p r noi

w ere longst nd ing p roble th t ffected u ltip le sp ects of the p tient’s life but never re ched p sychotic (d elu sion l) level. In this c se, the only psychotic sy pto d escribed is bod y od or. In ord er to eet criteri for d i gnosis of schizop hreni , the sy p to s w ou ld l st for t le st 6 onths, the d elu sion w ou ld likely be biz rre, nd she w ou ld h ve other p sychotic sy p to s su ch s u d itory h llu cin tions, d isorg nized speech, d isorg nized beh vior, nd / or fl t ffect. 8. (E) Delu sion l d isord ers re typ ic lly tre ted w ith n ntip sychotic. Of the choices listed , the only ntipsychotic is ol nz pine. C rb zep ine is n ntiepilep tic th t is lso u sed to tre t bipol r d isord er. ECT c n be used for the n ge ent of cu te psychosis, but h s not been show n to be benefici l in the tre tent of d elu sion l d isord er. Flu oxetine is n SSRI nd is ost co only used for d epressive nd nxiety d isord ers. Lithiu is ood st bilizer nd is not used s tre t ent for d elusion l d isord er. 9. (D ) N MS is ch r cterized by severe (“le d p ip e”) rigid ity, ch nge in ent l st tus, utono ic inst bility, elev ted CPK, nd elev ted w hite blood cou nt; slight elev tion in cerebrosp in l flu id p rotein cou nt is p ossible. N MS y be ind u ced by ny ntipsychotic ed ic tions (neuroleptics), inclu d ing the typic l ntip sychotics. N o one sy p to is necess ry for the d i gnosis; inste d , constell tion of sy pto s nd their severity, in setting of ntip sychotic exp osure, ke the d i gnosis ore or less likely. Anticholinergic synd ro e, resu lting fro overd osing on nticholinergic ed ic tions, d oes not p rod u ce rigid ity nd n elev ted CPK. A CN S infection w ou ld be better su pported by ch r cteristic find ings on the lu b r p u nctu re nd CT or MRI sc n; n elev ted CPK w ou ld be possible if the CN S infection c u sed seizu res. M lign nt hyp erther i , n cute uscu l r p thologic p rocess, rese bles N MS, bu t follow s the d inistr tion of inh led nesthetic gents, s in gener l su rgery. Prolonged i obiliz tion cou ld resu lt in n elev ted CPK, bu t w ou ld not ccou nt for the other find ings.

Answe rs : 5–15

10. (C) Co nd u d itory h llucin tions re p sychi tric e ergency nd ost clinici ns w ou ld gree th t this p tient should be rest rted on n ntip sychotic ed ic tion. ECT h s so e effic cy in tre t ent-resist nt schizophreni bu t is not first-line ther py. Bec u se they re long cting (i.e., w ith long h lf-life) nd given his risk of fu ture N MS, d ep ot injections of neu rolep tic re not first choice. As this p tient is not violent, p hysic l restr ints shou ld be void ed . 11. (B) This w o n likely su ffers fro jor d ep ressive d isord er w ith p sychotic fe tu res. Co nd u d itory h llu cin tions to co it suicid e, such s overd osing on ed ic tions, re psychi tric e ergency nd u su lly requ ire psychi tric d ission. Therefore, d isch rging to n outp tient setting w ould not be reco end ed . While she w ill need co plete ed ic l w orku p to rule out ed ic l c uses of d epression, there is no ind ic tion for d itting her to ed ic l unit t this ti e; the w orku p c n usu lly be cco plished on psychi tric unit. There is no ind ic tion, su ch s violent beh vior, th t w ould necessit te physic l restr ints. 12. (D ) This p tient requ ires n ntid epress nt nd ntip sychotic ed ic tion. In f ct, neither n ntid ep ress nt nor n ntip sychotic lone tre ts jor d ep ressive d isord er w ith p sychotic fe tu res s w ell s both together. Althou gh benzod i zep ine y help this p tient’s irrit bility, s sole gent it is p oor choice for tre ting her d ep ressive nd p sychotic sy p to s. Lithiu d oes h ve so e ntid ep ress nt effects, bu t in the bsence of convincing evid ence th t this p tient h s bip ol r d isord er, lithiu is not the p p rop rite choice (If this p tient h s only p rti l resp onse to the SSRI, lithiu y t so e p oint in the fu tu re be d d ed for u g ent tion.). A TCA w ou ld not be the first choice. First, there is su ggestion th t this p tient h s so e c rd i c history nd the tricyclics c n h ve p ro rrhyth i effect. Second , u nlike SSRIs, tricyclics c n be leth l in overd ose nd this p tient h s risk f ctors for su icid e tte p t, su ch s recent bre ku p w ith her

187

boyfriend , co nd u d itory h llu cin tions, nd feelings of help lessness. 13. (A) The p tient p resents w ith p sychotic sy p to s nd git tion, long w ith qu estion ble nic sy p to s (e.g., d ecre sed sleep , gr nd iosity, incre sed p rod u ctivity). The d ifferenti l d i gnosis inclu d es bip ol r d isord er, schizo ffective d isord er, nd schizop hreni ( ong others). Initi l ed ic tions in this c se re i ed t the t rget sy p to s of git tion, d elu sion l thinking, nd d isru p tive beh vior nd w ou ld re son bly inclu d e n ntip sychotic nd benzod i zep ine. Bu sp irone is n nxiolytic u sed ostly in gener lized nxiety d isord er. It is not ind ic ted for bip ol r d isord er nd w ou ld not d equ tely d d ress the t rget sy p to s seen in this c se. While ood st bilizers su ch s c rb zep ine nd lithiu w ou ld tre t bip ol r d isord er, they w ou ld not r p id ly tre t his p sychosis nd git tion. SSRIs u st be u sed w ith extre e c u tion in bip ol r illness bec u se they c n incite or ex cerb te ni . 14. (A) When the p tient retu rns for his first follow -u p visit, his lithiu level is ther p eu tic t 0.8 Eq/ L nd is not in need of d ju st ent. H ow ever, his u scle stiffness is likely n extr -p yr id l sy p to (EPS) of h lop erid ol, n ely cu te d ystoni . Other EPS inclu d e p rkinsoni n sy pto s (e.g., br d ykinesi , cogw heel rigid ity, pill-rolling tre or, sklike f ce). EPS is nor lly tre ted by low ering the d ose of the ntipsychotic or st rting nticholinergic ed ic tions su ch s benztropine (Cogentin), not w ith u scle rel x nt su ch s b clofen. The p tient w ould h ve to be follow ed closely for ree ergence of p sychotic sy pto s. 15. (A) Acu te stress d isord er is re ction th t y occur in p tient w ho w itnesses or experiences tr u tic event. The event ust involve d e th, thre tened d e th, serious injury, or sexu l viol tion. The typic l sy pto s includ e hyper rou s l st tes, d issoci tive st tes, void nce beh viors, nd intrusive reexperiencing of the event (e.g., fl shb cks, night res). PTSD is si il r in ost respects to cu te

188

6: Diffe re ntia l Diag nos is a nd Ma na g e me nt

d istress d isord er, but in PTSD the sy pto s persist for t le st 4 w eeks follow ing the tr u ; shorter d ur tion suggests d i gnosis of cu te d istress d isord er. The p tient d oes not ppe r to be suffering fro ny of the typic l criteri for d just ent d isord er or MDD, except for gu ilt. Althou gh she h s nxiety nd void nce sy pto s si il r to gor phobi , there is no evid ence of p nic tt cks or p nic d isord er, nd her sy pto s re better ccounted for by her cute stress d isord er. 16. (C) While the p tient initi lly h d sy p to s consistent w ith cu te stress d isord er, ll of these sy p to s resolved w ith p sychotherp y. H d these sy p to s p ersisted over 1 onth, she w ou ld h ve et criteri for PTSD. When she p resents g in ye r l ter, she d isp l ys sy p to s consistent w ith MDD su ch s d ep ressed ood , nhed oni , p oor sleep nd p p etite, nd feeling tired ll d y long. Ad ju st ent d isord er shou ld not be d i gnosed w hen d i gnosis of MDD c n be d e. The p tient d oes not p p e r to h ve ny p nic tt cks nd she d oes not d escribe fe rs of h ving nother p nic tt ck w hich w ou ld be consistent w ith d i gnosis of p nic d isord er. 17. (B) Ocu logyric crisis is sp ecific ex p le of n cu te d ystonic re ction in w hich there is sp s of the u scles of extr ocu l r otion. N MS, lso re ction to ntip sychotic ed ic tions, is ch r cterized by rigid ity, fever, leu kocytosis, u tono ic inst bility, nd d eliriu . Torticollis nd retrocollis re d d ition l d ystonic re ctions th t refer to u scle sp s s th t c u se bnor l p ositioning of the he d , w hile tris u s is sp s of the j w u scles. 18. (B) Benzod i zep ines, SSRIs, nd cognitivebeh vior l ther p y e ch h ve p l ce in tre tent of the nxiety d isord ers. In this c se, w hile the p tient d oes h ve history of GAD, she d oes not h ve history of p nic tt cks. It w ou ld be u nco on for 59-ye r-old w o n to su d d enly d evelop p nic tt cks. Therefore, fu rther ed ic l w orku p is necess ry. A short d ifferenti l d i gnosis list w ould

inclu d e rrhyth i , hyp oglyce i , seizu re, nd orthost tic hypotension.

ngin ,

19. (B) Reg rd less of p tient’s p st p sychi tric d i gnosis, ed ic l c u ses of physic l signs nd sy p to s u st lw ys be ru led ou t. The nxiety d isord ers, lthou gh intensely d istressing nd u nco fort ble to p tients, r rely, if ever, resu lt in u nconsciou sness. M ny sp ects of the p tient’s story see to be rel ted to nxiety nd cou ld , in f ct, occu r d u ring p nic tt ck; how ever, this p tient’s p resent tion is ore consistent w ith ische ic he rt d ise se w hich shou ld p ro p t c rd i c w orku p . Illness nxiety d isord er is the w orry of h ving or obt ining serious illness d espite re ssurnce by ed ic l p rovid ers. 20. (B) P r p hilic d isord ers d enote ny intense nd p ersistent sexu l interest other th n genit l sti u l tion or fond ling w ith nor l, tu re, or consenting hu n p rtners. P r p hili s re fu rther d ifferenti ted b sed on w h t beh vior p rovid es the u rges or f nt sies. Frotteu ristic d isord er is obt ining rou s l fro tou ching or ru bbing g inst nonconsenting p erson. Exhibitionistic d isord er is the d esire to exp ose one’s genit ls to nonconsenting p erson. Sexu l sochis d isord er is the d esire to be hu ili ted by nother p erson, w hile sexu l s d is d isord er is the d esire to inflict p hysic l or p sychologic l su ffering on nother p erson. Voyeu ristic d isord er is rou s l fro w tching u nsu sp ecting p eop le u nd ressing, n ked , or h ving sex. A few other p r p hili s inclu d e: p ed op hilic d isord er, sexu l f nt sies, u rges, or cts involving sex w ith p rep u bescent child ren; tr nsvestic d isord er, sexu l rou s l fro cross-d ressing; nd fetishistic d isord er, w hich is sexu l rou s l fro the u se of nonliving objects or nongenit l bod y p rts. 21. (E) While ny of the sid e effects listed y occu r w ith the u se of SSRIs, ost of the w ill d i inish or resolve over ti e. Sexu l d ysfu nction, either nifesting s d el yed org s or d ecre sed libid o, c n re in sid e effect in 25% to 80% of p tients fter 3 onths of tre t ent.

Answe rs : 16–27

22. (A) The ost co on c use of d e th w ith tricyclic ntid epress nt overd ose is c rd i c rrhyth i . The rrhyth i is d u e to the d ru g’s block d e of sod iu ch nnels in the yoc rd iu le d ing to c rd i c cond u ction slow ing. Seizu res, shock, resp ir tory f ilure, nd co c n lso occu r, bu t re not the ost co on c u ses of d e th. Strokes re typ ic lly not c used by overd osing on tricyclic ntid ep ress nts. 23. (E) The key p oints in this c se p resent tion re the bsence of previou s psychi tric history nd the d u r tion of sy pto s. Before d i gnosing pri ry ent l illness, it is necess ry to ru le ou t the effects of su bst nce. A u rine toxicology screen usu lly includ es opi tes, coc ine, c nn binoid s, p het ines ( nd so eti es PCP). A phet ines, coc ine, nd PCP intoxic tion c n i ic sy p to s of schizophreni . CBC, ESR, nd liver fu nction tests re u nlikely to id in the d i gnosis. Althou gh CT sc n is help fu l to ru le ou t ss lesion or bleed ing, these re not likely given his ge nd the nonfoc l neu rologic ex in tion. 24. (D ) Delu sions, h llu cin tions, nd d isorg nized beh vior re so e of the h ll rk sy pto s of schizophreni . Bec use the sy pto s h ve been present for ore th n 1 onth bu t less th n 6 onths ( nd there is no p reviou s p sychi tric history), the ost p p rop ri te d i gnosis is schizop hrenifor d isord er. Delu sion l d isord er u su lly nifests in id d le- ged ind ivid u l w ho, w hile fu nction l in society, exp eriences circu scribed d elusion of the je lou s, eroto nic, gr nd iose, p ersecu tory, or so tic typ e. As this p tient is lso su ffering fro pro inent u d itory h llu cin tions, this w ou ld exclu d e the d i gnosis of d elusion l d isord er. There is no history su ggestive of ood d isord er nd thu s jor d epression w ith psychotic fe tu res is not s likely. Since the toxicology screen is neg tive, the d i gnosis of su bst nce-ind u ced p sychotic d isord er is less likely. 25. (C) The p tient is ost likely exp eriencing lcohol w ithd r w l s d e onstr ted by the

189

f ct th t since being hospit lized 2 d ys go (his l st d rink) he h s h d n incre se in his blood p ressu re, he rt r te, nd h s incre sed nxiety, s w ell s d i p horesis nd tre ors. Oftenti es, p tients c n lso beco e febrile. Benzod i zep ines re the tre t ent of choice for lcohol w ithd r w l. Any benzod i zep ine c n be u sed —the choice d ep end s on the d esired rou te of d elivery or the p resence of liver d ysfu nction. With d ecre sed liver fu nction, lor zep (Ativ n) or ox zep (Ser x) is p refer ble bec u se they re p ri rily et bolized nd eli in ted ren lly inste d of throu gh the liver ( s re the other benzod i zep ines). Ad d ition lly, lor zep y be chosen bec use it c n be given IV, intr u scu l rly (IM), or by ou th (PO). C rb zep ine nd / or p ropr nolol h ve been u sed in so e settings for lcohol w ithd r w l, but neither re reco end ed for severe lcohol w ithd r w l, d eliriu tre ens, or seizu res. Clonid ine nd / or eth d one re u sed for op i te w ithd r w l. N ltrexone h s been u sed to help int in bstinence fro lcohol bu t h s no effic cy in w ithd r w l. 26. (E) Alcohol w ithdr w l c n be life thre tening. Tre ors begin bout 6 to 8 hours fter cess tion, follow ed by perceptu l bnor lities (h llucinosis) w ithin 8 to 12 hours. Seizures y occur nyw here fro 12 to 72 hours fter cess tion of lcohol. Deliriu tre ens (DTs) y occur ny ti e in the first w eek of bstinence. In the p st, before i proved d i gnosis nd n ge ent, untre ted DTs h d ort lity r te of pproxi tely 15%. Abd o in l p in nd uscle cr ps re co on sy pto s of opi te w ithd r w l. Cirrhosis nd f tty liver re long-ter consequences of lcohol use. 27. (E) This p tient is exp eriencing cu te intoxic tion w ith PCP s illu str ted by his git tion, p sychosis, vit l sign inst bility, nd tics. A toxicology screen is the best w y to d i gnose this cond ition. Urine toxicology screens co only includ e op i tes, coc ine, c nn bis nd PCP, but this v ries d epend ing on ind ivid u l institu tions. A CBC w ou ld not id in the d i gnosis of this p tient’s su bst nce-ind u ced psychotic d isord er. There is no ind ic tion for n

190

6: Diffe re ntia l Diag nos is a nd Ma na g e me nt

EEG t this ti e bec u se no seizu re history is elicited nd the p tient d oes not p p e r p ostict l. Althou gh structu r l br in lesions c n c u se psychotic sy pto s, subst nceind u ced sy p to s re ore co on in this ge grou p. As in this c se, nonfoc l neu rologic ex in tion w ou ld d ecre se the likelihood th t i ging stu d ies w ou ld lter the d i gnosis or tre t ent pl n. 28. (A) Unless the p tient is extre ely git ted nd t risk for hurting hi self or others, the best choice for tre t ent w ou ld be to p l ce hi in qu iet, d rk (low sti u l tion) roo . If the p tient is violent nd / or p sychotic, co bin tion of ntipsychotics (neu rolep tics) nd benzod i zepines cou ld be u sed to keep the p tient s fe. Methylphenid te is sti ul nt, w hich w ou ld likely w orsen his git tion. Benztrop ine, u sed to p revent extr p yr id l sy pto s ssoci ted w ith ntipsychotic use, ight w orsen the p tient’s d eliriu . Bec u se the p tient is not su ffering fro seizures, p henytoin is not ind ic ted . Bet blockers su ch s p rop r nolol re u sed for blood pressure nge ent s w ell s k thisi c u sed by ntip sychotics. There is no ind ic tion for its u se in this p tient. 29. (E) The first p rt of the vignette illu str tes so e of the fe tures of op i te intoxic tion, n ely p up ill ry constriction, d row siness, i p ired ttention nd e ory, nd slu rred sp eech. In the second inter ction, the p tient show s signs nd sy pto s of opi te w ithd r w l. Y w ning, u scle ches, d i rrhe , l cri tion or rhinorrhe , nd fever re ll typic l of opite w ithd r w l. Ad d ition lly, ild ly elev ted vit l signs y occu r; how ever, signific nt t chyc rd i nd hypertension shou ld lw ys sign l possible lcohol w ithd r w l. Ph r cologic n ge ent of opi te w ithd r w l c n inclu d e clonid ine, centr lly cting lp h 2- gonist, nd eth d one, synthetic longcting opi te. Lor zep c n be u sed to tre t elev ted vit l signs nd lop er id e c n be u sed to tre t d i rrhe . 30. (E) Tri zol is short- cting benzod i zep ine. Su d d en d iscontinu tion of ny sed tive,

hyp notic, or nxiolytic gent, su ch s benzod i zepines, c n c use signific nt w ithd r w l sy pto s, includ ing seizu res. Flu phen zine nd thiorid zine re ntipsychotics, nd i ipr ine nd nortriptyline re TCAs. N either ntipsychotics nor TCAs c use life-thre tening w ithd r w l sy p to s. 31. (B) As with the sudden discontinu tion of lcohol, benzodi zepine or b rbitur te withdr w l c n be life thre tening. The ost concerning discontinu tion side effect is seizures. Other withdr w l sy pto s include tre ors, nxiety, uditory, visu l, nd t ctile h llucin tions, nd utono ic hyper ctivity. The onset of sy pto s depends on the h lf-life of the edic tion, the dose, nd dur tion of use. Vo iting is not co on side effect of sed tivehypnotic withdr w l. 32. (B) The p tient is exhibiting typ ic l signs of d eliriu , includ ing n i p ired level of consciousness nd d ifficu lty w ith cognition, nifested s d row siness, p oor ttention, nd d isorient tion. Cl ssic lly, these sy pto s y fluctu te d u ring the d y. P tients w ho eet criteri for d eliriu y ppe r d ep ressed w ith d ecre sed level of rou s l or y p pe r overly ctive nd energetic, w ith the d ep ressed typ e occu rring ore co only. P tients lso show cognitive nd e ory i p ir ents, w hich y be nifested s d isorient tion or d ecre sed c p city to register, ret in, nd rec ll infor tion. An MMSE y lso be low d u e to n und erlying jor neu rocognitive d isord er (d e enti ) or d epressive illness. In this setting, it w ou ld be i p ort nt to g ther infor tion fro f ily or other ind ivid u ls w ho know the p tient in ord er to ev lu te for p ossible coexisting d e enti or jor d epression. Risk f ctors for d eliriu re extre es of ge, prior history of d eliriu , lcohol use d isord er, sensory i p ir ent ( u d itory or visu l), nd pre-existing br in d ge. There is bro d etiologic d ifferenti l for d eliriu . In this c se, c us tive f ctors y be infection, hypoxi , et bolic bnor lities, nd cu rrent ed ic tions. It is d ifficu lt to d i gnose n u nd erlying d e enti or MDD in d eliriou s

Answe rs : 28–38

p tient. There is no ind ic tion of n nxiety d isord er or p tient p u rp osely ind u cing sy pto s s in f ctitious d isord er. 33. (D ) A high-potency ntipsychotic such s h loperid ol is the d rug of choice for n ging the git ted or confused p tient w ith deliriu . Benztropine, d iphenhyd r ine, nd thiorid zine ll h ve nticholinergic properties th t y ctu lly w orsen the d eliriu . Lor zep , especi lly in older p tients, y h ve p r d oxic l d isinhibiting effect nd ex cerb te git tion nd confusion. H loperid ol in low d oses, such s 0.5 g every 8 hours, helps to n ge the git ted p tient. It lso h s the d v nt ge of IM, IV, or PO d inistr tion. It is gener lly reco ended to obt in n EKG to ev lu te for QTc prolong tion prior to initi tion. 34. (B) The cluster B person lity d isord ers re ch r cterized s the d r tic nd e otion l p erson lity grou p. Bord erline PD p tients d isp l y longst nd ing p ttern of inst bility in ny sp ects of their lives. They see things in either “bl ck or w hite,” w hich contribu tes to gre t d e l of ffect d ysregu l tion. Self- u til tion su ch s su p erfici l cu tting or bu rning is co on. H istrionic PD p tients typic lly d ispl y dr tic bu t su p erfici l e otions, w ith over-sexu lized beh viors. N rcissistic PD p tients d isp l y n overv lued sense of self, entitle ent, nd r ge. Bip ol r d isord er nd jor d epression re ffective d isord ers th t p resent s h ving p eriod s of good fu nctioning long w ith p eriod s of i p ir ent or d istress d uring the cu te episod es (e.g., d u ring nic or d epressed ph ses). 35. (D ) Di lectic l beh vior l ther p y is consid ered the tre t ent of choice for p tients w ith bord erline p erson lity d isord er. Cognitive beh vior l ther p y is p roble -solving p sychother p y specific lly u sed for d istorted ssu ptions nd neg tive feelings th t c n ex cerb te p sychi tric sy p to s like d ep ression nd nxiety. Psycho n lysis c n be used for d ifferent psychi tric illnesses; how ever, it is not consid ered first-line tre t ent for bord erline p erson lity d isord er. Med ic tions, inclu d ing ntip sychotics su ch s rip ip r zole,

191

nd SSRIs su ch s cit lop r , c n be effective in tre ting bord erline p erson lity d isord er; how ever, ed ic tions tend to be ore usefu l in conjunction w ith psychother py nd shou ld be t ilored to specific sy pto s. 36. (B) This p tient is exhibiting signs nd sy pto s consistent w ith MDD w ith p sychotic fe tu res. For ore th n 2 w eeks, the p tient h s d ispl yed d ep ressed ood , loss of interest in p le su r ble ctivities, w eight loss, in bility to concentr te, poor sleep, nd guilt bou t her current d epressive st te. She is lso su ffering d elusion l concerns bout bod ily fu nctioning nd infection. If the p tient were co pl ining of nond elu sion l cont in tion fe rs (obsessions) nd perfor ing ultiple co pu lsive t sks to rid herself of the p thogens, d i gnosis of OCD cou ld be consid ered . Persistent d ep ressive d isord er (for erly d ysthy ic d isord er) presents w ith chronic od er te level of d ep ressive sy p to s for t le st 2 ye rs; how ever, this p tient is d escribing n cu te, severe, ep isod ic d eco pens tion over the l st onth. A p tient w ith so tic sy p to d isord er presents w ith t le st one physic l sy pto th t c uses signific nt d istress, resu lting in soci l or occu p tion l d ysfu nction. 37. (E) Bec use of the d elu sion l qu lity to her p resent tion, the tre t ent of choice is both n ntid ep ress nt nd n ntip sychotic, or electroconvu lsive ther p y (ECT). Antid ep ress nts lone w ith or w ithout benzod i zepines w ou ld not help w ith her p sychotic sy p to s. Div lp roex sod iu nd lithiu re u sed s ood st bilizers in bip ol r d ep ression or ni , bu t not co only in the tre t ent of u nip ol r d ep ression. 38. (A) Brief psychotic d isord er is d i gnosed w hen sy pto s h ve been persistent for ore th n 1 d y bu t less th n 1 onth. Schizop hreni w ou ld be d i gnosed if the p tient’s sy p to s occu rred for longer th n 6 onths, nd schizop hrenifor d isord er w ou ld be d i gnosed if these sy pto s persisted for gre ter th n 1 onth bu t few er th n 6 onths. Delu sion l d isord er is cond ition usu lly beginning in id d le ge in w hich the p ri ry sy p to is

192

6: Diffe re ntia l Diag nos is a nd Ma na g e me nt

gener lly nonbiz rre d elu sion. Delu sion l d isord er is further cl ssified into subtypes, inclu d ing gr nd iose, je lou s, p ersecu tory, so tic, nd eroto nic. There is no cle r history or evid ence on physic l ex in tion of d rug use consistent w ith su bst nce-ind u ced p sychotic d isord er. 39. (E) The ost helpful test would be urine toxicology screen, since intoxic tion with v riety of subst nces, including coc ine, PCP, c nn bis, or phet ines y i ic the sy pto s of schizophreni . It is necess ry to first rule out these intoxic tion syndro es prior to offici lly king di gnosis of pri ry psychotic illness, such s schizophreni . There is no specific d i gnostic u tility for CBC or electrolytes for ny of the p ri ry p sychotic d isord ers. Bec u se the p tient d oes not p resent w ith signs or sy p to s of seizu re d isord er, n EEG is u nlikely to p rovid e infor tion th t w ill i p ct the d i gnosis. A CT sc n y help to ru le ou t ss lesion or bleed , bu t his ge nd nonfoc l neu rologic ex in tion ke this n u nlikely contribu tion to his p resent tion.

w hich is lso co on sy p to of d ep ression. Most of the bove sid e effects i prove fter sever l d ys. Ak thisi (inner sense of restlessness) nd constip tion re co on sid e effects of ntipsychotics. 42. (C) N or l grief c n ppe r si il r to jor d ep ressive ep isod e (MDE), bu t there re w ys to d istingu ish betw een the tw o. While bere ve ent is not p sychi tric d i gnosis, it is psychosoci l stressor th t c n precipit te n MDE in vu lner ble ind ivid u l. Non–bere ve ent-rel ted depression responds to the s e ed ic tions nd tre t ents s bere ve ent-rel ted d ep ression. MDE shou ld be consid ered if the d ep ressive sy p to s l st for longer th n 2 w eeks, nd ssoci ted w ith p sycho otor ret rd tion, ch nge in pp etite/ w eight, d ecre sed energy nd concentr tion, nd thou ghts of su icid e. The resp onse to norl bere ve ent exclu d es the d i gnosis of d ju st ent d isord er. A d i gnosis of p ersistent d ep ressive d isord er shou ld be consid ered for p tients w ho h ve d epressive sy pto s ore often th n not for 2 ye rs. Schizophreni w ou ld not be n p prop ri te d i gnosis s the p tient d oes not h ve previou s psychi tric history, d oes not h ve other p sychotic sy pto s, nd the h llu cin tions re better ccou nted for by bere ve ent.

40. (B) The p tient h s d i gnosis of MDD s ch r cterized by the d ep ressed ood , p oor concentr tion, p oor sleep , p oor p p etite, nd gu ilty feelings. Althou gh it p p e rs th t the p tient’s d ep ression w s triggered by d e th in the f ily, she nevertheless eets criteri for MDD. There re tw o choices for ntid ep ress nts in this grou p . Flu oxetine, n SSRI, is the best choice t this ti e. Phenelzine, n MAOI, is not the first choice of ntid ep ress nt s the SSRIs h ve ore f vor ble sid e effect p rofile, re s fer in overd ose, nd h ve si il r effic cy in the tre t ent of MDD. Flu zenil is benzod i zep ine nt gonist u sed for the tre t ent of benzod i zep ine overd ose, flu p hen zine is n ntip sychotic, nd lor zep is benzod i zep ine. N one re p p rop ri te first-line tre t ents for MDD.

43. (E) At this ti e, the p tient is exp eriencing nor l bere ve ent nd no fu rther interventions re necess ry. Shou ld the sy p to s continu e for longer period of ti e, w orsen, or the p tient beco es less fu nction l, initi ting n ntid ep ress nt su ch s flu oxetine ( n SSRI) or, less frequ ently, itriptyline ( TCA) w ou ld be p p rop ri te. The p tient’s u d itory h llu cin tions re still consid ered to be nor l p rt of bere ve ent t this ti e, so n ntipsychotic like h loperid ol should not be initi ted u nless the voices beco e ore d istressing nd p erv sive. Lithiu is ood st bilizer ind ic ted for bipol r d isord er.

41. (D ) SSRIs h ve nu ber of sid e effects, inclu d ing d i rrhe , constip tion, n u se , he d che, sexu l d ysfunction, nd git tion. A frequ ent, initi l sid e effect is inso ni ,

44. (B) Ad ju st ent d isord er is ch r cterized by the onset of e otion l or beh vior l d istu rb nces w ithin 3 onths of signific nt life event th t y nifest s rked ch nge in

Answe rs : 39–48

193

n ind ivid u l’s bility to fu nction in school, w ork, or interp erson l rel tionship s. The d istu rb nces re not so severe, how ever, s to su ggest the d i gnosis of nother d isord er such s MDD. The d ifferenti l d i gnosis for d ju st ent d isord er inclu d es ood d isord ers such s MDD, nd nxiety d isord ers su ch s PTSD, GAD, nd cute stress d isord er. In this c se, the p tient y be su ffering fro d ep ressed ood bu t d oes not h ve other neu roveget tive signs of d ep ression. Acu te stress d isord er is d i gnosed w hen n ind ivid u l w ho h s exp erienced n extre e stressor h s d issoci tive exp eriences nd nxiety shortly fter the tr u tic experience. In this c se, the p tient h s exp erienced signific nt psychosoci l stressor, bu t it is not life-thre tening event s is requ ired for the d i gnosis of cu te stress d isord er. In d d ition, this p tient is not d escribing these sy pto s. The p tient is not suffering nxiety bout nu ber of events for over 6 onths consistent w ith GAD. Grief is the nor l response to the loss or d e th of n ind ivid u l close to the p tient.

of h ving n tt ck or being u n ble to esc pe d uring n tt ck force p tients to re in in f ili r pl ces. The ed ic l w orku p for p nic d isord er inclu d es CBC, electrolytes, f sting glu cose, c lciu , liver fu nction tests, BUN , cre tinine, u rin lysis, toxicology screen, ECG, nd thyroid fu nction stu d ies. A c refu l su bst nce u se nd c ffeine int ke history should lso be obt ined . GAD occu rs in ind ivid u ls exp eriencing continu ed nxiety nd w orry over sever l events l sting for gre ter th n 6 onths. In contr st to p nic d isord er, there re not u su lly d iscrete ep isod es in GAD. Sep r tion nxiety d isord er is u su lly d i gnosed in you ng child ren w ho d isp l y excessive nxiety w hen sep r ted fro their pri ry c regiver. Soci l nxiety d isord er occu rs in the context of pu blic p pe r nces or p erfornces, w here ind ivid u ls exp erience nxiety nd fe r. These feelings c u se gre t d istress nd y c u se p eop le to void these settings. This d isord er d iffers fro p nic d isord er by h ving n id entifi ble trigger. Specific phobi s lso h ve n id entifi ble trigger.

45. (D ) The d i gnosis of d ju st ent d isord er d oes not u su lly require ph r cother peu tic intervention, bu t inste d y resp ond to su p p ortive, ind ivid u l, or grou p psychother p y. Di lect l beh vior l ther p y is the first-line tre t ent for bord erline person lity d isord er p tients, help ing d ecre se self-d estru ctive beh viors nd i p roving ffect regu l tion. Flu oxetine is n SSRI, nd nortrip tyline is TCA, both ntid ep ress nts. Risp erid one is n typic l or second -gener tion ntip sychotic.

47. (D ) A v riety of ed ic tions, inclu d ing SSRIs (sertr line), TCAs, MAOIs (tr nylcypro ine), nd benzod i zepines ( lpr zol ) h ve been used to tre t p nic d isord er. Althou gh ll h ve show n so e effic cy in the tre t ent of p nic, SSRIs re usu lly the first gent of choice bec use they re gener lly w ell toler ted , w ith few er sid e effects th n TCAs or MAOIs, nd l ck the d epend ency potenti l of benzod i zepines. Co on sid e effects inclu d e sexu l d ysfunction, g strointestin l d isturb nces, nd inso ni . Lithiu is used in the tre t ent of bipol r d isord er, nd propr nolol c n be used to n ge the p hysic l sy pto s of nxiety in soci l nxiety d isord er (soci l phobi ).

46. (B) This p tient is d escribing sy p to s ssoci ted w ith p nic d isord er. This d isord er begins in l te d olescence or e rly d ulthood nd is ore co on in w o en th n in en. Peop le typic lly d escribe recu rrent nd u np rovoked p nic tt cks, consisting of feelings of fe r, d re d , or intense d isco fort ssoci ted w ith v riety of so tic sy pto s, inclu d ing t chyc rd i , chest p in, shortness of bre th, tre u lousness, d i phoresis, n use , fe r of d ying, p resthesi s, light he d ed ness, nd hot or cold fl shes. The d i gnosis of gor p hobi is d d ition lly d e w hen fe rs

48. (B) The p tient likely h s obsessive-co pulsive d isord er (OCD) s she su ffers fro recu rrent, intru sive, u nw nted thou ghts or i ges (obsessions) nd repetitive beh viors or ent l cts (co pu lsions) th t re p erfor ed in resp onse to these obsessions. P tients re lize th t the obsessions nd co p u lsions re u nre son ble, nd they c u se rked i p irent in their lives. M ny p tients w ith OCD

194

6: Diffe re ntia l Diag nos is a nd Ma na g e me nt

hid e their sy p to s nd y first co e to the ttention of other ed ic l speci lists. GAD is suit ble d i gnosis in ind ivid u ls w ho h ve w orries nd concerns th t re not li ited to one p rticu l r sp ect of their lives, bu t w ith the nxiety signific ntly i p iring their bility to fu nction. OCPD is person lity d isord er in w hich p tients beco e p reoccup ied w ith perfectionis nd ord erliness. H ow ever, there re no recu rrent obsessions or co pu lsions, nd the sy pto s re egosyntonic. P nic d isord er nifests s recurrent, d iscrete ep isod es of extre e nxiety nd fe r th t occu r unexp ected ly nd re it spont neously. So tic sy pto d isord er p resents w ith t le st one p hysic l co p l int th t c nnot be fully expl ined by the results of ed ic l investig tions. 49. (A) A w id espre d nd effective tre t ent for OCD is cognitive-beh vior l ther py, d u ring w hich the p tient is exp osed to the thre t or fe r (obsession) w ith the response (h nd w shing) p revented for p rogressively longer p eriod s of ti e u ntil the beh vior is extingu ished . SSRIs h ve lso been su ccessfu l in the tre t ent of OCD. Lithiu is u sed in the tre t ent of bipol r d isord er nd h s not been show n to be effective in the tre t ent of OCD. Psychod yn ic p sychother p y is intensive, long-ter ther p y th t y be help fu l for p eople w ho w ish to better u nd erst nd the n tu re of their rel tionship w ith others, bu t it h s not d e onstr ted signific nt effic cy in OCD. Risp erid one is n typ ic l ntip sychotic th t is u sed to tre t psychotic d isord ers nd is not first line for tre ting OCD. 50. (A) Sy pto s of excessive w orry nd nxiety bou t sever l sp ects occu rring for over 6 onths re best ch r cterized by the d i gnosis of GAD. Other sy pto s of GAD inclu d e sleep d ifficu lties, irrit bility, nd d ifficulty concentr ting. P tients w ith GAD y lso su ffer fro u scle tension, f tigu e, nd restlessness. Althou gh this p tient d oes d it to nxiety, it d oes not occu r in resp onse to p nic tt cks or in response to p ublic p erfornces, s w ith p nic d isord er or soci l nxiety d isord er, resp ectively. Si il rly, lthou gh

the sy pto s of nxiety nd w orry re in excess of w h t w ou ld be exp ected , they d o not re ch the gnitu d e ch r cteristic for OCD, nor re ny sp ecific obsessions or co pu lsions elicited . There is no evid ence of p sychosis in this p tient consistent w ith schizophreni . 51. (A) The d i gnosis of conversion d isord er (fu nction l neurologic l sy pto d isord er) is ost likely given th t the p ri ry sy p to s re neurologic nd the w orkup is neg tive for evid ence of p thology th t w ould ccount for the d eficits. N eu rologic d eficits in conversion d isord er involve either otor or sensory od lities nd re believed to be resu lt of u nd erlying, u nconsciou s p sychologic l conflicts or stressors. H d the sy pto s been intention lly p rod u ced , the d i gnosis of f ctitious d isord er or lingering w ould be p p ropri te. If the p ri ry incentive for the beh vior w s to ssu e the sick role, the d i gnosis w ou ld be f ctitiou s d isord er. If the sy p to s re feigned or ex gger ted in ord er to g in onet ry rei bu rse ent or to void j il or w ork, the d i gnosis w ou ld be lingering. So tic sy pto d isord er is less ppropri te in this c se bec u se this p tient’s sy p to s ren’t d istressing to her nd re solely neu rologic co p l ints. An u nd i gnosed neu rologic cond ition is possibility, but given the p resent tion nd physic l ex in tion, it is less likely. More recent stu d ies h ve show n only 4% isd i gnosis of conversion d isord er w hen there is n u nd erlying neu rologic l d isord er c using the sy pto s. 52. (B) F ctitiou s d isord er (Mu nch u sen’s) is n ppropri te d i gnosis w hen physic l or psychologic l sy pto s re intention lly prod uced in ord er to ssu e the sick role. It ust be cle r th t these sy pto s re not intentionlly prod uced for other re sons, such s void ing w ork, g ining onet ry co pens tion, or void ing leg l issues. If ny of these re sons re present, the d i gnosis is lingering. So tic sy pto d isord er presents w ith t le st one or ore so tic sy pto s, w hich c n be p in. The d i gnosis of illness nxiety d isord er is not ppropri te s this p tient is not preoccupied w ith the id e of h ving severe d ise se th t

Answe rs : 49–58

is unrecognized d espite repe ted re ssur nces by ed ic l personnel. Cl ssic lly, people w ith f ctitious d isord er re ore likely to be fe le nd to h ve experience in he lth c re. Often, these p tients w ill h ve nu erous d issions to ny d ifferent hospit ls w ith v riety of sy pto s. The p sychologic l f ctors u nd erlying this d isord er re not w ell und erstood . N o effective for of psychother py or ph r cother py h s been id entified for the tre t ent of f ctitiou s d isord er. 53. (A) The ost likely d i gnosis in this p tient is norexi nervos nd is su p p orted by the f ct th t she p pe rs very u nd erw eight, excessively exercises nd continu es to feel th t she need s to lose ore w eight, thu s d e onstr ting her d istu rbed bod y i ge. Buli i nervos c n be exclu d ed bec u se she d oes not h ve ny binge e ting. While the p tient d oes h ve so e nxiety, it is not cle r th t her nxiety h s re ched to level th t it is c u sing ny typ e of d ysfu nction in her d ily ctivities, consistent w ith GAD. P tients w ith bod y d ysorphic d isord er gener lly focu s on one sp ecific d efect r ther th n so ething s gener l s w eight. While the p tient d oes h ve sy p to s su ggestive of MDD, su ch s proble s w ith sleep , p oor concentr tion, nd f tigu e, she d oes not ention feeling d ep ressed or h ving nhed oni , w hich re requ ired for d i gnosis of MDD. 54. (E) Obt ining height nd w eight is cruci l for est blishing the d i gnosis of norexi nervos , s p tients w ith norexi nervos re u n ble to chieve or int in gre ter th n 85% of exp ected bod y w eight for height nd fr e stru ctu re. Ad d ition lly, they h ve continu ed concerns bout thinness nd bod y i ge d esp ite being u nd erw eight. In contr st, ost p tients w ith bu li i nervos re of nor l height nd w eight or re slightly overw eight. While the other tests listed y, ind eed , be bnor l in norexi ( nd bu li i ), they re not necess ry or su fficient for the d i gnosis. 55. (E) Given the p tient’s history of nger nd ggressive outbursts w hen not und erst nd ing schoolw ork, d ifficulties w ith soci liz tion nd

195

c ring for hi self, s w ell s f ily history of le rning d is bilities, you shou ld su spect the d i gnosis of intellectu l d is bility (ID). ID is d efined by severely i p ired cognitive nd soci l fu nctioning. The severity of intellectu l d is bility is d eter ined now by d p tive fu nctioning r ther th n IQ score. Either the WISC or the St nford -Binet Intelligence Sc le is n d equ te instru ent for esti ting IQ. The Good enou gh-H rris Dr w -A-Person Test nd Kohs Block Test ssess visu l- otor coord in tion. The MMPI-2 is frequ ently u sed to ssess person lity stru cture. The Pe bod yVoc bu l ry Test is u sed w hen p tients su ffer fro l ngu ge b rrier. 56. (B) This p tient su ffers fro Tou rette d isord er, synd ro e of ultiple otor nd voc l tics th t occu r d ily for t le st 1 ye r, w ith the onset before ge 18. Tou rette d isord er occu rs ore co only in boys th n girls. First-line tre t ent for ild to od er te Tou rette d isord er is n lp h -2 gonist, su ch s gu nf cine or clonid ine. An ntipsychotic c n be used to tre t the d isord er, bu t is u su lly reserved for p tients w ith ore severe c ses. Ato oxetine is selective norep inep hrine reu p t ke inhibitor th t h s been used for the tre t ent of ADH D; it c n be effective for child ren w ith ADH D w ho d evelop tics fter u sing sti u l nts. L otrigine nd v lp ro te cid re ood st bilizers u sed for the tre t ent of bip ol r d isord er. Lor zep is benzod i zep ine nd is typ ic lly u sed for cu te nxiety d isord er nd git tion. 57. (D ) Given the signific nt i p rove ent d u e to the ed ic tion, it w ou ld not be ppropri te t this ti e to d iscontinu e or sw itch ed ic tions. Sed tion is the ost co on sid e effect of clonid ine nd w ill often go w y fter few w eeks. Gu nf cine, the other lph -2 gonist is nother op tion if the sed tion continu es or interferes in fu nctioning. A d os ge red u ction shou ld be tte pted before king ny of the other ch nges listed . 58. (B) Ep ilep sy is co p lex set of d isord ers th t y initi lly co e to the ttention of p sychi trist bec u se of u nu su l beh viors.

196

6: Diffe re ntia l Diag nos is a nd Ma na g e me nt

Co p lex p rti l seizures, in p rticul r, y be ssoci ted w ith w id e r nge of sy pto s, includ ing sensory ur s nd uto tic beh viors. Prior to the seizure, the p tient y notice b d s ell, n u nu su l t ste, or g strointestin l sy pto s. During the event, the p tient y look d zed or frightened nd y exhibit nu ber of uto tis s su ch s lip s cking, eye blinking, nd fu bling w ith his or her clothes. Ad d ition lly, there is i p ir ent of consciousness. Co plex p rti l seizu res re usu lly ssoci ted w ith te por l lobe epilepsy, but c n lso result fro lesions elsew here in the br in. Another d i gnosis to rule out w ould be phet ine intoxic tion. A phet ines re co only u sed to tre t ADH D nd y be d ru gs of buse in this ge group. Intoxic tion w ith phet ines y present s seizu res nd c n be e sily ru led ou t by urine toxicology screen, but the nor l vit l signs ke this less likely. The other d i gnoses w ou ld not ccount for this p resent tion. 59. (D ) Althou gh syphilitic infections h ve been on the d ecline since World W r I, they ust still be consid ered in the d ifferenti l for w id e v riety of psychi tric d isord ers, especi lly w hen co bined w ith neu rologic bnor lities. In this c se, tw o for s of neurosyphilis (t bes d ors lis nd gener l p resis) re co bined in one p tient. P tients d e onstr te sensory t xi w ith w id e-b sed g it, p ositive Ro berg sign, nd loss of vibr tory nd proprioceptive senses initi lly in the low er extre ities. Deep tend on reflexes re d ecre sed nd pu pil bnor lities re co on, lthou gh the fu ll Argyll–Robertson p up il ( pu p il th t cco od tes bu t d oes not re ct to light) is infrequ ent. Gener l p resis is u su lly ssoci ted w ith d e enting p rocess long w ith neu rologic sy p to s su ch s p u pil bnor lities, tre ors, d yscoord in tion, nd sp sticity in the low er extre ities. Testing for neu rosyp hilis shou ld inclu d e blood test for VDRL nd flu orescent trep one l ntibod y bsorp tion test (FTA-ABS). Consid er tion shou ld lso be given to p erfor ing lu b r p u nctu re to test cerebrosp in l flu id for evid ence of trep one l infection. A d i gnosis of ntipsychotic-ind uced d ystoni or N MS is

u nlikely s they d o not p resent w ith the bove yri d neu rologic l sy p to s nd s the p tient is not cu rrently t king n ntip sychotic ed ic tion. Although this p tient h s so e sy pto s consistent w ith Kors koff p sychosis nd Wernicke enceph lop thy (both d u e to thi ine d eficiency fro long-ter lcohol u se), neither d i gnosis lone w ou ld ccou nt for this p tient’s constell tion of neu ropsychitric sy p to s. 60. (D ) Pheochro ocyto is tu or of the chroffin cells, usu lly occu rring in the d ren l ed u ll . Sy pto s y d iffer ccord ing to the pred o in nt c techol ine rele sed . As norep inep hrine-secreting tu ors re the ost co on, b seline hypertension w ith p roxys l ex cerb tions re frequ ent. Anxiety, fe r, nd p lpit tions y cco p ny ex cerb tions of hyp ertension. Bec u se this y i ic sy pto s of p nic tt ck or GAD, it is essenti l to consid er this in the d ifferenti l d i gnosis w hen p tients present w ith nxiety cou pled w ith physiologic ch nges. Alcohol w ithd r w l, r ther th n intoxic tion, w ou ld p resent w ith nxiety, tre ors, nd elev ted vit l signs. H yperc lce i inclu d es sy pto s such s kid ney stones (“stones”), bone p in/ fr ctu res (“bones”), d ep ression/ irrit bility (“ o ns”), nd n u se / vo iting (“gro ns”). Sy pto s of hypothyroid is re d epression, w eight g in, constip tion, ed e , d ry skin, ho rseness, nd thinning h ir. A posterior circu l tion stroke w ou ld present w ith cute vision loss, confusion he d che, p resthesi s, w e kness, nd d izziness. 61. (B) H ep tic encep h lop thy c n p resent in ny w ys nd y h ve flu ctu ting cou rse, king it d ifficu lt to d i gnose w ithou t p erip her l stig t . N eu rologic lly, the p tient y exhibit sterixis, w hich, lthou gh ch r cteristic for hep tic encep h lop thy, is not sp ecific for the d isord er. P tients y d isp l y 6- to 9-H z tre ors, ild ly incre sed d eep tend on reflexes, nd n ltered sensoriu . Typ ic l p sychi tric sy p to s y inclu d e ch nges in p erson lity, bru p t ood sw ings, nd ch nges in cognitive bility. In n cu te d eco p ens tion, d ep ression, c t toni ,

Answe rs : 59–67

p sychosis, nd d eliriu y d evelop . Physic l stig t reflecting d ecre sed liver fu nction inclu d e ecchy osis, p erip her l ed e , scites for tion, p l r erythe , nd sp id er ngio s. 62. (C) H IV c n p rod u ce spectru of neuropsychi tric nifest tions s resu lt of opp ortu nistic infections, neopl s s, or d irect inv sion of the CN S by the viru s. M jor neurocognitive d isord er (d e enti ) d ue to H IV infection is one of the ost co on nifest tions of pri ry CN S inv sion. Criteri for jor neu rocognitive d isord er (d e enti ) includ e signific nt cognitive d ecline in one or ore cognitive d o ins (le rning nd e ory, ttention, executive function, l ngu ge, perceptu l- otor, or soci l cognition). It y present in v riety of w ys, includ ing otor bnor lities, cognitive d ecline, or beh vior l ch nges. This c se d escribes typic l beh vior l ch nges, for ex ple, leth rgy nd soci l w ithd r w l. M jor neurocognitive d isord er d ue to H IV infection y lso includ e neurologic bnor lities such s w e kness, i b l nce, or t xi , s w ell s irrit bility nd p thy. The results of the MMSE support the d i gnosis of jor neurocognitive d isord er. 63. (D ) This c se illu str tes present tion of hyp erthyroid is , w ith d ep ression, irrit bility, nd cognitive d ysfu nction. H yperthyroid is y present w ith sy pto s consistent w ith v riety of psychi tric d isord ers, inclu d ing d ep ression, nxiety, psychosis, ni , nd , t n extre e, d eliriu . P tients y rep ort hot fl shes nd / or incre sed sensitivity to he t. H yperthyroid is y lso c u se ild cognitive d eficits in c lcu l tion nd recent e ory. N eurologic lly, p tients y d e onstr te fine 8- to 12-H z tre or, lid l g, brisk d eep tend on reflexes, proxi l yop thy w ith uscle w sting, nd y lgi s. 64. (G) So tic sy p to d isord er is ch r cterized by t le st one so tic or p hysic l co p l int th t c nnot be verified by l bor tory nd p hysic l find ings. The d isord er h s its onset before ge 30 nd is ore co on in w o en th n in en. Di gnosis is b sed on the

197

occurrence of physic l sy pto s, w hich y be p red o in ntly p in, th t re d istressing nd c u se soci l or occu p tion l d ysfu nction. It is i p ort nt to d eter ine th t these sy p to s re not intention lly p rod uced or feigned for other pu rposes. Si il rly, it is i port nt to ensu re th t the p tient receive the p p rop ri te ed ic l w orku p , bec u se nu ber of d ise se p rocesses y present w ith constell tion of sy pto s. B sic tests th t shou ld be ord ered inclu d e blood tests, ECG, colonoscop y, or gynecologic l ex in tions. 65. (H) Depression is co on sy pto of SLE. Although neuropsychi tric cond itions y rise t ny ti e d u ring the cou rse of the d ise se, they re ost frequent d uring cute ex cerb tions or l te in the d ise se process. Other sy pto s inclu d e d eliriu , person lity ch nges, nxiety, nd psychosis. P tients y lso exhibit cognitive i p ir ent on neuropsychologic l testing. N eu rologic lly, p tients c n present w ith seizu res, stroke, cr ni l nerve bnor lities, nd he d che. 66. (E) This p tient is exhibiting signs nd sy p to s consistent w ith d eliriu second ry to hyp oglyce i . Signs of hyp oglyce i nifested by this p tient inclu d e t chyc rd i , tre or, hyp ertension, nd seizu re. Deliriou s st tes shou ld receive i ed i te w orku p for n etiology s they c rry signific nt r te of orbid ity nd ort lity. A st nd rd w orku p consists of CBC w ith d ifferenti l, electrolytes, BUN , cre tinine, VDRL, vit in B12, fol te, u rin lysis, thyroid -sti u l ting horone (TSH ), c lciu , gnesiu , p hosp horu s, glu cose, u rine toxicology screen, liver fu nction tests, p erip her l oxygen s tu r tion, chest x-r y, ECG, s w ell s ent l st tu s nd p hysic l ex in tion. 67. (C) P tients w ith jor neurocognitive d isord er d ue to Alzhei er d ise se typic lly present w ith e ory proble s th t gr d u lly w orsen, s w ell s one other cognitive d o in (e.g., p h si , pr xi , gnosi , or d istu rb nces in executive functioning). In this c se, the p tient de onstr tes e ory proble s nd n gnosi (un ble to n e pencil).

198

6: Diffe re ntia l Diag nos is a nd Ma na g e me nt

68. (F) M jor neu rocognitive d isord er d u e to Lew y bod y d ise se typ ic lly p resents w ith w ell-for ed visu l h llu cin tions (p ed u ncu l r h llu cinosis or Lillip u tionis ) l te in life w ithou t p reviou s p sychi tric d istu rb nces. Motor d istu rb nces re si il r to th t of P rkinson d ise se bu t in Lew y bod y d ise se occu r l ter th n the cognitive d eficits. Motor p roble s inclu d e cogw heel rigid ity, shu ffling g it, nd red u ced r sw ing. The typ ic l p illrolling tre or of P rkinson d ise se is less co on. 69. (D ) Pick d ise se is ch r cterized by ch nges in p erson lity e rly in the illness. Pri itive reflexes re seen, su ch s the su ck, B binski, nd snou t. Other cognitive fe tu res of the d e enti typ ic lly occu r l te in the d ise se. I ging is u su lly ch r cterized by p referenti l trop hy of the front l nd te por l lobes. Confir tion of the d i gnosis is d e vi biop sy or u top sy nd reve ls Pick inclu sion bod ies. 70. (A) Deliriu is synd ro e consisting of flu ctu tion in the level of ttention nd cognition, sy p to s of w hich c n w x nd w ne. It is essenti l to look for one or ore u nd erlying c u ses, su ch s electrolyte bnor lities, infections, nd sid e effects of ed ic tions. In contr st to jor neu rocognitive d isord er (d e enti ), d eliriu h s n cu te onset of sy pto s nd is typic lly reversible once the u nd erlying etiology is corrected . 71. (B) Pseudode enti is ter referring to the cognitive disturb nces (“de enti ”) c used by depressive illness, such s jor depression. P tients typic lly present with depressed ffect, neuroveget tive sy pto s, nd e ory proble s. During ex in tion, p tients de onstr te poor concentr tion nd effort such th t they h ve difficulty co pleting questioning. Pseudode enti is reversible once the depression is tre ted. 72. (E) M jor neu rocognitive d isord er d ue to H untington d ise se is n utoso l d o in nt d ise se c used by CAG repe t on the short r of chro oso e nu ber 4. The d ise se

typic lly c uses p sychi tric d istu rb nces su ch s d ep ression or nxiety, h llu cin tions, e ory proble s, nd choreifor ove ents. On i ging, p tients y h ve boxc r ventricles th t ppe r s h ving corners r ther th n rou nd ed ed ges. M jor neurocognitive d isord er d ue to Prion (Creu tzfeld t–J cob) d ise se is d e enti c used by prion, su ch s in “ d cow d ise se.” Ind ivid u ls p resent w ith cognitive d eficits, t xi , bnor l ove ents, nd often st rtle reflex. It h s r pid p rogression. M jor v scu l r neurocognitive d isord er (for lly referred to v scu l r or u lti-inf rct d e enti ) is c u sed by cerebrov scu l r events, such s strokes or e boli. In d d ition to the cognitive d eficits, it is not u nco on to d ispl y p erson lity nd ood ch nges. In contr st to Alzhei er d ise se, jor neu rocognitive d isord er d ue to v scu l r d ise se cl ssic lly p resents w ith step -w ise d eterior tion. 73. (C) Bec u se the d efense ech nis of d eni l is p ri ry in lcohol u se d isord ers, often the best p p ro ch in ev lu ting p tient is to explore how lcohol ffects his life, r ther th n d irect qu estions bou t d rinking beh vior. H e ring bou t his d rinking fro his friend s nd f ily y p rovid e ore ccu r te d escrip tion of p tient’s lcoholis th n the ind ivid u l w ou ld p rovid e of hi self. Althou gh l bor tory w ork ight provid e clu es to the presence of lcohol use d isord er, it c nnot be relied on to ke the d i gnosis. 74. (B) GAD is d efined by excessive nxiety or w orry occu rring for t le st 6 onths. Associ ted sy pto s includ e feeling restless, f tigu ed , or irrit ble; u scle tension; d ifficu lty w ith concentr tion; nd sleep d istu rb nce. Dep end ent p erson lity d isord er rep resents p ttern of the p erv sive need to be t ken c re of, le d ing to su b issive, clinging beh vior th t begins in e rly child hood . Althou gh d ep ression nd p ersistent d ep ressive d isord er sh re so e sy pto s w ith GAD, the d epressive sy p to s re ore severe in the ood d isord ers th n w ith the nxiety d isord ers. The onset of nxiety w ith p nic d isord er is ore sud d en nd intense, w ith the fe r of choking

Answe rs : 68–80

or d ying co on, nd the p nic tt cks re recu rrent nd u nexp ected . Soci l nxiety d isord er requ ires nxiety in soci l situ tions w ith fe rs of scru tiny or hu ili tion by others, such s perfor nce nxiety or fe rs of pu blic spe king. 75. (A) N eg tive thoughts reg rding the self, the w orld, nd the future re know n s the Beck cognitive tri d . Beck, w ho d eveloped the Beck Depression Inventory, pioneered the use of cognitive ther py for d epressive d isord ers. This tri d of sy pto s is helpful in both und erst nd ing nd tre ting d epression through cognitive ther py. The tri d is not ch r cteristic lly seen in the other d isord ers listed . 76. (B) Cognitive ther py c n help to t rget these neg tive w ys of perceiving the self nd the environ ent. While often u sed in conju nction w ith cognitive ther p y (i.e., cognitive-beh vior l ther p y), “p u re” beh vior l ther p y is only focu sed on beh vior od ific tion, not on the u nd erlying f ctors (such s cognitive errors) th t c u se or contribu te to the p roble tic beh viors. Cou p les ther p y w ou ld be less effic ciou s th n cognitive ther p y given this p tient’s sy pto s. The jor focus of interp erson l p sychother p y is the n lysis of co u nic tion styles nd one’s role in rel tionship s. In p r d oxic l ther py, d evelop ed by B teson, the ther p ist su ggests th t the p tient eng ge in the beh vior w ith neg tive connot tions (e.g., phobi or co pu lsion). 77. (A) Abou t 40% of those w ith lcohol u se d isord er w ill beco e severely d epressed w ithin the first w eek of bst ining fro lcohol. H ow ever, if they re ble to bst in fro lcohol for tot l of 4 w eeks, only 5% of p tients w ill continu e to h ve signific nt sy p to s of d ep ression. 78. (E) This p tient continu es to experience d ep ressed ood nd neu roveget tive sy p to s (inso ni , nergi , poor concentr tion) 6 w eeks fter the cess tion of lcohol. Dep ressive sy pto s d uring the first 4 w eeks of stopping the u se of lcohol re ore likely to be su bst nce ind u ced . H ow ever, the f ct th t

199

the d epressive episod e h s l sted longer th n 4 w eeks fter the cess tion of lcohol points to n und erlying (co orbid ) ood d isord er such s jor d epressive d isord er (MDD) r ther th n su bst nce-ind u ced ood d isord er. Sertr line is n SSRI nd is first-line gent for the tre t ent for MDD. While itriptyline ( tricyclic ntid epress nt) is n option, bec use of its sid e effect profile nd leth lity in overd ose, it is not u sed s firstline gent for d ep ression. Bu sp irone is n nxiolytic u sed for the tre t ent of gener lized nxiety d isord er. Lithiu is ood st bilizer typic lly u sed for bip ol r d isord er ( ni ) nd not co only for u nip ol r d epression. Lor zep ( benzod i zep ine) is lso n nxiolytic, bu t it w ou ld not d d ress the p tient’s d ep ressive sy p to s nd c n c use physiologic l d epend ence, esp eci lly in this p tient w ith history of lcohol d d iction. 79. (D ) This p tient is experiencing n cu te Wernicke enceph lop thy (d eliriu ) c u sed by d inistr tion of glu cose w ithou t thi ine. In p tients w ith long-ter lcohol u se d isord er w ho re likely to h ve low levels of thi ine, giving glu cose p rior to giving thi ine c n often precipit te this cond ition, s glucose oxid tion requires thi ine, thu s d epleting the re ining stores in the bod y; this d epletion c n then c use r p id d rop in the thi ine levels w hich c uses the encep h lop thy. 80. (A) Alcohol-ind u ced jor neu rocognitive d isord er (Kors koff synd ro e) is n nterogr d e nestic d isord er c u sed by chronic thi ine d eficiency, w hich cl ssic lly presents w ith conf bul tion, in w hich the p tients ke up sp ects of their life th t they c nnot re e ber. It is c u sed by d ge to the ill ry bod ies th t is u su lly irreversible. Conversion d isord er shou ld only be d i gnosed w hen there re sy pto s of ltered neurologic l fu nction (sensory or otor) not d equ tely expl ined by recognized neu rologic l or ed ic l cond itions. F ctitious d isord er w ou ld be d i gnosed if the p tient f bric ted his sy p to s in the bsence of extern l rew rd s (i.e., to ssu e the sick role). M jor neu rocognitive d isord er d ue to Alzhei er d ise se, w hich d oes present

200

6: Diffe re ntia l Dia g nos is a nd Ma na g e me nt

w ith e ory d isturb nces, does not usu lly present so cutely, nd the history of lcohol use d isord er ke Alzhei er’s less likely etiology. Wernicke enceph lop thy, lso c used by thi ine deficiency, is gener lly reversible cond ition resulting in t xi , confusion, nd ocul r bnor lities. Wernicke’s y progress to Kors koff synd ro e. 81. (C) Mood -congru ent d elu sions re co p tible nd consistent w ith the st te of ind of the p tient. In this c se, the p tient feels gu ilty nd p resu bly believes he d eserves p u nishent. H is d elu sions express these thou ghts. If he d escribed d elu sions of gr nd iosity, th t w ou ld be n ex p le of ood -incongru ent d elusion. Biz rre d elusions d escribe circu st nces th t re i possible to be tru e, for ex p le, schizop hrenic p tient believing co pu ter chip is i pl nted in his br in. The d elusions in this qu estion, how ever u nlikely, re in so e sense conceiv ble. Ego-syntonic d elusions re experienced by the su fferer s ccept ble; for ex ple, the nic p tient believing he is the gre test ctor in the w orld . This p tient’s d elu sions re ego-d ystonic, exp erienced s u n ccep t ble nd u np le s nt. So tic d elu sions focus on bod ily fu nctions nd integrity. 82. (C) Often, the n tu re of the d elu sion nd its rel tion to ood st te c n p rovid e clu es to d i gnosis. Delu sions rising out of severe jor d epressive episod e, s in this c se, re frequently ood congru ent. Bipol r d isord er, nic cl ssic lly p rod uces d elu sions th t re gr nd iose, rel ting to the infl ted self-estee . The d elu sions of schizop hrenic p tients c n be biz rre nd re often u nrel ted to ood st te. The p resence of d elu sions ru le ou t d ju st ent d isord er nd persistent d epressive d isord er s these ffective illnesses d o not re ch severity th t includ es d elu sion l thinking. 83. (C) P tients w ith schizoid person lity d isord er re typic lly soci lly isol ted nd d istinctly inept t for ing nd c rrying on interperson l rel tionships. They often co e to the ttention of physici ns s result of so e other issue, s seen in this c se. Their pp rent p thy nd

ind ifference y rese ble the neg tive sy pto s of schizophreni , nd ind eed , th t is in the differenti l here; but it would be difficult to ke d i gnosis of schizop hreni w ithout evid ence of psychosis. Although this p tient y, ind eed , be d epressed nd eet criteri for MDD, w e re given no d irect evid ence of this. H e exhibits none of the istrust nd suspiciousness seen in p r noid person lity d isord er. This p tient l cks the ctively od d nd pecu li r nneris s or eccentric w y of thinking seen in schizotyp l person lity d isord er. 84. (D ) Bip ol r d isord er h s one of the l rgest genetic co p onents of ll the ent l illnesses. H ving onozygotic tw in w ith bipol r d isord er incre ses the risk of d eveloping bipol r d isord er u p to 70%. 85. (C) Fro s fety st nd point, the ost i port nt fe tu re of h llucin tions of p tient w ith schizophreni is the n ture of w h t the voices re s ying. Co nd u d itory h llu cin tions telling p tient to h r or kill hi or herself or so eone else re often psychi tric e ergency, u su lly requ iring hosp it liz tion. A possible excep tion y be w here n exp erienced clinici n know s the p tient w ell nd believes the situ tion to be s fe. All of the other choices in this qu estion spe k to i port nt ch r cteristics of u d itory h llu cin tions th t shou ld be ev lu ted , bu t the p resence of co nd u d itory h llu cin tions requ ires i ed i te ttention. 86. (D ) N rcolepsy is ch r cterized by p ersistent d yti e hyperso nolence th t is te por rily relieved by brief n ps. N rcolepsy is often cco p nied by pheno en co only ssoci ted w ith in ppropri te REM sleep, includ ing hypn gogic or hypnopo pic h llucin tions (vivid h llucin tions upon f lling sleep or w king up), c t plexy ( sud d en dr tic loss of uscle tone, usu lly follow ing n intense e otion l re ction), sleep p r lysis ( loss of volunt ry uscle tone t the beginning or end of sleep, s seen in this c se), nd is ssoci ted w ith hypocretin d eficiency. H yperso nolence d isord er is lso ch r cterized by persistent sleepiness bu t c n be d istingu ished

Answe rs : 81–92

fro n rcolepsy by the f ct th t the n ps cco p nying it re often long nd unrefreshing. Circ d i n rhyth sleep–w ke d isord er, d v nced sleep-ph se type refers to ind ivid uls w hose soci l/ occup tion l functioning is d isturbed by their d row siness d uring the evening, w here s d el yed sleep-ph se type refers to those w hose soci l/ occup tion l functioning is d isturbed by d row siness in the orning nd lertness in the evening. P r so ni s re beh viors th t re w orsened or only occur d uring sleep nd includ e night re d isord er nd non-REM sleep rous l d isord ers (e.g., sleepw lking or sleep terrors). 87. (C) The p tient in this qu estion h s p reviou s opi te u se d isord er. Bec u se of this, choosing non ddictive edic tions would be preferred over ddictive edic tions. Diphenhydr ine is hist ine blocker nd is not d d ictive. It c n be help fu l for p tients w ith inso ni . Alpr zol nd zolp id e , w hile they c n be benefici l, d o h ve n d d ictive potenti l. Cod eine is n opi te, nd , w hile one sid e effect is sed tion, it shou ld be u sed for the relief of p in. Qu eti p ine is n ntip sychotic w hich c u ses sed tion. H ow ever, d u e to its sid e effect profile su ch s ove ent d isord ers, w eight g in, incre sed cholesterol, nd incre sed f sting su g rs, it shou ld not be u sed s first-line gent for inso ni . 88. (C) The p tient in this vignette h s d i gnosis of ADH D, w ith sy pto s of in ttention nd hyp er ctivity. Tre t ent for ADH D is u lti od l nd inclu d es ed ic tions nd beh vior l od ific tions. First-line p h r cother py is ethylphenid te, sti ul nt. Clonid ine, n lp h -2 gonist, is second -line ed ic tion for ADH D. I ip r ine, TCA, nd sertr line, n SSRI, re ntid ep ress nts u sed to tre t nxiety nd d ep ression. Risperid one is n typ ic l ntip sychotic. 89. (D ) This vignette illu str tes p tient w ho is show ing signs of ild to od er te op i te w ithd r w l. Sy p to s of op i te w ithd r w l inclu d e l cri tion, rhinorrhe , y w ning, u scle ches, bd o in l cr p ing, nd d i rrhe . Anticholinergic toxicity p resents w ith

201

fevers, d il ted p u p ils, d ry skin, nd confu sion. H yp ertensive crisis occu rs w hen p tient is on n MAOI nd consu es tyr ine-rich food s resu lting in elev ted blood p ressu re, he d ches, n u se , nd vo iting. Influ enz is viru s resu lting in fevers, y lgi s, nd resp ir tory sy p to s. Serotonin synd ro e is life-thre tening cond ition d u e to i trogenic serotonin excess; this p tient is only t king one serotonergic ed ic tion—venl f xine. Serotonin synd ro e is ch r cterized by lethrgy, confu sion, flu shing, clonu s, co , nd p ossibly d e th. 90. (E) P tients on high-dose steroids re t risk for developing steroid-induced psychotic disorder or ni . This p tient is h ving uditory h llucin tions nd is thus experiencing psychosis r ther th n ni . Bipol r disorder due to nother edic l cond ition (hyperthyroidis ) is less likely given the p tient’s nor l thyroid studies. In order to h ve di gnosis of brief psychotic disorder, edic tion or edic lly induced psychosis ust be ruled out; steroids re well-known c use of psychosis. MDD with psychotic fe tures requires pri ry ood disorder which this p tient does not h ve. This p tient h s cute onset of uditory h llucin tions nd does not ppe r to h ve pri ry thought disorder, such s schizophreni . 91. (A) As steroid -ind uced psychosis is co only seen in p tients requ iring high-d ose steroid s. The first step in n ge ent of this p tient w ould be to d ecre se the steroid d os ge. H er TSH nd T4 levels re nor l nd she d oes not require d just ent in her levothyroxine. There is no ind ic tion to st rt nd SSRI or ood st bilizer. ECT c n be used for the n ge ent of cute psychosis, how ever, w e h ve n id entifi ble c use of this p tients psychosis nd w ould first d ecre se the steroid s. 92. (E) Serotonin syndro e is life-thre tening cond ition d ue to i trogenic serotonin excess. This p tient is on ultiple serotonergic ed ic tions, includ ing phenelzine, n MAOI, tr d ol, nd linezolid . Together these edic tions c used the sy pto s of leth rgy, confusion, flushing, clonus, di phoresis, co , nd y

202

6: Diffe re ntia l Dia g nos is a nd Ma na g e me nt

c use d e th. Bipol r d isord er is pri ry ood d isord er; w hile the p tient h s p reviou s d i gnosis of MDD w ith psychotic fe tu res, there is no history or evid ence of ni . H ypertensive crisis occurs w hen p tient t king n MAOI consu es tyr ine-rich food s resu lting in rked ly incre sed BP, he d ches, n use , vo iting, nd confu sion. Migr ines w ith ur re type of he d che w ith sy pto ( ur ) preced ing the he d che. N euroleptic lign nt synd ro e is n id iop thic re ction fro ntipsychotic ed ic tions. It is psychitric e ergency ch r cterized by confusion or d eliriu , fluctu ting vit l signs, nd “le d pipe” rigid ity. 93. (B) Multiple ed ic tions in d d ition to psychotropic ed ic tions (e.g., SSRIs, TCAs, MAOIs) c n incre se serotonin nd re i port nt to be w re of. These ed ic tions includ e tr d ol, linezolid , nd tript ns. They shou ld ll be d iscontinu ed in p tient w ith serotonin synd ro e. The other choices re either not i ed i te or not p p rop ri te. 94. (B) This p tient d e onstr tes ho rd ing d isord er, n in bility to d isc rd p ossessions reg rd less of v lu e, so u ch so th t he h s cre ted n u nliv ble situ tion in his ho e. The p tient find s hi self u n ble to w ork second ry to incre sing stress levels. First-line tre t ent for ho rd ing d isord er is CBT. GAD is d i gnosis reserved for p tients w ho h ve incre sed nxiety over u ltip le d o ins, signific ntly i p iring their bility to fu nction in soci l or occu p tion l settings. P tients w ith OCD h ve recu rrent, intru sive thou ghts (obsessions) resu lting in rep etitive beh viors (co p u lsions) s resp onse. OCPD is p erson lity d isord er in w hich p tients re rigid nd p erfectionistic, w ith no recu rrent obsessions or co p u lsions. P nic d isord er involves recu rrent, u np rovoked p nic tt cks nd p eriod s of extre e nxiety l sting t le st 1 onth. 95. (C) The p tient in this vignette eets criteri for d issoci tive nestic, w ith d issoci tive fu gue, w hich is ch r cterized by the sud d en tr vel fro ho e cco p nied by forgetting

self or other u tobiogr p hic l infor tion. These p tients often h ve signific nt ou nt of tr u or stressfu l event in their history. Bord erline p erson lity d isord er is p erv sive d isord er w ith ffect d ysregu l tion, p tterns of unst ble rel tionships, nd su icid l/ self- util tion beh viors. Deperson liz tion d isord er is d escribed s recu rrent feelings of d et ch ent fro self s w ell s h ving occ sion l “ou t of bod y” experiences. Dissoci tive id entity d isord er, for erly know n s u ltip le p erson lity d isord er, is d i gnosis of h ving tw o or ore d istinct person lities. Multiple ch nges of id entity usu lly co-occu r w ith e ory d eficits in p erson l d t nd d ily events. M jor neu rocognitive d isord er d u e to Alzhei er d ise se is d e enti d efined by gr d u l d ecline in cognitive fu nctions of e ory, le rning, nd l ngu ge. 96. (E) Restless legs synd ro e nifests s u nco fort ble sens tions in the legs, w hich re relieved w ith ove ent, ostly t rest nd in the evening. This cond ition is p ro inent c use of sleep d istu rb nce. Iron d eficiency is know n risk f ctor ssoci ted w ith restless legs synd ro e. GAD is d i gnosis inclu d ing signific nt nxiety over sever l re s of life th t i p irs fu nctioning. Inso ni d isord ers re collection of sy pto s th t d isru p t sleep d ur tion nd qu lity. N on-REM sleep rou s l d isord ers inclu d e sleep w lking nd sleep terrors. While they y includ e ove ents, they re p rt of beh viors such s w lking, e ting, nd sitting u p in bed . Obstru ctive sleep p ne , chronic sleep cond ition, is ch r cterized by p neic ep isod es resu lting in excessive d yti e sleepiness. 97. (B) Iron d eficiency is know n c u se of restless legs synd ro e. Rep leting iron w ou ld be the first step in n ging this p tient before consid ering other ph r cother py options. 98. (E) The p tient in this vignette eets criteri for inter ittent explosive d isord er, d i gnosis d escribed by frequent verb l or physic l ou tbu rsts tow rd people or p roperty. These ou tbu rsts of ggression re ou t of proportion to the situ tion or stressor nd c u se d istress

Answe rs : 93–108

to the p tient. P tients w ith ntisoci l p erson lity d isord er h ve p ttern of viol ting the rights of others nd the f ilure to confor to soci l nor s. While this p tient d ispl ys nger ou tbu rsts, there re no other viol tion of rights nor leg l proble s consistent w ith ntisoci l p erson lity d isord er. Bip ol r II d isord er inclu d es ep isod es of n MDE and hyp o ni ; ood d isord er is not d escribed in this vignette. Bord erline p erson lity d isord er p tients d escribe p ttern of inst bility d e onstr ted throu gh poor ffect regu l tion, self- util tion beh viors, nd unst ble rel tionship s. Cond uct d isord er is d i gnosed in child hood / d olescence nd involves n irrit ble/ ngry ood , d efi nt beh vior, nd vind ictiveness th t y progress to ntisoci l p erson lity d isord er in d ulthood . 99. (A) Person lity d isord ers re ego-syntonic, l d p tive, p erv sive beh viors th t c u se signific nt i p ir ent in soci l or occu p tion l fu nctioning. Antisoci l person lity d isord er requ ires p tient to h ve history of cond u ct d isord er prior to d ulthood . These ind ivid u ls h ve no reg rd for b sic hu n rights, re frequ ently violent, often f il to bid e by the l w , ke i p u lsive d ecisions, nd re reckless. They h ve little to no re orse for their ctions. 100. (D ) Depend ent person lity d isord er d escribes p tient w ith n excessive need to be t ken c re of resulting in d ifficulty king everyd y d ecisions nd the need for support fro others. 101. (I) Schizoid p erson lity d isord er p tients re soci lly w ithd r w n nd d o not d esire close rel tionship s. They p refer solit ry ctives nd p pe r to h ve restricted e otions th t sp n w id e r nge of environ ents. 102. (C) P tients w ith bord erline p erson lity d isord er h ve chronic p tterns of i pu lsivity nd u nst ble rel tionship s, ffect d ysregu l -

203

tion, nd often d e onstr te recu rrent suicid l thre ts, tte pts, or self- u til tion. 103. (E) H istrionic p erson lity d isord er inclu d es p tients w ith excessive e otion lity nd ttention-seeking beh viors, w ith focu s on p p e r nce nd ttr ctiveness. These inclu d e sed u ctive beh viors, over-ex gger ted e otion l resp onses, nd f lse p ercep tions s to the extent of inti cy of their rel tionship s. 104. (F) P tients w ith n rcissistic p erson lity d isord er h ve n overinfl ted sense of self-w orth, sense of entitle ent, nd requ ire excessive d ir tion. 105. (H) P r noid p erson lity d isord er involves p tient th t h s consistent d istru st of others w ithou t su bst nti l re soning. P tients often h ve d oubts of loy lty nd perceive tt cks on their ch r cter. While not overtly p sychotic, they c n be extre ely p r noid nd litigiou s. 106. (B) Avoid nt p erson lity d isord er is d i gnosed in p tients w ho re extre ely sensitive, often void ing situ tions (occu p tion l or soci l) w here they re exp ected to for new rel tionship s or friend ship s. These p tients re extre ely shy nd feel inferior to others. This c u ses the to be relu ct nt to eng ge in new ctivities for fe r of e b rr ss ent or rejection. 107. (G) P tients w ith obsessive-co pulsive person lity d isord er re const ntly preoccupied w ith p erfectionis nd control t the exp ense of efficiency nd flexibility. This results in u nw illingness to d eleg te, excessive d evotion to w ork, nd in bility to d pt. 108. (J) P tients w ith schizotyp l person lity d isord er re often looked t s eccentric nd h ving od d beh vior. While not d elusion l, they h ve unu su l beliefs, such s gic l thinking, cl irvoy nce, or the p r nor l.

This page intentionally left blank

CHAPTER 7

Prac tic e Te s t 1 Que s tions

D IRECTION S (Questions 1 through 107): For each of the multiple-choice questions in this section, select the lettered answ er that is the one best response in each case. Questions 1 and 2 A 29-year-old m an w ith a history of bip olar d isord er presents to the psychiatric em ergency d ep artm ent saying that he is the king of “Pum bar” and need s everyone’s allegiance for the u p com ing w ar w ith the Martians. In the p ast few d ays, he has slep t a total of 3 hou rs bu t says that he is not tired . H e has spent all of his m oney soliciting phone sex. N ow , he is agitated , d em and ing, and threatening. 1. Which of the follow ing is the best treatm ent for this p atient in the acu te setting? (A) (B) (C) (D) (E)

Carbam azep ine (Tegretol) Divalp roex sod iu m (Dep akote) H alop erid ol (H ald ol) H yd roxyzine (Atarax) Lithiu m

2. After treating the p atient acu tely, a m ed ication is requ ired for ongoing treatm ent of his bip olar d isord er. You find that he has a history of agranulocytosis. Which of the follow ing m ed ications w ou ld be the best choice for his treatm ent? (A) (B) (C) (D) (E)

Antip sychotic m ed ication Carbam azep ine Divalp roex sod iu m Lithiu m Lorazep am (Ativan)

Questions 3 and 4 A 44-year-old w om an p resents to her p rim ary care d octor w ith m u ltip le com p laints, inclu d ing w eakness in her low er extrem ities, bloating, head aches, interm ittent loss of ap p etite, and back p ain. A carefu l review of sym p tom s reveals m any other vagu e sym p tom s. H er com p laints d ate back to ad olescence and she has seen m any d octors. Thorou gh w orku p s, inclu d ing an exp loratory lap arotom y, have failed to u ncover any clear, anatom ic, or p hysiologic cau se. 3. Which of the follow ing is the best ap proach to this patient? (A) Tell her any physical w orku p is u nnecessary. (B) Tell her to com e back in 1 m onth and , if the sym ptom s are still present, you w ill initiate a p hysical w orku p . (C) Tactfu lly ask her w hy she is inventing sym ptom s. (D) Assess her for other p sychiatric illnesses. (E) Initiate a p hysical w orku p and arrange for follow -u p in a year’s tim e. 4. Which of the follow ing p ersonality d isord ers w ou ld m ost likely be com orbid in the above p atient? (A) (B) (C) (D) (E)

Avoid ant Bord erline Obsessive-com p u lsive Schizoid Schizotyp al 205

206

7: Pra c tic e Te s t 1

Questions 5 and 6 A 50-year-old sin gle m an h as in creasin gly becom e a con cern to his n eigh bors. H e w orks at a com icbook store, d resses in od d , ou td ated cloth es, an d d isp lays p oor eye con tact. Althou gh h e tend s to keep to him self, h e h as told neighborhood child ren th at th ere are w itch es w h o live d ow n th e street. 5. Which of the follow ing is the m ost likely d iagnosis? (A) (B) (C) (D) (E)

Bipolar d isord er Bord erline p ersonality d isord er Schizoid p ersonality d isord er Schizop hrenia Schizotyp al p ersonality d isord er

6. The patient’s brother brings him to a d octor because, since the d eath of their mother, the patient’s paranoia has cau sed him to qu estion his neighbors’ activities. H e has since m oved into a hotel that he could not afford in ord er to get aw ay from the “spies” living next d oor. Which of the follow ing is the most appropriate intervention? (A) (B) (C) (D) (E)

Antip sychotic Benzod iazep ine N o treatm ent Psychod ynam ic p sychotherapy Selective serotonin reu p take inhibitor (SSRI)

Questions 7 and 8 A 25-year-old fem ale college grad u ate is brou ght to her d octor by her m other. Described as “od d ” since she lost her job a year ago, the p atient has com p lained of hearing voices and believes that her bod y is receiving Wi-Fi com m u nications for a cou nterterrorist op eration. H er m other notes she has been isolating herself in her room . She is alert and oriented bu t su sp iciou s and gu ard ed on exam ination. H er affect is flat and her sp eech reveals loose associations. A com p lete m ed ical w orku p is negative.

7. Which of the follow ing sym p tom s is consid ered a “negative sym ptom ” in this patient’s p resentation? (A) (B) (C) (D) (E)

Au d itory hallu cinations Delu sions Flat affect Loose associations Paranoia

8. The p atient is started on m ed ication and m any of her sym p tom s im p rove. She begins a new job and d oes w ell. One year later, she is brou ght to her d octor florid ly p sychotic, actively hearing voices, and extrem ely p aranoid . She believes that her boss is trying to kill her. She has an u p p er respiratory viral illness that she believes to be the w ork of a foreign governm ent. She d iscontinu ed her m ed ication 4 w eeks ago becau se she felt too sed ated . In the p ast year, her cigarette sm oking habit has d ecreased to one p ack p er d ay. What is the m ost likely cau se of her exacerbation? (A) (B) (C) (D) (E)

A reaction to the viral illness Decreased cigarette sm oking Med ication noncom p liance Stress from w ork Treatm ent refractory illness

Questions 9 and 10 A 44-year-old man complains to his doctor that he is alw ays tired and is having d ifficulty getting out of bed in the morning. Upon questioning, he reveals he has three or four d rinks each night and perhaps more on the w eekend s, but d enies he has any problem w ith alcohol. A d iagnosis of alcohol use d isord er is mad e and the patient comes to your office in acute alcohol w ithd raw al. He subsequently has a w ithd raw al seizure and is ad mitted to the intensive care unit. 9. Which of the follow ing laboratory find ings w ou ld be m ost likely fou nd in this p atient? (A) (B) (C) (D) (E)

Decreased p rothrom bin tim e Elevated or d ep ressed liver enzym es H igh blood -alcohol level H yp erm agnesem ia Throm bocytosis

Que s tions : 5–14

10. Which of the follow ing m ed ications w ou ld be m ost im portant to ad m inister? (A) (B) (C) (D) (E)

Diazepam H alop erid ol Lorazep am Phenytoin Valp roic acid

11. A 28-year-old woman is brought to the emergency department for active suicidal ideation with a plan to overdose on acetaminophen. She has no history of a psychiatric illness but endorses many criteria for major depressive disorder (MDD), including poor sleep for the past 2 weeks. She recently lost her job and fears that she may not be able to pay her rent. Attempts to obtain collateral information have been unsuccessful. You believe the patient requires inpatient evaluation but her insurance company denies authorization for inpatient care, alternatively authorizing eight outpatient visits with a psychiatrist. You speak to the weekend on-call physician-reviewer and report that the patient remains unsafe and wishes to be discharged. Upon learning that the patient does not have a history of psychiatric illness, the reviewer fails to authorize inpatient care, despite your assessment. Which of the following is the most appropriate intervention? (A) Ad m inister an antipsychotic m ed ication and reevalu ate the p atient in 1 hou r. (B) Ad m it the p atient on an em ergency certificate to an inpatient facility. (C) Begin antid ep ressant therap y and arrange for outp atient follow -u p the next d ay. (D) Explain to the p atient that her insu rance com p any d id not au thorize hosp italization and d ischarge her w ith follow -u p care. (E) Prescribe a m ed ication to help her sleep, arrange for follow -up care, and d ischarge her from the hospital. Questions 12 and 13 A 34-year-old w om an suffering from severe m ajor d epressive d isord er is ad m itted to the hospital d u e

207

to w orsening d epressive sym ptom s and acute suicid ality. She has had m u ltiple trials of m ed ications w ithou t significant im p rovem ent. A cou rse of electroconvulsive therap y (ECT) is d eterm ined to be the next ap p ropriate step. 12. While u nd ergoing the ECT, the treatm ent team w ishes to m onitor im provem ent in her d epression. Which of the follow ing tests has the greatest reliability and valid ity for this p u rp ose? (A) Beck Dep ression Inventory (B) Draw -A-Person Test (C) H alstead -Reitan N eu rop sychological Battery (D) Rorschach Test (E) Them atic Ap p ercep tion Test (TAT) 13. After one of her treatm ents, the p atient com p laints of m em ory im p airm ent. Which of the follow ing tests w ou ld be the m ost ap p rop riate to assess her com plaint? (A) (B) (C) (D) (E)

Beck Dep ression Inventory Brow n-Peterson Task Bu lim ia Test—Revised Eating Disord er Inventory 2 (EDI-2) State-Trait Anxiety Inventory

14. A 67-year-old w oman w ith a history of d epression presents to your office for evaluation. Her symptoms of poor appetite, insomnia, low energy, and feelings of hopelessness have w orsened recently. She has been on several d ifferent serotonin-specific reuptake inhibitors (SSRIs), w hich you learn have not resulted in complete remission of her symptoms. Which of the follow ing med ications w ould be the most appropriate to prescribe? (A) (B) (C) (D) (E)

Citalopram Flu voxam ine Paroxetine Sertraline Venlafaxine

208

7: Pra c tic e Te s t 1

Questions 15 and 16 A 29-year-old w om an p resents to the em ergency d ep artm ent w ith her 3-year-old child reporting that the child suffered a seizu re w hile at hom e. H ospital record s verify that this is the third em ergency d ep artm ent visit in as m any w eeks for the sam e p resentation. N eu rologic w orku p for seizu re d isord er w as negative. Initiation of an anticonvu lsant has been ineffective. The m other becom es very fru strated , d em and ing that her son be ad m itted to the hosp ital for further testing. 15. Based on the above p resentation, w hich of the follow ing d iagnoses is m ost likely in the child ? (A) (B) (C) (D) (E)

Conversion d isord er Factitiou s d isord er N o d iagnosis Seizu re d isord er Sep aration anxiety d isord er

16. The m other is ad d itionally m ost likely to su ffer from w hich of the follow ing? (A) (B) (C) (D) (E)

Bipolar d isord er Ep ilep sy Major d ep ressive d isord er Posttrau m atic stress d isord er (PTSD) Schizop hrenia

Questions 17 and 18 A 26-year-old m an is being evalu ated in the em ergency d ep artm ent for su d d en onset of chest p ressure and d yspnea. This is his third em ergency d ep artm ent visit for sim ilar sym p tom s for w hich he rep orts “I feel like I’m going to d ie.” An electrocard iogram (ECG) and stress test w ere norm al. Urine toxicology w as negative. The p atient d enies risk factors for heart d isease and d oes not have a fam ily history of heart d isease. 17. Which of the follow ing is the m ost likely d iagnosis? (A) Acu te m yocard ial infarction (B) Acu te stress d isord er (C) Deliriu m

(D) H yp ochond riasis (E) Panic d isord er 18. Which of the follow ing m ed ications is the m ost efficaciou s in the treatm ent of this illness? (A) (B) (C) (D) (E)

Clonid ine (Catap res) H alop erid ol Lithiu m Prop ranolol Sertraline (Zoloft)

19. A w om an being treated for m ajor d ep ression is brou ght to the em ergency d ep artm ent after being fou nd u nconsciou s by a neighbor. The neighbor states that over the p ast few d ays the w om an had been com plaining of severe head aches. She also m entions that the w om an often enjoys red w ine. The w om an’s blood pressu re is record ed as 220/ 110 m m H g. Which of the follow ing shou ld be ad m inistered im m ed iately? (A) (B) (C) (D) (E)

Alp ha-blocker Beta-blocker Brom ocrip tine (Parlod el) Calciu m channel blocker Dantrolene sod iu m (Dantriu m )

Questions 20 and 21 A 38-year-old m an w ith schizop hrenia has had nu m erou s hosp italizations and m any trials w ith variou s antip sychotic m ed ications. H e continues to be sym p tom atic, w ith d erogatory au d itory hallu cinations, p aranoia regard ing his neighbors, and poor self-care. H e d enies su icid al or hom icid al id eation. H e has recently been started on clozap ine (Clozaril), w ith significant im p rovem ent in his cond ition. H e d oes not d rink alcohol or u se illicit d rugs, and he d enies ad d itional m ed ical history. 20. Which of the follow ing ad verse effects is associated w ith this m ed ication? (A) (B) (C) (D) (E)

Brad ycard ia Galactorrhea H yp ertension Seizu res Weight loss

Que s tions : 15–27

21. Which of the follow ing blood tests w ill requ ire frequ ent, regu lar m onitoring for this p atient? (A) (B) (C) (D) (E)

Calciu m level Com p lete blood cou nt w ith d ifferential Electrolytes Thyroid fu nction tests Urinalysis

Questions 22 and 23 You ad m it an 83-year-old w id ow ed fem ale for fu rther evalu ation as she is no longer able to care for herself at hom e. She has lost 30 lb in the p ast year, has p oor hygiene, and ad m its to increasing forgetfulness. 22. Which of the follow ing tests w ou ld best help to m ake the correct d iagnosis? (A) Blessed Rating Scale (B) Folstein Mini-Mental State Exam ination (MMSE) (C) Geriatric Rating Scale (D) Glasgow Com a Scale (E) Mental Statu s Exam ination (MSE) 23. Which of the follow ing d isord ers w ou ld be m ost im p ortant to ru le ou t as a cau se of her clinical p resentation? (A) (B) (C) (D) (E)

Generalized anxiety d isord er Major d ep ressive d isord er Obsessive-com p u lsive d isord er Panic d isord er Posttrau m atic stress d isord er

24. A 35-year-old patient is encou raged to seek “p rofessional help” by his cow orkers. H e d enies pervasive d epression or anxiety, bu t u pon interview he is od d ly d ressed , expresses u nu sual beliefs and thinking, som e paranoia regard ing his cow orkers’ m otivations, and has few close friend s. H e d enies d elu sions or hallu cinations, and there is no su icid al or hom icid al id eation. Which of the follow ing d iagnoses is the m ost appropriate to consid er? (A) Avoid ant p ersonality d isord er (B) N arcissistic p ersonality d isord er

209

(C) Paranoid p ersonality d isord er (D) Schizoid p ersonality d isord er (E) Schizotyp al p ersonality d isord er 25. A 28-year-old sep arated w om an is referred from her p rim ary care d octor for evalu ation for d ep ression. While she ad m its to occasional p eriod s of d ysp horia, she claim s to “alw ays feel em p ty insid e.” Up on fu rther qu estioning she d em onstrates a p ervasive p attern of u nstable relationship s, p oor self-im age, im p u lsiveness, and irritability. Which of the follow ing d iagnoses is the m ost ap p rop riate to consid er? (A) (B) (C) (D) (E)

Antisocial p ersonality d isord er Bord erline p ersonality d isord er Dep end ent p ersonality d isord er H istrionic p ersonality d isord er Schizoid p ersonality d isord er

Questions 26 and 27 A 28-year-old single m an w ith a 10-year history of schizophrenia has been taking his m ed ications regu larly. H e now p resents w ith w orsening hallucinations and prom inent thought d isorganization. 26. Which of the follow ing neu rop sychological tests w ou ld best d eterm ine his ability to organize and correctly p rocess inform ation? (A) Bend er Gestalt Test (B) Draw -A-Person Test (C) Lu ria-N ebraska N eu rop sychological Battery (D) Mini-Mental State Exam ination (E) Wisconsin Card Sorting Test (WCST) 27. The above test m easu res fu nctioning at w hich of the follow ing lobes of the brain? (A) (B) (C) (D) (E)

Cerebellar Frontal Occip ital Parietal Tem p oral

210

7: Pra c tic e Te s t 1

Questions 28 and 29 A 30-year-old w om an w ithout prior p sychiatric history is brought to the em ergency d epartm ent by the p olice after being arrested for “breach of the p eace.” The w om an w as observed acting irrationally at a local bu siness w here she d em and ed to sp eak w ith the presid ent of the com p any claim ing that she had new id eas for prod u ct d evelopm ent. The patient rep orts that she has not slep t for d ays and that her m ood is “fabulou s.” Urine hum an chorionic gonad otrop in is p ositive. Illicit su bstances w ere not d etected . 28. Which of the follow ing ad d itional find ings w ou ld m ost likely be p resent in her history or m ental status exam ination (MSE)? (A) (B) (C) (D) (E)

Daytim e sleepiness Dep ressed affect Racing thou ghts Visu al hallu cinations Weight loss

29. Which of the follow ing m ed ical d isord ers can also present w ith sim ilar sym ptom s? (A) (B) (C) (D) (E)

Cirrhosis Diabetes m ellitu s H yp erglycem ia Rheu m atoid arthritis Thyroid d isord er

Questions 30 and 31 A 24-year-old m an w ith a history of seizu re d isord er, alcohol, and cocaine u se d isord er has been incarcerated for assau ltive behavior. The p atient is evaluated by a neurologist, who prescribes phenytoin (Dilantin). 30. Which of the follow ing sid e effects is m ost likely associated w ith this m ed ication? (A) (B) (C) (D) (E)

Ebstein anom aly Gingival hyp erp lasia H ep atic failu re H yp ertension Leu kocytosis

31. The p atient retu rns 1 m onth later for a follow u p exam ination and rep orts th at he exp erienced a gen eralized seizu re. Laboratory investigation reveals that the p henytoin level is 6.5 m g/ d L (norm al, 10–20 m g/ d L). Which of the follow ing is the m ost ap p rop riate intervention at this tim e? (A) (B) (C) (D)

Ad d a benzod iazep ine. Ad d p henobarbital. Assess com p liance. Discontinu e p henytoin and begin d ivalproex sod ium . (E) Increase the p henytoin d ose.

32. A 24-year-old w om an w ith a history of schizophrenia tells you that she w ou ld like to becom e p regnant. H er husband has no history of m ental illness. What d o you tell her is the chance of her offsp ring d evelop ing schizophrenia? (A) (B) (C) (D) (E)

1% 2% 5% 8% 12%

33. A 28-year-old m arried fem ale p atient is ad m itted to the hospital w ith bizarre behavior and d isorganized thinking. Consid eration is given to a d iagnosis of schizophreniform d isord er. To help w ith the d iagnosis, the patient is ad m inistered a test that consists of view ing a set of 10 inkblots sequ entially. The exam iner scores the patient’s responses to the blots by noting the content of the p ercep tion, the area of the blot that form s the basis of the response, and the asp ects of the area that are u sed to form the resp onse. In w hich of the follow ing classes d oes this projective test belong? (A) (B) (C) (D) (E)

Associations Choice of ord ering Com p letions Constru ctions Self-exp ression

Que s tions : 28–39

34. A m other and her 17-year-old son p resent to you r p sychiatric clinic seeking fam ily therap y. The m other rep orts that she and her son have a tum ultu ou s relationship and are constantly argu ing w ith each other. The son ad m its that abou t a year ago he d isclosed to his m other that he is gay. H e feels that because of his m other ’s strong religiou s beliefs, she has not accep ted his sexu ality and this has been the sou rce of tension in their relationship. The m other firm ly believes that her son is “ju st confu sed ” and need s cou nseling. She has heard of a particular therapy that can change the sexu al orientation of an ind ivid u al and asks that you cond u ct this on her son. Which of the follow ing is the appropriate next step? (A) Ask the son if he is w illing to p u rsu e this treatm ent. (B) Engage the son in this therap y based off on w hat you know . (C) Inform the m other that su ch therap y is u nethical. (D) Learn m ore abou t the treatm ent first and ask the fam ily to retu rn w hen you are know led ge able enou gh to cond u ct this. (E) Refer the fam ily to a therap ist w ho specializes in such treatm ents. 35. An 18-year-old m an is referred to his college counseling center d ue to failing grad es. It is d eterm ined that he has stop ped attend ing classes, preferring to rem ain in his d orm room . H is room m ates claim that he stays u p “all night,” w ith little ap parent sleep. The p atient claim s that he has d iscovered som ething that w ill m ake him a m illionaire. On m ental statu s exam ination, he app ears d iap horetic w ith p sychom otor agitation. H is speech is rap id and his affect is eu phoric. H e d enies su icid al or hom icid al id eation but d isp lays significant grand iosity. H is insight is p oor. Which of the follow ing is the m ost likely d iagnosis? (A) (B) (C) (D) (E)

Cannabis intoxication Cocaine (stim u lant) intoxication MDD w ith p sychotic featu res Schizoaffective d isord er Schizop hrenia

211

36. An 89-year-old m arried w om an w ith no p rior p sychiatric history bu t a history of m u ltiple m ed ical p roblem s is ad m itted for failu re to thrive. Upon history, the patient d oes not seem acclim ated to her su rrou nd ings. Which of the follow ing tests w ou ld best help to d eterm ine her level of confu sion? (A) (B) (C) (D) (E)

Fargo Map Test Sp atial Orientation Mem ory Test Stroop Test Tem p oral Orientation Test Wisconsin Card Sorting Test

37. A Malaysian m an is brou ght into cu stod y by the police after m u rd ering his friend . While he d oes not rem em ber com m itting the act, he d oes recall being insu lted by him at an earlier tim e. Which of the follow ing cond itions is m ost associated w ith this presentation? (A) (B) (C) (D) (E)

Am ok Dhat Ganser synd rom e Koro Latah

Questions 38 and 39 A 26-year-old w oman is d iagnosed w ith schizophrenia. The psychiatrist d ecid es to treat her symptom s w ith a high-potency antipsychotic med ication. While her symptom s appear to improve on the med ication, she d evelops acu te, painfu l muscle spasm s of her jaw . 38. Which of the follow ing m ed ications shou ld be ad d ed ? (A) (B) (C) (D) (E)

Benztrop ine (Cogentin) Cholinergic agonist Clozap ine Methylp henid ate (Ritalin) Risp erid one (Risp erd al)

39. Despite compliance with the above regimen, the patient continues to suffer with extrapyramidal symptoms. As a result of this, she abruptly stops taking her antipsychotic medication. It is decided to switch to another medication. Which

212

7: Pra c tic e Te s t 1

of the following medications would be the most appropriate choice? (A) (B) (C) (D) (E)

H alop erid ol Loxapine (Loxitane) Perp henazine (Trilafon) Pim ozid e (Orap ) Qu etiapine

Questions 40 and 41 A 30-year-old w om an presents w ith a history of chronic but rapid and intense m ood sw ings, feelings of em ptiness, low self-esteem , chronic suicid al id eation, as w ell as frequ ent but brief sexu al relationships, anger outbu rsts, and self-m u tilation. 40. Which of the follow ing d isord ers is this p atient m ost likely to be su ffering from ? (A) (B) (C) (D) (E)

Avoid ant p ersonality d isord er Bipolar d isord er Bord erline p ersonality d isord er Depend ent personality d isord er N arcissistic personality d isord er

41. The above p atient is referred for d ialectical behavioral therapy (DBT). Which of the follow ing techniqu es is the m ost im portant in her therapy? (A) Aversion (B) Eye-m ovem ent d esensitization and rep rocessing (EMDR) (C) Flood ing (D) H om ew ork (E) System atic d esensitization 42. A 21-year-old man presents w ith a 7-month history of bizarre behavior, paranoid delusions, and auditory hallucinations that comment on his appearance. After a thorou gh evaluation, you d iagnose him w ith schizophrenia and prescribe haloperid ol 5 m g bid . One w eek later, the patient retu rns for a follow -up exam ination and reports that, althou gh his symptom s have imp roved , he now experiences mu scle stiffness in his arms and neck. You p rescribe benztropine 1 mg bid and sched ule a follow -up ap pointment in 2 w eeks. One w eek later, the

patient’s mother calls you and reports that her son is more agitated and confused. Physical examination reveals tachycardia, dilated pupils, and flushed skin. Which of the following would be the most appropriate next step? (A) Discontinu e benztropine and p rescribe am antad ine (Sym m etrel). (B) Increase haloperid ol to 5 m g tid . (C) Increase benztropine to 2 m g bid . (D) Discontinu e halop erid ol and p rescribe risp erid one. (E) Prescribe lorazep am 1 m g bid . Questions 43 and 44 A 25-year-old single w om an w ho carries the p rovisional d iagnosis of d epend ent personality d isord er is referred for p sychological testing. You d ecid e to ad m inister the Rorschach. 43. Which of the follow ing asp ects of this typ e of psychological testing w ou ld be the m ost im p ortant in this case? (A) Ask sp ecific qu estions w ith item ized resp onses. (B) Determ ine how a p atient feels abou t you . (C) Lack of stru ctu re allow s for a variety of resp onses. (D) Provid e nu m erical scores. (E) Resu lts allow for easy statistical analysis. 44. You ad d itionally d ecid e to ad m inister the Minnesota Mu ltiphasic Personality Inventory 2 (MMPI-2). Which of the follow ing aspects of this typ e of p sychological testing w ould be the m ost im portant in this case? (A) Assesses test-taking attitud es. (B) Instru ctions are u nam bigu ou s in natu re. (C) Item s d esigned to sep arate norm al subjects from those w ith psychiatric illness. (D) More sensitive in p icking u p gend erspecific issu es. (E) Most researched and w ith norm ative d ata.

Que s tions : 40–49

45. A 28-year-old m ale m ed ical stu d ent is fou nd to have an enlarged testicle d u ring a rou tine p hysical exam ination. The stu d ent reports that it has been grad ually enlarging for several m onths. The physician asks w hy he d id not report these find ings earlier. “I’m su re it’s nothing,” the stud ent replies. Which of the follow ing typ es of resp onses d oes this m ost likely rep resent? (A) (B) (C) (D) (E)

Confid ence Denial Rationalization Repression Su p p ression

46. A 45-year-old Asian-Am erican w om an is brou ght to the em ergency d ep artm ent by her hu sband , w ho rep orts that for the past 3 d ays his w ife has not been sleeping w ell, has been experiencing bad d ream s, and ap pears “in a d aze” w ith a sense of feeling “nu m b.” The patient end orses feeling anxiou s bu t d oes not know w hy. She has been u nable to p erform her u su al activities of d aily living, poorly concentrating on tasks and frequently “ju m p ing” w hen startled . One w eek ago, the p atient w as d ischarged from the hosp ital after exp eriencing an anap hylactic reaction to IV contrast d ye w hile u nd ergoing an im aging p roced u re for sinusitis. Althou gh she cannot recall specifics, her husband verifies the history, ad d ing that the d octors “thought she w as going to d ie.” Upon retu rning to the hosp ital, she exp eriences intense fear abou t revisiting the sam e hospital from w hich she w as recently d ischarged . Which of the follow ing is the m ost app rop riate d iagnosis? (A) (B) (C) (D) (E)

Acu te stress d isord er Ad ju stm ent d isord er Generalized anxiety d isord er (GAD) Major d ep ressive d isord er (MDD) Posttrau m atic stress d isord er (PTSD)

47. A 40-year-old w om an w ithou t p ast p sychiatric history is ad m itted to the hospital for the treatm ent of d epression. Du ring m orning rou nd s, the patient appears unresp onsive and d oes not, ind eed , respond to verbal stim u li. There

213

are no signs of trau m a or overd ose. A review of her chart reveals that the p atient w as w ell the night before and w ent to sleep w ithou t incid ent. You d eterm ine that the p atient’s u nresponsiveness is psychogenic. Which of the follow ing find ings is m ost likely to be ap p arent on her exam ination? (A) (B) (C) (D) (E)

Abnorm al electroencep halogram (EEG) Cold -caloric–ind u ced nystagm u s Decreased resp irations Elevated tem p eratu re N onsaccad ic eye m ovem ents

48. A 24-year-old w om an is referred by her fam ily p ractice d octor after the Christm as holid ay. She gives a significant history of fatigu e, w eight gain, and hyp erp hagia d u ring the w inter m onths. She d escribes a “sad m ood ,” poor sleep, problem s concentrating, and “can’t w ait” u ntil her vacation to Florid a. Which of the follow ing is the m ost likely d iagnosis? (A) Acu te stress d isord er (B) Generalized anxiety d isord er (C) Major d ep ressive d isord er w ith seasonal p attern (D) Persistent d ep ressive d isord er (d ysthym ia) (E) Su nd ow ning synd rom e 49. A 27-year-old w om an is brou ght to the em ergency d ep artm ent by her p arents, w ho rep ort that their d au ghter is u nable to recall her nam e. The em ergency d ep artm ent p hysician rep orts that a com p lete neu rologic w orku p is w ithin norm al lim its. Collateral inform ation reveals that the p atient had ep isod es in w hich she w ould take u nplanned trips, som etim es for d ays or w eeks, w ithou t notice, and w ou ld retu rn u nable to recall the ep isod e. A review of her m ed ical chart notes a p ast history of possible sexual abu se as a child . Urine toxicology is negative and she d oes not take any m ed ications. Which of the follow ing is the m ost likely d iagnosis? (A) Deliriu m (B) Dissociative am nesia w ith d issociative fu gu e

214

7: Pra c tic e Te s t 1

(C) Major d ep ressive d isord er (D) Partial com p lex seizu res (E) Posttrau m atic stress d isord er Questions 50 and 51 A 20-year-old college stu d ent is referred for testing to evaluate poor acad em ic perform ance. H e rep orts that he has alw ays “stru ggled ” to pass his classes d espite stu d ying for m any hours. H e attend s all of his lectures and is able to pay attention, yet he d oes not seem to be able to ad equ ately learn the m aterial. You susp ect he has a learning d isord er. 50. Which of the follow ing tests w ou ld be the m ost ap prop riate to help d eterm ine this p atient’s p roblem ? (A) Draw -A-Person Test (B) Minnesota Mu ltip hasic Personality Inventory 2 (MMPI-2) (C) Wechsler Ad u lt Intelligence Scale— Revised (WAIS-R) (D) Wechsler Intelligence Scale for Child ren (WISC) (E) Wechsler Mem ory Test (WMT) 51. Which of the follow ing form u las is used to calcu late the intelligence quotient (IQ)? (A) Actu al IQ/ theoretical IQ × 100 (B) Chronological age/ p erform ance IQ × 100 (C) Mental age/ chronological age × 100 (D) Mental age/ full-scale IQ × 100 (E) Perform ance IQ/ verbal IQ × 100 52. A 26-year-old female graduate student reports to you a 4-week history of a depressed mood that has caused her significant difficulty in attending her classes. The patient reports difficulty falling asleep at night, red uced appetite and weight loss, poor energy, and passive suicidal ideation. A careful review of her history reveals that for the past 2 years she also experienced brief and distinct periods of an elevated and expansive mood, a decreased need for sleep, and an increase in activities, although she was still able to function adequately. Which of the following is the most likely diagnosis?

(A) (B) (C) (D) (E)

Bip olar I d isord er Bip olar II d isord er Cyclothym ic d isord er Major d ep ressive d isord er Persistent d ep ressive d isord er (d ysthym ia)

53. You are asked to evalu ate a 30-year-old m ale p risoner w ho reports a d ep ressed m ood and suicid al id eation. Du ring your interview and m ental statu s exam ination, you note that the prisoner d oesn’t give correct or sp ecific answ ers, respond ing instead to you r qu estions w ith approxim ate answ ers. Which of the follow ing is the m ost ap p rop riate d iagnosis? (A) (B) (C) (D)

Cap gras synd rom e Dissociative am nesia Ganser synd rom e Major neu rocognitive d isord er (d em entia) (E) Mu nchau sen synd rom e

54. A 75-year-old m an is ad m itted to the hospital follow ing a serious su icid e attem pt. The p atient exhibits clinical featu res of d ep ression w ith severe neu rovegetative sym ptom s and has a p ast history of su icid e attem p ts. The m ed ical chart reveals that he is p rescribed levod op a/ carbid op a, atorvastatin, asp irin, and nifed ip ine for a card iac arrhythm ia. H e rem ains extrem ely help less and hop eless, w ith little ap p etite, and ongoing su icid al id eation w ith several lethal p lans. Which of the follow ing treatm ents w ou ld be the m ost ap p rop riate? (A) (B) (C) (D) (E)

Diazep am (Valiu m ) Electroconvu lsive therap y (ECT) N ortrip tyline (Pam elor) Risp erid one Su p p ortive p sychotherap y

55. A 38-year-old w om an presents to your office w ith an 8-w eek history of sym ptom s of d ep ressed m ood w ith an increased appetite and 10-lb w eight gain, hyp ersom nia, a “heavy feeling” in her bod y, and rejection sensitivity. Up on fu rther qu estioning she ad m its to being

Que s tions : 50–60

able to brighten w hen sp end ing tim e in activities she u sually enjoys. She rep orts to you that she recently enrolled in grad uate school and is having trou ble w ith m any of her classes. Which of the follow ing d iagnoses is the m ost likely? (A) Ad ju stm ent d isord er (B) Major d epression w ith atypical features (C) Major d ep ression w ith m elancholic featu res (D) Persistent d epressive d isord er (d ysthym ia) (E) Sleep d isord er 56. A 33-year-old m an changes his first nam e to honor a m u sician w hom he id olizes. H e recently bou ght the sam e gu itar as the m u sician and form ed a rock band to play his m usic. Du ring p ractice, the m an d resses like his id ol. Which of the follow ing d efense m echanism s d oes this behavior best rep resent? (A) (B) (C) (D) (E)

Fixation Id ealization Id entification Projection Regression

57. A 60-year-old m an w ith schizop hrenia sits m otionless in his chair. The patient is m u te and reacts very little to his environm ent. H is eyes ap pear fixated on a d istant object. At tim es, he assum es bizarre postu res and im itates the m ovem ents of others. Which of the follow ing is the best d escrip tion of his behavior? (A) (B) (C) (D) (E)

Absence seizu re Catalep sy Catap lexy Catatonia Partial com plex seizu re

58. A 37-year-old m arried m ale presents w ith a history of m ultiple d epressive ep isod es. For the past 5 w eeks he has felt sad , w ith crying spells, term inal insom nia, low energy, but increased ap p etite and w eight gain. H e has had passive su icid al id eation of “w anting to

215

d ie” bu t d enies any plan or intent. Upon d iscussion w ith the w ife present, they both confirm a p ast episod e w here for several w eeks last year he w as “u p ,” w ith increased energy, little sleep , w anting to have sex several tim es p er d ay, talking fast, gam bling a large su m of m oney, and attem pting to rem od el the hom e d esp ite having no experience. H e d enies hallu cinations or p aranoia. H e occasionally d rinks one to tw o glasses of w ine on w eekend s and d enies any illicit d ru g u se. H e has not been taking any m ed ications recently as he thou ght he cou ld “tou gh it ou t.” Which of the follow ing m ed ications w ou ld likely be the m ost ap p rop riate to begin? (A) (B) (C) (D) (E)

H alop erid ol Lam otrigine Lithiu m Sertraline Valproic acid

Questions 59 and 60 A 42-year-old healthy m an is u nd ergoing a lot of stress at his w ork. As a result, he is having a d ifficu lt tim e falling and staying asleep . H is p rim ary care d octor has since p rescribed him a m ed ication to help w ith his insom nia. While he is sleep ing better, he calls one m orning several w eeks later w ith com plaints of a very painfu l penile erection w hich has lasted throu ghout the evening. 59. Which of the follow ing m ed ications has m ost likely been p rescribed ? (A) (B) (C) (D) (E)

Flu oxetine (Prozac) N ortrip tyline (Pam elor) Paroxetine (Paxil) Sertraline (Zoloft) Trazod one (Desyrel)

60. Which of the follow ing is the m ost ap propriate next step ? (A) Tell him to d ecrease the d ose. (B) Tell him to stop the m ed ication and restart in 3 d ays. (C) Tell him to stop the m ed ication and m onitor his cond ition.

216

7: Pra c tic e Te s t 1

(D) Tell him to stop the m ed ication and go im m ed iately to the em ergency room . (E) Tell him to stop the m ed ication and have sexual intercourse or m asturbate. Questions 61 and 62 A 66-year-old man reports a history of excessive w orry about his d aughter since she moved aw ay from the area 1 year ago. H is w ife of 43 years verifies his complaint, ad d ing “he w orries about everything.” Recently, his w ife mad e plans to travel abroad to visit friend s. The patient is u nable to accomp any his w ife because of chronic obstructive pulmonary d isease cau sed by years of heavy smoking and is very anxious about her leaving. H e reports su bsequent d ifficulty falling asleep, excessive d aytim e fatigue, and trou ble concentrating at w ork. 61. Which of the follow ing m ed ications w ou ld be the m ost ap propriate to treat his sym p tom s? (A) (B) (C) (D) (E)

Alprazolam (Xanax) Busp irone (Bu Spar) Diazep am (Valiu m ) Lorazep am (Ativan) Propranolol (Ind eral)

62. Several w eeks later, the patient rep orts that the m ed ication you prescribed has helped everything bu t his d ifficu lty falling asleep . H e rep orts that d u ring the p ast w eek he has slep t only 4 hours p er night. After carefu l consid eration, you d ecid e to begin a trial of a 1-w eek su pply of m ed ication to help his insom nia. Which of the follow ing m ed ications w ou ld be the m ost app rop riate to prescribe? (A) (B) (C) (D) (E)

Diphenhyd ram ine (Benad ryl) Lorazepam (Ativan) Oxazep am (Serax) Tem azep am (Restoril) Zaleplon (Sonata)

Questions 63 and 64 A 25-year-old w om an w ith a history of m ajor d epressive d isord er, single episod e, in rem ission is ad m itted to the hosp ital for rem oval of a fibroid . H er p roced u re is com pleted w ithout d ifficu lty, but u p on

d ischarge, she is still exp eriencing significant p ain. She has no other m ed ical problem s or allergies. She is cu rrently taking oral contracep tives and a m onoam ine oxid ase inhibitor (MAOI) for her d epression. 63. Which of the follow ing analgesics shou ld be m ost avoid ed in this p atient? (A) (B) (C) (D) (E)

Acetam inophen Cod eine Ibu p rofen Mep erid ine Oxycod one

64. Tw o m onths later the patient retu rns for a follow -u p visit and rep orts that she is p regnant. Which of the follow ing interventions is the m ost app rop riate? (A) Continu e the MAOI. (B) Discontinu e the MAOI. (C) Discontinu e the MAOI and initiate treatm ent w ith flu oxetine after 2 w eeks. (D) Discontinu e the MAOI d u ring the first trim ester only. (E) Discontinu e the MAOI and begin m aintenance ECT. 65. A 62-year-old hom eless m an p resents to the em ergency d epartm ent w ith confu sion, agitation, im p aired gait, and nystagm u s. H is vital signs are stable and an ophthalm ologic exam ination is w ithin norm al lim its. The p atient is u nable to recall the d ate and has d ifficu lty su staining attention. Which of the follow ing is the m ost appropriate initial intervention? (A) Ad m inister IV d extrose. (B) Ad m inister p arenteral thiam ine. (C) Obtain a com p u ted tom ograp hic scan of the brain. (D) Perform a breathalyzer. (E) Perform a lu m bar p u nctu re. 66. You are a community psychiatrist who is seeing a patient for the first time. The patient informs that he has ju st m oved into the area and need s to establish him self w ith a p sychiatrist

Que s tions : 61–71

to continue his treatm ent for d ep ression and anxiety. On fu rther evalu ation, you learn that he w as previou sly arrested for child p ornograp hy and served several years in p rison. H e is cu rrently in therap y to ad d ress this issu e. You are su rp rised to find that after the d isclosure, you feel angry tow ard the patient as you have tw o you ng d au ghters of you r ow n. You are concerned abou t w hether it m ay be d ifficult for you to treat the patient in an u nbiased , nonjud gm ental m anner becau se of his crim inal history. Which of the follow ing is the next m ost ap prop riate step ? (A) Continu e to see the patient bu t lim it the tim e spent w ith him . (B) Im m ed iately refer the patient to another p hysician in you r clinic. (C) Inform the p atient that you are u nable to see him and refer him back to the com m unity. (D) Obtain the patient’s therap y record s so that you are aw are of his progress. (E) Seek a consu ltation w ith an exp erienced colleague regard ing you r feelings tow ard the p atient. 67. A 39-year-old execu tive w ithout p sychiatric history bu t a history of hyp ertension rep orts d rinking up to six cu ps of caffeinated coffee p er d ay. H e board s a p lane sched u led for an 18-hou r flight on w hich only d ecaffeinated beverages are served . Which of the follow ing sym ptom s is he m ost likely to experience? (A) (B) (C) (D) (E)

Depressed m ood H ead ache Irritability Mu scle cram p ing N au sea

Questions 68 and 69 A 25-year-old m an w ith a history of schizophrenia p resents to the em ergency d ep artm ent w ith severe m u scle stiffness and an elevated tem p eratu re. H is casew orker claim s that he is com pliant w ith his m ed ications and d enies that he u ses alcohol or d rugs. H is vital signs d em onstrate elevated tem p erature, blood p ressu re, and p u lse. The p atient

217

ap p ears confu sed and is d iap horetic. A u rine toxicology screen is negative. 68. Which of the follow ing is the m ost im portant intervention? (A) (B) (C) (D) (E)

Ad m inister brom ocrip tine. Ad m inister IV d antrolene sod iu m . Ad m inister naloxone (N arcan). Ap p ly cooling blankets. Discontinu e m ed ications.

69. Which of the follow ing laboratory abnorm alities is m ost likely to be p resent in this patient? (A) (B) (C) (D) (E)

Anem ia Decreased transam inases Elevated blood u rea nitrogen (BUN ) Elevated creatine p hosphokinase (CPK) Leu kop enia

70. An 18-year-old m an is brou ght to th e emergency department by the paramedics after being involved in a motor vehicle accident. His medical chart reports a history of substance use. Which of the following tests can most likely confirm the diagnosis of chronic substance use with physical dependence? (A) (B) (C) (D) (E)

Breath analysis Elevated heart rate N aloxone injection Seru m liver function stu d ies Urine toxicology screen

71. A 72-year-old man without prior psychiatric history presents to the outpatient clinic with a recent history of memory difficulty. The patient has had a stable level of consciousness and denies any current or past substance use. He has not been prescribed any new medications. The medical chart reveals a history of megaloblastic anemia and a subtotal gastrectomy for severe peptic ulcer disease. Which of the following initial interventions is the most appropriate? (A) Discu ss long-term extend ed -care facility ad m ission. (B) Obtain a forensic exam ination to evalu ate com p etency.

218

7: Pra c tic e Te s t 1

(C) Obtain a neu rologic consu lt. (D) Ord er a m agnetic resonance im aging scan of the brain. (E) Ord er a thorou gh laboratory w orku p . 72. A 37-year-old w om an presents to the em ergency d ep artm ent com p laining of severe d iarrhea, nau sea, and a coarse trem or. The m ed ical chart notes a history of bip olar I d isord er. Which of the follow ing w ou ld be the m ost ap prop riate initial intervention? (A) (B) (C) (D) (E)

Obtain renal fu nction stu d ies. Obtain a seru m m ed ication level. Obtain a u rine p regnancy test. Obtain a u rine toxicology screen. Prescribe antid iarrheal m ed ication.

Questions 73 and 74 A 72-year-old m an is brou ght into the em ergency room via am bu lance after su ffering w hat is su spected to be a stroke in his right p arietal region. 73. Which of the follow ing areas w ould be m ost likely to have d eficits d u e to the stroke? (A) (B) (C) (D) (E)

Long-term im plicit m em ory Long-term p roced u ral m em ory Tactile m em ory Verbal m em ory Visu al nonverbal m em ory

74. Which of the follow ing tests w ould best id entify the abnorm ality in that region? (A) Minnesota Mu ltip hasic Personality Inventory 2 (B) Rey–Osterrieth Test (C) Rorschach Test (D) Wechsler Intelligence Scales (E) Wisconsin Card Sorting Test 75. A 32-year-old m arried m an is d iagnosed w ith bip olar I d isord er. Variou s treatm ent op tions are d iscussed , and he agrees to begin lithiu m carbonate to treat his sym ptom s of m ania. Which of the follow ing tests is the m ost im p ortant to ord er p rior to initiating treatm ent w ith lithiu m ?

(A) (B) (C) (D) (E)

BUN and creatinine CBC ECG Liver fu nction stu d ies Thyroid -stim u lating horm one

76. A 27-year-old w om an has been prescribed flu oxetine for d ep ression for the p ast year. She com es to you com plaining of m ed ication sid e effects and w ishes to d iscontinu e her antid ep ressant. As her p hysician, you d iscu ss w ith her the ap p rop riate risks and benefits, and you d iscontinu e her treatm ent. Tw o w eeks later, she retu rns com p laining of d ep ressive sym p tom s characterized by sleep and appetite changes, poor concentration, and a d epressed m ood for m ost of the d ay. You d eterm ine that she is experiencing a recurrent m ajor d ep ressive episod e and d ecid e to prescribe her a d ifferent class of m ed ication to treat her sym p tom s. After d iscussing the appropriate precau tions w ith her, you prescribe an MAOI. Within 1 w eek, she begins exp eriencing irritability, abd om inal pain and d iarrhea, and au tonom ic instability characterized by hyp ertension and tachycard ia. H er tem p eratu re is 103.5°F. The p atient also rep orts exp eriencing jerking of her m uscles and vivid visu al hallucinations of colorfu l flow ers spinning tow ard the sky. She states that she has follow ed your d irections carefu lly w hile taking this m ed ication. Which of the follow ing cond itions m ost likely accou nt for this p resentation? (A) (B) (C) (D) (E)

H allu cinogen intoxication Malignant hyp ertherm ia N eu rolep tic m alignant synd rom e (N MS) Serotonin synd rom e Tyram ine-ind u ced hyp ertensive crisis

Questions 77 and 78 A 17-year-old girl is referred to the school nu rse for frequ ent ep isod es of vom iting in the bathroom d u ring lu nch breaks. H er friend s rep ort that, d esp ite alw ays talking abou t w anting to lose w eight, she eats “tw ice as m u ch as anybod y else.” The p arents are called to the school and recall that a recent bill from their charge accou nt at the local p harm acy

Que s tions : 72–82

ind icated a large nu m ber of laxative p u rchases. The girl rep orts that her m ood is fine, and her record s at school ind icate that she is an above-average stu d ent. 77. Which of the follow ing m ed ical com p lications w ou ld be m ost likely in this p atient? (A) (B) (C) (D) (E)

Acid osis Dental carries Diarrhea H yp erkalem ia H yp erchlorem ia

78. Which of the follow ing m ed ications w ou ld be the m ost ap propriate initial treatm ent for her sym p tom s? (A) (B) (C) (D) (E)

Bu propion Citalop ram Lithiu m Risp erid one Valp roic acid

79. A 70-year-old m an w ith m u ltip le m ed ical problem s is su sp ected to have had a stroke, affecting his ability to speak. Which of the follow ing tests w ou ld best assess the natu re of his speech d ifficu lty? (A) (B) (C) (D) (E)

Bend er Gestalt Test Boston Diagnostic Ap hasia Exam ination Folstein MMSE Sentence Com p letion Test Stroop Test

Questions 80 and 81 A 16-year-old fem ale high school stu d ent is referred by her parents for an evalu ation. One year prior to the evalu ation, the girl began restricting her food intake and started a rigorou s exercise program to im prove her appearance. Aspiring to be a m od el, the girl lost 25 lb bu t rem ained preoccu pied w ith her app earance d espite w eighing only 85 lb. H er friend s rep orted that she constantly referred to herself as being “fat” and d id not seem interested in d ating. The girl continu ed to lose w eight and w as relu ctant to d iscu ss her cond ition.

219

80. Which of the follow ing com p lications w ou ld be the m ost likely in this p atient? (A) (B) (C) (D) (E)

Diarrhea H eat intolerance Leu kocytosis Menorrhagia Osteop orosis

81. Which of the follow ing w ou ld be the m ost likely p red iction of a good ou tcom e in this patient? (A) (B) (C) (D) (E)

10% 20% 50% 75% 90%

Questions 82 and 83 A 70-year-old w om an rep orts a d ep ressed m ood an d insom n ia 1 m on th follow in g th e d eath of her h u sban d . Im m ed iately after h is d eath , sh e began to feel th at sh e w ou ld be “better off d ead ,” alth ou gh d en ies an y active su icid al id eation . For th e first several w eeks sh e h ad a red u ced ap p etite w ith a several p ou nd w eigh t loss, bu t this h as im p roved . At tim es, sh e believes th at sh e can h ear his voice calling to her. Sh e d en ies any feelin gs of w orthlessness bu t feels gu ilty abou t n ot bein g able to d o th e right th in gs for h im before h e d ied . Sh e occasionally gets d istracted and forgets that he h as d ied . Sh e is able to brigh ten w hen sp eakin g abou t sp en d in g tim e w ith h er gran d ch ild ren . Wh en talkin g w ith oth ers, sh e believes th at h er feelin gs are n orm al. 82. Which of the follow ing is the m ost likely d iagnosis? (A) Ad ju stm ent d isord er w ith d ep ressed m ood (B) Bereavem ent (C) Major d ep ressive ep isod e (D) Persistent d ep ressive d isord er (d ysthym ia) (E) Schizoaffective d isord er

220

7: Pra c tic e Te s t 1

83. Which of the follow ing is the m ost app rop riate course of action? (A) (B) (C) (D) (E)

Ad m ission to a hosp ital. Evalu ate for a sleep d isord er. N o treatm ent. Prescribe antid ep ressant m ed ication. Prescribe sed ative-hyp notic m ed ication.

84. A 25-year-old m an rep orts a 5-year history of excessive hand w ashing and a p reoccu p ation w ith feeling clean. The thou ght of contracting an infectiou s d isease p ersists throu ghout the d ay even thou gh he m akes attem p ts to ignore it. H is cond ition has p rogressively w orsened and has cau sed significant im p airm ent w hile at w ork and at hom e. Which of the follow ing m ed ications w ou ld be the best initial choice to treat his sym ptom s? (A) (B) (C) (D) (E)

Antid epressant Antiep ilep tic Antip sychotic Benzod iazep ine Lithiu m

85. A 28-year-old w om an w ith no p sychiatric history is arrested for shop lifting. She claim s she d oes not rem em ber d etails of the crim e, and that for “a w hile” she has had trou ble recalling d etails from things she’s recently read . You are called to assess her, bu t w ant to ensure she’ll give good effort on testing. Which of the follow ing tests w ou ld be the m ost ap p rop riate to ad m inister? (A) (B) (C) (D) (E)

Benton Visu al Retention Test Clock Draw ing Test of Mem ory Malingering Wechsler Intelligence Scale Wisconsin Card Sorting Test

Questions 86 and 87 A 52-year-old obese m an exp eriences excessive d aytim e sleepiness and a d epressed m ood . H is w ife rep orts that he snores lou d ly and is restless w hile sleeping. There is no evid ence of regu lar alcohol or illicit su bstance u se, and the p atient d oes not have a p rior p sychiatric history.

86. Which of the follow ing d iagnoses w ou ld be the m ost likely cause of his sym ptom s? (A) (B) (C) (D)

Circad ian rhythm sleep –w ake d isord er Major d ep ressive d isord er Obstru ctive sleep ap nea hyp op nea Rap id eye m ovem ent (REM) sleep behavior d isord er (E) Restless legs synd rom e

87. Which of the follow ing is the treatm ent of choice for his sym ptom s? (A) (B) (C) (D) (E)

Antid ep ressant m ed ication Benzod iazep ine m ed ication Breathing air u nd er p ositive p ressu re N asal su rgery Uvu lop alatop lasty

Questions 88 and 89 A 60-year-old m ale p atient w ith schizop hrenia w ho has been stable for years on a low -p otency antip sychotic agent begins exp eriencing parkinsonian-like sid e effects. H is p hysician p rescribes a d ru g to alleviate som e of these sid e effects. One w eek later, the p atient is seen in the em ergency d ep artm ent w ith d ilated pup ils, d ry m ou th, w arm skin, and tachycard ia. H e is also exp eriencing the new onset of visu al hallu cinations. 88. Which of the follow ing w ou ld be the m ost ap p rop riate m ed ication to ad m inister? (A) (B) (C) (D) (E)

Anticholinesterase Atrop ine Benzod iazep ine Dantrolene sod iu m H alop erid ol

89. After the ap p rop riate intervention, the p atient exp eriences nau sea and vom iting and su bsequ ently has a seizu re. Which of the follow ing m ed ications shou ld be ad m inistered next? (A) (B) (C) (D) (E)

Atrop ine Ep inep hrine Lorazep am Physostigm ine Prochlorp erazine (Com p azine)

Que s tions : 83–95

90. A 40-year-old w om an com p lain s of 3 to 4 m onths of feeling sad , w ith trou ble falling asleep , d ecreased ap petite and som e w eight loss, d ifficu lty concentrating at w ork, and fatigu e. She ad m its to su icid al id eation w ithou t sp ecific plan or intent. There is no history of m ania. She also m entions longstand ing and repetitive thoughts w here she w orries abou t “germ s.” As a resu lt, she w ashes her hand s very frequ ently, som etim es end ing u p w ith cracked and chapp ed skin. She realizes that her w orries are unrealistic, but she cannot keep from d oing it. Which of the follow ing m ed ications w ou ld be the m ost app rop riate to p rescribe? (A) (B) (C) (D) (E)

Busp irone Clom ip ram ine (Anafranil) Doxep in (Sinequ an) Flu voxam ine (Lu vox) Phenelzine (N ard il)

Questions 91 and 92 A 56-year-old m an w ith a long history of alcohol u se d isord er and elevated liver function tests is ord ered by the court to enroll in an inp atient d etoxification program for his alcohol u se d isord er after nu m erou s DWIs. After 2 d ays in treatm ent, he begins to exp erience trem ors, sw eating, flu shing, and anxiety. 91. Which of the follow ing m ed ications w ou ld be the m ost approp riate to prescribe? (A) (B) (C) (D) (E)

Alp razolam Chlord iazep oxid e (Libriu m ) Disu lfiram (Antabu se) Lorazepam Phenobarbital

92. Shortly after this tim e, the m an begins to rep ort visu al hallu cinations and becom es agitated . Which of the follow ing m ed ications shou ld be prescribed to treat these sym ptom s? (A) (B) (C) (D) (E)

Fluoxetine H alop erid ol Lithiu m Lorazep am Sertraline

221

93. A 48-year-old w oman successfu lly completes an inpatient program for alcohol d etoxification for w hich she w as prescribed chlord iazepoxid e. Upon d ischarge, the patient is prescribed d isulfiram. Soon after d ischarge, the patient attend s an office party w here she ad m its to having a few d rinks. She has been comp liant w ith her prescribed med ication and d oes not have any active med ical problem s. Which of the follow ing symptom s is she most likely to experience? (A) (B) (C) (D) (E)

Blu rred vision Eu p horia H igh blood p ressu re Urinary retention Vom iting

94. A 22-year-old m an w ith a history of bipolar d isord er is prescribed lithium carbonate to treat his sym p tom s. Du ring a w eekend ru gby tou rnam ent, he hu rts his knee and an orthoped ic physician p rescribes a m ed ication to red u ce his sym p tom s of p ain and sw elling. Although the patient reports relief from this m ed ication, he begins to exp erience abd om inal pain, d iarrhea, and d row siness. Which of the follow ing m ed ications w ou ld m ost likely contribu te to the prod u ction of these sym p tom s? (A) (B) (C) (D) (E)

Acetam inophen Asp irin Cod eine Ibup rofen Mep erid ine

95. A 32-year-old new ly m arried m an w ith a p ast history of m ajor d ep ressive d isord er p resents w ith a 4-w eek history of d ep ression, insom nia, anergia, p oor concentration, and anhed onia. H e has had p assive su icid al id eation w ithou t a p lan, as w ell. H e has not taken citalop ram for several years, and , d esp ite rem ission of his sym p tom s, he had significant sexu al d ysfu nction w hen taking it. H e w ishes to restart a m ed ication bu t is greatly concerned abou t the im p act on his m arriage. Which of the follow ing m ed ications w ou ld be the m ost ap p rop riate?

222

7: Pra c tic e Te s t 1

(A) (B) (C) (D) (E)

Citalop ram Flu voxam ine Paroxetine Mirtazapine Venlafaxine

96. A patient w ho is hu m an im m u nod eficiency viru s (H IV)-p ositive rep orts 4 w eeks of a d epressed m ood , low energy, p oor sleep, and hopelessness. H is app etite is negligible, and he has been refu sing to eat or d rink anything for 2 d ays. Which of the follow ing m ed ications w ou ld be the m ost appropriate to p rescribe? (A) (B) (C) (D) (E)

Bupropion Buspirone Flu oxetine Methylphenid ate Sertraline

97. A 43-year-old w om an p resents w ith a new history of d ep ressive sym p tom s, inclu d ing insom nia, poor ap petite and w eight loss, low energy, and d istractibility. On m ental statu s exam ination she is asked to cou nt backw ard by 7’s, beginning at 100. Which of the follow ing asp ects is being assessed by this test? (A) (B) (C) (D) (E)

Attention Fu nd of know led ge Mathem atics skills Rem ote m em ory Verbal m em ory

Questions 98 and 99 A 19-year-old college stu d ent rep orts that he and his friend s have been experim enting w ith “hu ffing” on cam pu s. H is room m ate rep orts that he has been accu mu lating typew riter correction fluid , nail polish rem over, and m od el airp lane glu e. 98. Which of the follow ing w ou ld he m ost likely exp erience d u ring intoxication? (A) (B) (C) (D) (E)

Conju nctival injection Depressed reflexes Dim inished response to pain Increased ap petite Staring into space

99. Which of the follow ing is the m ost effective treatm ent for this type of d rug abuse? (A) (B) (C) (D) (E)

Abstinence Antid ep ressant agents Antip sychotic agents Dialectic behavioral therapy Exp osu re and resp onse p revention

100. A 30-year-old man w ith schizophrenia is brought in w ith his mother, w ith w hom he lives. His med ical chart d ocuments numerous but adequate medication trials including risperidone, olanzapine, quetiapine, and haloperid ol. Despite his compliance he continues to have significant paranoia, d elusions, and aud itory hallucinations, telling him that he is going to die. H e has no other medical problems, and both he and his mother d eny any alcohol or d rug use. Which of the follow ing medications w ould be the most appropriate to prescribe for this patient? (A) (B) (C) (D) (E)

Arip ip razole Clozap ine Flu p henazine Qu etiap ine Zip rasid one

101. You are treating a 32-year-old w om an for bip olar I d isord er w ith a com bination of m ed ications. She d evelops hair loss w hich eventually resolves on its ow n. Which of the follow ing m ed ications is m ost likely resp onsible for this sid e effect? (A) (B) (C) (D) (E)

Carbam azepine Clozap ine Divalp roex sod iu m Olanzap ine (Zyp rexa) Zip rasid one (Geod on)

102. A 34-year-old man is referred by his internist for depression. For the past 2 months he has been suffering from anhedonia, crying spells, frequent aw akenings, poor appetite, and low energy. He is subsequently diagnosed w ith a major d epressive episod e and recommend ed to begin citalopram 20 mg daily. After discussion of the risks and benefits of the med ication, he

Que s tions : 96–107

expresses concern about possible sexual d ysfunction as he is currently in a new relationship w ith a cow orker. Which of the follow ing symptoms are the most likely w ith this med ication? (A) (B) (C) (D) (E)

Decreased libid o Prem ature ejaculation Priap ism Retrograd e ejaculation Sexually transm itted d isease

Questions 103 and 104 A 21-year-old m ale college stud ent w ithou t p rior m ed ical or psychiatric history is evalu ated for p oor w ork p erform ance. The stu d ent rep orts to the d ean that he find s it hard to follow through w ith assignm ents and is easily d istracted by environm ental stim uli. The d ean com m ents to the stu d ent abou t how frequ ently he interru p ts others d u ring conversations and observes that it seem s that the stud ent is not p aying attention to w hat he is saying. Overall, his grad es are p oor excep t in the class he consid ers “the m ost interesting class I have ever had .” H e often forgets requ ired m aterials for classes and his assignm ents are frequ ently late. The stu d ent’s p arents report that he w as evaluated for sim ilar d ifficu lties w hile in p u blic school, bu t he seem ed to im p rove w hen the fam ily m oved ou t of the area and he attend ed p rivate school. 103. Which of the follow ing is the m ost likely d iagnosis? (A) Attention-d eficit/ hyperactivity d isord er (ADH D) (B) Bipolar I d isord er (C) Bip olar II d isord er (D) Cond u ct d isord er (E) Learning d isord er 104. The p atient is p rescribed m ethylp henid ate for the above sym ptom s. After an ad equate trial and d ose, he continu es to d isp lay som e sym p tom s w hich interfere w ith his school fu nctioning, althou gh he d enies significant sid e effects. Which of the follow ing actions w ou ld be the m ost ap prop riate next step? (A) Ad d a low -d ose am phetam ine. (B) Ad d an antid ep ressant.

223

(C) Ad d an anxiolytic m ed ication. (D) Discontinu e m ethylp henid ate and p rescribe a m ixed am phetam ine com pou nd . (E) Discontinu e m ethylp henid ate and p rescribe an antid ep ressant. 105. A 35-year-old w om an presents w ith ep isod ic anxiety and com p laints that occasionally w hen hearing som ething for the first tim e, that she has actually heard it before. She expresses her concern that she is “going crazy.” Which of the follow ing term s best d escribes this p henom enon? (A) (B) (C) (D) (E)

Déjà entend u Déjà vu Folie à d eu x Jam ais vu La belle ind ifférence

106. A 19-year-old m an is brou ght to the em ergency d ep artm ent by the p olice for an evalu ation. The w ritten p olice rep ort states that the p atient has been calling 911 for the past 5 w eeks, reporting that he is being spied on by aliens from a d istant p lanet. The p atient rep orts that he is receiving m essages from the aliens throu gh his com pu ter and that he hears voices in his head com m enting on his appearance. H e has not been sleep ing w ell at night becau se he has been “gu ard ing his bed room .” You note that his affect is flat, and he ap p ears tired d uring your examination. A urine toxicology screen is negative. Which of the follow ing is the m ost likely d iagnosis? (A) Bip olar I d isord er w ith p sychotic featu res (B) Brief p sychotic d isord er (C) Major d ep ressive d isord er (MDD) w ith p sychotic featu res (D) Schizoaffective d isord er (E) Schizop hreniform d isord er 107. An 18-year-old w oman w ith recently d iagnosed schizophrenia is acutely psychotic and in labor w ith her first child . The obstetrics service requests a psychiatric consultation for an appropriate and safe med ication to use in ord er

224

7: Pra c tic e Te s t 1

to control the patient’s psychotic symptoms. Which of the follow ing w ould be the best choice?

110. A 7-year-old boy w ith d ifficu lty in school need s fu rther evalu ation of acad em ic p otential.

(A) (B) (C) (D) (E)

111. You w ou ld like to ad d ress the d eficits of a 69-year-old m an w ith nonflu ent sp eech and form u late a sp ecific treatm ent p lan.

Chlorp rom azine (Thorazine) H aloperid ol (H ald ol) Loxap ine (Loxitane) Perphenazine (Trilafon) Thiorid azine (Mellaril)

D IRECTION S (Questions 108 through 116): The follow ing group of questions are preceded by a list of lettered options. For each questions, select the one lettered option that is most closely associated w ith it. Each lettered option may be used once, multiple times, or not at all. Questions 108 through 111 Match the clinical presentation w ith the ap propriate neu ropsychological test. (A) (B) (C) (D) (E) (F) (G) (H ) (I) (J) (K)

Beck Depression Inventory Bend er Gestalt Test Blessed Rating Scale Boston Diagnostic Aphasia Exam ination Mini-Mental state Exam ination (MMSE) Minnesota Mu ltiphasic Personality Inventory 2 (MMPI-2) Rey–Osterrieth Test Rorschach Test Wad a Test Wechsler Ad u lt Intelligence Scale— Revised (WAIS-R) Wechsler Intelligence Scale for Child ren (WISC)

108. A 65-year-old fem ale p hysician w ho is now concerned about m em ory d ifficulties. 109. A 35-year-old w om an w ith a p rotein S d eficiency show s evid ence of left sid e hem i-neglect.

Questions 112 through 116 Select the d ru g m ost likely to cause the associated sym ptom s. (A) (B) (C) (D) (E) (F)

Cocaine Lysergic acid d iethylam ine (LSD) Mariju ana N icotine Op iates Phencyclid ine (PCP)

112. An 18-year-old high school student with conjunctival redness, increased appetite, dry mouth, tachycardia, and a sensation of slowed time. 113. A 31-year-old m an w ith m iosis, brad ycard ia, hyp otension, hypotherm ia, and constipation. 114. An assau ltive 26-year-old m an w ith vertical nystagm u s, echolalia, paranoid id eation, and hallucinations. 115. A 16-year-old girl w ith abd om inal cram ps, confu sion, palpitations, and m u scle tw itching. 116. A 21-year-old m an w ith tachycard ia, d ilated p upils, hallucinations, and com p laints of chest p ain.

Ans we rs a nd Expla na tions

1. (C) Antip sychotics are ind icated for acu te treatm ent of agitation and violence som etim es seen in m anic patients. H aloperid ol w orks relatively qu ickly (20–30 m inu tes). Doses of 2 to 5 m g by m ou th (PO) or intram u scu larly (IM) are u su al, althou gh it m ay be given IV as w ell, especially in the intensive care u nit (ICU). H yd roxyzine is an antihistam ine; it is not effective in m ania. Lithium , d ivalproex sod ium , and carbam azepine have all been show n to control m ood fluctu ations in m anic p atients. H ow ever, these agents take d ays to w ork and are not effective in the im m ed iate m anagem ent of this patient. 2. (D ) Lithium is not associated w ith significant blood d yscrasias, althou gh it can cau se a m od est benign increase in the w hite blood cell (WBC) cou nt. Carbam azepine is com m only associated w ith a benign red u ction in WBC cou nt, bu t severe blood d yscrasias only occu r in app roxim ately 1 in 125,000 p atients. Agranulocytosis is also a rare com p lication w ith d ivalproex sod iu m , although a benign throm bocytop enia is m ore com m on. Agranulocytosis can also occu r in the setting of antipsychotic m ed ication u se in rare cases (clozap ine has a higher incid ence). Lorazep am is not associated w ith WBC abnorm alities bu t is not a m ood stabilizer. 3. (D ) This patient is m ost likely suffering from som atic sym ptom d isord er, w hich often coexists w ith other p sychiatric illnesses su ch as anxiety, d ep ressive, and p ersonality d isord ers. The su sp ected p resence of a som atic sym ptom d isord er should prom pt a search for other treatable illnesses. In m anaging patients

w ith som atic sym p tom d isord er, it is im p ortant to u nd erstand that sym ptom s are not consciou sly prod u ced and to let p atients know that you realize their sym p tom s are a source of great consternation. Regard less of history, any p atient presenting to a p hysician w ith p hysical com plaints d eserves a reasonable physical investigation; p atients w ith som atic sym p tom d isord er are as likely, if not m ore likely, to d evelop id entifiable m ed ical cond itions. H ow ever, rather than rep eat tests, it m ay be necessary to contact previous treaters. Send ing this p atient aw ay for a year w ou ld not be help ful; rather regular, frequ ent checku ps and fostering a therap eu tic alliance and su p p ort are in ord er. 4. (C) Ap p roxim ately 50% of ind ivid u als w ith som atic sym p tom d isord er have ad d itional p sychiatric illnesses, esp ecially anxiety and d ep ression. Personality traits or a d isord er is not u ncom m on. Obsessive-com pu lsive personality d isord er is the m ost likely com orbid p ersonality d isord er in som atic sym ptom d isord ered p atients. 5. (E) This patient is most likely suffering from schizotypal personal disorder, which as with all personality disorders, represents a lifelong maladaptive approach to life and does not suddenly develop in ad ulthood . Schizotypal personality disorder with its attendant social isolation and subtle distortions of reality may ind eed resemble schizophrenia; the diagnosis can be sorted out by a thorough history. Frank hallucinations of any type are uncommon; only subtle distortion of environmental cues is seen. There is no evidence of mania to justify a diagnosis of 225

226

7: Pra c tic e Te s t 1

bipolar disorder. While under stress, patients with borderline personality disorder can have so-called “micropsychotic episodes,” they display chronic feelings of emptiness, mood/ affect lability, self-mutilation, and intense anger and impulsivity. Schizoid personality disorder may appear similar to schizotypal personality disorder, although it is characterized more by feelings of d etachment, lack of friends, choosing solitary activities, and anhedonia. 6. (A) In schizotyp al p ersonality d isord er, the subtle d isconnection from reality, w hich m ay be exacerbated in tim es of stress as in this case, can be treated w ith low d oses of antip sychotic m ed ication. These patients lack the cap ability, stable sense of self, and tru st to be able to engage in, or benefit from , p sychod ynam ic p sych oth erap y. An tid ep ressant m ed ication of any type, includ ing tricyclics and SSRIs, is helpful in the schizotypal patient w ho d isplays significant affective (mood ) symptoms, w hich are not seen in this case. Benzodiazepines are not ind icated in this patient as there is no evid ence of anxiety. 7. (C) This p atient is m ost likely su ffering from schizophrenia. Sym ptom s of schizop hrenia are com m only d ivid ed into positive and negative sym ptom s. Flat affect, a negative sym ptom , represents an absence of a norm ally reactive and variable affect. Other negative sym ptom s inclu d e alogia and avolition. Au d itory hallucinations, d elu sions (inclu d ing p aranoid d elu sions), and loose associations are all positive sym p tom s. 8. (C) The exacerbation seen is m ost likely p recip itated by m ed ication noncom p liance. A viral illness or stress from w ork m ay ind eed contribu te to a relap se, bu t they are less strongly p red ictive of a reem ergence of p sychotic sym p tom s than m ed ication noncom p liance. N icotine has been show n to low er neu rolep tic levels, w hich has been offered as a reason cigarette sm oking is ram p ant am ong p atients w ith schizop hrenia. H ow ever, this p atient’s sm oking d ecreased , w hich if anything w ou ld be exp ected to increase antip sychotic levels. Since she has not been

com p liant w ith her m ed ication, her exacerbation is not likely d u e to treatm ent refractory schizop hrenia. 9. (B) Classically, p atients w ith hep atitis second ary to alcohol use d isord ers have elevated liver enzym es (su ch as gam m a-glu tam yl transp ep tid ase, aspartate transam inase, or alanine transam inase). H ow ever, in ad vanced alcoholism , the liver m ay be “bu rnt ou t” and liver fu nction tests m ay reveal low or norm al levels of these enzym es. As a resu lt of liver d am age, prothrom bin tim e is typ ically increased . H ypom agnesem ia, not hyperm agnesem ia, is m ore likely to be fou nd in alcohol u se d isord er, usu ally as a resu lt of d ietary d eficiency. In the p atient having a w ithd raw al seizure, the p roblem is the absence, not presence, of alcohol. H is blood alcohol w ou ld be exp ected to be low or zero. A p atient w ith alcohol u se d isord er is m ore likely to have throm bocytop enia rather than throm bocytosis. 10. (C) The p atient has alcohol u se d isord er and has su ffered a w ithd raw al seizu re. The m ainstay of alcohol d etoxification rem ains benzod iazep ines, w hich m ay be ad m inistered p arenterally. Lorazep am is com m only u sed , since it can be given PO, IM, or IV, and it is not d ep end ent on ad equ ate liver fu nctioning (w hich m ay be com p rom ised in alcoholics) for its m etabolism . While d iazep am (Valiu m ) can be u sed , it has erratic absorp tion w hen given IM, and , in a p atient w ith liver d am age, its p rolonged half-life can create a large bu ild u p w ith su bsequ ent oversed ation and p otential resp iratory d ep ression. H alop erid ol is an antip sychotic m ed ication w hich m ay help w ith agitation seen in alcohol w ithd raw al, bu t it d oesn’t treat the u nd erlying p roblem and m ay fu rther low er the seizu re threshold . While antiseizu re m ed ications su ch as carbam azep ine have been show n to be effective in the m anagem ent of alcohol w ithd raw al, they are not consid ered to be the first-line treatm ent. 11. (B) While the p atient d oes not have a history of m ental illness, w hich provid es inform ation abou t p ast behaviors, she is actively su icid al

Answe rs : 6–17

w ith a p lan, and attem p ts to contact collaterals have been unsu ccessfu l. Collateral inform ation w ou ld be help fu l to valid ate the p resent history as w ell as to p rovid e d ata abou t baseline m ental statu s. Given the absence of collateral inpu t and the fact that she rem ains suicid al, the p atient shou ld not be d ischarged from the em ergency d epartm ent and shou ld be involu ntarily ad m itted u nd er certificate and petition. The p atient d oes not exhibit p sychotic behavior and is not a m anagem ent p roblem ; the ad m inistration of an antip sychotic m ed ication can cau se sid e effects and m ay affect fu tu re com pliance w ith p sychotropic m ed ications. The other choices d o not ad equ ately ad d ress her safety. 12. (A) The Beck Dep ression Inventory is a w id ely u sed test that allow s clinicians to follow the severity of previously d iagnosed d ep ression. The Draw -A-Person Test, Rorschach Test, and TAT are all typ es of p rojective testing. Projective tests, although useful clinical tools, often suffer low reliability and valid ity. Projective tests requ ire a person skilled at this type of evalu ation and d o not alw ays have rigorou s em p irical d ata and grou p com p arison. The evalu ation of d ep ression u sing the H alstead Reitan Test is lim ited by the fact that m any d ep ressed patients fail to show d eficits on such classic neu ropsychological batteries. In ad d ition, these tests, even w hen d em onstrating d eficits in cognitive d om ains su ch as attention and learning, still have very lim ited u sefulness in evaluating the severity of the d ep ression.

227

Utilizing an antid ep ressant w ith a d ifferent m echanism of action (affecting d ifferent neu rotransm itters) m ay be beneficial. Venlafaxine is a serotonin and norepinephrine reu ptake inhibitor, w hich, in higher d oses, w ill increase the intrasynap tic levels of both neu rotransm itters. All the other choices are SSRIs. 15. (C) This scenario represents a case of factitiou s d isord er im posed on another (form ally factitious d isord er by proxy), in w hich one person falsifies illness in another to vicariously gain m ed ical attention. Com m only, the victim is a child and the perp etrator is the child ’s m other. Ap p arent bleed ing, seizu res, and central nervous system (CN S) d epression are typical presentations. The d isord er is und erd iagnosed d ue both to the elu sive and crafty p lanning of the perp etrator and the u nw illingness of health care p rovid ers to accu se the ostensibly caring parent. Given the age of the child , it is extrem ely u nlikely that the child , him self, is intentionally p rod u cing the sym p tom s, and there is no evid ence that he is experiencing p hysiologic seizu res. Conversion d isord er is characterized by the p resence of one or m ore neu rologic sym p tom s that are incom p atible w ith a recognized m ed ical or neu rologic d isord er. Paralysis, blind ness, and pseud oseizu res are com m on conversion d isord er sym p tom s. Sep aration anxiety d isord er is characterized by excessive and inap p rop riate anxiety concerning sep aration from hom e or to w hom ind ivid u als are attached .

13. (B) The Brow n-Peterson Task is a test sp ecifically d esigned to evalu ate short-term m em ory, a cap acity that can be affected d u ring ECT. The Bulim ia Test—Revised and the EDI-2 are both u seful for the evalu ation of bu lim ia and eating d isord ers, resp ectively. The Beck Depression Inventory and State-Trait Anxiety Inventory are u sed to evaluate d epression and anxiety d isord ers, respectively.

16. (C) Mothers w ith factitiou s d isord er im p osed on another are m ost likely to have u nd erlying m ajor d ep ressive d isord er. Bord erline p ersonality d isord er is the m ost com m on p ersonality d isord er associated w ith factitiou s d isord er im p osed on another. Bip olar d isord er and PTSD have not been clearly d escribed in the literatu re. Ep ilep sy is not associated w ith the d isord er to any significant extent, and schizop hrenia has been rep orted in only a few cases.

14. (E) The patient has m u ltiple episod es of d ep ression, u nfortunately w ithou t rem ission d esp ite several m ed ication trials w ith SSRIs.

17. (E) Panic d isorder is characterized by the sudd en unexpected occurrence of panic attacks (period s of intense anxiety or fear accompanied

228

7: Pra c tic e Te s t 1

by som atic sym p tom s), com m only cau sing the m isd iagnosis of a m ed ical illness such as m yocard ial infarction. The frequ ency of p anic attacks varies w id ely from m any per d ay to a few per year. Panic d isord er is often associated w ith agorap hobia, the fear of being in p laces w here escap e is d ifficu lt. The lifetim e p revalence is up to 4%. Concerns of d eath from card iac or respiratory d isord ers occu r frequ ently. Acu te stress d isord er is d iagnosed in ind ivid u als w ho have experienced a trau m atic event and subsequ ently d evelop sym p tom s w ithin 4 w eeks of the event that rem it after 1 m onth. The trau m atic event m u st be reexp erienced by the p atient, and this cau ses the p atient to avoid stim uli that arouse recollections of the trau m a. Other criteria necessary for the d iagnosis are the presence of d issociative sym p tom s and m arked anxiety or increased arou sal not better accounted for by other m ed ical or p sychiatric illnesses. Delirium is characterized by the su d d en onset of a d istu rbance attention and aw areness, as w ell as a change in cognition, cau sed by another m ed ical cond ition, substance (alcohol, d ru g, m ed ication), and / or toxin. 18. (E) Of the choices listed , sertraline, an SSRI, is the m ost efficaciou s. In ad d ition, tricyclic antid ep ressants, MAOIs, and the benzod iazep ines are also effective in the treatm ent of p anic d isord er. Beta-ad renergic d ru gs like p rop ranolol are not effective for the treatm ent of p anic d isord er. H alop erid ol, an antip sychotic agent; lithiu m , a m ood stabilizer; and clonid ine, an alp ha-2-ad renergic agonist, are ineffective. 19. (A) H yp ertensive crisis is a p otentially lifethreatening com plication that occu rs w hen p atients taking an MAOI eat tyram ine-containing food s su ch as w ine, beer, pickled food s, and aged cheese. Clinical featu res includ e hyp ertension, severe occipital head ache, stiff neck, nausea, vom iting, and sw eating. IV p hentolam ine, an alp ha-ad renergic recep tor blocker, is the treatm ent of choice. It has been show n to be m ore effective and safer than beta blockers or calciu m channel blockers. Ad m ission to an ICU and su pportive m easu res are

ind icated . N either brom ocrip tine, a d op am ine agonist, nor d antrolene, a m u scle relaxant, is ind icated . 20. (D ) Clozap ine is an effective antip sychotic m ed ication that has been associated w ith few er extrapyram id al sid e effects than the conventional antipsychotics (w hich prim arily act by blocking d op am ine typ e 2 recep tors). Over 4% of p atients taking m ore than 600 m g/ d of clozap ine experience clozapine-associated seizu res. Tachycard ia, hyp otension, sed ation, fatigue, and w eight gain have all been associated w ith clozapine treatm ent. Clozapine, u nlike the conventional antip sychotic agents, d oes not affect prolactin secretion and thus d oes not cause galactorrhea. 21. (B) Agranu locytosis is a p otentially lifethreatening sid e effect of clozapine treatm ent. It is d efined as a d ecrease in the nu m ber of WBCs, w ith a sp ecific d ecrease in the nu m ber of neu trop hil granu locytes. It occu rs in ap p roxim ately 1% of all p atients treated w ith clozapine. Therefore, a com plete blood cou nt w ith d ifferential is requ ired w eekly in the beginning of treatm ent and frequ ently thereafter. Rou tine m onitoring w ith the other tests are not requ ired or necessary w ith clozap ine treatm ent. 22. (B) The Folstein MMSE is a frequently u sed screening assessm ent for m ajor neu rocognitive d isord er (d em entia), althou gh m ore in d ep th and form al neu ropsychological testing shou ld be p erform ed at som e p oint. It is a 30-p oint scale w ith d ed u ctions for incorrect answ ers. Scores less than 25 are su ggestive of cognitive im p airm ent. Scores less than 20 ind icate d efinitive im pairm ent. The Blessed Rating Scale is a tool that typ ically asks a p atient’s friend s or relatives to assess the ability of the p atient to fu nction in his or her cu rrent environm ent. The Geriatric Rating Scale is a rating scale for nonp rofessional staff to evalu ate p atients’ abilities to p erform their activities of d aily living and interact w ith others. It m ay be m ost help ful in evalu ation of the m od erately to severely d em ented ind ivid ual. The Glasgow Com a Scale is an easy-to-perform instrum ent

Answe rs : 18–27

that evalu ates level of consciou sness. There are three general categories that the exam iner tests: eye op ening, verbal resp onse, and best m otor resp onse. Each category receives a num ber for p atient resp onse. Overall scores range from 3 to 15, w ith low er scores reflecting m ore severely im p aired consciou sness. A m ental statu s exam ination is the form al p sychiatric exam ination that includ es m any item s such as appearance, behavior, sp eech, assessm ent of m ood and affect, p resence of p sychosis, and evaluation of insight and jud gm ent. It is not a su bstitu te for the Folstein MMSE. 23. (B) When evaluating the MMSE score in elderly patients, it is critical to consid er that major d epression (also know n as “pseud od ementia”) may prod uce similar impairments in cognition. The other d iagnoses listed w ould not commonly red uce the Folstein MMSE score. 24. (E) Schizotyp al p ersonality d isord er is characterized by a p ervasive p attern of social and interp ersonal d eficits. Ind ivid u als d em onstrate a red u ced cap acity to establish any close relationship s, and eccentric behavior is p resent. The p resence of od d beliefs or m agical thinking sep arates schizotyp al p ersonality d isord er from schizoid p ersonality d isord er. Paranoid id eation is com m on in both schizotyp al and p aranoid p ersonality d isord ers bu t not in the others. Avoid ant p ersonality d isord er rep resents a p ervasive p attern of behavior characterized by social inhibition, feelings of inad equ acy, and hyp ersensitivity to negative evalu ation. N arcissistic p ersonality d isord er rep resents a p ervasive p attern of grand iosity, lack of em p athy, and a need for ad m iration. Schizoid p ersonality d isord er rep resents a p ervasive p attern of d etachm ent from social relationship s and a restricted range of exp ressed em otions. 25. (B) Bord erline p ersonality d isord er is a pervasive pattern of instability of interpersonal relationships, self-im age, affect, and marked impulsivity. Antisocial personality d isord er d escribes ind ivid uals w ith long histories of d isregard for the rights of others and often d ishonesty used in attem pts to gain som ething

229

for themselves. Patients w ith d epend ent personality d isord er require an excessive amount of ad vice, reassu rance, and approval from others. Ind ivid u als w ith histrionic personality d isord er, like those w ith bord erline personality d isord er, may d isplay excessive emotionality and attention-seeking behavior, but their core sym ptoms center around su perficial sed u ctiveness and theatricality. Schizoid p ersonality d isord er represents a pervasive pattern of d etachment from social relationships and a restricted range of expressed emotions. 26. (E) In the WCST, exam inees are asked to sort card s d ep icting variou s p ictu res and sym bols accord ing to a variety of d ifferent criteria that change over tim e w ithou t the su bject’s know ing. The WCST assesses a p erson’s ability to sw itch sets, reason abstractly, and solve p roblem s. These cap acities are also know n as execu tive fu nctions and are localized in the frontal lobes. Cu rrent research regard ing the p sychop athology of schizop hrenia su ggests that there are abnorm alities in the frontal lobes, sp ecifically in the d orsolateral p refrontal cortex, that are reflected in p oor p erform ance on the WCST. Peop le w ith schizop hrenia p erform m ore p oorly on the WCST than p eop le w ithou t schizop hrenia; how ever, ind ivid u als w ith d am age to their frontal lobes from a variety of cau ses also show execu tive fu nction d eficits. The Bend er Gestalt Test involves cop ying figu res, w hich help s d eterm ine if organic brain d isease is p resent. The Draw -A-Person Test requ ires the exam inee to d raw a p erson. It w as initially d evised to test intelligence in child ren, bu t is now u sed p rim arily as a screening test for brain d amage. The Luria-N ebraska N europsychological Battery is a com prehensive set of neu ropsychological tests used to assess specific cortical areas and aid s in assessm ent of hem ispheric d om inance. The Mini-Mental State Examination is a com monly used scale to assess the possibility of a major neurocognitive d isord er (d em entia). 27. (B) The WCST assesses a p erson’s ability to sw itch sets, reason abstractly, and solve p roblem s. These cap acities are also know n as

230

7: Pra c tic e Te s t 1

execu tive fu nctions and are localized in the frontal lobes. The cerebellum is responsible for coord ination. The occipital lobe is d ed icated to the sense of sight. The parietal lobe hand les sensory inp u t, m otor outpu t, and visu ospatial processing. The tem p oral lobe is resp onsible for the sense of sou nd , taste, and m em ory storage. 28. (C) This p atient is m ost likely su ffering from bip olar I d isord er. Racing thou ghts, p ressu red speech, expansive m ood , a d ecreased need for sleep, and an increase in goal-d irected activity are all com m on m anifestations of m ania. The p atient is u su ally energized d u ring the d ay even after only a few hou rs of sleep . Dep ressed m ood and w eight loss are m ore characteristic of a d ep ressive d isord er. Visu al hallu cinations cou ld be present in severe cases of m ania but are m ore com m only a sym ptom of a p rim ary p sychotic d isord er su ch as schizophrenia. 29. (E) Many d isord ers can m im ic sym p tom s of m ania. A com p lete history, physical exam ination, and rou tine laboratory tests can su fficiently ru le ou t m ost m ed ical cau ses of m ania. They inclu d e end ocrine d isord ers su ch as thyrotoxicosis and Cu shing synd rom e, hypoglycem ia, electrolyte d isord ers, su bstance u se and w ithd raw al, m ed ications su ch as steroid s and anticholinergic agents, nu tritional d eficiencies, and CN S insu lts.

m ay be as high as 12%, com p ared w ith a > 1% risk in the general p op u lation. 33. (A) The p atient is being evalu ated w ith the Rorschach Test, w hich w as d evelop ed in 1921 by H erm ann Rorschach. It consists of show ing a su bject a set of 10 inkblot stim u li in a sequ ential m anner w hile noting (1) the resp onses in relationship to the content of the p ercep tion; (2) the area of the blot that form s the basis of the resp onse; and (3) the asp ects of the area that are u sed to form the resp onse. In 1961, Lind zey p rop osed a m ethod of classifying p rojective tests based on the typ e of activity. The Rorschach is classified in the category of associations. Another test in this category is the Word Association Test. In the choice of ord ering category, p atients p lace objects in a rank ord er w ith resp ect to p reference. Tests su ch as the Sentence Com p letion Test fall into the category of com p letions, in w hich a p erson com p letes an u ncom p leted stim u lu s. The constru ction category requ ires the su bject to constru ct content based on a stim u lu s, su ch as a story in the Them atic Ap p ercep tion Test. The self-exp ression category consists of tests su ch as the Draw -APerson Test, in w hich the su bject p rod u ces a resp onse w ithou t a stim u lu s.

31. (C) Before any p harm acologic changes are consid ered , the physician m u st assess com p liance w ith m ed ications. The subtherap eu tic p henytoin level m ay be d ue to p atient noncom pliance, especially if sid e effects are being exp erienced .

34. (C) It is not ethical to engage in therapy (i.e., conversion therapy) to change the sexual orientation of a patient. The “American Psychiatric Association opposes any psychiatric treatment, such as reparative or conversion therapy w hich is based u p on the assu m p tion that h om osexu ality p er se is a m en tal d isord er or based u p on th e a p riori assu m p tion that th e p atien t shou ld chan ge his/ h er sexu al h om osexu al orien tation .” Ultim ately, p er APA’s p rin cip les of m ed ical eth ics “a p sych iatrist sh ou ld not be a p arty to any typ e of p olicy that exclu d es, segregates, or d em ean s the d ign ity of any p atient becau se of ethn ic origin , race, sex, creed , age, socioecon om ic statu s, or sexu al orien tation .” All the other ch oices w ou ld be consid ered u nethical.

32. (E) The risk of schizophrenia among first-degree relatives w here one p arent has schizop hrenia

35. (B) Cocaine (stim u lant) intoxication can ap p ear sim ilar to a m anic ep isod e associated

30. (B) Gingival hyperplasia is associated w ith ad m inistration of phenytoin. Other, d oserelated sym p tom s inclu d e nystagm u s, d izziness, slu rred speech, ataxia, m ental confu sion, and d ecreased coord ination. Ebstein anom aly is associated w ith lithiu m therap y, and hep atic failure is associated w ith d ivalproex sod iu m .

Answe rs : 28–38

w ith bip olar I d isord er w ith an increase in energy, eu p horia, grand iosity, p ressu red speech, and im p aired ju d gm ent. While cannabis intoxication can p resent w ith eup horia, p aranoia, and im paired ju d gm ent (in association w ith physical sym ptom s of conju nctival injection, increased ap p etite, d ry m ou th, and tachycard ia), a d ecreased need for sleep, d iap horesis, and rap id speech are not consistent w ith cannabis intoxication. The acu te onset of sym ptom s m akes the d iagnosis of m ajor d ep ression u nlikely. A d iagnosis of schizoaffective d isord er requires that d u ring the p eriod of illness there are sym ptom s that su ggest both a schizophrenia and a m ood d isord er (m ajor d ep ressive or m anic episod e). Schizophrenia requ ires the p resence of significant p sychotic sym ptom s d uring a 1-m onth p eriod , w hich is not evid ent in this case; the p atient exp erienced eu p horia w hich is not characteristic of schizophrenia. N egative sym ptom s (e.g., flattened affect, avolition, alogia) are also m ore com m on in schizophrenia. 36. (D ) The Tem poral Orientation Test asks the patient to id entify the appropriate d ay, m onth, d ay of the w eek, and cu rrent tim e. Deviation from the correct resp onse is d ifferentially scored in each category. Total score and the d ata incorrectly id entified sep arate the p atients into tw o grou ps: p atients w ith brain d am age and patients w ithou t brain d am age. The Tem p oral Orientation Test is also sensitive to cognitive abnorm alities in d em enting illnesses. The Fargo Map Test assesses recent and rem ote sp atial m em ory and visu osp atial orientation by using m aps of the United States and d ifferent regions w ithin it. Patients are asked to id entify certain areas. Ed u cation level and age influ ence the score on this test. The Spatial Orientation Mem ory Test evalu ates the ability to im m ed iately recall the orientation of figu res and is u sed to evalu ate im m ed iate m em ory. The Stroop Test has a nu m ber of d ifferent form ats, but the general concep t is that it takes longer to correctly id entify a color than to read w ord s and longer yet to correctly id entify a w ord (e.g., nam e of a color) w hen that w ord is in a color d ifferent from

231

that w ord . This test seem s to be prim arily an assessm ent of the ability to concentrate. The WCST is u sed to evalu ate “set shifting,” or execu tive fu nctioning of the brain. 37. (A) Culture-bound synd rom es d enote recu rrent, locality-sp ecific p atterns of behavior. The synd rom e of am ok is a cu ltu re-bou nd synd rom e of Malaysian origin that refers to a violent or fu riou s ou tbu rst w ith hom icid al intent. Fou r d efining characteristics are p rod rom al brood ing, a hom icid al ou tbu rst, p ersistence in reckless killing w ithou t an ap p arent m otive, and a claim of am nesia. The attack typ ically resu lts in m u ltip le casu alties and is m ost com m on in you ng m en w hose self-esteem has been inju red . Dhat is a Sou th Asian term referring to anxiety regard ing the loss of sem en. Ganser synd rom e is characterized by a patient w ho resp ond s to qu estions by giving ap p roxim ate or ou tright rid icu lou s answ ers. Ad d itional features m ay also inclu d e altered consciou sness, hallu cinations, conversion p henom enon, and am nesia for the ep isod e. Koro is another Malaysian term that refers to su d d en anxiety that the p enis (or vu lva in fem ales) w ill reced e into the bod y and cau se d eath. Latah is also of Malaysian or Ind onesian origin, m ore frequ ent in m id d le-aged w om en, and is characterized by su d d en fear, often w ith d issociation and catatonic-like featu res (e.g., echolalia, echop raxia). 38. (A) Antip sychotic (neu roleptic)-ind u ced extrapyram id al sid e effects d u e to the blockad e of d opamine are comm on in the treatm ent of psychosis. The higher-potency antipsychotic med ications are more likely to cau se extrapyram id al sid e effects than the low er-potency ones. Extrap yram id al sid e effects includ e acu te d ystonic reactions (as in this case), akathisia (restlessness), pseud oparkinsonism, and tard ive d yskinesia. Acute d ystonia is treated w ith anticholinergic agents such as benztropine and antihistamines. Other op tions inclu d e red ucing the d ose or sw itching to a low er-p otency agent or atypical antipsychotic (such as risperid one). N either of these are preferable at this point as the patient’s sym ptoms have im proved .

232

7: Pra c tic e Te s t 1

39. (E) Becau se of this p atient’s continu ed extrap yram id al sym ptom s d espite treatm ent w ith an anticholinergic, the m ost app rop riate choice w ou ld be to sw itch to an atypical antip sychotic m ed ication. Atypical, or second generation antip sychotics, su ch as qu etiap ine, are believed to act on variou s su bp op u lations of d opam ine receptor su btyp es as w ell as on other neurotransm itter system s, inclu d ing serotonergic. They are consid ered “atyp ical” agents because they are associated w ith a low er risk of extrapyramid al sym ptoms. Clozapine, introd uced to the United States in 1989, is the first antipsychotic to be labeled atypical. Other atyp ical or second -generation m ed ications includ e risperid one, olanzapine, ziprasid one, lu rasid one, asenap ine, and arip ip razole. The other choices are all consid ered to be typical antipsychotics. 40. (C) Bord erline p ersonality d isord er is a p attern of instability in interpersonal relationships, self-im age, affect, and m arked im p u lsivity, inclu d ing inap p rop riate anger and self-m u tilation. Avoid ant personality d isord er rep resents a p ervasive p attern of behavior characterized by social inhibition, feelings of inad equ acy, and hyp ersensitivity to negative evaluation. Bipolar d isord er is an affective illness characterized by alternating p eriod s of m ania and m ajor d epression. Featu res of d epend ent personality d isord er are an excessive need to be taken care of, clinging behavior, and fears of sep aration. N arcissistic p atients d em onstrate a pattern of grand iosity, need for ad m iration, and lack of em pathy. 41. (D ) DBT is a m anu alized treatm ent for chronically parasuicid al patients su ch as those w ith bord erline p ersonality d isord er. It incorp orates elem ents of cognitive, behavioral, and su pportive therapies, and it includ es techniqu es of ad vice, confrontation, and u se of hom ew ork. Aversion therapy is a controversial techniqu e in behavioral therap y w here an aversive stim u li is p aired w ith an u nd esired behavior, hopefully su ppressing the u nw anted behavior. EMDR is a cognitivebehavioral techniqu e w hereby a p atient w ith p osttrau m atic stress d isord er is told to think

abou t a stressful event w hile tracking an object back and forth across their vision. Flood ing is another behavioral intervention u sed for phobias, w here a p atient is exp osed to a feared object d irectly. This is in contrast w ith system atic d esensitization, w here a p hobic p atient is grad u ally exp osed to increasingly feared situ ations u sing a hierarchy of anxiety-p rovoking triggers. 42. (A) The p atient is exp eriencing anticholinergic toxicity as evid enced by d ilated p up ils, d ry or flu shed skin, agitation, confu sion, and tachycard ia. Ad d itional m anifestations includ e d isorientation and u rinary retention. More severe toxicity m ay resu lt in hypertherm ia or com a. In this case, the p atient rep orted benefit from halop erid ol bu t experienced extrap yram id al sid e effects, com m on to the high-p otency antip sychotic m ed ications. Benztrop ine, an anticholinergic m ed ication, is p rescribed to alleviate extrap yram id al sid e effects, bu t in this patient it cau sed anticholinergic sid e effects. Am antad ine is an antiviral m ed ication that is also used to help alleviate extrapyram id al sid e effects and should be institu ted to rep lace benztrop ine. The other choices w ou ld not be appropriate in this case. 43. (C) Objective tests u sually involve questions w ith lists of p ossible resp onses. These tests p rovid e num erical scores on w hich statistical analyses are easily perform ed . Exam ples inclu d e the United States Med ical Licensing Exam inations. The m ain ad vantage to p rojective tests (such as the Rorschach) is that there are a variety of resp onses w ithou t a single correct answ er. The id ea behind projective tests is that w hen presented w ith an am biguou s stim u lu s, patients cannot help bu t reveal som ething about them selves both in how they ad d ress the stim u lu s as w ell as the content of their answ ers. Many of these tests, inclu d ing the Rorschach Test, the Them atic Ap perception Test (TAT), the Sentence Com pletion Test, and the Draw -A-Person Test, requ ire sp ecific training in giving the test and interpreting the resu lts. Projective tests d o not necessarily tell the interview er how the patient feels about him or her.

Answe rs : 39–48

44. (C) The MMPI and MMPI-2 are objective tests that have been u sed for over 50 years in the assessm ent of p ersonality stru cture. They consist of m ore than 500 statem ents, w hich are cond ensed into clinical subscales. Item s on the su bscales w ere d erived em pirically and w ere chosen becau se d ata show ed they cou ld separate p sychiatric p atients from norm al control su bjects. Ad d itionally, qu estions are asked at the tim e of exam ination to evaluate attitu d es w hen taking the test. These qu estions help to p rovid e inform ation abou t the valid ity of the exam ination. Interpretation of the resu lts requ ires an exp erienced evalu ator. Together w ith the clinical history, this test p rovid es valu able inform ation abou t personality stru ctu re. Its accuracy d oes not d epend on the p atient’s gend er, and the results d o not reflect how the exam inee feels about test taking. 45. (B) Denial, a narcissistic d efense, is an (unconsciou s) em otional d efense m echanism used to avoid becom ing aw are of a painfu l aspect of reality. Denial can be used in both norm al and p athologic states. As a d efense m echanism it serves to keep internal or external reality ou t of the consciou s to avert stress and anxiety. Rationalization is a neu rotic d efense m echanism in w hich unaccep table behavior, feelings, or thou ghts are logically ju stified by elaborate and reassuring answ ers. Regression, an im m ature d efense, is an em otional and p hysical retreat from ad u lt stand ard s of behavior tow ard an infantile level of p assivity and d ep end ence. Su p p ression, a m atu re d efense, is a consciou s act of controlling and inhibiting u naccep table im p u lses, em otions, or id eas. 46. (A) The essential featu re of acu te stress d isord er is the d evelop m ent of anxiety and d issociative sym p tom s w ithin 1 m onth of exp osu re to an extrem ely traum atic stressor. Diagnosis shou ld be consid ered if the sym ptom s persist at least 3 d ays and cau se significant d istress or im p airm ent. Du ring or after experiencing the stressor, the ind ivid u al m ust experience at least nine associated sym ptom s (e.g., sleep d istu rbance; absence of em otional resp onsiveness; d issociative sym ptom s; reexp eriencing

233

the event in som e w ay, su ch as nightm ares); anxiety or increased arou sal; avoid ance of the stim u li that arou se recollections of the trau m atic event. The d iagnosis of PTSD requ ires m ore than 1 m onth of sim ilar sym p tom s. Ad ju stm ent d isord er can be consid ered for those ind ivid u als w ho d o not m eet criteria for acu te stress d isord er bu t d evelop sim ilar sym ptom s in excess of w hat is to be expected given the natu re of the stressor. GAD is characterized by excessive anxiety and w orry that occu r for at least 6 m onths. MDD can exist in the context of an acute stress d isord er. H ow ever, the sym p tom s are not su ggestive of a d ep ressive illness as there is no clear d ep ressed m ood , change in ap p etite, energy, or concentration. 47. (B) Psychogenic u nresp onsiveness can be d elineated from com a by obtaining an EEG, w hich is norm al in a p sychogenic state. On the p hysical exam ination of a com atose patient, d eep tend on reflexes m ay be su p p ressed . In p atients w ho are aw ake, cold w ater introd u ced into the ear p rod u ces nystagm u s w ith the fast com p onent aw ay from the ear. In com a, the eyes either d o not react or they m ay slow ly and sm oothly d eviate tow ard the ear in w hich the cold w ater w as introd u ced . Volu ntary eye m ovem ents called saccad es are rap id and sm ooth. Saccad ic eye m ovem ents are elicited in p atients by asking them to stare at an object at one sid e of the visu al field and then ask them to shift their gaze to the op posite visual field . Typically, eye m ovem ents in com a or persistent vegetative states are sp ontaneou s and rand om . Abnorm al vital signs su ch as tem perature and respiration w ou ld not likely be affected in a p sychogenic state. 48. (C) A seasonal pattern can be associated with major d epressive episodes in MDD, bipolar I disorder, and bipolar II disorder. The essential feature is the onset of depressive symptoms at characteristic times of the year, most episod es beginning in the fall or winter and remitting in the spring. It occurs more commonly in women and in higher latitudes. Major depressive episodes that occur in a seasonal pattern are typically characterized by anergia, hypersomnia,

234

7: Pra c tic e Te s t 1

overeating, weight gain, and carbohydrate craving. Age is a strong predictor, with young persons at higher risk. Acute stress disorder is characterized by the development of anxiety, reexperiencing, dissociative, and other symptoms within 1 month of exposure to a traumatic event. Generalized anxiety d isorder patients describe chronic anxiety over a number of events lasting over 6 months. Persistent depressive disorder (dysthymia) is a mood disorder where the depressed mood occurs most of the time for at least 2 years. Sundowning refers to drowsiness, confusion, and falling, typically seen in patients with major neurocognitive disorders (dementias) when lights or other orienting cues are removed. 49. (B) The essential featu res of d issociative am nesia w ith fu gu e are the inability to recall im p ortant autobiographical inform ation, along w ith p urp oseful travel or w and ering associated w ith the am nesia. The d isord er is m ore com m on in w om en and is often associated w ith a history of abu se in child hood . Delirium is characterized by a d istu rbance in attention and aw areness, along w ith a change in cognition that d evelop over a short period of tim e. Major d ep ression is d iagnosed in ind ivid uals w ith 2 w eeks of d epression and neu rovegetative sym p tom s (changes in sleep , ap p etite, energy). Partial com p lex seizu res tend to be brief and d o not last as long as the clinical p icture presents. For the d iagnosis of p osttrau m atic stress d isord er, an ind ivid u al m u st be exp osed to a trau m atic stressor, along w ith sym ptom s of intrusion, alterations in cognition and m ood , d istu rbances of arou sal, and avoid ance of stim u li. 50. (C) The DSM -5 recognizes three sp ecific learning d isord ers: im p airm ent in read ing, w ritten com m u nication, and m athem atics. Learning d isord ers are id entified w hen a p atient’s acad em ic skills in that area are below those exp ected for the ind ivid u al’s chronological age, as confirm ed by ad m inistered stand ard ized achievem ents m easu res and clinical assessm ent. The Wechsler Ad ult Intelligence Scale w as initially p u blished in 1955 and has u nd ergone a series of revisions to reach the

current scale, the WAIS-R. The test is com p osed of 11 d ifferent su btests—6 verbal and 5 p erform ance—that allow for the calculation of the fu ll-scale IQ, p erform ance IQ, and verbal IQ. The WAIS-R has high reliability, w hich m eans that in norm al su bjects retesting d oes not lead to significantly d ifferent evaluations; therefore it can be u sed to follow peop le over tim e. The Draw -A-Person Test is u sed to evalu ate for organic brain d isease, and the MMPI-2 is a test u sed for the assessm ent of p ersonality stru ctu re. The WISC is used to d eterm ine IQ in child ren from ages 6 to 16. The WMT w as d esigned to evalu ate a variety of aspects of m em ory fu nction in ad u lts. 51. (C) The IQ is calculated by d ivid ing the m ental age by the chronological age and mu ltip lying this by 100. 52. (B) While this patient m eets criteria for a m ajor d ep ressive ep isod e, she has a history of sym ptom s consistent w ith a hyp om anic ep isod e (a d istinct p eriod of a p ersistently elevated , exp ansive, or irritable m ood that lasts at least 4 d ays). Bipolar II d isord er d iffers from bipolar I d isord er in that, although the m anic and hyp om anic episod es of these tw o d isord ers have id entical criteria, bipolar II d isord er is not severe enou gh to cau se m arked im p airm ent in social or occu p ational fu nctioning or to cau se hosp italization, and d oes not have p sychotic sym p tom s. Cyclothym ic d isord er consists of chronic, flu ctuating m ood ep isod es of hyp om ania alternating w ith period s of d epression w hich d o not m eet criteria for m ajor d ep ressive episod es. Persistent d epressive d isord er (d ysthym ia) is characterized by chronically d ep ressed m ood m ost of the tim e for 2 years w ithou t hypom anic or m anic ep isod es. 53. (C) The hallmark of Ganser synd rome is that the patient respond s to qu estions by giving ap proximate or rid iculous answ ers. It involves the prod u ction of answ ers to qu estions that are close bu t not qu ite correct (e.g., 4 × 5 = 19). Ganser synd rome is primarily d escribed in m ales in the prison population, most likely related to malingering or factitiou s d isord er w ith

Answe rs : 49–57

p sychological sym ptom s. Capgras synd rom e d escribes a content-specific d elu sion in w hich the patient believes that a significant other, u su ally a fam ily m em ber, has been rep laced by an id entical im p oster. Major neu rocognitive d isord ers (d em entias) are cognitive d isord ers associated w ith m em ory d eficits and p roblem s w ith execu tive fu nctioning. Althou gh p atients w ith d em entia m ay confabu late, this p atient’s age is too you ng, and he has obviou s second ary gain. Mu nchausen synd rom e is a factitious d isord er w ith physical sym p tom s, w here the ind ivid u al feigns an illness in ord er to assu m e the sick role, and is often associated w ith traveling to d ifferent hosp itals. 54. (B) ECT provid es a safe and effective treatm ent of MDD in the eld erly, especially in a p atient w ith a high risk of su icid e or m ed ical contraind ications (e.g., heart d isease) to the use of certain psychotropic m ed ications. It is also ind icated w hen a m ore rap id onset of action (several d ays) is im p ortant (com p ared w ith several w eeks for antid ep ressant m ed ications). Diazepam is a benzod iazep ine w hich is u sefu l in the treatm ent of sym p tom atic anxiety bu t not for d epression. N ortrip tyline, a tricyclic antid ep ressant (TCA), is lethal in overd ose and shou ld not be p rescribed in an acutely su icid al p atient. Certain card iac arrhythm ias are also contraind ications to TCA u se. Risperid one is an antip sychotic m ed ication u sed prim arily for psychotic d isord ers such as schizophrenia and schizoaffective d isord er, as w ell as for bipolar d isord er or m ajor d epression w ith psychotic features. Supportive psychotherapy may be helpful in conjunction w ith med ications or ECT, but it is not indicated as monotherapy for severe depression w ith suicid ality. 55. (B) This p atient is su ffering from a m ajor d ep ressive ep isod e w ith atyp ical featu res. The sp ecifiers of m elancholic featu res and atypical featu res can be ap plied to a cu rrent or recent m ajor d epressive episod e that occurs in the course of MDD, and to a major d ep ressive episod e in bipolar I or bipolar II d isord er. The atypical featu res sp ecifier can also be applied to persistent d epressive d isord er (d ysthymia).

235

The essential features of the atypical specifier are mood reactivity and tw o of the follow ing four features: increased appetite or w eight gain, hypersom nia, lead en paralysis, and interpersonal rejection sensitivity. The essential featu re of a major d epressive episod e w ith melancholic featu res is a loss of interest or p leasure in all or almost all activities or a lack of reactivity to usually pleasurable stimuli, along w ith ad d itional sym ptoms of d epression (e.g., early morning aw akening, d epression w orse in the morning). The patient’s d epressive symp toms are too acute and severe for ad ju stment d isord er, and d on’t last 2 years w hich w ould be necessary for persistent d epressive d isord er (d ysthymia). Although hypersomnia (and insom nia) may be d ue to a sleep d isord er, the patient’s symptom s are better accounted for by a major d epressive d isord er. 56. (C) Id entification is an u nconsciou s d efense m echanism in w hich the p erson incorporates the characteristics and qu alities of another p erson or object into his or her ow n ego system . The d efense serves to strengthen the ego. Fixation refers to an overactive attachm ent to a p erson or object; id ealization is the attribution of near-perfect, u nrealistic attribu tes to that person or object. Projection is the false attribu tion of one’s ow n u nacceptable feelings to another. Regression, an im mature d efense, is an em otional and physical retreat from ad ult stand ard s of behavior tow ard an infantile level of passivity and d ep end ence. 57. (D ) The essential featu re of the catatonia observed in p atients w ith schizop hrenia (as w ell as m ood d isord ers w ith p sychotic featu res) is a m arked p sychom otor d istu rbance involving m otor im m obility, excessive m otor activity, mutism, negativism, peculiar voluntary movements, echolalia (parrot-like senseless repetition of w ord s or phrases), or echopraxia (im itation of movem ents of another person). Catalepsy, a term for an imm obile position that is maintained , is one potential symptom of catatonia, and is not a com p lete answ er. Catap lexy is a su d d en and brief loss of m u scle tone involving either a few m u scle grou p s or m ost of the antigravity m u scles of the bod y;

236

7: Pra c tic e Te s t 1

it is not u ncomm on in patients w ith narcolepsy. H is d iagnosis of schizophrenia, and the fact that the patient occasionally m imics the movements of others, makes seizu re activity unlikely. 58. (B) This patient w ou ld be d iagnosed w ith bip olar d isord er, typ e I, cu rrent ep isod e d ep ressed . The treatm ent of bipolar d epression is tricky. Lam otrigine is likely the m ost effective in treating bip olar d ep ression and as m aintenance treatm ent to prevent relapse of d epression. First-generation antipsychotics like halop erid ol are not effective in treating bip olar d ep ression; how ever, atyp ical antip sychotics (e.g., qu etiap ine) can be beneficial. While lithiu m is first-line treatm ent for acu te m ania and p reventing fu rther m anic ep isod es, it is not as effective in treating the d ep ressed p hase of bipolar d isord er. Monotherapy w ith an antid ep ressant like sertraline shou ld be avoid ed as it can p rovoke a m anic ep isod e in p atients w ith bip olar d isord er. Valp roic acid (Depakote) is another m ood stabilizer prim arily u sed to treat m ania and as m aintenance, bu t it is not as efficaciou s in the treatm ent of d ep ression in this popu lation. 59. (E) Priapism can occu r w ith a num ber of antid ep ressant or antip sychotic agents bu t occurs m ore frequ ently w ith the u se of trazod one (1 in 1,000 to 1 in 10,000 incid ence). The three serotonin-sp ecific reu ptake inhibitors (fluoxetine, p aroxetine, and sertraline) listed can cause d ecreased sex d rive and im potence, bu t are u nlikely to cau se priap ism . N ortrip tyline is a tricyclic antid ep ressant and not associated w ith p riap ism . 60. (D ) While p riap ism u su ally occu rs w ithin the first 4 w eeks of treatm ent, it can occu r at any tim e. It is also ind ep end ent of d ose. Priap ism , esp ecially lasting over several hou rs, is a m ed ical em ergency and m ay requ ire su rgery to p revent p ossible necrosis, scarring, and im p otence. Ejacu lation d oes not alw ays red u ce the erection. Priap ism m ay requ ire d raining blood from the corp u s cavernosu m or intracavernosal injection of an alp haad renergic agonist to p rom ote d etu m escence

and constriction of veins. The offend ing m ed ication shou ld be im m ed iately stop p ed and not restarted , as p rior p riap ism p red isp oses to fu tu re ep isod es. 61. (B) The p atient is m ost likely su ffering from generalized anxiety d isord er (GAD). Treatm ent of GAD inclu d es antid ep ressants su ch as serotonin-sp ecific reup take inhibitors, benzod iazep ines, bu sp irone, and cognitivebehavioral therap ies. H ow ever, benzod iazep ines (su ch as alp razolam , d iazep am , and lorazep am ) shou ld be avoid ed in p atients w ith resp iratory com p rom ise, becau se these m ed ications can exacerbate breathing-related d isord ers, and they carry the risk of ad d iction. Bu sp irone, a nonbenzod iazep ine anxiolytic, is a logical alternative and has been d em onstrated to be effective in the treatm ent of GAD. Beta-blocking agents such as propranolol are effective in certain anxiety d isord ers, such as social anxiety d isord er (social phobia), bu t shou ld not be first-line therap y in p atients w ith chronic obstru ctive p u lm onary d isease or other respiratory d isord ers. 62. (E) The long-term u se of benzod iazepines (lorazepam , oxazepam , and tem azepam ) for sleep is controversial, especially in p atients w ith su bstance u se or breathing-related d isord ers. Zaleplon is not a m em ber of the benzod iazepine class but d oes prod u ce its effects at the gam m a-am inobu tyric acid (GABA)-benzod iazepine recep tor and has a low er risk of p hysical d ep end ence. A nonbenzod iazep ine m ed ication is a logical alternative in cases in w hich resp iratory fu nction is com p rom ised . Dip henhyd ram ine shou ld be avoid ed in the old er population d ue to its anticholinergic effects and p otential d eliriu m . 63. (D ) H yp ertensive crisis is a life-threatening em ergency that m ay resu lt from the com bination of m ep erid ine (Dem erol) and an MAOI. The com bined u se of these m ed ications is absolu tely contraind icated . The treatm ent of hypertensive crisis is w ith IV alp ha-ad renergic blocking agents and ICU ad m ission and supp ort. All of the other m ed ications are safe in com bination w ith MAOIs.

Answe rs : 58–69

64. (C) The use of any m ed ications in pregnancy requ ires exam ining the risks and benefits of treatm ent. The u se of MAOIs is contraind icated in pregnancy partly because they can exacerbate pregnancy-ind u ced hypertension. Althou gh ECT is safe in p regnancy, m aintenance ECT is ind icated only after an initial cou rse of ECT is com pleted . The safety of new er antid epressants has not yet been com p letely established , bu t stu d ies of flu oxetine su ggest that it is relatively safe. In this case, after d iscontinu ing the MAOI, there w ou ld have to be a 14-d ay w ashou t before initiating flu oxetine. Most recom m end ations regard ing the course of treatm ent for a single episod e of MDD are for 9 to 12 m onths of p harm acotherapy. The retu rn of sym ptom s su ggests a recu rrent MDD that requ ires longer-term p harm acotherap y. Of cou rse, ongoing d iscussion w ith and ed ucation of the patient are essential. 65. (B) The patient presents w ith clinical sym p tom s of Wernicke encephalopathy, a m ed ical em ergency, characterized by op hthalm op legia, ataxia, and d eliriu m , cau sed by acu te thiam ine d eficiency. If the thiam ine d eficiency is not corrected , Korsakoff synd rom e can em erge characterized by chronic p ersisting am nesia. Korsakoff synd rom e resu lts from chronic thiam ine d eficiency w ith resu ltant p athology in the m am m illary bod ies and the thalam u s. A history of heavy alcohol use is com m on bu t other cau ses of thiam ine d eficiency su ch as starvation, p rolonged vom iting, and gastric carcinom a can cause this synd rom e. The u se of IV d extrose prior to the ad m inistration of thiam ine m ay aggravate Wernicke encephalop athy. The other interventions m ay be ap prop riate if thiam ine d oes not resolve the sym ptom s. 66. (E) Accord ing to the AMA’s cod e of ethics “A p hysician shall be d ed icated to provid ing com p etent m ed ical care, w ith com p assion and resp ect for hu m an d ignity and rights.” In the event a p hysician is concerned abou t w hether he or she is able to rem ain im p artial and p rovid e ad equ ate p sychiatric treatm ent, the physician shou ld seek a consu ltation w ith

237

an u nbiased colleagu e or su p ervisor to d iscuss the issues and ultim ately d eterm ine w hat w ou ld be in the best interest of the p atient. It m ay be that another physician in the clinic or com m u nity w ou ld better serve the p atient’s need s, but this w ould best be d eterm ined after a consu ltation. To lim it the tim e spent w ith the p atient is not ap prop riate and can constitu te d iscrim ination. While it is im portant to be aw are of the p atient’s p rogress in therap y, the physician m u st first process his or her ow n p artiality to d eterm ine w hether a therap eu tic alliance can be established and m aintained . 67. (B) Caffeine withdrawal occurs after the abrupt cessation or a m arked red uction in the use of caffeinated prod ucts. Stud ies estimate that approximately 50% of caffeine users w ill experience symptoms of w ithdraw al at some point. Caffeine w ithd raw al sym p tom s have their onset 12 to 24 hou rs after the last d ose, and symptoms peak in 24 to 48 hours. Symptoms typ ically rem it in 1 w eek. Althou gh all the p ossible choices are associated w ith caffeine w ithdraw al, up to 50% of sufferers experience head ache, the most common symptom. 68. (E) This p atient is m ost likely su ffering from neu roleptic m alignant synd rom e (N MS). N MS is characterized by m u scu lar rigid ity, confu sion, m u tism , agitation, and elevated au tonom ic signs. Death is rep orted in u p to 20% of cases, u su ally from a failu re to recognize the synd rom e or from d elayed treatm ent. Discontinu ation of the offend ing agent and sup portive treatm ent in an ICU setting is im p erative. IV d antrolene sod iu m and oral brom ocrip tine have been u sed bu t their benefits are u nclear. Cooling measures may reduce hyperthermia. The exact frequency of NMS remains unknow n, w ith some studies suggesting up to 2.4% of all patients treated w ith antipsychotics experiencing this synd rome. NMS may be more frequent in men and younger patients. N aloxone d oes not have a place in the treatment of N MS. 69. (D ) An elevated seru m CPK d em onstrates m uscle inju ry. Since liver transam inases (AST, ALT) are also fou nd in m u scle, these m ay be elevated . Elevated BUN m ay occu r d u e to

238

7: Pra c tic e Te s t 1

d ehyd ration resu lting from N MS but is not as com m on as creatinine or CPK elevation. Leu kocytosis is also seen in N MS. Anem ia is not fou nd in N MS. 70. (C) Physical d epend ence emp hasizes the presence of tolerance or w ithd raw al. N aloxone injection can precipitate opiate w ithd raw al in d epend ent persons. Positive urine toxicology or breath analysis can id entify only su bstance use, not necessarily physiological d ep end ence. Elevated heart rate can be p resent in m any w ithd raw al synd rom es or m ed ical d isord ers and is not sp ecific for any su bstance. Elevated liver fu nction stu d ies can be p resent in ind ivid u als w ho u se heavy am ou nts of alcohol, bu t can also be p resent in other non– su bstance-related d isord ers. 71. (E) Major neu rocognitive d isord ers (d em entias) are characterized by m em ory d eficits and cognitive d istu rbances w ithou t im p aired consciousness. (This d ifferentiates it from d elirium characterized by a w axing and w aning consciou sness.) The m ost com m on typ e of m ajor neu rocognitive d isord er (d em entia) is d ue to Alzheim er d isease (60%). Major vascu lar neu rocognitive d isord er is the second m ost com m on typ e of d em entia, occu rring in 10% to 20% of cases. Other d egenerative p rocesses that cause m ajor neurocognitive d isord ers (d em entias) are Parkinson d isease, H u ntington d isease, and Pick d isease. Infectious, m etabolic, end ocrine, neop lastic, and toxic p rocesses can also cau se neurocognitive d isord ers (d em entias), so a com p lete w orkup for a reversible cau se is essential. This p atient’s history reveals a m egaloblastic anem ia, the cau se of w hich m ay be a vitam in B12 d eficiency, likely second ary to a gastrectom y. Folate d eficiency can also cau se m egaloblastic anem ia. Vitam in d eficiencies su ch as B12 and folate can cause clinical signs of m ajor neu rocognitive d isord ers (d em entias), and they represent one of a few possible reversible cau ses of this p rocess. A com p lete reversible w orku p for neurocognitive d isord ers (d em entias) shou ld also inclu d e thyroid fu nction stu d ies and a rap id p lasm a reagin to test for syp hilis. While an

MRI w ou ld be im portant, it is not necessarily d iagnostic of a p articu lar neu rocognitive d isord er (d em entia). 72. (B) Lithiu m , com m only u sed to treat bip olar d isord er, can be d angerou s in intoxication and can lead to d eath or p erm anent d am age to the nervou s system , p articu larly the cerebellu m . Card iovascu lar and renal m anifestations m ay also be p resent. Lithiu m has a narrow therap eu tic w ind ow w ith a therap eu tic range of 0.6 to 1.2 m Eq/ L. Plasm a concentrations above the therap eu tic range, esp ecially greater than 2.0 m Eq/ L, p recip itate severe CN S and renal impairm ent. Early clinical signs of intoxication includ e d ysarthria, coarse trem or, and ataxia. Im paired consciou sness, fascicu lations or m yoclonu s, seizu res, and com a are om inou s m anifestations. The goal of treatm ent is rem oval of lithiu m from the bod y. Gastric lavage, rehyd ration, and hem od ialysis are possible interventions. Given the d anger of lithiu m intoxication, a seru m d rug level should be obtained before any of the other interventions. 73. (E) A right parietal stroke w ould most likely d emonstrate abnormalities in visual, nonverbal m em ory. 74. (B) The Rey–Osterrieth Test is a com p lex figu re that the p atient is asked to cop y w hile looking at the figu re. The figu re is then taken aw ay and the p atient is asked to d raw the p ictu re from im m ed iate m em ory. The p atient is again asked to d raw the figu re 5 m inu tes and 30 m inu tes after the figu re has been rem oved . The tasks assess visu al nonverbal m em ory. Peop le w ith right p arietal lesions u su ally show abnorm alities in cop ying the figu re correctly by neglecting the item s in the left visu al field . Conversely, a p atient w ith a right tem p oral lobectom y m ay have no d ifficu lty in cop ying the figu re, bu t show m arked abnorm alities in d raw in g th e item from m em ory. Th e Min n esota Mu ltip h asic Person ality In ven tory, Rorschach Test, Wechsler Intelligence Scales, and Wisconsin Card Sorting Test w ou ld not be as u sefu l in assessing a right p arietal lesion.

Answe rs : 70–80

75. (A) Baseline m ed ical tests should be obtained and d ocum ented before starting m any psychotropic med ications, includ ing the mood stabilizers lithium , carbam azepine, and d ivalproex sod ium. As lithiu m has a narrow therapeu tic w ind ow and is renally excreted , assessm ent of kid ney function w ith BUN and creatinine is essential prior to starting lithium. Lithium can cause a benign leukocytosis, but a CBC isn’t necessary prior to initiation. Reversible T-w ave changes on the ECG are also com mon, but an ECG is only necessary in an old er patient or one w ith card iac d isease. Liver function tests are not necessary, but monitoring thyroid function during treatment w ith lithium is im portant. 76. (D ) The serotonin synd rome occurs w hen an SSRI is combined w ith another d rug that can potentiate serotonin, particularly the monoamine oxid ase inhibitors. Symptoms of the synd rome includ e abd ominal pain, d iarrhea, d iaphoresis, hyperpyrexia, tachycard ia, hypertension, myoclonus, irritability, agitation, seizu res, and d eliriu m. Com a, card iovascu lar collapse, and d eath have been reported . In this case, the serotonergic effects of fluoxetine are potentiated by the MAOI. Fluoxetine has a prolonged half-life w ith active metabolites that require up to 6 w eeks clearance time. Treatment w ith the MAOI w as initiated before ad equate w ashout of the fluoxetine and resulted in the serotonin syndrome. Management involves d iscontinuation of the offending agent and supportive care. There is no evid ence that the patient w as not compliant w ith a tyramine-free d iet, w as using hallucinogens, or that she w as taking antipsychotics, w hich w ould be consistent w ith the remaining choices. 77. (B) Bu lim ia nervosa, m ore com m on than anorexia nervosa, consists of rep eated ep isod es of binge eating of large am ou nts of food associated w ith the feeling of being ou t of control. The p atient u su ally p u rges by self-ind u ced vom iting, rep eated laxative or d iu retic u se, or excessive am ou nts of exercise to p revent w eight gain. Unlike ind ivid u als w ith anorexia nervosa, ind ivid u als w ith

239

bu lim ia nervosa u su ally m aintain a norm al or above-norm al bod y w eight. Med ical com p lications of bu lim ia nervosa are u su ally second ary to either chronic vom iting or laxative abu se. They inclu d e flu id and electrolyte im balances (su ch as alkalosis, hyp okalem ia, hyp ochlorem ia, and d ehyd ration); d ental caries and enam el loss; gastrointestinal d istu rbances su ch as esop hagitis, Mallory–Weiss tears, and constip ation; and sore throat. 78. (B) SSRIs, TCAs, MAOIs, and top iram ate have all been show n to be effective in red u cing bingeing and p u rging behavior, even in the absence of a m ood d isord er. Bu p rop ion, w hile an antid ep ressant, shou ld be avoid ed d u e to the increased risk of seizu res from electrolyte abnorm alities. N either antim anic agents, su ch as lithiu m and valp roic acid , nor antip sychotics, su ch as risp erid one, are beneficial in treating bu lim ia nervosa. 79. (B) The Boston Diagn ostic Ap hasia Exam in ation is a com p rehensive set of tests given by a skilled interview er to evalu ate ap hasic d isord ers and to help d efine fu rther interventions to im p rove sp eech. The Bend er Gestalt Test is a constru ctional test for evalu ation of brain d am age and h as som e ability to d ifferentiate the location of th e lesion. The Folstein MMSE is u sed for rap id screening for neu rocogn itive d isord ers (d em en tias) and d eliriu m . The Sentence Com p letion Test is a p rojective test u sed to d escribe p ersonality stru ctu re. The Stroop Test aid s in the evalu ation of concentration. 80. (E) Anorexia nervosa is characterized by a d eliberate self-im p osed starvation in the p u rsu it of being thin, d riven by a fear of being fat. The p atient refu ses to m aintain accep ted norm al bod y w eight appropriate for age and height. Und erd eveloped breasts, lanugo (fine, d ow ny hair), abnorm al insu lin secretion, end ocrine d isord ers, and am enorrhea are also associated w ith this d isord er. Com plications inclu d e osteop orosis, constip ation, cold intolerance, leu kop enia, thyroid d ysfu nction, and card iac arrhythm ias.

240

7: Pra c tic e Te s t 1

81. (C) Anorexia nervosa has a fair p rognosis overall; app roxim ately 50% of p atients have good ou tcom es (inclu d ing significant w eight gain), 25% have interm ed iate ou tcom es, and 25% have poor ou tcom es. 82. (B) Bereavem ent is consid ered to be a norm al response to the d eath of a loved one, and it is not a m ental illness. It is typ ical for the bereaved to have intense feelings of em p tiness that w ax and w ane. It is also not u nu su al to believe they hear the voice of the d eceased , although these experiences are brief and insight is m aintained . If this p atient m et criteria for m ajor d epression, then she w ou ld be d iagnosed as su ch; how ever, in this case she d oes not m eet the fu ll criteria (e.g., ap p etite change, poor concentration, anhed onia). Up to 30% of bereaved ind ivid u als d evelop a m ajor d ep ressive ep isod e, so it is im p ortant to m onitor sym ptom s closely. Ad ju stm ent d isord er cannot be d iagnosed if the ind ivid u al su ffers from norm al bereavem ent. Persistent d ep ressive d isord er (d ysthym ia) requ ires 2 years of sym ptom s m arked by a d epressed m ood . There is no evid ence of a p rim ary p sychotic d isord er su ch as schizoaffective d isord er, nor have the fu ll sym p tom s been present for the ap prop riate am ou nt of tim e. In ad d ition, her age of onset w ou ld be extrem ely u nu su al for su ch a d iagnosis. 83. (C) Most bereaved d o fine w ithou t treatm ent. Sup port and reassurance u sually com e from fam ily, friend s, or clergy. When the bereaved feel u nable to m ove beyond the grief, p sychotherapeu tic m easu res are appropriate. The treatm ent of anxiety or insom nia w ith m ed ications rem ains controversial, bu t m ed ications m ay be w arranted if cognitive or other higher fu nctions of d aily living are su bstantially affected . 84. (A) The treatm ent of p atients w ith obsessivecom pu lsive d isord er (OCD) requ ires integrating m u ltip le treatm ent m od alities inclu d ing m ed ications, p sychotherapy, and collateral su pp ort system s. One of the first m ed ications show n to be effective for OCD w as clom ipram ine, a TCA, althou gh the sid e effects and

risk of overd ose m ake this a second -line m ed ication. SSRIs are also very effective in treating sym p tom s of OCD, bu t u su ally at d oses higher than those used to treat d ep ression. Antip sychotic agents are u sed in conju nction w ith antid ep ressants in very severe, intractable cases, bu t are not u sually ind icated as ind ivid u al agents. Antiep ilep tics and lithiu m are not ind icated for the treatm ent of OCD. Anxiolytic agents su ch as benzod iazep ines are of little u se in treating the obsessions or com pu lsions of OCD. 85. (C) The concern in this case is that the p atient is lying abou t her m em ory p roblem , therefore m alingering. When given tests of m em ory, m alingerers tend to not put forth genuine effort. The Test of Mem ory Malingering can help assess this effort. It consists of a subject being p resented 50 p ictures; they are then p resented card s and asked if they had seen a given card before. Perform ance below the od d s of chance ind icates high likelihood of intentionally changing a correct answ er to an incorrect one in ord er to falsify resu lts. The Benton Visu al Retention Test m easu res visu al perception and m em ory, and is u sed to help id entify learning d isabilities. The Clock Draw ing is u sed as p art of a screen for cognitive im pairm ent and spatial d ysfu nction in neu rocognitive d isord ers, such as d em entias. The Wechsler Intelligence Scale is an IQ test for late ad olescents and ad u lts, w hile the Wisconsin Card Sorting Test is a neuropsychological test for set-shifting, d etecting frontal lobe d am age from a variety of cau ses. 86. (C) Obstructive sleep apnea hypopnea is d iagnosed w ith polysomnographic evid ence of either: (1) Five obstructive apneas or hypopneas per hour w ith snoring/ gasping or d ay times sleepiness/ fatigue or (2) 15 or more obstructive apneas or hypopneas per hour regard less of associated symptoms. It is increased in males, old er ad ults, and the obese. Circad ian rhythm sleep–w ake d isord ers are characterized by patterns of sleep d isruption d ue to alterations of the circad ian rhythm system (e.g., shift w ork). There is no other evidence for major depression in this patient. The criteria for REM sleep

Answe rs : 81–91

behavior disorder include repeated episodes of arousal during sleep w ith vocalization and/ or motor behaviors occurring only d uring REM sleep. Patients w ith restless legs synd rome have an urge to move the legs accompanied by feelings of uncomfortableness. The urges are w orsened at rest and at night, and relieved w ith movement. 87. (C) N asal continu ou s p ositive airw ay p ressu re (CPAP) is the treatm ent of choice for obstructive sleep apnea. N o m ed ications are consistently effective in norm alizing sleep in these p atients. The use of nasal CPAP ap p ears to p revent long-term m orbid ity and m ortality and provid es restorative sleep . Su rgical p roced ures su ch as those listed are reserved for CPAP treatm ent failures. 88. (A) Anticholinergic toxicity can be cau sed by many psychotropic medications. The low p otency agen ts chlorp rom azine and thiorid azine and the TCAs am itrip tyline an d im ip ram ine are p articu larly anticholinergic. The synd rom e often resu lts w hen an anticholinergic m ed ication, su ch as benztrop ine, is coad m inistered w ith an antipsychotic m ed ication in ord er to prevent or treat extrap yram id al sid e effects su ch as p arkinsonism and d ystonia. Clinical features of anticholinergic toxicity includ e d ecreased secretions, agitation, d ry skin, flu shing of the skin, hyp ertherm ia, tachycard ia, d ilated p upils, urinary retention, constip ation, and hyp otension. Seizu res, hallu cinations, and com a m ay resu lt from severe intoxication. Drugs that inhibit the enzym e that breaks d ow n acetylcholine (anticholinesterase m ed ications such as physostigm ine) reverse the synd rom e and are u su ally ad m inistered IV w ith ap p rop riate m onitoring. Atrop ine w ou ld fu rther increase the heart rate; benzod iazep ines are reserved for u se in agitation and anxiety, and haloperid ol m ay w orsen anticholinergic sym ptom s. 89. (A) Anticholinesterase toxicity, m anifesting as nausea, vom iting, brad ycard ia, and seizu res, resu lts from excessive action of acetylcholine at specific receptors. Atropine, an antim uscarinic agent, can reverse the symptoms. Epinephrine

241

w ould treat the brad ycard ia w ithout treating the und erlying cond ition. Physostigm ine w ould w orsen the toxicity. Although prochlorperazine w ould com bat vomiting, it w ou ld not treat the u nd erlying cond ition and also might w orsen the anticholinergic toxicity. 90. (D ) This w om an appears to suffer from both a major d epressive d isord er (MDD), single episod e, as w ell as obsessive-comp ulsive d isord er (OCD). Serotonin-specific reuptake inhibitors (SSRIs) are used to treat a variety of psychiatric d isord ers, includ ing both MDD and OCD. Fluvoxamine is an SSRI p articu larly ind icated for both d epression and OCD. Buspirone is a nonbenzod iazepine m ed ication used to treat generalized anxiety d isord er but is not efficacious for OCD. Clomipramine is a tricyclic antid epressant (TCA) w ith significant serotonergic effects and is particu larly u sefu l in OCD; d oxepin is also a TCA, bu t given the tolerability and sid e effect profile and safety in overd ose, SSRIs are preferred as first-line therapy for MDD and OCD. Phenelzine is a m onoam ine oxid ase inhibitor (MAOI) w hich can be u sed for d epression. H ow ever, the d rug–d rug interactions and d ietary restrictions make MAOIs a second - or third -line therapy. 91. (D ) The p atient is beginning to have sym p tom s of alcohol w ithd raw al. Because they act at the sam e GABA receptors as alcohol, the benzod iazepines are the preferred m ed ications used in d etoxification from alcohol. For u ncom p licated d etoxification, the long-acting benzod iazepines su ch as chlord iazepoxid e and d iazepam are appropriate because they are essentially self-tap ering. The shorter-acting benzod iazep ines su ch as lorazep am or oxazep am are u sed in p atients w ith im p aired liver fu nction, such as in this case, becau se they are not d ep end ent on liver fu nction for their m etabolism and d o not accu m u late. Patients w ith u nstable m ed ical p roblem s, cognitive im p airm ent, or old age are also cand id ates for the shorter-acting m ed ications. Disu lfiram , w hich interferes w ith the m etabolism of alcohol, is reserved for long-term behavioral therapy of alcohol u se d isord er. Phenobarbital w as form erly u sed for d etoxification and

242

7: Pra c tic e Te s t 1

has been replaced by benzod iazepines d u e to safety concerns and the risk of respiratory d ep ression. 92. (D ) The patient is experiencing hallucinations likely due to alcohol w ithd raw al. Benzod iazepines w ill treat the underlying w ithdraw al as w ell as the hallucinations. The patient show s no evid ence of either a d epressive d isord er that should be treated w ith sertraline or fluoxetine (SSRIs), or a bipolar disord er that w ould benefit from lithium, a mood stabilizer. An antipsychotic, such as haloperid ol, is unnecessary and may also red uce the seizure threshold . 93. (E) Ethanol is metabolized in the liver via alcohol d ehyd rogenase to acetald ehyd e, w hich is then enzymatically converted to acetate by the enzyme acetaldehyde dehydrogenase. Disulfiram is a med ication that irreversibly inhibits the action of ald ehyd e d ehyd rogenase. Acetald ehyde accumulates and accounts for the aversive effects associated w ith the disulfiram reaction. Flushing, sw eating, d yspnea, hyperventilation, tachycard ia, hypotension, nausea, and vomiting are common symptoms. Extreme but rare reactions can result in respiratory d epression, card iovascular collapse, myocard ial infarction, seizures, and d eath. Blurred vision an d u rinary retention are an ticholin ergic effects associated w ith an tip sych otic, antid epressant, and antip arkinsonian m ed ications. Eup horia w ou ld not be exp erienced w hen alcohol is com bined w ith d isu lfiram . H ypertension m ay be associated w ith, bu t is not characteristic of, the d isu lfiram -ethanol reaction. Disu lfiram is not recom m end ed for p atients w ith m od erate to severe liver d isease, renal failu re, severe card iac d isease, p regnancy, or peripheral neu ropathy. 94. (D ) Many nonsteroid al anti-inflam m atory d rugs can d ecrease the clearance of lithium and p rod u ce significant increases in serum levels. Ibu p rofen, ind om ethacin, ketop rofen, d iclofenac, nap roxen, and p iroxicam have all been rep orted to p rod u ce su ch p otential d angerou s interactions. The sym ptom s of abd om inal p ain, d iarrhea, and d row siness ind icate m ild to m od erate (1.5–2.0 m Eq/ L) lithiu m toxicity.

Acetaminophen, aspirin, and the opiates do not interfere w ith lithium clearance. Meperid ine, how ever, can cause d angerous interactions w ith MAOIs. 95. (D ) Given the p atient’s p rior m ajor d ep ressive ep isod e and cu rrent sym p tom s, treatm ent w ith antid ep ressants is w arranted . H ow ever, the patient is concerned abou t sexual d ysfunction, a very com m on sid e effect (up to 80% in som e stu d ies) of serotonin-sp ecific reup take inhibitors su ch as citalop ram . Mirtazap ine is an antid ep ressant w hich has the effect of increasing both ad renergic and serotonergic neu rotransm ission, thu s im p roving d epression, bu t w ithout cau sing appreciable sexual d ysfunction. All the other m ed ication choices w ou ld likely equ ally cau se p roblem s w ith sexual functioning. 96. (D ) While the u su al antid ep ressants (e.g., TCAs, SSRIs) are all effective in treating d ep ression in a patient w ith H IV, the rapid onset of action of stim ulants like m ethylphenid ate w ou ld be p articu larly beneficial, esp ecially in a m ed ically com prom ised p atient w ho is at risk of d ehyd ration and starvation. Bu p rop ion, flu oxetine, and sertraline m ay take u p to 4 to 6 w eeks for fu ll benefit. Bu sp irone is a nonbenzod iazep ine anxiolytic m ed ication not ind icated in the treatm ent of d ep ression as m onotherap y. 97. (A) Attention refers to the ability to give focu s to a cognitive task. Perform ing “serial sevens” (as in this case) and spelling “w orld ” backw ard are tests of attention. Althou gh a certain facility w ith the rem aining choices is necessary to perform each task (no cognitive function is tested in absolu te isolation), the serial sevens test p rovid es a w ind ow on a p atient’s concentration. Fu nd of know led ge inclu d es inform ation the p atient read ily has available to him or her; know led ge of cu rrent events is often u sed to assess this. The m ental statu s exam ination m ay contain tests of m athem atics skills, bu t testing m athem atics skills is not the p u rp ose of the serial sevens test. Recent m em ory is recall of events occu rring in the last several m inu tes. Rem ote m em ory

Answe rs : 92–103

involves the recall of events long p ast, for exam p le, inform ation from a p atient’s child hood . Any test of cognitive fu nction m u st take into accou nt the p atient’s cu ltu ral, ed u cational, and social backgrou nd . 98. (B) Inhalant use is commonly a social activity und ertaken in groups. Volatile or toxic solvents such as toluene (found in glues and ad hesives), trichloroethane (found in correction fluid ), or hyd rocarbons (gasoline) are placed in a bag and the fumes are inhaled . The practice is know n as huffing, sniffing, or bagging. Effects are almost immed iate because the substances are rapid ly absorbed . Signs and symptoms of intoxication includ e visual d isturbances, d yscoord ination (a d runken appearance), d epressed reflexes, euphoria, and nystagmus. Conjunctival injection and increased appetite are consistent w ith cannabis use. A d iminished response to pain, euphoria, and staring into space are characteristic of PCP intoxication. 99. (A) Abstinence is the only treatm ent for inhalant u se d isord er. Antid ep ressants have not show n utility in inhalant u se d isord er. Antip sychotic agents are also not effective and m ay aggravate the p resentations of inhalant intoxication. Agitation can be m anaged by benzod iazep ine m ed ications su ch as lorazep am . Dialectical-behavioral therap y is a form of cognitive-behavioral therap y used in bord erline personality d isord er. Exposu re and resp onse p revention is a cognitive-behavioral therapy u sed in the treatm ent of anxiety d isord ers su ch as obsessive-com pu lsive d isord er. 100. (B) This p atient su ffers from treatm ent refractory schizophrenia. H e has had fou r m ed ication trials, includ ing both typical and atypical antip sychotics. Given his ad equ ate trials, prior com pliance, and ongoing sym ptom s, he is an ap propriate cand id ate for clozap ine, the first atypical antip sychotic and the only one that d em onstrates su p erior efficacy in treatm entrefractory sch izop hren ia. While taking it requ ires frequ ent m onitoring of his com p lete blood cou nts, that fact that he lives w ith his m other w ill increase the likelihood of his com p liance. N either aripiprazole and zip rasid one,

243

atyp ical or second -generation antip sychotics, nor flu p henazine, a typ ical antip sychotic, w ou ld be as effective as clozap ine in this case. Given his ongoing sym p tom s, restarting qu etiap ine w ou ld not be ap p rop riate in this p atient. 101. (C) Divalp roex sod iu m , ind icated for the treatm ent of seizu res as w ell as bip olar d isord er, can occasionally cau se significant hair loss that is usu ally reversible. Other com m on sid e effects associated w ith d ivalp roex sod iu m are nausea, vom iting, and ind igestion. Sed ation and m ild ly elevated seru m transam inase levels m ay also be seen. Rare occu rrences of hep atic failu re resu lting in fatalities have hap p ened , u sually d u ring the first 6 m onths of treatm ent. H air loss is not associated w ith carbam azep ine, clozap ine, olanzap ine, or zip rasid one, althou gh these m ed ications have their ow n sid e effects. 102. (A) Sexual d ysfu nction is a very com m on sid e effect associated w ith the serotonin-sp ecific reu p take inhibitors (SSRIs), su ch as citalop ram , w ith an incid ence of ap proxim ately 50% to 80%. Decreased libid o and d elayed orgasm are the m ost com m on sym p tom s, bu t erectile d ysfunction is also not uncom m on. All SSRIs ap p ear to be equally as likely to cau se sexu al d ysfunction. Priapism is a rare bu t seriou s sid e effect of trazod one. Retrograd e ejacu lation is a know n sid e effect of certain antipsychotic m ed ications. Sexually transm itted d iseases are not increased w ith SSRI (or any antid ep ressant) use. Variou s p harm acologic strategies aim ed at treating sexu al d ysfu nction inclu d e bu p rop ion and p hosp hod iesterase-5 inhibitors, and less commonly methylphenid ate and bu sp irone. 103. (A) ADH D is a d isord er of u nclear etiology. As m any as 5% of child ren su ffer from this d isord er, and ap proxim ately half of those d iagnosed carry the d iagnosis into ad u lthood . The d isord er is characterized by sym p tom s of inattention, hyperactivity, and im p ulsivity that are inappropriate for age. Although the DSM -5 criteria require that sym p tom s m ust have been present before the age of 12 years,

244

7: Pra c tic e Te s t 1

m any cases are not d iagnosed u ntil ad olescence or ad u lthood . In these cases, a retrospective history from both the patient and the fam ily often su pport the presence of early life sym p tom s. Althou gh som e sym p tom s are rem iniscent of the m anic com p onent of bip olar d isord er, these d isord ers rep resent tw o separate entities. A carefu l history by astu te p hysicians w ill secu re the p rop er d iagnosis. Dep ressive, anxiety, and su bstance-u se d isord ers are com m on com orbid cond itions that often m ake the d iagnosis of ADH D m ore d ifficult. The absence of prom inent m ood sym ptom s and the relative d egree of im pairm ent help d istingu ish ADH D from other d isord ers. 104. (D ) The app roach to the treatm ent of the core sym ptom s of ADH D has been the initiation of a trial of a p sychostim u lant m ed ication, su ch as m ethylp henid ate. Often, if one stim u lant fails to treat sym p tom s, another stim u lant is p rescribed . Many patients benefit from one stim ulant and not another. Given his partial response and tolerability, the appropriate choice in this case w ould be to try a second type of stimulant before the alternative strategies are im plem ented . 105. (A) Déjà entend u is the feeling that one is hearing som ething one has heard before. It is u su ally associated w ith anxiety states or fatigu e. Déjà vu is a sim ilar exp erience, bu t refers to the sensation that som ething has been seen before. Jam ais vu is the op p osite of d éjà vu in that it refers to som ething that shou ld be fam iliar bu t seem s qu ite u nfam iliar. Folie à d eu x is a shared d elu sion arou sed in one person by the influ ence of another. La belle ind ifférence is the ind ifference show n tow ard a d eficit or loss of fu nction, classically (bu t not alw ays) seen in a conversion (fu nctional neu rologic sym ptom ) d isord er. 106. (E) The ep id em iology and p rognosis of schizop hreniform d isord er are p oorly u nd erstood , althou gh the lifetim e p revalence rate is ap proxim ately 0.2% and app roxim ately tw o-third s eventu ally go on to d evelop schizop hrenia or schizoaffective d isord er. The d iagnostic criteria are sim ilar to those of schizophrenia,

but the episod e lasts betw een 1 and 6 months. Distinguishing schizophrenia from mood disord ers w ith psychotic symptoms (e.g., bipolar I d isorder w ith psychotic features, MDD w ith psychotic features) and schizoaffective d isord er can often be d ifficult in that a mood disturbance is not uncommon d uring episod es of schizophrenia and schizophreniform d isord ers. If psychotic symptoms only occur w ith the presence of mood symptoms, then MDD w ith psychotic features is more likely. Brief psychotic d isord er is d efined by the presence of d elusions, hallucinations, d isorganized speech or behavior, or catatonic behavior lasting for at least 1 day but less than 1 month. Schizoaffective d isord er d escribes the presence of a major d epressive or manic episod e concurrent w ith symptoms that are characteristic of schizophrenia: d elusions, hallucinations, d isorganized speech or behavior, or negative symptoms. 107. (B) In the context of p regnancy and labor, high-p otency typ ical antip sychotics such as halop erid ol are consid ered relatively safe to u se to control psychosis. For a m other su ffering from p sychosis d u ring d elivery, the benefit of u sing high-p otency antip sychotics ou tw eighs the risk. All of the other antipsychotics are low - or m ed ium -potency and m ay low er blood p ressu re significantly d u e to their alpha-blocking p rop erties. 108–111. [108 (E), 109 (G), 110 (K), 111 (D )] The MMSE is a qu ick, easily ad m inistered test that allow s for im m ed iate screening for neu rocognitive d isord ers (e.g., d em entias). Scores of less than 24 are su ggestive of a d em enting p rocess. The Rey–Osterrieth figu re is sensitive to d eficits in copying and lack of attention to d etail in people w ith right-sid ed parietal lobe lesions. It ap p ears that this you ng w om an m ay have had a stroke in this area resu lting from her protein S d eficiency, a hypercoagu lable state. The ap p rop riate test to evalu ate the intelligent qu otient (IQ) for child ren ages 5 to 15 is the WISC. The Boston Diagnostic Ap hasia Exam ination is a series of tests given by an exp erienced clinician to evalu ate and m ake treatm ent recom m end ations for ind ivid u als w ith ap hasia. The Beck Dep ression Inventory

Answe rs : 104–116

(A) is a 21-item test, w ith three responses p er item , that is an easily u sed screening tool to evalu ate for d ep ression. The Bend er Gestalt Test (B) involves copying figu res, w hich helps assess neurological function or brain d isease. The Blessed Rating Scale (C) is a tool that asks friend s or fam ilies of the patient to assess the ability of the p atient to fu nction in his u su al environm ent. MMPI-2 (F) is an objective test interp reted by skilled evalu ators u sed in p ersonality assessm ent. It is the m ost w id ely u sed and highly stand ard ized test of p ersonality stru ctu re. The Rorschach Test (H) is a p rojective test u sed to assess p ersonality stru ctu re. The Wad a Test (I) is u sed to evalu ate hem ispheric langu age d om inance prior to surgical am elioration of seizu re focu s. Whereas m ost right-hand ed ind ivid u als show left hem ispheric d om inance for langu age, left-hand ed ind ivid u als m ay either be right or left d om inant. The test consists of injecting sod iu m am ytal into the carotid artery and observing the transient effects on sp eech. Injection into the left carotid artery w ill anesthetize the left sid e of the brain; those w ith left hem ispheric langu age d om inance w ill show interru p ted speech. The WAIS-R (J) is u sed to d eterm ine the IQ for ind ivid u als age 15 and old er. 112. (C) Cannabis (m ariju ana) is the m ost com m only abu sed illicit d ru g in the United States. The onset of action after sm oking is im m ed iate, and sym p tom s inclu d e conju nctival injection, m ild sed ation, d ose-d ep end ent hyp otherm ia, d ry m ou th, increased ap p etite, tachycard ia, and eu p horia. A sensation of slow ed tim e and p aranoid id eation can also occu r.

245

113. (E) The onset of action for opiates can be almost immediate when smoked , about 5 minutes w hen injected , an d lon ger if taken orally. Sym p tom s inclu d e m iosis, brad ycard ia, hyp otension, hypotherm ia, constipation, and eu phoria. Intoxication can lead to fatal respiratory d ep ression. 114. (F) PCP is a hallu cinogen that p rod u ces a d issociative anesthesia. It can cause u npred ictable beh avior, assau ltiveness, an d belligeren ce. Agitation, nystagm u s (vertical or horizontal), tachycard ia, a num bed resp onse to pain, m u scle rigid ity, hyperacu sis (increased sensitivity to sou nd ), hypertension, echolalia, and anticholinergic effects are associated sym p tom s. 115. (D ) N icotine fou nd in tobacco sm oke p rod u ces sym p tom s of excitem ent. Intoxication w ith large am ou nts m ay p rod u ce sym p tom s of confu sion, m u scle tw itching, w eakness, abd om inal cram p s, d epression, p alp itations, com a, and respiratory failu re. 116. (A) As a sym p athom im etic agent, cocaine increases the heart rate and d ilates the p u p ils. It can also cau se constriction of the coronary arteries. The hallm ark of cocaine intoxication is eu p horia. Use of LSD (B) or other hallu cinogens (e.g., MDMA [3,4-m ethylened ioxyN -m ethylam p hetam ine], or “Ecstasy”) cau ses sym p tom s that u su ally begin 1 hou r after ingestion and generally last 8 to 12 hou rs. Most hallucinogenic d rugs have stim u lanttype effects and prod u ce elevated vital signs and increased activity. H allu cinations and a heightened sense of percep tion of objects and colors are typical.

This page intentionally left blank

CHAPTER 8

Prac tic e Te s t 2 Que s tions

D IRECTION S (Questions 1 through 4): For each of the follow ing vignettes, select the one lettered option that is most closely associated w ith it. Each lettered option may be used once, multiple times, or not at all. (A) (B) (C) (D) (E) (F)

Cit lo r Flu oxetine Mirt z ine P roxetine Sertr line Venl f xine

1. A 29-ye r-ol new ly rrie le w ith signific nt e ressive sy to s, inclu ing inso ni n w eight loss, w s nonco li nt w ith e ic tions in the st u e to sexu l ysfunction. 2. A 45-ye r-ol n w ith severe jor e ression on h igh ose of n n ti e ress nt evelo s incre se bloo ressu re. 3. A 50-ye r-ol ivorce fe le being tre te for jor e ression oesn’t refill her rescri tion n 2 ys l ter h s he che, u scle ches, n n use . 4. A 33-ye r-ol w o n w ith ersistent e ressive isor er ( ysthy i ) h s history of oor co li n ce w ith e ic tion s, occ sion lly issing oses. D IRECTION S (Questions 5 through 41): For each of the multiple-choice questions in this section select

the lettered answ er that is the one best response in each case 5. An 18-ye r-ol w o n recently i gnose w ith first e iso e of schizo hreni grees to t ke n nti sychotic e ic tion to hel ecre se her h llu cin tions, bu t she is ntly o ose to t king ny e ic tion th t y c u se her to g in excessive w eight. Which of the follow ing e ic tions w ou l be the ost ro ri te to rescribe? (A) (B) (C) (D) (E)

Cloz ine (Cloz ril) Ol nz ine (Zy rex ) Qu eti ine (Seroqu el) Ris eri one (Ris er l) Zi r si one (Geo on)

6. A 40-ye r-ol rrie fe le w ithout st sychi tric history is referre by her internist. She h s been feeling “ ow n” for sever l w eeks, w ith little bility to enjoy herself. Since th t ti e, her slee h s been isru te , her etite re uce , n she’s felt “tire .” She h s lso been king ist kes t w ork n is w orrie bou t being fire . She its to h ving thou ghts of killing herself, bu t she enies ny l n or intent to o so. She enies h llu cin tions or elu sions. Which of the follow ing e ic tions w ou l be the ost ro ri te for this tient? (A) (B) (C) (D) (E)

Ari i r zole Cit lo r Methyl heni te N ortri tyline Tr nylcy ro ine 247

248

8: Pra c tic e Te s t 2

7. A 25-ye r-ol w o n resents to her ri ry c re octor co l ining of su en onset of intense fe r n feeling th t she w s going to ie, w hich occurre w hile she w s sto e in tr ffic e rlier in the w eek. At th t ti e she bec e short of bre th, i horetic, n tre u lous, n she cou l feel her he rt oun ing. H er initi l i u lse w s to rive to n e ergency e rt ent, bu t her istress su bsi e on its ow n in bou t 20 inu tes. Which of the follow ing shou l be the next ost ro ri te cou rse of ction? (A) Con u ct thorou gh e ic l screening for he rt n lu ng ise se. (B) Prescribe selective-serotonin reu t ke inhibitor (SSRI) n sk to see the tient b ck in 1 w eek. (C) Prescribe short- cting benzo i ze ine s nee e in c se of nother e iso e. (D) Re ssu re the tient th t her sy to s re ost likely benign n not c u se for concern. (E) Refer the tient to sychi trist. 8. You re seeing 45-ye r-ol n w ith history of schizo hreni . H e is cu rrently co li nt w ith his nti sychotic e ic tions bu t re ins sy to tic, w ith u itory h llu cin tions n r noi . On ent l st tu s ex in tion (MSE), he s e ks coherently n rticul tely, bu t kes no sense bec u se ny of the w or s he uses re of his ow n invention. Which of the follow ing ter s best escribes this MSE sign? (A) (B) (C) (D) (E)

Cl ng ssoci tions Echol li Flight of i e s N eologis s Wor s l

9. A 34-ye r-ol w o n resents to sychi trist w ith 6-ye r history of e tiness n e ression for “ ost of the ti e,” w ith low self-estee , chronic inso ni , n ifficulty king ecisions. She enies ny roble s w ith her etite or energy, n , w hile she

oes not feel like her con ition w ill i rove, she enies ny su ici l i e tion. She rinks one to tw o rinks once or tw ice er onth, bu t enies illicit ru gs. She is su bsequ ently begu n on flu oxetine, w hich is gr u lly incre se to 60 g. She enies si e effects, lthou gh she h s not ex erience signific nt lessening of her e ressive sy to s. H ow long shou l her cu rrent os ge be continu e before sw itching to nother e ic tion? (A) (B) (C) (D) (E)

3 ys 1 w eek 2 w eeks 4 w eeks 6 w eeks

10. A 56-ye r-ol w o n w ith schizo hreni h s been st ble on h lo eri ol for ny ye rs. She enies ny current sychotic sy to s n h s been co li nt w ith her e ic tions. She resents for rou tine follow -u ointent. Which of the follow ing tests shou l be inistere ? (A) Abnor l Involu nt ry Move ent Sc le (AIMS) (B) Beck De ression Inventory (BDI) (C) Brief Psychi tric R ting Sc le (BPRS) (D) P tient H e lth Qu estionn ire (PH Q) (E) Positive n N eg tive Sy to Sc le (PAN SS) 11. A n brings his w ife to the octor bec u se she h s beco e ore sy to tic over the st sever l w eeks. She is i gnose w ith schizo hreni n h s recently sto e her e ic tions. She h s e thre ts tow r her hu sb n bec u se she believes th t he is n i ostor w ho looks ex ctly like her hu sb n . Which of the follow ing ter s best escribes this sy to ? (A) (B) (C) (D) (E)

A ok C gr s syn ro e Cot r syn ro e Cou v e syn ro e Koro

Que s tions : 7–17

12. A 69-ye r-ol w o n w ithout rior sychi tric history is seen by her ri ry c re hysici n ue to “crying s ells.” She st tes th t she h s felt s since the su en e th of her husb n 5 w eeks go. Since th t ti e, she h s h ifficulty slee ing n f tigue. She h s lso lost sever l oun s ue to not enjoying her foo . She is ble to cheer u w hen s en ing ti e w ith her chil ren, but feels es eci lly lonely t night. She is so ew h t nxious bec use she occ sion lly he rs her ece se husb n ’s voice, es eci lly e rly in the orning or l te in the evening. While she feels her future is “ble k,” she enies ny suici l i e tion or l n. Which of the follow ing i gnoses is ost likely? (A) (B) (C) (D)

Bere ve ent Bi ol r isor er M jor e ressive isor er (MDD) Persistent e ressive isor er ( ysthy i ) (E) Schizo ffective isor er

Questions 13 and 14 You re ske to ev lu te 12-ye r-ol boy with history of Tourette isor er n obsessive-co ulsive isor er. He is brought in to the oint ent with his rents, who escribe the severity of his illness n how he is often te se in school. During the interview the boy inter ittently kes obscene gestures. 13. Which of the follow ing ter s best this sy to ? (A) (B) (C) (D) (E)

escribes

Ble h ros s Bru xis Co ro r xi Echo r xi Torticollis

14. While obt ining the history, the tient is lso note to re e t hr ses you u se i e i tely fter you s y the . Which of the follow ing ter s best escribes this sy to ? (A) Co rol li (B) Dys rthri

249

(C) Echol li (D) P lil li (E) P r r xis Questions 15 and 16 A w e lthy, ivorce 48-ye r-ol w o n resents to you fter being rreste for sho lifting. The tient its th t she h s been ste ling for ye rs, lthou gh she is e sily ble to ffor the objects stolen. She st tes th t she ste ls “on the s u r of the o ent” n th t these i u lses see foreign n istressing. 15. Which of the follow ing i gnoses is the likely? (A) (B) (C) (D) (E)

ost

Antisoci l erson lity isor er Inter ittent ex losive isor er Kle to ni Pyro ni Trichotillo ni

16. Which of the follow ing ter s lies to the f ct th t these i u lses re istressing to her? (A) (B) (C) (D) (E)

Delu sion l Ego- ystonic Ego-syntonic Moo congruent Moo incongru ent

17. A 49-ye r-ol n co es to you co l ining of he ches, e ory loss, isorient tion , n occ sion l r lysis th t ffects his r s n l sts sever l hou rs. Du ring th e MSE, you notice th t th e tient is giving v gu e nsw ers to ny qu estions (e.g., th ere re six toes on th e foot n 2 + 2 = 5). Which of th e follow in g ter s best escribes th is resen t tion? (A) (B) (C) (D) (E)

A ok Dere liz tion G nser syn ro e N eg tivis Piblokto

250

8: Pra c tic e Te s t 2

18. A sychi trist iscovers th t she is fru str te n e sily ngere w ith one of her tients for no obvious re son. While t lking to colle gue, she its th t the tient re in s her of her busive f ther. Which of the follow ing best escribes the clinici n’s re ction? (A) (B) (C) (D) (E)

Countertr nsference Dis l ce ent Projection Re ction for tion Tr nsference

believe her thoughts coul be he r through the w lls. She feels th t this is n inv sion of her riv cy. 21. Which of the follow ing ter s best escribes the belief th t the tient’s thoughts tow r the neighbor w ere res onsible for the lost regn ncy? (A) (B) (C) (D) (E)

Dis l ce ent I e s of reference M gic l thinking Projection Re ction for tion

Questions 19 and 20 A 19-ye r-ol w o n resents w ith co l ints of fe r, rehension, n tre bling w ithou t ny know n re is osing situ tions. On ex in tion, you note brow n skin, s oky brow n rings on the ou ter corne , n occ sion l r i , jerky, u r oseless sw inging of the r s w hich e rs to w orsen w ith volu nt ry ove ent. 19. Which of the follow ing ter s best the tient’s ocul r fin ings? (A) (B) (C) (D) (E)

escribes

Arcus senilis Bru shfiel s ots K yser–Fleischer rings Su bconju nctiv l he orrh ge X nthel s

20. Which of the follow ing ter s best escribes the bnor l ove ents seen in this tient? (A) (B) (C) (D) (E)

Athetoi Choreifor H e ib llis u s Myoclonu s Myotoni

Questions 21 and 22 A 21-ye r-ol w o n is brought to the sychi tric e ergency e rt ent fter c lling the olice to tu rn herself in. She cl i s th t she w s res onsible for the loss of her neighbor’s regn ncy. She believes her neg tive thoughts tow r the w o n c u se her isc rri ge. On further qu estioning, she tells you th t she felt thre tene by her neighbor bec u se she

22. Which of the follow ing ter s best escribes the tient’s fe r th t her thoughts cou l be overhe r ? (A) (B) (C) (D) (E)

Echol li Thou ght bro c sting Thou ght control Thou ght insertion Tr nsference

Questions 23 and 24 A 25-ye r-ol n resents to the e ergency roo fter being brou ght in by his girlfrien . She h s been concerne s “he h s not been cting right.” She st tes he h s been st ying u ll night over the l st sever l ys but still “very ctive” uring the yti e. H e h s re rr nge the fu rniture n tte te to re o el their b throo , es ite his l ck of ex erience w ith constru ction. H e h sn’t show n u for w ork recently, n she is fr i he w ill be fire . While he its th t he h sn’t sle t, he cl i s to feel “gre t!” H e is so ew h t ifficu lt to interview s he w lks roun the roo , h nling ll the equ i ent. When confronte w ith his not going to his w ork, he st tes “Why shou l I . . . I on the verge of iscovering cu re for AIDS, w hich lso h ens to be for u l for econo ics, w hich w ill lso get ri of overty.” H e enies lcohol or ru g u se, bu t u on qu estioning, its to erio in his l te teens w hen he felt e resse , w ith ifficu lty slee ing, w eight loss, low energy, gu ilt, n hel lessness/ ho elessness. H e w s su bsequ ently tre te w ith “so e sort of nti e ress nt” for roxi tely 18 onths, bu t he never follow e u fter th t ti e.

Que s tions : 18–29

23. Which of the follow ing is the i gnosis for this tient?

ost

ro ri te

(A) (B) (C) (D)

Bi ol r I isor er Bi ol r II isor er Cyclothy ic isor er M jor e ressive isor er w ith sychotic fe tu res (E) Schizo hreni

A itri tyline Cloz ine Flu oxetine H lo eri ol V l roic ci

25. A 36-ye r-ol w o n w s begu n on l r zol (X n x) 3 ye rs go for nic isor er. After w tching new s re ort on television, she bec e frightene bou t iction. She bru tly sto e the e ic tion 2 ys go n h s since been ex eriencing cu te, intense nxiety, tre ors, sw e ting, n l it tions. Which of the follow ing e ic tions w ou l be the ost ro ri te to rescribe? (A) (B) (C) (D) (E)

Di ze Flu zenil Pro r nolol Sertr line V l roic ci

(A) (B) (C) (D) (E)

A evou t hu sb n fin s th t his w ife is h ving n ff ir w ith his best frien . One w eek l ter, he fin s th t he c nnot w lk. A thorou gh neurologic w orku f ils to reve l c u se to his su en r legi . H is neu rologic ex in tion is not consistent w ith u er or low er otor neuron fin ings. Des ite this r tic is bility, he see s qu ite un ffecte by it e otion lly. be

Déjà enten u Déjà vu Folie à eu x J is vu L belle in ifférence

28. A 67-ye r-ol w o n w ith u l on ry c rcino n secon ry br in et st ses recently r fte w ill in the resence of her f ily ttorney. She h s history of jor e ressive isor er (MDD) th t is now in re ission. She eci es th t her chil ren w ho re w ell est blishe in their c reers o not nee ny inherit nce n th t her est te w oul best serve ch rity. To secure the v li ity of the w ill, the tient sks her sychi trist to sub it letter to her ttorney reg r ing her co etency. Which of the follow ing w ou l be the ost i ort nt f ctor in eter ining this w o n’s test ent ry c city? (A) (B) (C) (D) (E)

Questions 26 and 27

26. Which of the follow ing i gnoses w ou l the ost ro ri te in this tient?

Conversion isor er F ctitiou s isor er Illness nxiety isor er M lingering So tic sy to isor er

27. Which of the follow ing ter s best escribes this tient’s ini iz tion of the severity of his sy to s?

24. Which of the follow ing e ic tions w ou l be the ost ro ri te to rescribe? (A) (B) (C) (D) (E)

(A) (B) (C) (D) (E)

251

Actu s reu s A history of MDD Know le ge of her n tu r l heirs The resence of conserv tor of erson The resence of ju ge to w itness the signing of the w ill

29. You re consu lte to see 77-ye r-ol H isnic le w ith schizo ffective isor er, e ressive ty e, itte to the hos it l for chest in. H is w orku h s reve le u l on ry e bolis , n the e ic l te w ishes to l ce chest tube in the tient, bu t the tient refu ses. The ri ry te h s ske you to eter ine his c city to refu se the roce u re. Which of the follow ing fe tures is

252

8: Pra c tic e Te s t 2

the

ost i king c

ort nt in eter ining city in this tient?

ecision-

(A) Ability to co u nic te (B) Age of tient (C) Agree ent w ith tre t ent te reco en tions (D) Presence of sychosis (E) S nish s e king 30. A e uty sheriff serves you su b oen for the recor s of one of your tients w ho is the efen nt in civil li bility l w su it. Which of the follow ing is the ost ro ri te cou rse of ction? (A) Rele se the tient’s recor s to the l intiff bec u se the su b oen overri es tient consent. (B) H n over only the infor tion th t is relev nt to the c se. (C) Cont ct the tient n sk if she or he w ou l like the infor tion rele se . (D) Rele se the tient’s recor s irectly to the cou rt. (E) Refu se to s e k to the sheriff in or er to int in confi enti lity. 31. A 23-ye r-ol gr u te stu ent you h ve been seeing in long-ter sychother y h s sto e ying his bills, es ite being rei bu rse by his insu r nce co ny. Which of the follow ing is the next ost ro ri te cou rse of ction? (A) Cont ct the tient’s insu r nce co ny n requ est th t they issue you nother y ent. (B) Directly ress this issu e w ith the tient t the next sche u le oint ent. (C) Inqu ire w ith f ily e bers w hether the tient h s fin nci l roble s. (D) N otify collection gency to obt in rei bu rse ent. (E) Sen the tient letter of ter in tion.

Questions 32 and 33 You re ske to erfor “co etency to st n tri l” ev lu tion of n 18-ye r-ol n w ho w s rreste for violently ss u lting his girlfrien . The tient h s no st sychi tric history, n frien s re ort th t he h s never e onstr te ny violent ten encies. The tient re orts th t his rel tionshi w ith his girlfrien “w ent ow nhill” shortly fter they gr u te fro high school n his girlfrien sto e c lling hi . On MSE, his ffect is constricte n his oo is re orte s “cr zy.” Thou ght rocesses re go l irecte , n he enies ny h llu cin tions or elu sions. The tient is u n ble to correctly erfor si le c lcu l tions. E ch ti e he is ske to u lti ly, su btr ct, or ir of nu bers, his nsw ers re w rong by one or tw o igits. For ex le, he res on s “22” w hen ske to u lti ly 7 by 3. 32. Which of the follow ing i gnoses is the likely in this c se? (A) (B) (C) (D) (E)

ost

Conversion isor er F ctitiou s isor er M jor neu rocognitive isor er M lingering Schizo hreni

33. Which of the follow ing st te ents best escribes the bove tient’s ro u ction of sy to s n otiv tion? (A) Consciou s ro u ction of sy to s to ssu e the sick role. (B) Consciou s ro u ction of sy to s to obt in secon ry g in. (C) Unconsciou s ro u ction of sy to s u e to u nconsciou s conflicts. (D) Unconsciou s ro u ction of sy to s to ssu e the sick role. (E) Unconsciou s ro u ction of sy to s to obt in secon ry g in. 34. You re ske to ev lu te 68-ye r-ol n on the in tient e icine service for incre sing confu sion. The tient w s itte 2 ys e rlier for neu oni . After erfor ing ent l st tu s ev lu tion, you su s ect eliriu . Fu rther history fro the tient’s

Que s tions : 30–39

w ife in ic tes th t the tient ro e ou t of school in the seventh gr e. Which of the follow ing tests w ou l best ssess his bility to int in n focu s ttention? (A) (B) (C) (D) (E)

Cou nting by 2’s to 20 R n o letter test Seri l sevens Seri l threes Si le c lcu l tions

35. A 7-ye r-ol girl is brou ght to the e ergency e rt ent for ev lu tion of sore thro t n fever. H er rents took her to the e i trici n’s office bou t 1 w eek go n he reco en e flu i s n be rest. Within the l st 2 ys, the tient evelo e ys h gi n severe b o in l in. H istory fro the rents in ic tes the tient h s been nxiou s n i u lsive over the l st onth. There h s lso been rke ecline in school erfornce, n she h s not been intereste in l ying w ith her frien s. Physic l ex in tion is re rk ble for signific nt erythe over the osterior h rynx w ith gr y exu te. There re br sions in the region of the tient’s l bi . Co lete bloo cou nt show s w hite bloo cell (WBC) cou nt of 14,000/ µL w ith left shift. Which of the follow ing is the next ost ro ri te ste ? (A) Arr nge for f ily eeting to eter ine s fe is osition. (B) Cont ct the st te’s Chil Protective Services w hile kee ing the tient s fe. (C) Det in the rents w hile you notify the olice. (D) N otify the rob te court to h ve the tient leg lly re ove fro the llege er etr tor. (E) Refer the tient to the f ily’s e i trici n. Questions 36 and 37 A 48-ye r-ol le is brou ght into the e ergency roo (ER) vi n bu l nce. H e s ells of lcohol, is covere in vo it, n not res onsive to qu estions. H e is unste y n u ncoo er tive w ith the hysic l ex in tion, bu t he is note to h ve isconju g te eye ove ents.

253

36. Which of the follow ing w ou l be the ost ro ri te next ste in the n ge ent of this tient? (A) A inister thi ine or lly (PO) before IV flu i s n glu cose. (B) A inister thi ine intr venou sly (IV) before IV flu i s n glu cose. (C) A inister thi ine IV fter IV flu i s n glu cose. (D) A inister n loxone IV before IV flu i s n glu cose. (E) A inister n loxone IV fter IV flu i s n glu cose. 37. The bove tient is st bilize n itte to the ICU. After 3 ys, he begins to beco e confu se g in, w ith visu l h llu cin tions, tre ors, i horesis, n elev te bloo ressu re n u lse. Which of the follow ing is the ost ro ri te tre t ent for this tient? (A) (B) (C) (D) (E)

A A A A A

inister b rbitu r te. inister benzo i ze ine. inister ition l thi ine. inister n nti sychotic. inister hy r l zine.

38. A you ng w hite le, ge u nknow n, is brou ght into the e ergency roo u nres onsive to qu estioning. H is vit ls e onstr te nor l te er tu re, low u lse n bloo ressure, n ecre se res ir tions. H e e rs le, w ith u ils th t re constricte n ini lly res onsive. A inistr tion of w hich of the follow ing w ou l be ost likely to i rove his con ition? (A) (B) (C) (D) (E)

Disu lfir (Ant bu se) Benzo i ze ines Flu zenil N loxone Thi ine

39. A 77-ye r-ol w o n w ithout rior sychi tric history is brought into her f ily hysici n’s office w ith her husb n . H e is concerne th t she is “ e resse ” n nee s tre t ent. H e escribes her w ith r w ing e otion lly over the st ye r or tw o, w ith gr u l ecline in

254

8: Pra c tic e Te s t 2

her bility to c re for herself. They h ve been un ble to t ke rt in their u su l soci l ctivities, n the tient “ju st w n ers roun the hou se uring the y.” H e h s lso notice th t she is forgetful, often is l cing ite s, n ixing u n es of cqu int nces. When ske , the tient enies ny ifficulties, st ting she feels “fine,” n th t her husb n w orries too uch. She ntly refutes roble s w ith her e ory, inste bl ing hi for oving things roun in their house. She h s no signific nt e ic l roble s. U on MSE, she is le s nt n coo er tive w ith questions, lthough efensive t ti es. H er ffect is neutr l but full. There is no su ici l or ho ici l i e tion, n she enies ny sychotic sy to s. H er Mini-Ment l St te Ex in tion is 20/ 30. H er hysic l is essenti lly u nre rkble. Which of the follow ing w oul be the ost ro ri te ri ry tre t ent for this tient? (A) (B) (C) (D) (E)

Cit lo r G l nt ine Ginkgo bilob Me ntine Zi r si one

Questions 40 and 41 You re ske to see 37-ye r-ol w hite C tholic fe le w ith en -st ge ov ri n c ncer bec u se she h s tol her oncologist th t she w nts to ie. When you ro ch the tient’s be , you see c chectic bu t s iling w o n su rrou n e by her hu sb n n tw o you ng u ghters. She h s history of e ression bu t is not un er ny tre t ent for it now . “I re y to go, Doctor,” she s ys to you . H er oncologist h s tol you th t he w oul like to try new che other y for w hich the tient is goo c n i te, bu t the tient h s refu se ; he is concerne bou t her suici l otenti l. 40. Which of the follow ing ch r cteristics incre se this rticu l r tient’s risk of co itting su ici e? (A) Age (B) Gen er (C) M rit l st tu s

(D) R ce (E) Religion 41. The tient w its u ntil her f ily n ri ry c re te le ve the roo . She then sks you to hel her co it su ici e, s her rognosis is very oor. Wh t is the ost ro ri te next course of ction? (A) Discu ss her re sons for su ici e in ore et il. (B) Ex l in how she c n obt in leth l oses of e ic tion. (C) Infor the f ily i e i tely. (D) Pl ce the tient in restr ints. (E) Refer her to hysici n w ho erfor s eu th n si . D IRECTION S (Questions 42 through 49): Match the clinical presentation w ith the appropriate neuropsychological test. Each lettered heading may be selected once, more than once, or not at all. (A) (B) (C) (D) (E) (F) (G) (H ) (I) (J) (K) (L)

Beck De ression Inventory (BDI) Ben er Gest lt Test Blesse R ting Sc le Boston Di gnostic A h si Ex in tion Folstein Mini-Ment l St te Ex in tion (MMSE) Minnesot Mu lti h sic Person lity Inventory 2 (MMPI-2) Rey–Osterrieth Test Rorsch ch Test Stroo Test W Test Wechsler A u lt Intelligence Sc le— Revise (WAIS-R) Wisconsin C r Sorting Test (WCST)

42. A 40-ye r-ol w o n w ho scores 26/ 30 on the Folstein MMSE g ve ny nsw ers of, “I on’t know, I’ too tire to nsw er.” You w nt to ssess for the ossibility of e ression. 43. A 65-ye r-ol n h s ifficu lty org nizing, sequ encing, n l nning ctivities but no i ir ent in e ory.

Que s tions : 40–51

44. The f ily of n 80-ye r-ol n w ith neu rocognitive isor er ( e enti ) h s ske you to ev lu te his bility to continu e to live in his cu rrent environ ent. You w ish to sk his f ily n frien s their o inion of how he h s been oing. 45. A 16-ye r-ol boy w ith f ily history of intellectu l is bility resents w ith longst n ing oor school erfor nce n ggressive beh vior tow r eers. 46. A 37-ye r-ol n h s history of voi ing soci l situ tions, no close frien s, n reference for being lone. H e h s been escribe by others s u ne otion l n et che . You woul like to ev lu te this tient’s erson lity style w ith rojective test. 47. You w oul like to quickly ssess for otenti l neurocognitive isor er in 75-ye r-ol wo n itte to the e ergency e rt ent for f ilure to thrive. 48. A 45-ye r-ol w o n h s long history of u nst ble rel tionshi s, self-injuriou s beh vior, n ffective inst bility. She oes not eet criteri for jor e ressive, bi ol r, or nxiety isor er. You w ish to e loy n objective test to give fu rther evi ence for erson lity isor er. 49. You re ske to reo er tively ev lu te he is heric o in nce in n 18-ye r-ol left-h n e w o n w ith history of seizu re isor er w ho is bou t to u n ergo su rgery to re ove seizu re focu s in her left he is here. DIRECTIONS (Questions 50 through 83): For each of the multiple-choice questions in this section select the lettered answer that is the one best response in each case. 50. A 16-ye by her consents you kee

r-ol girl is brou ght to you r office rents to get regn ncy test. She for the ev lu tion bu t requ ests th t the resu lts confi enti l. Bet –hu n

255

chorionic gon otro in is ositive. The tient’s rents e n to know the resu lts. Which of the follow ing is the ost ro ri te cou rse of ction? (A) Disclose the resu lts in eeting w ith the tient n her rents. (B) Encou r ge the tient to iscu ss the resu lts w ith her rents. (C) Invoke testi oni l rivilege. (D) N otify the rents of the resu lts bec u se they y her e ic l bills. (E) Re ort the resu lts to Chil Protective Services. Questions 51 and 52 The tient is 31-ye r-ol , ivorce le, w ithou t rior sychi tric history, referre by his w ork for ev lu tion. H e st tes he h s felt “ ow n” since his se r tion n ivorce sever l onths go. H e h s been slee ing oorly, frequ ently w king u u ring the night, then feeling exh u ste u ring the y. He h s h little interest in foo , losing 10 lb, n he h s been e sily istr cte t w ork, resu lting in ist kes n re ri n . While he feels hel less n th t “I’ll never feel better,” he enies ny su ici l i e tion or ho ici l i e tion. H e rinks 1 to 2 beers on the w eeken s n enies illicit ru g u se. H e s okes 1/ 2 ck of cig rettes er y n h s no jor e ic l roble s exce t for bor erline hy ertension. H e t kes u ltivit in bu t no other e ic tions regul rly. After fu rther iscu ssion, he grees to begin tri l of e ic tion. The tient is begu n on flu oxetine 20 g ily. H e is co li nt w ith the e ic tion n follow s u regu l rly. After 4 w eeks he re orts slight i rove ent in his sy to s. Of note, he initi lly h nxiety n loose stools, bu t he enies current si e effects. 51. Which of the follow ing w ou l ro ri te next ste ? (A) (B) (C) (D) (E)

be the

A ri i r zole. A lithiu . Discontinu e the flu oxetine. Incre se flu oxetine to 40 g. Sw itch to sertr line.

ost

256

8: Pra c tic e Te s t 2

52. H e eventu lly chieves fu ll re ission of his e ressive sy to s n continu es on the e ic tion. H ow ever, he is concerne bou t his rognosis n otenti lly sto ing the nti e ress nt in the fu tu re. Wh t o you tell hi is his roxi te risk of evelo ing fu rther e iso e in the fu tu re? (A) (B) (C) (D) (E)

10% 20% 30% 40% 50%

Questions 53 and 54 A 35-ye r-ol w o n resents to you r office w ith referr l fro sychologist for sychi tric tre tent. She h s been su ffering “the blues” for sever l onths, inclu ing crying s ells, low energy, n hy erso ni . She is convince th t her fi ncé is bout to bre k u w ith her bec use she h s been e ting ore th n usu l. So eti es, how ever, she feels “ju st fine.” 53. Wh ich of th e follow in g w ou l be th e ost ro ri te tre t en t of choice for th is tient? (A) (B) (C) (D) (E)

A itri tyline (El vil) Div l roex so iu (De Flu oxetine (Proz c) Phenelzine (N r il) Zi r si one (Geo on)

kote)

54. The tient res on s w ell to tre t ent, n fter sever l onths retu rns to you r office for follow -u visit. She tells you she h s just been rrie n w ou l like to beco e regn nt. She n her hu sb n h ve been h ving u n rotecte sex for onth. Which of the follow ing w ou l be the ost ro ri te next cou rse of ction? (A) A nother nti e ress nt to revent ost rtu e ression. (B) Ask how i ort nt regn ncy is to her. (C) Discu ss the risks n benefits of continu ing her tre t ent u ring regn ncy.

(D) Refer her for electroconvu lsive ther (ECT). (E) Tell her to sto her e ic tion i e i tely.

y

55. A 45-ye r-ol n is brought to the sychi tric e ergency roo by olice fter being foun scre ing w hile cutting ow n tree in the loc l rk. H e tells you th t the ghost of his w ife, w ho ie 3 onths go, is living in the tree. The n h s no revious sychi tric history, but he re orts being very e resse since the e th of his w ife n h s h suici l i e tion. H e is ishevele n e rs to h ve ignore his erson l hygiene for so e ti e. H is vit l signs re nor l. Which of the follow ing is the ost ro ri te next course of ction? (A) (B) (C) (D)

Obt in u rine toxicology screen. Prescribe n nti e ress nt. Prescribe n nti sychotic. Prescribe n nti sychotic n n nti e ress nt. (E) Prescribe oo st bilizer.

Questions 56 and 57 You re seeing 56-ye r-ol n w ith history of lcohol u se isor er. H e co l ins bou t not being ble to slee n requ ests slee ing i . H e enies erv sive e ression or ny ch nge in his etite, energy, or concentr tion. H e cl i s th t otherw ise “everything is going w ell.” 56. Which of the follow ing e ic tions w ou l be the ost ro ri te to rescribe for this tient? (A) (B) (C) (D) (E)

Al r zol (X n x) Di ze (V liu ) Tr zo one (Desyrel) Z le lon (Son t ) Zol i e (A bien)

57. H e grees to begin the ro ri te bove e ic tion. Which of the follow ing si e effects w oul be ost i ort nt to w rn hi bout? (A) Anorg s i (B) I otence

Que s tions : 52–61

(C) Incontinence (D) Pri is (E) Retrogr e ej cul tion Questions 58 and 59 58. You re seeing 9-ye r-ol boy w ith history of e ression n suici l i e tion. H e h s been t king flu oxetine (Proz c) for 6 onths n h s h goo res onse. H is other sks to see you fter his oint ent n tells you th t she is concerne bou t new s re orts s ying th t serotonin-s ecific reu t ke inhibitors (SSRIs) incre se the risk of su ici e in chil ren. She sks you to consi er sto ing the e ic tion. Which of the follow ing is the best cou rse of ction? (A) Ask the other, “Wh t’s re lly bothering you bou t you r son being on nti e ress nts?” (B) Discu ss the risks n benefits of flu oxetine n other SSRIs in the e i tric o u l tion. (C) Sto the flu oxetine i e i tely s the tient is inor n it is his other’s w ish. (D) Sw itch to itri tyline (El vil). (E) Sw itch to roxetine (P xil). 59. A onth l ter, the boy n his other return to your office. She infor s you th t fter her son’s l st oint ent, she h s been giving hi St. John wort bec use she believes he nee s “n tur l re e y.” Which of the following woul be the ost ro ri te course of ction? (A) Incre se the ose of his nti e ress nt s St. John w ort w ill low er bloo concentr tions of the nti e ress nt. (B) Refer the c se to soci l services for chil bu se. (C) Tell the other there is no evi ence th t St. John w ort is n effective tre t ent for e ression. (D) Tell the other to sto the St. John w ort i e i tely. (E) Tell the other to w tch ou t for si e effects inclu ing hotosensitivity n izziness.

257

Questions 60 and 61 A 38-ye r-ol tient w ith history of bi ol r isorer h his first ( n only) nic e iso e 18 onths go. H e w s tre te for 1 ye r w ith oo st bilizer, n h s not t ken ny sychotro ics since th t ti e. H e h s co e in for n ev lu tion t the urging of his w ife, w ho is concerne bout his frequent crying s ells. The tient its th t he h s felt ore e resse over the l st sever l w eeks, w ith ssocite inso ni , low energy, oor etite n 5-lb w eight loss, ifficu lty concentr ting, n nhe oni . H e enies ny su ici l i e tion. Fortu n tely, these sy to s h ve not interfere w ith his w ork. H e h s no jor e ic l roble s n is on no e ic tions. H e rinks one to tw o gl sses of w ine sever l ti es er ye r n enies illicit ru g u se. 60. Which of the follow ing e ic tions w ou l be the ost ro ri te to rescribe this tient? (A) (B) (C) (D) (E)

Bu ro ion Flu oxetine H lo eri ol L otrigine V l roic ci

61. Which of the follow ing si e effects w ou l be the ost i ort nt to iscu ss w ith the tient rior to st rting? (A) (B) (C) (D) (E)

At xi N u se N eu tro eni R sh Se tion

Questions 62 and 63 A 60-ye r-ol w hite fe le w i ow is brought to you r clinic by her ughter, w ho re orts th t her other s i to her this fternoon th t she “w nte to en it ll.” She re orts thou ghts of over osing on her ills, w hich she h s stock ile . This tient h s no rior history of su ici e tte ts bu t h s h thou ghts of su ici e on ulti le rior occ sions over the cou rse of her ne r-lifelong history of e ression. She lost her hu sb n to su en he rt tt ck 1 ye r go, but her u ghter s ys th t she still h s ny very su ortive frien s w ith w ho she s en s signific nt

258

8: Pra c tic e Te s t 2

ou nts of ti e. She still c rries on her vi hobby of g r ening, lthough t slow er ce bec u se of her w orsening rthritis. She ex resses w orries over recent “big ro ” in the v lu e of retire ent stocks she h s. 62. Which of the follow ing is the ost signific nt f ctor itig ting her risk for su ici e? (A) (B) (C) (D) (E)

G r ening hobby N o rior su ici e tte ts R ce Slow er ove ents secon ry to rthritis Soci l su orts

63. Which of the follow ing e ic l con itions w ou l be the ost i ort nt to ru le ou t s otenti l c u se of e ression in this tient? (A) (B) (C) (D) (E)

H y ercholesterole i Phos holi i et bolis isor ers Thin b se ent e br ne ise ses Thyroi illness Try to h n eficiency

Questions 64 and 65 A 20-ye r-ol college so ho ore is brou ght to the e ergency e rt ent fter being fou n sse ou t in her b throo . She h been vo iting, w hich she its now w s self-in u ce . H er history is not ble for bre ku 4 onths go w ith her boyfrien of 2 ye rs. H er w eight is in the 82n ercentile for her height. She is enorrheic. There is no history of binge e ting. 64. Which of the follow ing o you ost ex ect to fin on her hysic l ex in tion? (A) (B) (C) (D) (E)

C fé u l it s ots Dent l ec y P l ble s leen Pectus exc v tu Pitting fingern ils

65. Wh t is the likelihoo th t with tre t ent she will ke full recovery over the next 10 ye rs? (A) 5% (B) 10%

(C) 25% (D) 50% (E) 75% 66. A 24-ye r-ol gr u te stu ent resents to her University He lth C re Syste w ith the chief co l int of “ e ression.” U on et ile questioning she reve ls history of intense oo sw ings since e rly olescence, w ith short (hours to ys) erio s of ys hori , ltern ting w ith ti es w here she feels “nor l.” She its th t she is e sily ngere , occ sion lly bre king ite s n su erfici lly cutting on herself w hen or bore . She h s been un ble to kee boyfrien for ore th n sever l onths, but st tes th t she w ill frequently rtici te in “one-night st n s” bec use she often feels lonely n e ty. She enies ersistent inso ni , nergi , or nhe oni , lthough gets e sily istr cte w hile in lectures. While w hen cutely u set she y feel ssively suici l, she enies ny st or current l n. Which of the follow ing tre t ents w oul be the ost ro ri te to begin for this tient? (A) (B) (C) (D) (E)

Ari i r zole Cit lo r Di lectic l beh vior l ther Lithiu V l roic ci

y (DBT)

67. A 24-ye r-ol n w ith history of schizohreni resents to the e ergency e rtent. H e h reviou sly t ken h lo eri ol (H l ol), but now refuses to t ke it. Bec use of this, you eci e to try flu hen zine (Prolixin), w hich see s to itig te his sy to s. One w eek l ter, he le ves the hos it l. H e continues the e ic tions you rescribe n on follow -u visit you notice he oves slow ly n h s festin ting g it. Block e of w hich of the follow ing rece tors ost likely ccounts for the bove si e effect? (A) (B) (C) (D) (E)

B s l g ngli cetylcholine rece tors D 4 rece tors 5-hy roxytry t ine-2 (5-H T2) Mesoli bic D 2 rece tors N igrostri t l D 2 rece tors

Que s tions : 62–74

68. A 16-ye r-ol boy beg n h ving re etitive eye blinking t the ge of 6. By ge 12, he beg n blu rting stereoty e hr ses su ch s “B lls!” n “Shitty!” s w ell s king kissing noises. Conco it nt w ith this, he h incre sing ifficu lty w ith ying ttention in school n occ sion lly is l ye w il , isinhibite beh vior. Which of the follow ing w ou l be the ost effective tre t ent for the ri ry (initi l) con ition? (A) (B) (C) (D) (E)

Cloni ine (C t res) Flu oxetine (Proz c) Methyl heni te (Rit lin) Ris eri one (Ris er l) To ir te (To x)

Au tis s Con u ct Intellectu M jor e Tic isor

ectru isor er isor er l is bility ressive isor er er

71. The chil un ergoes lengthy tri l of the reco en e ther y s bove, but his con ition continues without signific nt i rove ent. Which of the following woul be the ost ro ri te h r cologic tre t ent of choice? (A) (B) (C) (D) (E)

C rb ze ine (Tegretol) Cloni ine (C t res) Des o ressin (D v ) Di henhy r ine (Ben ryl) I i r ine (Tofr nil)

An 8-ye r-ol boy is referre by his e i trici n for fu rther ev lu tion. H is history is not ble for being o te t ge 6 onths. H is f ily history is signific nt for biologic l other w ith severe lcohol use isor er. H e h s stru ggle w ith ll of his su bjects n h s been in s eci l e u c tion cl sses s resu lt. H e cts ore concrete th n his eers, n h s few frien s. Of note, his intelligence qu otient (IQ) h s recently been e su re t 60. 72. Which of the follow ing w ou l be this chil ’s ost ro ri te i gnosis?

Questions 70 and 71 A 9-ye r-ol boy w ith history of n or l evelo ent l ilestones h s been w etting his be for the l st 6 on ths. H is ren ts re ort th t th e be w etting initi lly occu rre s or ic lly, bu t for the l st 2 onths it h s been h enin g bou t every oth er n ight. This h s c u se hi sign ific nt ou nt of sh e n h s been li iting his soci l inter ctions s he is u n ble to go to slee overs or w y to c . 70. Which of the follow ing interventions w ou l be the ost ro ri te? (A) Beh vior l ther y (B) Cognitive ther y (C) Di lectic l beh vior l ther

y

Questions 72 and 73

69. A 12-ye r-ol boy h s recently been rreste for setting fire to neighbor ’s b rn. H e h s been in trou ble before for l ying w ith firew orks in the neighborhoo , n w hen he w s 10 ye rs ol he w s su s en e for lighting tches t his school. Which of the follow ing i gnoses w ou l ost likely be co orbi in this tient? (A) (B) (C) (D) (E)

(D) Psycho yn ic sychother (E) Pu nish ent

259

y

(A) (B) (C) (D) (E)

Au tistic s ectru isor er Mil intellectu l is bility Mo er te intellectu l is bility Severe intellectu l is bility Profou n intellectu l is bility

73. Given the bove tient’s history, w hich of the follow ing hysic l fin ings w ou l ost likely be ex ecte ? (A) (B) (C) (D) (E)

Cleft l te Congenit l blin ness H y erextensible joints Microce h ly Pro inent j w

74. A 5-ye r-ol girl h s co e un er st te Chil Protection Services. H er know n to be using cr ck coc ine n n su orting her h bit w ith

the c re of other w s IV heroin, rostitution.

260

8: Pra c tic e Te s t 2

The girl’s te or ry foster rents re orte to st te w orkers th t she w s in iscri in tely soci l w ith u lts bu t often oesn’t e r to seek or res on to co forting. In her soci l inter ctions w ith other 5-ye r-ol chil ren t her reschool, she is note to often st y by herself but is escribe s being in ro ri tely ngry w ith her eers. Which of the follow ing w ou l be the ost ro ri te i gnosis in this c se? (A) Attention- eficit/ hy er ctivity isor er (ADH D) (B) Au tis s ectru isor er (C) Con u ct isor er (D) L ngu ge isor er (E) Re ctive tt ch ent isor er 75. A 39-ye r-ol n is rreste fter being fou n in e rt ent store, stu rb ting by ru bbing w o n’s shoe long his enis. H e relu ct ntly its th t he h s co itte this ct on nu erou s occ sions lthou gh h s not gotten c u ght or rreste . H e feels gre t e l of sh e n guilt, bu t he is u n ble to sto the beh vior. Which of the follow ing is the ost likely i gnosis? (A) (B) (C) (D) (E)

Fetishistic isor er Frotteu ristic isor er Pe o hilic isor er Tr nsvestic isor er Voyeu ristic isor er

Questions 76 and 77 A 27-ye r-ol l w stu ent resents to you s referr l for the tre t ent of first e iso e of jor e ression, initi lly i gnose by his ri ry c re hysici n 8 w eeks go. On his ent l st tu s ex in tion, the tient’s s eech is extre ely r i , ifficult to interru t, n he quickly ju s fro one to ic to nother. H e s ys th t his oo h s i rove gre tly since he l st s w his ri ry c re octor: “In f ct, I’ on to of the w orl .” The tient re orts 3 to 4 hou rs of slee er night for the l st w eek, “not th t I nee th t uch slee nyw y.” The tient h s been s en ing ll of his oney buying first e ition textbooks n re rr nging the fu rniture in the l w school inistr tion buil ing. H ow ever, he h s

been ski ing cl sses n not turning in ssign ents s he believes he w ill u to tic lly gr u te n ss the b r bec use of his bilities n intelligence. 76. Which of the follow ing w ou l be the likely i gnosis for this tient? (A) Bi ol r isor er, cu rrent e e resse (B) Bi ol r isor er, cu rrent e (C) Cyclothy ic isor er (D) Persistent e ressive isor ( ysthy i ) (E) Schizo ffective isor er, bi

ost

iso e iso e

nic

er ol r ty e

77. Which of the follow ing w ou l be the ost ro ri te h r cologic tre t ent for this tient? (A) (B) (C) (D) (E)

Cit lo r (Celex ) Div l roex so iu (De Flu oxetine I i r ine Lor ze (Ativ n)

kote)

Questions 78 and 79 A 36-ye r-ol w o n resents w ith 7 w eeks of e resse oo ost of the ti e ( lthou gh w s h y for short erio w hen she w on $150 in the lottery), sense of ho elessness, n thou ghts of su ici e for the l st onth. She lso re orts incre se etite n h s been slee ing u to 16 hou rs y. On ent l st tus ex in tion, her oo n ffect re ys horic ost of the ti e, but she l u ghs in res onse to hu orou s st te ents. 78. Which of the follow ing i gnoses w ou l the ost ro ri te in this tient?

be

(A) Bi ol r I isor er, cu rrent e iso e e resse (B) Bi ol r II isor er, cu rrent e iso e e resse (C) M jor e ressive isor er, w ith ty ic l fe tu res (D) M jor e ressive isor er, w ith el ncholic fe tu res (E) Persistent e ressive isor er ( ysthy i )

Que s tions : 75–82

79. Which of the follow ing cl sses of e ic tions w ou l be the ost effective in this tient? (A) (B) (C) (D) (E)

Anticonvu ls nts Lithiu Mono ine oxi se inhibitors (MAOIs) Tricyclic nti e ress nts (TCAs) Ty ic l (first-gener tion) nti sychotics

80. A 42-ye r-ol ccount nt resents for longter sychother y tre t ent. In the first session, you fin th t he h s been ivorce three ti es. The oor qu lity of his ro ntic n other rel tionshi s re the focus of his concerns. Over the course of onths, you fin out th t he is erfectionist n inflexible w ithin rel tionshi s. He ty ic lly i e lizes his rtner, then ev lues her, before en ing the rel tionshi su enly. At ti es, he loses “ sense of self” in rel tionshi s, not know ing w ho he is or w h t he w nts. During this ti e, he relies on others to vise hi on w h t he shoul o. H ow ever, if he t kes this vice n his l ns f il, he uses this to ev lue the one w ho vise hi . The tient kes it very cle r th t he sought your tre t ent s sychother ist “bec use I’ll only cce t the best in the city.” The tient lso re orts to you th t he h s so e unique bilities of erce tion th t llow hi to etect w h t or w ho w ill rovi e the best ossible outco e for hi . For ex le, he icke you s ther ist not only bec use of your re ut tion, but lso bec use of your office hone nu ber, w hich h s igits in scen ing v lue or er. Which of the following i gnoses is the ost ro ri te for this tient? (A) (B) (C) (D) (E)

Bor erline erson lity isor er N rcissistic erson lity isor er Obsessive erson lity isor er Other s ecifie erson lity isor er Schizoty l erson lity isor er

81. A 43-ye r-ol w o n is hos it lize uring sychotic e iso e in w hich she h the elusion th t she is being controlle w ith co uter rogr evelo e by the De rt ent of Ho el n Security. Accor ing to her f ily, she sto e t king her regi en of queti ine n lithiu 4 w eeks before this ission to

261

the hos it l, n she h s been telling her f ily bout her concerns reg r ing the governent controlling her for the l st 2 w eeks. The tient’s st sychi tric history is signific nt for three rior hos it liz tions over the l st 20 ye rs. At the ti e of her first ission, she resente w ith 3-w eek history of nic sy to s n sychosis. During her secon hos it liz tion, she w s suici l n note to be e resse w ith sychotic sy to s. During her thir ission, she w s g in e resse w ith ro inent isorg nize beh vior. Her ent l st tus ex in tion u on resent tion to the hos it l reve ls looseness of ssoci tion n elusion l sy to s w ithout ro inent oo co onent. Which of the follow ing is the ost ro ri te i gnosis for this tient? (A) Bi ol r isor er, w ith sychotic fe tu res (B) M jor e ressive isor er (MDD) w ith sychotic fe tu res (C) Schizo ffective isor er (D) Schizo hreni (E) Uns ecifie sychotic isor er Questions 82 and 83 The e ic l te on n in tient service c lls you to hel the w ith 72-ye r-ol fe le tient w ho w s itte for congestive he rt f ilu re. They re ort th t t night she beco es confu se , thinking th t she is in hotel. She kes u nre son ble e n s of the nu rsing st ff n beco es isru tive on the w r , interfering in the nu rses’ w ork. She h s the ost ifficu lty fro 10 pm u ntil bou t 2 a m, w hen she u su lly f lls slee . In the orning, she is better oriente n u ch ore coo er tive. A review of her e ic tions reve ls th t she is t king ci eti ine 400 g PO bi , fu rose i e 40 g PO q , tenolol 50 g PO q , igoxin 0.125 g PO q , n i henhy r ine 50 g PO qhs n 25 g PO qhs (secon ose 3 hou rs ost first ose for inso ni ). 82. Which of the follow ing e ic tions is likely to c u se this tient’s confu sion? (A) (B) (C) (D) (E)

Atenolol Ci eti ine Digoxin Di henhy r Fu rose i e

ine

ost

262

8: Pra c tic e Te s t 2

83. Which of the follow ing neu rotr ns itter syste s is ost irectly i lic te in the etiology of her confu sion? (A) (B) (C) (D) (E)

Acetylcholine Do ine G - inobu tyric ci (GABA) Serotonin N ore ine hrine

D IRECTION S (Questions 84 through 89): The follow ing question is preceded by a list of lettered options. For each of the questions in this section match the clinical vignette with the most appropriate medication to prescribe. M tch the clinic l vignette w ith the e ic tion to rescribe. (A) (B) (C) (D) (E) (F) (G) (H ) (I) (J) (K) (L) (M) (N )

ost

ro ri te

Ari i r zole Bu ro ion Chlor ro zine Cloz ine H lo eri ol L otrigine Lithiu N ortri tyline P roxetine Ris eri one Ris eri one Const Sertr line V l roic ci Venl f xine

84. A 32-ye r-ol n with schizo ffective isor er h s goo benefit with nti sychotic e ic tions but is oorly co li nt ue to l ck of insight n re uce cognition. 85. A 48-ye r-ol ivorce w o n h s history of schizo hreni n is co li nt w ith her e ic tion, bu t she resents to the e ergency roo fter evelo ing severe su nbu rn es ite only being ou tsi e for sever l hou rs. 86. A 41-ye r-ol w o n w ith chronic schizo hreni h s been trie on sever l first- n secon gener tion nti sychotics t ther eu tic oses,

bu t she re ins sychotic, w ith u itory h llu cin tions telling her to kill herself. 87. A 53-ye r-ol rrie le with history of jor e ressive isor er n oor co li nce ue to signific nt sexu l ysfunction now resents with recurrence of e ressive sy to s with rke hy erso ni n low energy. 88. A 22-ye r-ol fe le gr u te stu ent w ith bi ol r isor er beco es regn nt. 89. A 37-ye r-ol n w ith history of nic e iso es now h s 3 w eeks of e ression, w ith inso ni , oor etite, w eight loss, low energy, n ecre se concentr tion. DIRECTIONS (Questions 90 through 112): For each of the multiple-choice questions in this section select the lettered answer that is the one best response in each case. 90. A clinic l rese rch sychi trist t u niversity A h s been running ou ble-blin l cebocontrolle tri l of new nti e ress nt. The sychi trist conclu es fter st tistic l n lysis of the t th t there is no better effect of the ru g co re to l cebo. The sychi trist’s colle gu es t five se r te institu tions (u niversities B, C, D, E, n F) h ve ru n i entic l stu ies n eter ine th t there is tre t ent effect of the ru g co re to l cebo. Given this infor tion, one y conclu e th t the rese rch tri l t u niversity A resu lte in w hich of the follow ing? (A) (B) (C) (D) (E)

H igh v ri nce Low - re ictive v lu e St n r error Ty e I error Ty e II error

Questions 91 and 92 A 56-ye r-ol n w ith sever l ec e history of schizo hreni h s been t king nu erou s ifferent nti sychotics, lbeit regu l rly, for 27 ye rs. H is sy to s re re son bly w ell controlle , w ith occ sion l, nonco n h llu cin tions n so e

Que s tions : 83–96

r noi . n lives evelo e fingers th

H e h s been ble to w ork s volu nteer in grou ho e. Abou t 5 ye rs go, he w rithing ove ents of his w rists n t is e r w hen he goes to slee .

91. Which of the follow ing si e effects is consistent w ith this c se? (A) (B) (C) (D) (E)

ost

Ak thisi Dystoni N eu role tic lign nt syn ro e (N MS) P rkinsonis T r ive yskinesi

92. Which of the follow ing n to ic stru ctu res is ost likely i lic te in the etiology of his ove ents? (A) (B) (C) (D) (E)

B s l g ngli Cerebellu Front l cortex Mi br in Motor cortex

93. You re seeing 66-ye r-ol Vietn veter n t Veter n’s A inistr tion hos it l. H e h s been hos it lize nu erous ti es w ith si il r resent tions. H e co l ins of ongoing night res reg r ing fighting in Vietn , w ith frequ ent fl shb cks, intru sive thou ghts, oor slee , n incre se st rtle. Other th n co ing in for his oint ents, he h s been u n ble to go out in ublic bec u se he often feels th t he is u n er tt ck, fe ring th t so eone is going to ju ou t of the bu shes n “ bush” hi . H e h s been tre te w ith the follow ing e ic tions, ll w ith only o erte i rove ent in his sy to s: flu oxetine, sertr line, roxetine, cit lo r , irt z ine, i i r ine, n itri tyline. H e h s been co li nt w ith venl f xine 375 g ily. H e h s been sober fro lcohol n heroin for 20 ye rs. Which of the follow ing e ic tions w ou l be the ost benefici l for this tient? (A) (B) (C) (D) (E)

Bu ro ion H lo eri ol Lor ze Pr zosin V l roic ci

263

94. A 29-ye r-ol rrie n resents to you r out tient sychi tric clinic co l ining of nxiety n e ression. H is soci l history reve ls th t he is ju nior f cu lty e ber of loc l u niversity, w orking bout 60 hours er w eek. H e enjoys his job but re orts stress rising fro his rit l rel tionshi bec use his w ife ccuses hi of not s en ing “qu lity ti e” w ith her n his being “ rrie to the job.” Accor ing to Erik Erikson, in w hich st ge of evelo ent oes this tient currently h ve conflict? (A) (B) (C) (D) (E)

Au tono y versu s sh e n I entity versu s role iffu sion In u stry versu s inferiority Inti cy versu s isol tion Tru st versu s istru st

ou bt

Questions 95 and 96 A 46-ye r-ol n resents w ith e iso es of “bl cking out,” w here he w ill not re e ber erio s l sting for sever l inutes. During those ti es, he is tol by others th t he h s n o ex ression on his f ce n oesn’t res on to his n e. In ition, he co l ins of ifficulty in rel tionshi s u e to ecre se sex rive n irrit bility. H e s en s gre t e l of ti e w riting ow n his thoughts in his e oirs. 95. Which of the follow ing ition l sy to s w ou l be ost likely resent in this tient? (A) (B) (C) (D) (E)

De ression H y erreligiosity Obsessive-co u lsive beh viors Slee isor ers Urin ry incontinence

96. A slee - e rive electroence h logr is erfor e on the tient s rt of his co lete w orku . Which of the follow ing lobes of the br in w oul ost likely e onstr te bnor lities in this tient? (A) (B) (C) (D) (E)

Front l Cerebell r Occi it l P riet l Te or l

264

8: Pra c tic e Te s t 2

97. After being c lle to the e ergency e rtent to ev lu te you ng w o n w ho overose on b rbitu r tes, you fin her in be w ith her eyes close . She o ens her eyes briefly in res onse to in n e onstr tes flexion fro in bu t kes no sou n s. Which of the follow ing best re resents her Gl sgow Co Sc le score? (A) (B) (C) (D) (E)

7 6 5 4 3

Questions 98 and 99 You h ve been tre ting 34-ye r-ol n for jor e ression w ith serotonin-s ecific reu t ke inhibitor (SSRI) for the l st 2 onths. H e w s initi lly st rte on flu oxetine 20 g/ for the first onth, n then you r ise the ose to 40 g/ for the l st onth. While he h s toler te the e ic tion w ithout signific nt si e effects, his e ressive sy to s h ve re ine refr ctory es ite the incre se in osge. You h ve eci e to sw itch his e ic tion to henelzine (N r il), ono ine oxi se inhibitor (MAOI), in ho es of eliciting better nti e ress nt res onse. 98. Which of the follow ing w ou l be the best str tegy in king the tr nsition fro the SSRI to the MAOI? (A) Begin t ering the ose of the SSRI w hile incre sing the ose of the MAOI si ult neou sly. (B) St rt the MAOI u ntil ther eu tic levels h ve been re che , then t er the SSRI. (C) Sto the SSRI bru tly n begin the MAOI t n equi otent ose the follow ing y. (D) Sto the SSRI bru tly n i e i tely begin the MAOI. (E) T er the SSRI n 5 w eeks fter the l st ose begin the MAOI. 99. After he is sw itche to the henelzine, w hich of the follow ing foo s/ rink shoul the tient ost voi ?

(A) (B) (C) (D) (E)

Beets Li be ns Liver Pe nu ts White w ine

100. A 66-ye r-ol n w ith history of schizo hreni h been int ine on thiori zine (Mell ril) 700 g q for ny ye rs, fter nu erous unsuccessful tri ls w ith v rious nti sychotics. Over the l st 8 onths, he h s been co l ining of he ring erog tory voices, n his ose h s been gr u lly incre se to 1,000 g q . H e h s no signs of t r ive yskinesi . Which of the follow ing si e effects w oul be the ost concerning t this ose? (A) (B) (C) (D) (E)

Consti tion Dry eyes N e hrogenic i betes insi i u s Pig ent ry retino thy Urin ry retention

101. A 24-ye r-ol n w ithou t rior sychi tric history is itte to the sychi tric w r w ith isorg niz tion, r noi , git tion, n co n u itory h llu cin tions telling hi to kill hi self. H e is given tw o oses of h lo eri ol 5 g or lly. A roxi tely 8 hou rs l ter, he evelo s torticollis. Which of the follow ing w ou l be the ost ro ri te tre t ent? (A) (B) (C) (D) (E)

Acet ino hen Benztro ine Cyclobenz rine Ibu rofen Pro r nolol

102. A 28-ye r-ol w o n w ith history of bi ol r isor er is itte to the e ic l service bec u se of w e kness, ent l st tu s ch nges, n seru so iu of 154 ol/ L. Accor ing to her boyfrien , for the l st 2 w eeks she h s been rinking “lo s n lo s of w ter” bu t co l ining of being thirsty often, n lso frequ ently u rin ting. Which of the follow ing e ic tions w ou l be ost likely res onsible for this tient’s resent tion?

Que s tions : 97–107

(A) (B) (C) (D) (E)

C rb ze ine H lo eri ol Lithiu c rbon te Qu eti ine V l roic ci

enies ny suici l i e tion. H is st e ic l history is signific nt for i betes ellitu s, tw o yoc r i l inf rctions, tri l fibrill tion, n coron ry rtery by ss gr ft. The tient’s w ife w ishes hi to st rt e ic tion. 105. Which of the follow ing nti e ress nts shou l be ost voi e in this rticu l r tient?

Questions 103 and 104 A 58-ye r-ol n w ith long history of o i te u se isor er h s been u sing intr venou s heroin ily, roxi tely $50–60 er y. H e h s not been ble to int in ny signific nt sobriety es ite nu erous in tient n resi enti l reh bilit tion rogr s. H e l st u se heroin 12 hou rs go n h s su bsequ ently been itte to the in tient sychitric w r for etoxific tion. 103. Which of the follow ing e ic tions w ou l be the ost ro ri te to tre t his w ith r w l sy to s? (A) (B) (C) (D) (E)

A nt ine (Sy Cloni ine H lo eri ol Lor ze N loxone

etrel)

104. After the tient co letes his etoxific tion on the u nit, he continu es to be otiv te to bst in fro heroin u se. H e is referre to n out tient ictions rogr , bu t w ishes to t ke e ic tion to ecre se his heroin u se fter isch rge. Which of the follow ing e ic tions w ou l be the ost ro ri te to rescribe? (A) (B) (C) (D) (E)

265

D ntrolene Disulfir Flu zenil Meth one Sertr line

Questions 105 and 106 A 75-ye r-ol n w ith jor neu rocognitive isor er u e to Alzhei er ise se resents w ith his w ife to you r clinic for ev lu tion. She is concerne bec u se he h s been “ ow n” recently, co l ining of e ression. H is slee h s been w orse, n his etite n energy h ve been oor. The tient

(A) (B) (C) (D) (E)

Bu ro ion I i r ine Mirt z ine Sertr line Venl f xine

106. Which of the follow ing nti e ress nts w ou l ost likely resu lt in cognitive ecline in this tient? (A) (B) (C) (D) (E)

Bu ro ion Cit lo r Flu oxetine P roxetine Venl f xine

107. A 35-ye r-ol w o n w ith bi ol r isor er n frequ ent hos it liz tions resents to the e ergency e rt ent fter t king n overose of lithiu in su ici e tte t. Which of the follow ing signs w ou l be ost likely to occur? (A) (B) (C) (D) (E)

Ab o in l in Acu te ystoni Leg in P r noi elu sions P resthesi s

Questions 108 and 109 A 67-ye r-ol w o n resents to you r office w ith long history of jor e ression. She h s trie nu ber of ifferent nti e ress nt e ic tions in the st bu t h s not h re ission w ith ny of the . Currently, she is on lithiu , venl f xine (Effexor), nortri tyline (P elor), lor ze , riseri one, n benztro ine. She is st rting to believe th t her next- oor neighbors re ou t to h r her. You eci e to begin tri l of electroconvulsive thery (ECT).

266

8: Pra c tic e Te s t 2

108. Which of the follow ing the seizure l st for o ti tient? (A) (B) (C) (D) (E)

ini u ti es shou l l effectiveness in this

10 secon s 15 secon s 60 secon s 120 secon s 180 secon s

109. Which of the follow ing verse effects w ou l be the ost likely u ring the cou rse of ECT? (A) (B) (C) (D) (E)

Br in ge C t toni Fr ctu res Me ory i ir ent Vo iting

Questions 110 and 111 A 26-ye r-ol n w ithou t rior sychi tric history resents to sychi tric e ergency e rt ent w ith r noi , visu l h llu cin tions, feelings of u nre lity, e erson liz tion, n extre e git tion. A u rine toxicology screen is ositive for hencycliine (PCP). 110. Which of the follow ing w ou l be the best i e i te tre t ent for the git tion in this tient? (A) (B) (C) (D) (E)

Chlor ro zine Flu oxetine Lor ze Tr zo one Trihexy heni yl (Art ne)

111. Which of the follow ing tre t ents w ou l be the ost ro ri te if this tient w ere not git te ? (A) (B) (C) (D) (E)

Aci ific tion of the urine Cheese Su ortive c re Vit in B12 Vit in E

112. A 45-ye r-ol w o n w ith rior history of jor e ression is itte to the sychitric in tient w r . She h s been severely e resse , w ith oor slee , ecre se etite, low energy, n su ici l i e tion. She now believes th t her neighbors re cons iring to u r er her, n occ sion lly c n he r n’s voice th t tells her she shou l ie. After s e king w ith the neighbors, you fin ou t th t they brou ght her to the hos it l. They h ve been goo frien s for 20 ye rs n h ve notice the tient h s not been le ving the hou se uch, h s t ken to looking out of her w in ow s w ith su s iciou s gl nces, n h s not cle ne her hou se in the l st 2 onths. Which of the follow ing e ic tion co bin tions w ou l be the ost ro ri te tre t ent for this tient? (A) (B) (C) (D) (E)

Flu oxetine Lor ze P roxetine Sertr line Sertr line

n ris eri one n cloz ine n thyroi u g ent tion n lithiu n lor ze

Questions 113 through 116 M tch the clinic l vignette w ith the ost likely su bst nce-in uce isor er. E ch lettere o tion y be u se once, u lti le ti es, or not t ll. (A) (B) (C) (D) (E) (F) (G) (H ) (I) (J) (K) (L) (M) (N )

Alcohol intoxic tion Alcohol w ith r w l C nn bis intoxic tion Coc ine intoxic tion Coc ine w ith r w l H eroin intoxic tion H eroin w ith r w l Inh l nt intoxic tion Meth het ine intoxic tion N icotine w ith r w l N itrou s oxi e intoxic tion PCP intoxic tion Psilocybin w ith r w l 3,4-Methylene ioxy eth het (MDMA) (Ecst sy) intoxic tion

ine

Que s tions : 108–118

113. A 23-ye r-ol n is brou ght to the e ergency e rt ent by the olice. H e show s git tion, vertic l nyst g us, n n lgesi . 114. A 62-ye r-ol ho eless n itte to sychi tric unit begins h ving elev te vit l signs, visu l h llucin tions, i horesis, tre or, n seizu res. 115. A 43-ye r-ol w o n resents to the sychitric e ergency e rt ent co l ining of e ression n suici lity. She is observe to be f tigue , irrit ble, n with ys horic ffect. 116. A 30-ye r-ol u sici n resents in the ou t tient clinic. H e co l ins of fever n chills, runny nose, n use , bo y ches, i rrhe , n b o in l cr s for the l st 24 hours. Questions 117 and 118 For the follow ing scen rios, choose the e ic tion ost likely ssoci te w ith the si e effects. E ch

lettere o tion or not t ll. (A) (B) (C) (D) (E) (F) (G) (H ) (I) (J) (K)

y be u se

once,

Benztro ine Cloz ine Desi r ine (N or r Flu oxetine H lo eri ol Lithiu Lor ze Ol nz ine Phenelzine V l roic ci Venl f xine

267

u lti le ti es,

in)

117. A 34-ye r-ol n w ith schizo hreni resents w ith fever n chills. H e is foun to be b ctere ic n h s WBC cou nt of 900/ µL. 118. A 41-ye r-ol w o n on e ic tions for her bi ol r isor er resents w ith f tigu e, w eight g in, col intoler nce, consti tion, n ecre se concentr tion.

Ans we rs a nd Expla na tions

1. (C) Mirt z ine is n ty ic l nti e ress nt w ith nor renergic n serotonergic ctivity. Its ech nis of ction inclu es block e of the α 2- renergic uto- n heterorece tors, s w ell s block e of the 5-H T2 n 5-H T3 serotonin rece tors n hist inergic (H 1) rece tors. This unique co bin tion results in incre sing serotonin, n , in higher oses, nore ine hrine; how ever, irt z ine oesn’t crete sexu l ysfunction n , es eci lly t low er oses, c uses se tion n incre se etite. 2. (F) Venl f xine is n nti e ress nt th t inhibits the reu t ke of both serotonin, n , in higher oses, nore ine hrine. Bec u se of this, it y c u se incre se bloo ressure, rticu l rly in oses bove 300 g ily. 3. (D ) Although ost nti e ress nts c n c use w ith r w l syn ro e (ch r cterize s flulike sy to s) if bru tly iscontinue , those w ith shorter h lf-lives, es eci lly roxetine, re ore likely to o so. 4. (B) Flu oxetine h s signific ntly longer h lflife th n the other liste nti e ress nts. This tr nsl tes into few er “w ith r w l” sy to s if su enly sto e n y lso be n v nt ge in c ses of occ sion l oor co li nce. 5. (E) Most of the secon -gener tion nti sychotics c u se w eight g in, inclu ing cloz ine, ol nz ine, qu eti ine, n ris eri one. H ow ever, zi r si one, lu r si one, n ri i r zole re w eight neu tr l. 6. (B) This tient is su ffering fro jor e ressive isor er (MDD), single e iso e. While 268

there re ny effic ciou s nti e ress nts to choose fro , serotonin-s ecific reu t ke inhibitors (SSRIs) su ch s cit lo r re consi ere first-line tre t ent. Ari i r zole is secon -gener tion (or ty ic l) nti sychotic. It w ou l be ro ri te to u se ri i r zole if this tient h sychotic sy to s (su ch s h llu cin tions or elu sions) or tre t ent refr ctory e ression (w hich is not the c se in this tient). Methyl heni te is sti ul nt u se for ttention- eficit hy er ctivity isorer, lthou gh it y occ sion lly be u se s n ju nct in the tre t ent of severe e ressions or in cert in c ses w here r i onset is necess ry (such s ehy r te tient). N ortri tyline is tricyclic nti e ress nt (TCA). While TCAs re ju st s effic cious s SSRIs, their si e effects n leth lity in over ose resu lt in their not being u se s first-line e ic tions for jor e ression. Tr nylcyro ine is ono ine oxi se inhibitor (MAOI), rticu l rly u sefu l in MDD w ith ty ic l fe tu res (incre se etite, w eight g in, n oo re ctivity), lthou gh the ru g– ru g inter ctions n iet ry restrictions of MAOIs li it their use. 7. (A) This tient h s escribe nic tt ck. P nic tt cks re ch r cterize by su en onset of intense fe r, re , or nxiety, n cco nie by v riety of hysic l sy to s. Althou gh virtu lly ny org n syste c n see to be the source of istress in nic isor er, co on sy to s inclu e i horesis, l it tions, shortness of bre th, tre u lou sness, flu shing, chest in, n izziness. Reg r less of the clinic l setting or sychi tric history, tients resenting w ith sy to s

Answe rs : 1–12

suggestive of nic isor er ust be thorou ghly w orke u to ru le ou t hysiologic thology s the sou rce of the sy to s. Re ssu r nce y be re tu re u ntil other e ic l c u ses re ru le ou t. If no other e ic l con itions re c u sing the sy to s n the nic tt cks continu e, rescribing n SSRI n / or referring her to sychi trist y be ro ri te. Since nti e ress nts t ke sever l w eeks to re ch fu ll benefit, benzo i zeines y be u se in the interi . Untre te , nic isor er ten s to ru n chronic cou rse n c n c u se severe is bility. 8. (D ) N eologistic s eech, ositive sy to of schizo hreni , consists of w or s eu by the tient, h ving no est blishe e ning. Cl ng ssoci tions re w or s or hr ses stru ng together bec u se of the soun s they ke, not bec u se of the e ning they convey. Echol li is sy to of c t toni , w here the tient w ill re e t w h t is s i by so eone else, reg r less of the context, n often in re etitive nner. Flight of i e s is the r i shifting of one i e to nother, n is ch r cteristic of ni . Wor s l is n illogic l, incoherent collection of w or s or hr ses. 9. (E) This w o n suffers fro ersistent e ressive isor er ( ysthy i ), low -gr e, chronic e ression. In ults, the sy to s ust be resent for t le st 2 ye rs. Like jor e ressive isor er (MDD), other th n subjective feeling of e resse oo , the key fe tures of ysthy i re lter tions in etite n slee , feelings of ho elessness, ifficulty concentr ting or king ecisions, low self-estee , n low energy or f tigue. While ersistent e ressive isor er y not res on s robustly to e ic tions s MDD, n equ te ther eutic tri l (on xi u ose) shoul l st t le st 6 w eeks before eci ing th t it is f ilure. 10. (A) The AIMS is st n r clinic l ex in tion n r ting sc le th t shoul be inistere to tients receiving nti sychotics (neu role tics) su ch s h lo eri ol. It is u sefu l for etecting the evelo ent of t r ive yskinesi s w ell s tr cking ch nges over ti e. The BDI is e ression screening tool. The

269

BPRS is short sc le evelo e to ssess sychi tric sy to tology ri rily in tients w ith sychosis or severe i ir ent. It is ore often use in rese rch settings. The PH Q is selfinistere screening tool focu se on co on ent l roble s th t often resent to the ri ry c re hysici n (e.g., e ressive, nxiety, lcohol use, e ting, n so tic sy to isor ers). The PAN SS is st n r test to ssess tre t ent ou tco es in stu ies of schizo hreni n other sychotic illnesses; it u tilizes three subsc les: ositive sy to s, neg tive sy to s, n gener l sycho thology. Like the BPRS, it is ostly use for rese rch r ther th n clinic l r ctice. 11. (B) P tients w ith C gr s syn ro e su ffer fro the elusion th t so eone f ili r is n i entic le ring re l ce ent. A ok is M l ysi n ter for issoci tive e iso e ch r cterize by erio of broo ing le ing to fit of violence follow e by nesi . Cot r syn ro e is nihilistic elu sion involving believing one is e , or th t the bo y or org ns re ying. Cou v e syn ro e occurs w hen the husb n of regn nt w o n h s si il r sy to s of regn ncy n / or l bor. Koro is tr u tic fe r th t the enis is shrinking into the bo y c vity, seen in so e Asi n cu ltu res. 12. (A) Bere ve ent is the sychologic l re ction to loss, often the e th of signific nt other. It is not consi ere ent l isor er or thologic l re ction. While sy to s y overl w ith MDD ( e ression, etite/ slee / energy ch nges), the intense sy to s of e ression in bere ve ent re often short-live n self-li iting, n there is little over ll fu nction l i ir ent. Occ sion l illu sions or h llucin tions of the ece se re not u nco on, bu t the in ivi u l ret ins insight reg r ing these. Th t being s i , if the e ressive sy to s continue or w orsen, consi ertion for MDD shou l be given. There is no evi ence of rior or cu rrent nic e iso es to ju stify the i gnosis of bi ol r isor er. Persistent e ressive isor er ( ysthy i ) is chronic, low -level e ression th t l sts t le st 2 ye rs in u lts. There is lso no evi ence of

270

8: Pra c tic e Te s t 2

ri ry sychotic illness consistent w ith schizo ffective isor er. 13. (C) Co ro r xi is co lex otor tic occ sion lly seen in Tou rette isor er, ch r cterize by king obscene gestu res. Bru xis is the grin ing of the teeth. Ble h ros s is the s s of the eriorbit l u scles c using su st ine or ex gger te blinking. Echo r xi is lso co lex otor tic w here ove ents of nother re re e te . It is cl ssic sy to of c t toni . Torticollis is s s of the neck u scles su ch s the sternoclei o stoi on one or both si es, n c n be si e effect of nti sychotic e ic tions. 14. (C) Echol li refers to re e ting nother’s w or s or hr ses. Co rol li is so eti es seen in Tou rette’s, w here tient w ill blu rt ou t obscenities s rt of voc l tic. Dys rthri refers to roble of w or rticu l tion, u su lly secon ry to cerebell r or otor control bnor lities. P lil li is lso re etition of w or s bu t is the re etition of one’s ow n w or s s if the erson “gets stu ck” on the s e w or or hr se. P r r xis is the ter for “sli of the tongu e” or Freu i n sli , ch r cterize by n error in s eech occu rring u e to the interference of the u nconsciou s. 15. (C) Kle to ni is n i u lse control isorer w here n in ivi u l ste ls objects not out of nee or onet ry v lu e. Antisoci l erson lity isor er is ch r cterize by l ck of e thy n chronic, l tive ttern of viol ting the rights of others; in this c se, the tient is istu rbe by the ste ling n show s no other evi ence of ntisoci l beh vior. Inter ittent ex losive isor er is lso n i u lse control isor er w here ggressive i u lses resu lt in ss u lt or the estru ction of ro erty. Pyro ni is n i u lse control isor er of eliber te, recurrent fire setting. Trichotillo ni (h ir- u lling isor er) is isor er rel te to obsessive-co ulsive isor er, w here tients co u lsively u ll their h ir resu lting in reci ble h ir loss. 16. (B) Ego- ystonic n ego-syntonic refer to w hether thou ghts or beh viors re istressing

or not istressing to the tient, res ectively. Delu sions re fixe f lse beliefs th t re in es ite evi ence to the contr ry n re not cultu r lly s nctione . Moo congru ent n oo incongru ent refer to elu sions or h llu cin tions in sychi tric isor ers th t re consistent w ith or not consistent w ith the ri ry oo istu rb nce, res ectively. 17. (C) G nser syn ro e w s thou ght to be ty e of issoci tive isor er, origin lly escribe in risoners; how ever, it is now believe to be consistent w ith lingering. It is ch r cterize by giving roxi te nsw ers. A ok n iblokto re issoci tive tr nce isor ers th t re escribe s istu rb nces in consciou sness n re in igenou s to rticu l r cu ltu res. Dere liz tion is n ex erience of u nre lity or et ch ent, often seen in nic tt cks n issoci tive isor ers. N eg tivis is ch r cterize by o osition to su ggestion or vice, inclu ing irect resist nce to being ove ; it is co on sy to of c t toni . 18. (A) Cou ntertr nsference refers to feelings the tient elicits in the ther ist, versu s tr nsference, w hich refers to feelings the ther ist elicits in the tient. Dis l ce ent is consi ere neu rotic efense ech nis w here feeling is irecte tow r nother in ivi u l w ho is less thre tening th n the origin l object of th t feeling. Projection is ri itive efense ech nis ch r cterize by re cting to u n cce t ble feelings or i u lses by l cing the in nother erson. Re ction for tion is nother neurotic efense ech nis w here n u n cce t ble i u lse is tr nsfor e into its o osite. 19. (C) This tient likely h s Wilson ise se, n utoso l recessive error of co er et bolis . Wilson ise se c n resent with ulti le hysic l s well s sychi tric sy to s n signs, inclu ing e ression, irrit bility, l bility, n erson lity ch nges. K yser–Fleischer rings re one such hysic l sign: gol en brown or gr y-green rings of ig ent t the corne l li bus. Arcus senilis is light gr y ring beginning su eriorly n exten ing to the li bus. It c n rese ble corne l rcus, which is ssoci te

Answe rs : 13–24

with hy ercholesterole i , but in n el erly tient it is often nor l fin ing. Brushfiel s ots re lighter-colore re s in the outer thir of the iris th t c n be ssoci te with Down synro e, but c n lso be nor l v ri nt. Subconjunctiv l he orrh ge is the fin ing of bloo in the re s of the eye surroun ing the iris. Also ssoci te with hy ercholesterole i is x nthel s , r ise yellow re s roun the eyeli s. 20. (C) H e ib llis us is n u ncontrolle sw inging of n extre ity ssoci te w ith Wilson ise se. It is u su lly su en, n once initi te it c nnot be controlle . Athetoi ove ents, or thetosis, re slow , sn ke-like ove ents of the fingers n h n s. Choreifor oveents re lso involu nt ry, irregu l r, n jerky bu t l ck the b llistic-like n tu re of he ib llis u s. Choreo thetoi ove ents re cl ssic lly seen in t r ive yskinesi , longter si e effect of nti sychotic e ic tions. Myoclonus is su en u scle s s , n yotoni is rolonge u scle contr ction. 21. (C) M gic l thinking n i e s of reference re tw o ty es of elusions th t c n be fou n in sychotic isor ers, su ch s schizo hreni . M gic l thinking is the belief th t one’s thoughts c n control outsi e events. I e s of reference re beliefs th t other in ivi u ls (e.g., govern ent, entert iners, the e i ) re referring to or t lking bou t the erson ex eriencing the elu sion. Dis l ce ent, rojection, n re ction for tion re ll ty es of efense ech nis s. Dis l ce ent is the tr nsferring of feeling tow r n object th t is less thre tening, su ch s the f ily et or one’s s ou se or chil ren. Projection is the f lse ttribution of one’s ow n u n cce t ble feelings to nother. Re ction for tion is the for tion of th ou gh ts th t re o osite to nxietyrovoking feelings. 22. (B) Thou ght bro c sting is the elu sion th t one’s thou ghts c n be he r by others. It is often seen in schizo hreni . Echol li is the re etition of nother’s w or s or hr ses, so eti es seen in c t toni . Thou ght control n thou ght insertion lso c n be seen in schizo hreni , n re both elu sions th t

271

others c n control the tient’s thoughts or insert thoughts into tient’s in , res ectively. Tr nsference, in strict ter s, is the reexeriencing of st ex eriences with the ther ist in the setting of sychother y. In gener l, this ter h s co e to e n the tr nsferring of e otions n feelings th t one h s fro one’s st to the hysici n or c re rovi er. 23. (A) This tient is cu rrently ex eriencing nic e iso e. While this tient likely h rior jor e ressive e iso e (MDE), w ith one nic e iso e he eets criteri for bi ol r I isor er. Bi ol r II isor er consists of n MDE n hy o nic e iso e. Criteri for hy o ni re the s e s for ni , bu t the severity is less (i.e., the tient is not sychotic, oes not nee hos it liz tion, or oes not h ve signific nt soci l or occu tion l i ir ent). This tient h s sychotic sy to s (gr n iose elusions) n signific nt i ir ent (re o eling hou se, not going to w ork). Cyclothy ic isor er consists of inor e ressive e iso es (i.e., th t o not eet criteri for MDE) ltern ting w ith hy onic e iso es. While this tient h s history of n MDE, he is currently su ffering fro nic e iso e n not e ressive e iso e. While he is sychotic ( elu sion l), his rior fu nctioning n signific nt ffective sy to s ke ri ry sychotic isor er su ch s schizo hreni u ch less likely. 24. (E) First-line h r cother y for tre ting nic e iso e of bi ol r isor er is either oo st bilizer (such s v l roic ci or lithiu ) or secon -gener tion (“ ty ic l”) nti sychotic (e.g., ris eri one, queti ine, ol nz ine). Anti e ress nts such s itri tyline ( tricyclic) or flu oxetine ( serotonins ecific reu t ke inhibitor) shou l be voi e , s they y w orsen the nic e iso e. While cloz ine is secon -gener tion nti sychotic, it is not first-line e ic tion given the risk of gr nu locytosis n nee for rou tine bloo onitoring. Although first-gener tion (“ty ic l”) nti sychotics such s h lo eri ol c n be use , they re not consi ere to be s effic ciou s in cute ni s oo st bilizers or the secon -gener tion nti sychotics.

272

8: Pra c tic e Te s t 2

25. (A) This tient is su ffering fro benzo i ze ine w ith r w l. The ost effic cious tre tent for benzo i ze ine w ith r w l is to use benzo i ze ine w ith long h lf-life (e.g., i ze ) n to slow ly t er it over ti e. Flu zenil is nons ecific co etitive nt gonist of the benzo i ze ine rece tor, u se in c ses of benzo i ze ine over ose. Giving it to this tient w oul severely w orsen her sy to s of w ith r w l n ut her t risk of seizu res. The other choices re not consi ere to be s effective s benzo i ze ines in tre ting w ith r w l. 26. (A) A conversion isor er (fu nction l neu rologic l sy to isor er) is often r tic set of neu rologic sy to s th t likely result fro u nconsciou s conflicts. F ctitiou s isorers involve the consciou s f bric tion of n illness in or er to resent oneself s ill (i.e., ssu e the sick role n be t ken c re of). Illness nxiety isor er (hy ochon ri sis) is the reoccu tion w ith fe rs of h ving seriou s ise se b se on isinter ret tion of bo ily sens tions. M lingering is the conscious f bric tion of n illness in or er to obt in secon ry g in (e.g., voi ing ilit ry u ty, w ork or j il, or obt ining is bility), not i entifie in this vignette. So tic sy to isor er is ch r cterize by one or ore so tic sy to s th t re istressing or resu lt in isru tion of life; it is likely ore rev lent in w o en. 27. (E) L belle in ifférence is the cl ssic l ck of concern show n tow r eficit or loss of function so eti es seen in conversion isor er (fu nction l neu rologic l sy to isor er). Déjà enten u is the feeling th t one is he ring w h t one h s he r before. It is u su lly ssocite w ith nxiety st tes or f tigu e. Déjà vu is si il r ex erience but refers to the sens tion th t so ething h s been seen before. J is vu is the o osite of éjà vu in th t it refers to so ething th t shou l be f ili r but see s qu ite unf ili r. Folie à eux is sh re elusion rou se in one erson by the influ ence of nother. 28. (C) Test ent ry c etence requ ire to

city (the level of co ke v li w ill) is b se

on the resence of ll of the follow ing: (1) n u n erst n ing of the n tu re of the w ill; (2) know le ge of one’s ssets; (3) know le ge of n tu r l heirs; (4) bsence of cu te sychosis (i.e., elusions) w hich ight co ro ise r tion l ecision king; n (5) free o fro u n u e influ ence or coercion. The v li ity of the w ill y be u n er ine by e onstr ting th t the in ivi u l f ile to eet ny of the bove criteri . Actu s reu s refers to the volu nt ry ct of co itting cri e n is n ele ent u se in eter ining cri in l res onsibility. A history of ent l illness is not su fficient to inv li te one’s test ent ry c city. The resence of conserv tor of erson n history of ent l illness re i ort nt only insof r s they be r u on ny of the bove note f ctors. The resence of ju ge t the signing of w ill is not requ ire . 29. (A) Co etency is leg l eter in tion e by the cou rt. C city, w hich is situ tion- n ti e-s ecific, is frequ ent sychi tric consu lt qu estion. Ele ents of c city inclu e the follow ing bilities: to co u nic te one’s w ishes, to u n erst n one’s illness, to u n erst n the risks n benefits of tre t ent n nontre tent, n to re son bou t one’s o tions. The ge of tient h s no be ring on the eter in tion of c city. Whether the tient grees w ith the tre t ent te reco en tions is not the issu e; how ever, qu estions of c city o not u su lly co e u u nless the tient is grees w ith the tre t ent te . The resence of sychotic illness oes not, in n of itself, eter ine c city. A tient being too isorg nize to co unic te equ tely or h ving elu sions involving the tre t ent ro ose y in ic te l ck of c city. A l ngu ge b rrier w ou l not e onstr te l ck of c city, so long s the tient int ins the bility to co u nic te his or her references n re sons. 30. (C) Confi enti lity is n integr l rt of the hysici n– tient rel tionshi n is es eci lly i ort nt in est blishing tru st in sychother eutic rel tionshi . The hysici n– tient rivilege shou l be bre che only in s ecific inst nces, such s the tient’s

Answe rs : 25–35

w iving of the rivilege (e.g., tient initites litig tion, consent to rele se infor tion is obt ine , there is u ty to rotect, e ergency situ tion, or in c ses in w hich the cou rt or ers rele se). In this c se, the infor tion shou l not be rele se unless the tient h s w ive his hysici n– tient confi enti lity rivilege. Lying to or isle ing the cou rt is u nethic l n y su bject one to rosecu tion. The re ining choices f il to int in confienti lity. 31. (B) When tient h s sto e ying his or her bill, the hysici n shoul irectly ress the issu e w ith the tient. F ilu re to y one’s bills y reflect u n erlying issu es for w hich the tient h s entere into sychother y or th t re rel te to sychi tric sy to s. Cont cting collection gency before ressing the issu e w ith the tient is in ro ri te. If tient continu es to refu se y ent fter iscu ssing the issu e, the tient shou l be notifie in w riting of the ou tst n ing b l nce, n referr l to collection gency ight be e. To reserve confi enti lity, c re shou l be t ken to isclose the ini u infor tion nee e for collection (i.e., te of service n ch rge) w ithou t referring to i gnosis or tre t ent ren ere . Asking the insu r nce co ny to y you fter check h s been isbu rse to the tient is fu tile bec u se the insu r nce co ny’s oblig tion tow r the insu re h s been fu lfille . Disclosing the existence of rofession l rel tionshi w ith the tient to his f ily is bre ch of confi entility. Ter in ting the ther eu tic rel tionshi before ex loring the yn ics behin the tient’s y ent elinqu ency ight h r ny fu tu re ther eu tic lli nce. 32. (D ) M lingering is the intention l ro u ction of sy to s for secon ry g in (e.g., to voi w ork, to ev e cri in l rosecu tion, or to g in fin nci l rew r s). A i gnosis of lingering shou l be su s ecte in e ico-leg l c ses, in in ivi u ls w ith ntisoci l erson lity isor er, w hen sy to s re ou t of ro ortion to objective fin ings ( s in this c se), or w hen tient in istress oes not coo er te w ith ev lu tion or tre t ent. Conversion isor er

273

(fu nction l neurologic l sy to isor er) escribes the ro u ction of neu rologic sy to s thought to resu lt fro intr sychic nee s or conflicts. F ctitiou s isor er c n be ifferenti te fro lingering b se on the bsence of extern l incentives. In f ctitiou s isor er, sy to s re intention lly feigne to s tisfy n intr sychic incentive to ssu e the sick role. A jor neu rocognitive isor er ( e enti ) is unlikely in this c se given the ge n resenting co l ints of the tient. There is no evi ence to su ggest isorg niz tion or sychotic sy to s consistent w ith schizo hreni . 33. (B) In ivi u ls w ith lingering consciou sly feign sy to s for fin nci l incentives or to voi w ork/ soci l oblig tions (i.e., secon ry g in). The consciou s ro u ction of sy to s to ssu e the sick role is seen in f ctitiou s isor er. Conversion isor er is the u nconsciou s or involu nt ry ro uction of sy to s likely ue to intr sychic, or u nconsciou s, nee s or conflicts. The unconscious ro u ction of sy to s to ssu e the sick role or to obt in secon ry g in is not consistent w ith sychitric illnesses. 34. (B) In this inst nce, the r n o letter test, w hich relies on concentr tion, coo er tion, n the bility to he r, is the test of choice. It consists of telling tient letter n then in onotone voice listing r n o string of letters. The tient res on s by r ising finger to in ic te w hen he or she he rs the key letter. The other tests rely not only on ttention bu t lso on c lcu l tion bilities, n therefore e u c tion l level. 35. (B) Sexu l bu se is esti te to occu r w ith n inci ence of 1% of chil ren er ye r. Know n cqu int nces (e.g., f thers, ste f thers, n le rel tives) re often the er etr tors. In this vignette, the tient likely h s cqu ire N eisseria gonorrhoeae fro le er etr tor. Other hysic l fin ings y inclu e inju ries to the genit li (i.e., hy en, v gin ) or the erineu . Psychi tric nifest tions of sexu l buse y inclu e nxiety, git tion, ggressive or i u lsive beh vior, n exhibitionis .

274

8: Pra c tic e Te s t 2

When sexu l buse of chil is su s ecte , the first ste is to ensure the s fety of the chil n i e i tely notify the st te’s Chil Protective Services. Re orting is n tory in c ses of hysic l n sexu l chil bu se. The other choices f il to rotect the chil n o not s tisfy st te- n te re orting st tu tes. 36. (B) This c se re resents Wernicke ence h lo thy, con ition seen in chronic lcohol u se isor ers. The etiology is cute thi ine eficiency n it resents w ith the cl ssic tri of confu sion, t xi , n o hth lo legi . Althou gh it is u su lly reversible w ith thi ine re l ce ent, it is i ort nt to inister the thi ine renter lly, rior to giving flu i s n glu cose. A inistr tion of glu cose n flu i s first y rovoke w orsening of sy to s resu lting in er nent ge. The inistr tion of n loxone is u se for o i te w ith r w l. 37. (B) The tient is ex eriencing sy to s consistent w ith severe lcohol w ith r w l, n ely eliriu tre ens (DTs), ch r cterize by eliriu , confu sion, co b tiveness, n elev te vit l signs. DTs c rry signific nt ort lity if not tre te . The tre t ent of choice is IM or IV benzo i ze ines. Although b rbitur tes y lso tre t lcohol w ith r w l, they h ve n rrow er ther eu tic in ex, w ith higher risk of se tion n res ir tory e ression. N either thi ine nor n nti sychotic w ill tre t the un erlying lcohol w ith r w l, n giving hy r l zine y be ngerou s s it y sk the u tono ic sy to s of the lcohol w ith r w l. 38. (D ) The tient is resenting w ith evi ence of over ose w ith o i tes, w hich is ch r cterize by the tri of so nolence, res ir tory e ression, n in oint u ils. O i te intoxic tion c n be r i ly reverse w ith IV inistr tion of n o i te nt gonist, su ch s n loxone. Ant bu se blocks cet l ehy e ehy rogen se n is given to lcoholics s n versive sti u lu s to voi lcohol consu tion. Benzo i ze ines w ou l be u se in c ses of severe lcohol w ith r w l, w hich w oul e onstr te elev te vit l signs. Flu zenil is

benzo i ze ine nt gonist use in the e ergency tre t ent of over ose w ith benzo ize ines, w hich oes not c use u ill ry constriction. Thi ine w oul be given in c ses of Wernicke ence h lo thy, c u se ost co only by chronic, he vy lcohol consu tion. It resents w ith sy to s of confusion, t xi , n o hth l o legi . 39. (B) This w o n is show ing signs of jor neu rocognitive isor er ( e enti ), likely ue to Alzhei er ise se, the ost co on c use of neu rocognitive isor ers ( e enti ). The ri ry tre t ent o lity is to begin n nticholinester se inhibitor, su ch s g l nt ine. Me ntine, n N - ethyl d - s rt te (N MDA) rece tor nt gonist, is e ic tion th t y be e to the regi en itionlly, bu t it is not s effective in il e enti s. Anti e ress nts (e.g., cit lo r ) n nti sychotic e ic tions (e.g., zi r si one) y be u se for ssoci te e ression or git tion/ sychosis, res ectively, bu t they re not ri ry tre t ents. Ginkgo bilob h s not been show n to be effective for the e ory eficits in neu rocognitive isor ers. 40. (D ) Risk f ctors for co lete su ici e inclu e (1) ge > 45, (2) le gen er, (3) never rrie > w i ow e / se r te / ivorce > rrie , (4) w hite r ce > bl ck r ce, n (5) religiou s > not religiou s. Wo en tte t su ici e tw ice s frequently s en; how ever, en re three ti es ore likely to co lete su ici e th n re w o en. 41. (A) Physici n- ssiste suici e involves the octor’s f cilit ting erson’s e th by rovi ing the infor tion or equi ent necess ry to en his or her life. The AMA’s Co e of Me ic l Ethics strongly con e ns such ction s “funent lly inco tible w ith the hysici n’s role s he ler.” R ther, the ethic l oblig tion of the hysici n is to equ tely res on to tient’s en -of-life issues. Choices (B) n (E) w oul both be ssisting the tient to co it suici e n re u nethic l. Infor ing the f ily w oul be bre ch of confi enti lity n not ro ri te t this oint, n l cing the tient in restr ints is not in ic te .

Answe rs : 36–51

42–49. [42 (A), 43 (L), 44 (C), 45 (K), 46 (H), 47 (E), 48 (F), 49 (J)] The Beck De ression Inventory (A) is 21-ite test w ith three res onses er ite th t is n e sily use screening tool to ev lu te for e ression. Bec u se it w ou l be u nu su l for n in ivi u l so you ng to h ve neu rocognitive isor er ( e enti ) n bec u se ny of the nsw ers reflect l ck of interest, the Beck De ression Inventory, in conju nction w ith the Folstein MMSE, y hel istingu ish e ression fro e enti . The WCST (L) ssesses execu tive fu nctions of the br in su ch s org niz tion l bilities, ent l flexibility, n the bility to bstr ct n re son. These c cities re believe to be loc te in the front l lobes. D ge to the front l lobes c n le to bnor lities on this test. The Blesse R ting Sc le (C) is tool th t sks frien s or f ily of the tient to ssess the bility of the tient to fu nction in his or her u su l environent. The WAIS-R (K) is n ssess ent of IQ u se in ges 16 onw r ; it c n be e loye to hel better u n erst n the in ivi u l’s intellectu l level of fu nctioning n ru le ou t n intellectu l is bility. IQ ssess ent w ill lso i the school in evelo ing n in ivi u lize stu y rogr for the stu ent. The Rorsch ch Test (H) is rojective test th t y be u se to ssess erson lity stru ctu re. The tient h s ch r cteristics of schizoi erson lity isor er, w hich y be better elu cite w ith skille ssess ent in the context of his clinic l history. The Folstein MMSE (E) is qu ick, e sily inistere test th t llow s for i e i te ssess ent of jor neu rocognitive isor er ( e enti ). Scores of less th n 24 re su ggestive of e enti . The MMPI-2 (F) is n objective test consisting of sever l hu n re tru e/ f lse qu estions u se to ssess n in ivi u l’s erson lity. It is the ost w i ely use n highly st n r ize test of erson lity. The W Test (J) is u se to ev lu te he is heric l ngu ge o in nce rior to surgic l elior tion of seizure focus. Where s ost right-h n e in ivi u ls show left he is heric o in nce for l ngu ge, left-h n e in ivi u ls y be either right or left o in nt. The test consists of injecting so iu yt l into the c roti rtery n observing the tr nsient effects on s eech.

275

Injection into the left c roti rtery nesthetizes the left si e of the br in, n those w ith left he is heric l ngu ge o in nce show interru te s eech. The Ben er Gest lt Test (B) involves co ying figu res, w hich hel s to screen for evelo ent l isor ers or br in ge. The Boston Di gnostic A h si Ex in tion (D ) is series of tests given by n ex erience clinici n to ev lu te n ke tre t ent reco en tions for in ivi u ls w ith h si . The Rey–Osterrieth figure (G) is sensitive to eficits in co ying n l ck of ttention to et il in in ivi u ls w ith rightsi e riet l lobe lesions. The Stroo Test (I) ssesses execu tive function s the tient h s to e l w ith istr ctions n h s to su ress n incorrect re ly before rovi ing the correct one. 50. (B) Physici n– tient confi enti lity in the tre t ent of inors u st be int ine u nless otherw ise n te by l w (e.g., chil bu se or c ses of bortion in so e st tes) or w hen rent l involve ent is necess ry in king co lex n life-thre tenin g e ic l ecision s. In this c se, th e hysici n u st res ect the tient’s confi enti lity bu t shou l re in cogniz nt of the i lic tions this w ill h ve on the f ily syste . Therefore, it is ost ro ri te to encou r ge the inor to iscu ss the issu e w ith her rents. Testi oni l rivilege refers to the rivilege invoke to rotect hysici n– tient confi enti lity w hen e ic l infor tion is su b oen e w ithou t tient consent. At this oint, there is no in ic tion of chil h r or neglect th t w ou l n te re orting to Chil Protective Services. The re ining tw o choices f il to int in hysici n– tient confi enti lity. 51. (D ) This tient is ex eriencing jor e ressive isor er (MDD), single e iso e. Anti eress nts re the tre t ent of choice. As this tient is show ing so e i rove ent n toler ting the e ic tion, the ost ro rite next ste w ou l first be to xi ize the ose. While u g enting w ith n ty ic l ntisychotic (e.g., ri i r zole) or lithiu c n be hel ful in tre t ent refr ctory e ression, this tient h s not h n equ te ose or tri l

276

8: Pra c tic e Te s t 2

of his cu rrent nti e ress nt. Discontinuing his fluoxetine w ou l gre tly incre se his risk of rel se. While sw itching to nother ntie ress nt (e.g., sertr line) y be in ic te , this w ou l only be reserve in c ses w here he either i not res on to n equ te tri l of xi l ose of n nti e ress nt, or if he suffere signific nt si e effects fro the current nti e ress nt. 52. (E) MDD ten s to be chronic illness. P tients w ith one e iso e of e ression h ve 50% risk of h ving fu tu re e iso es, lthou gh ntie ress nts hel to ini ize the risk. 53. (C) The tient is su ffering fro jor e ressive isor er (MDD), w ith ty ic l fe tures (i.e., incre se etite/ slee , oo re ctivity). Bec u se of their si e effect rofile, the tre t ent of choice in jor e ression is the serotonin-s ecific reu t ke inhibitors (SSRIs), such s flu oxetine (Proz c). A itri tyline n other tricyclic nti e ress nts (TCAs) inclu e the risk of rrhyth i s n c n be leth l in over ose, u nlike the SSRIs. While the tient re orts erio s u ring w hich she is “ju st fine,” there is nothing to in ic te cycling ttern (i.e., bi ol r isor er) th t w oul require oo st bilizer su ch s iv l roex so iu . Si il rly, being convince th t her fi ncé is bou t to le ve her is not likely sychotic, bu t is inste istortion th t oes not requ ire n nti sychotic su ch s zi r si one. Althou gh MAOIs such s henelzine re thou ght to be so ew h t ore effective (th n TCAs) in ty ic l e ression, they c rry the risk of hy ertensive crisis w hen co bine w ith foo s cont ining try to h n. 54. (C) Flu oxetine is cl ss C ru g, e ning th t it is otenti lly h r fu l to the evelo ing fetu s, ccor ing to ni l stu ies, lthou gh SSRIs s cl ss re not believe to be ter togenic or c u se birth efects. A iscu ssion of the risks n benefits of tre t ent is therefore ost ro ri te. There is no in ic tion th t she is t rticu l r risk for ost rtu e ression n no evi ence in the liter ture th t ing secon nti e ress nt in regn ncy w ill revent this isor er.

Asking the tient how i ort nt regn ncy is to her w ou l be in ro ri te, since it is erson l choice, n , ccor ing to the infor tion rovi e in the vignette, it is not so ething th t u ts her in nger of ny kin . ECT y be n o tion s it is s fe in regn ncy, bu t not before iscu ssion of the risks n benefits of her cu rrent tre t ent. While fluoxetine oes not h ve the w ith r w l synro e know n to occu r w ith roxetine, sto ing it i e i tely shou l be voi e u e to the risk of rel se. 55. (A) This tient is exhibiting sychosis (cu tting ow n tree th t his w ife’s ghost lives in) n e ression. There re ny ossible etiologies, inclu ing jor e ressive isorer w ith sychotic fe tu res, bi ol r isor er, schizo ffective isor er, n schizo hreni . Reg r less of the eventu l i gnosis, it is i ort nt to first ru le ou t other c u ses of his beh vior su ch s illicit or rescribe rug intoxic tion. Therefore, it is re tu re t this oint to rescribe ny e ic tions for hi . 56. (C) Tr zo one, w hich w s evelo e s n nti e ress nt, h s been show n to be n effective slee ing i n w ou l be useful in this tient. The benzo i ze ines, such s l r zol n i ze , overl w ith lcohol in bin ing to GABA rece tors—hence their u se in lcohol w ith r w l— n shoul be voi e in tients w ith history of lcohol u se isor er. Z le lon is rel tively contr inic te in those w ith severe liver ysfu nction, w hich is otenti l issu e in tient w ith history of chronic lcohol u se. Zol i e h s lso been show n to h ve si il r ictive otenti l to the benzo i ze ines. 57. (D ) Tr zo one c rries 1 in 1,000 to 1 in 10,000 risk of ri is . The other sexu l si e effects ong the choices re know n to occur w ith the serotonin-s ecific reu t ke inhibitors, but not rticul rly w ith tr zo one. Incontinence is not frequent si e effect of ny nti e ress nt. 58. (B) All of the SSRIs h ve been in the new s bec u se of controversy over t on incre se suici l gestures ong chil ren t king these

Answe rs : 52–65

e ic tions. As resu lt of nu erou s stu ies n review , the FDA h s l ce bl ck box w rning for ll nti e ress nts u e to the s ll incre se risk of suici l thou ghts or beh vior in chil ren t king nti e ress nts co re w ith l cebo. H ow ever, flu oxetine is not only the ost stu ie in chil ren, bu t, long w ith escit lo r (Lex ro), re the only e ic tions FDA rove for the tre t ent of chil hoo e ression, w hich kes sw itching to roxetine or itri tyline in ro ri te in this setting. Asking the chil ’s other w h t is “re lly bothering her” is u nlikely to be ro u ctive, s it is so ew h t ccu s tory n not ressing her legiti te concerns. Fluoxetine oes not h ve the w ithr w l syn ro e know n to occu r w ith roxetine, bu t sto ing it i e i tely shou l still be voi e . 59. (D ) Althou gh ny in ivi u ls think of n tu r l re e ies su ch s St. John w ort s s fe, these su le ents c n h ve signific nt si e effects. In rticu l r, St. John w ort y c u se incre se hotosensitivity, sto ch u set, r shes, f tigu e, restlessness, he che, ry outh, izziness, n confu sion. These si e effects re ore likely w hen St. John w ort is co bine w ith SSRIs. The recise h r cokinetics of St. John w ort re u nknow n, bu t St. John w ort is believe to ct like onoine oxi se inhibitor (MAOI) n h s conflicting evi ence reg r ing its effic cy in tre ting il e ression. H ow ever, oses of SSRIs shou l be ecre se w hile tients re t king the su le ent bec u se of ossible otenti tion of si e effects. As w ith ny nti e ress nt, it is not goo i e to sto it bru tly. There is no re son to think th t chil buse is involve , s the chil ’s other is not utting her son in nger. 60. (D ) This tient is resenting w ith bi ol r isor er, ost recent e iso e e resse . Des ite the l ck of t reg r ing bi ol r e ression, the i ct of e ressive e iso es y be signific ntly w orse th n th t of nic e iso es. L otrigine, n ntie ile tic e ic tion, h s e onstr te effic cy in ( n h s been rove for) the tre t ent of

277

e ressive e iso es of bi ol r isor er. Antie ress nts su ch s bu ro ion n fluoxetine shou l be voi e (es eci lly w ithou t n ition l oo st bilizer) s they y reci it te nic e iso e or le to r i cycling (> 3 oo e iso es er ye r). While secon -gener tion nti sychotics (e.g., qu eti ine) re likely benefici l in the tre t ent of bi ol r e ression, first-gener tion ntisychotics (neu role tics), su ch s h lo eri ol, re not effective n y w orsen e ressive sy to s. V l roic ci is consi ere firstline tre t ent of bi ol r ni , bu t the t re f r less convincing for e resse bi ol r tients. 61. (D ) L otrigine u se h s 7% risk of r sh; u nfortu n tely, Stevens–Johnson syn ro e ( life-thre tening r sh) occu rs in 0.1% of tients, n bec u se of this l otrigine u st be titr te slow ly. Other signific nt bu t less concerning verse events inclu e t xi , n u se , n se tion. N eu tro eni is otenti lly ngerou s si e effect of c rb ze ine, not l otrigine. 62. (B) H er l ck of rior su ici e tte ts is her ost signific nt rotective f ctor; lesser rotective f ctor w ou l be her soci l su orts. The other f ctors o not l y s u ch of role. 63. (D ) There re ny otenti l e ic l c u ses of e ression, inclu ing thyroi ysfunction, vit in B12 eficiency, ren l ise se, n v scu l r ise se. The other choices o not likely c u se e ressive illness. 64. (B) This tient is ost likely su ffering fro norexi nervos . Excessive ent l ec y is co on in tients w ith norexi or bu lii w ho urge by self-in u ce vo iting. Althou gh the other choices y be resent in severe st rv tion, excessive ent l ec y is the ost co on n e rs the e rliest. 65. (D ) Anorexi nervos h s v rie course, lthough oor rognosis over ll. Stu ies ocu ent th t only bout 50% of tients h ve goo outco e; 25% h ve oor ou tco e.

278

8: Pra c tic e Te s t 2

Unfortun tely, the nervos is roxi

ort lity r te of norexi tely 4%.

66. (C) This tient likely h s bor erline erson lity isor er. The first-line tre t ent for bor erline erson lity isor er re ins sychother y, such s DBT. DBT is w ell-stu ie v ri tion of CBT th t focu ses on suici l beh vior, consisting of in ivi u l sychothery, grou ther y, n ho ew ork ssignents. A v riety of e ic tions h ve been stu ie in bor erline erson lity isor er, inclu ing nti sychotics (e.g., ri i r zole), nti e ress nts (e.g., cit lo r ), n oo st bilizers (e.g., lithiu n v l roic ci ). H ow ever, w hile the liste e ic tions h ve e onstr te so e benefit in so e of the sy to s of bor erline erson lity isor er, in the bsence of nother sychi tric isor er (e.g., MDD, bi ol r isor er), sychother y is the inst y of tre t ent. 67. (E) N igrostri t l D 2 rece tors th t re blocke by nti sychotic e ic tions c use n i b lnce betw een o ine n cetylcholine, w hich ccounts for the rkinsoni n sy to s seen in this c se. Block e of cetylcholine rece tors c n hel tre t extr yr i l sy to s (EPS). D 4 or 5-H T2 rece tor blocke oes not c u se EPS. Mesoli bic D 2 rece tor block e is believe to ccou nt for the nti sychotic effect of nti sychotics n oes not c u se extr yr i l sy to s. Ty ic l nti sychotics block both nigrostri t l n esoli bic o ine, w here s ty ic l (secon -gener tion) nti sychotics referenti lly block esoli bic o ine, ccou nting for the over ll ecre se extr yr i l sy to li bility. 68. (D ) This tient is ost likely suffering fro Tou rette isor er. Aty ic l nti sychotic e ic tions (such s ris eri one) w hich block o ine rece tors re consi ere the ost effic ciou s t ecre sing voc l n otor tics. Cloni ine is benefici l in tre ting Tou rette’s, es eci lly in il c ses, lthough it y c u se excessive se tion n izziness. Flu oxetine n to ir te y be effective for co orbi con itions foun w ith Tourette isor er, but

h ve not been fou n to be s ecific lly hel fu l for this ri ry con ition. Althou gh ethyl heni te y hel this tient’s rent ttention- eficit hy er ctivity isor er sy to s, it y lso ex cerb te his tic isor er. 69. (B) Fire setting in re e te ttern in chilren often cco nies other beh viors th t re efi nt of ru les n u thority, s in conu ct isor er. Althou gh fire setting y occu r in chil ren i gnose w ith u tis s ectru isor er, intellectu l is bility, or tic isorer, it is u ch ore u nu su l. Chil ren w ith jor e ressive isor er o not eng ge in fire-setting s co on beh vior. 70. (A) This chil is suffering fro nocturn l enu resis. Utilizing bell/ bu zzer n evice ( for of beh vior l ther y) is the ost effective intervention, hel fu l in gre ter th n 50% of chil ren. Cognitive ther y, i lectic l beh vior l ther y ( for of cognitive-beh vior l ther y esigne for the tre t ent of bor erline erson lity isor er), n sycho yn ic ther y h ve not been foun to be effective in the tre t ent of enuresis. Pu nish ent is not ty e of beh vior l conitioning use to tre t sychi tric illnesses. 71. (C) While i i r ine w s consi ere the tre t ent of choice, bec use of its si e effects (e.g., c r iotoxicity) es o ressin is consi ere first-line h r cother y for enu resis. Althou gh rel se is high fter sto ing the e ic tion, roxi tely 30% of tients chieve tot l ryness, w ith nother 40% exhibiting re u ction in be w etting. C rb zeine is n nticonvu ls nt so eti es use s oo st bilizer, n cloni ine is n ntihy ertensive often use to tre t o i te w ith r w l n ADH D; neither re ro ri te for the tre t ent of enuresis. Di henhy r ine is n ntihist ine w ith nticholinergic ro erties, often use for inso ni or to tre t extr yr i l sy to s, n r rely use for enuresis. 72. (B) B se on his c e ic ifficu lties n IQ, this tient w ou l be cl ssifie s h ving il intellectu l is bility (intellectu l evelo ent l isor er). Intellectu l is bility is

Answe rs : 66–77

279

ch r cterize by eficits in intellectu l fu nctions confir e by clinic l ssess ent n st n r ize intelligence testing (e.g., IQ testing), s w ell s eficits in tive fu nctioning th t resu lt in f ilure to eet evelo ent l st n r s. While intellectu l is bilities re co on in tients w ith u tistic s ectru isor er, there is no history of restrictive, re etitive tterns of beh vior or interests consistent w ith u tis .

is lso r hili w hereby the in ivi u l s tisfies sexu l u rges by ru bbing g inst nonconsenting in ivi u ls. Pe o hilic isor er involves sexu l ctivity w ith re u bescent chil . The focu s of tr nsvestic isor er involves cross- ressing, n is u su lly seen in heterosexu l les. The r hili of observing u nsu s ecting eo le n ke or involve in sexu l ctivity is i gnose s voyeu ristic isor er.

73. (D ) Microce h ly, short l ebr l fissu res, fl t i f ce, n thin u er li re ll ssoci te w ith fet l lcohol syn ro e (FAS), w hich w ou l be su s ecte in this tient w ith tern l history of chronic, severe, lcohol u se. Cleft l te is not s ecific lly ssoci te w ith FAS. Congenit l blin ness is not ssoci te w ith FAS, lthou gh ethyl lcohol consu tion c n c use blin ness in u lts. H y erextensible joints n ro inent j w re ssoci te w ith fr gile X syn ro e.

76. (B) Des ite his recent i gnosis of jor e ressive isor er (MDD), this tient h s sy to s consistent w ith cu rrent nic e iso e (e.g., eu hori , ecre se nee for slee , i u lsivity, ressu re s eech, i irent in fu nctioning), n therefore w oul be i gnose w ith bi ol r isor er, current e iso e nic. Bi ol r isor er, cu rrent e iso e e resse w oul be i gnose in tient w ith history of ni bu t cu rrent jor e ressive e iso e. Criteri for cyclothy ic isor er re e ressive e iso es (w hich o not eet criteri l for MDD) ltern ting w ith hy o nic e iso es (sy to s of ni w ithou t i ir ent in fu nctioning, sychotic sy to s, or the nee for hos it liz tion). Persistent e ressive isor er ( ysthy i ) is chronic, e ressive illness, l sting for t le st 2 ye rs. Althou gh the tient h s nic sy to s n qu estion ble sychosis ( gr n iose elusion), there is no in ic tion th t he h s h erv sive sychotic sy to s in the bsence of his oo sy to s, w hich w ou l be consistent w ith schizo ffective isor er, bi ol r ty e.

74. (E) Chil ren w ith re ctive tt ch ent isor er is l y rke ly istu rbe n in ro ri te tt ch ent beh viors ssoci te w ith thologic l c re, su ch s seen in this c se. Althou gh ADH D n con u ct isorer re often co orbi in those chil ren w ho h ve histories of re ctive tt ch ent isorer, there is not enou gh evi ence to i gnosis either t this ti e. P tients w ith u tis s ectru isor er e onstr te ersistent i ir ent in reci roc l soci l co u nic tion (si il r to re ctive tt ch ent isor er) bu t lso h ve restricte , re etitive tterns of beh vior, interests, or ctivities, not evi ent in this c se. The criteri for l ngu ge isorer inclu e ifficu lties in the cqu isition n u se of l ngu ge u e to eficits, n o not inclu e bnor lities in e otion l co unic tion (e.g., is l ys of ggression tow r other chil ren) fou n in re ctive tt ch ent isor er. 75. (A) This tient w ou l ostly likely be i gnose w ith fetishistic isor er, r hili ch r cterize by the use of nonliving objects for sexu l gr tific tion. Frotteuristic isor er

77. (B) As this tient h s sy to s consistent w ith bi ol r isor er, ni , he w ou l ost benefit fro lithiu or nother oo st bilizer su ch s v l roic ci , or n ty ic l ntisychotic (e.g., ol nz ine). Anti e ress nts such s cit lo r , flu oxetine, n i i r ine c n ex cerb te ni n shou l be voi e . Lor ze , benzo i ze ine, y be u se to tre t sycho otor git tion n inso ni , bu t it oes not tre t the u n erlying isor er n h s risk of iction if u se in the long ter .

280

8: Pra c tic e Te s t 2

78. (C) A jor e ressive e iso e w ith ty ic l fe tu res is ch r cterize by oo re ctivity, rejection sensitivity, hy erso nolence, incre se etite, n “le en r lysis” (i.e., he vy feeling in extre ities). Mel ncholic fe tures inclu e signific nt nhe oni , e ression w orse in the orning, ter in l inso ni , rke sycho otor bnor lities, signific nt w eight loss, n in ro ri te guilt. To i gnose bi ol r I isor er there ust be t le st one nic e iso e. Bi ol r II isor er is ch r cterize by n e iso e of jor e ression n hy o nic e iso e. Persistent e ressive isor er ( ysthy i ) is chronic, often less-intense e ression, l sting 2 ye rs. 79. (C) Althou gh not co only u se bec u se of the risk of si e effects, MAOIs re consi ere to be the ost effective cl ss of ru gs for tre ting ty ic l e ression, es eci lly co re to TCAs. SSRIs re lso quite effective for tients w ith ty ic l fe tures. So e nticonvu ls nts n lithiu re u se for the tre t ent of bi ol r isor er, bu t not ty ic lly for uni ol r e ression. First-gener tion nti sychotics (e.g., h lo eri ol) re u se to tre t sychotic isor ers su ch s schizo hreni n schizo ffective isor er, s w ell s jor e ression w ith sychotic fe tu res, but they re not ro ri te s onother y for MDD. 80. (D ) This c se eliber tely brings together ny ch r cter tr its r w n fro “ u re” i gnoses of erson lity (ch r cter) isor ers, inclu ing bor erline, n rcissistic, e en ent, obsessive-co ulsive, n schizoty l erson lities. In clinic l r ctice, ny tients o not eet criteri for ny one s ecific erson lity isor er; su ch ixe c ses re referre to s other s ecifie erson lity isor er. P tients w ith i gnose bor erline erson lity isor er re know n for their intense erson l rel tionshi s, oo l bility, self- estru ctive beh viors, nger n ge ent issues, n oor sense of self-i ge. N rcissistic erson lity isor er is rke by gr n iose sense of self-w orth, entitle ent, l ck of re l e thy, n ten ency to use others for one’s ow n en s. P tients w ith obsessive-co ulsive

erson lity isor er re erfectionists, inflexible, n overly e n ing of the selves n others. Schizoty l tients re know n for their qu irkiness n see ingly gic l thinking styles. 81. (C) The essenti l fe tu res of schizo ffective isor er re ro inent oo e iso es ( jor e ression, ni , or ixe e iso e) concu rrent w ith sy to s consistent w ith schizohreni . In ition, the i gnosis requ ires erio of 2 w eeks of sychotic sy to s w ithou t ffective sy to s w hich ru les ou t bi ol r isor er or MDD w ith sychotic fe tu res. While tients w ith schizo hreni y ( n often o) beco e e resse , the oo sy to s re not resent for su bst nti l ortion of the illness. Uns ecifie sychotic isor er is u su lly reserve for situ tions in w hich the clinici n oes not h ve enou gh historic l t to know w hether the tient h s schizo hreni , oo isor er w ith sychotic fe tu res, su bst nce-in uce sychotic isor er, or sychotic isor er u e to nother e ic l con ition. 82. (D ) This w o n is su ffering fro eliriu , likely c u se by u lti le e ic tions. The el erly re rticu l rly vu lner ble to e ic tions know n to h ve sycho ctive ro erties or si e effects. This is in rt ue to the ecre se bility of geri tric tients to et bolize e ic tions n the ten ency of the rugs to bu il u to higher levels. A ition lly, el erly ersons re gener lly ore vu lner ble to the effects of e ic tions th t ct on the centr l nervou s syste (CN S). The tient in this c se is on nu ber of e ic tions th t y h ve CN S si e effects t high oses. H ow ever, the signific nt nticholinergic ro erties of i henhy r ine ut this tient t rticu l r risk of e ory i ir ent n confu sion. Ci eti ine n fu rose i e re lso otenti l contribu tors to eliriu , bu t re uch less likely th n i henhy r ine. Bet blockers like tenolol h ve been ssoci te w ith e ression, resu bly ssoci te w ith block e of centr l renergic rece tors. A ition lly, in so e vu lner ble tients, they y re u ce c r i c

Answe rs : 78–91

ou t u t enou gh to c u se function l hy o erfu sion of the br in c using confusion n izziness. Digoxin y ctu lly ecre se so iu ion con u ction enou gh in CN S neu rons to ro uce eliriu in so e tients. 83. (A) Deliriu rel te to block e of cholinergic rece tors is the ost co on c u se of e ic tion-in uce eliriu in the el erly. Acetylcholine is u niqu ely involve w ith e ory rocesses in the br in, n its involve ent in the tho hysiology of jor neu rocognitive isor er ( e enti ) ue to Alzhei er ise se h s been he vily rese rche . Dru gs th t lter o ine neu rotr ns ission re ost ssoci te w ith ove ent isor ers n sychosis-rel te si e effects. Me ic tions ffecting GABA rece tors (su ch s benzo i ze ines n b rbitu r tes) y c u se se tion n t xi s w ell s e ory istu rb nces. Dru gs cting on the serotonergic syste y lter slee , etite, n oo , lthou gh few non sychi tric ru gs w ork on this syste . Me ic tions ffecting centr l nore ine hrine tr ns ission (e.g., ntihy ertensives) y r rely be ssoci te w ith e ression. 84–89. [84 (K), 85 (C), 86 (D ), 87 (B), 88 (E), 89 (F)] P tients w ith ent l illnesses y be nonco li nt for v riou s re sons. Long- cting nti sychotics su ch s ris eri one Const (K) re goo choices in sychotic tients w ho benefit fro n toler te nti sychotics, bu t w ho re nonco li nt u e to l ck of insight, confu sion, or forgetfu lness (not u e to si e effects). Chlor ro zine (C) is ty ic l nti sychotic (neu role tic) w hich h s been ssoci te w ith sensitivity to su nlight, w hich c n c u se n ex gger te sunburn. Cloz ine (D ) w s the first “ ty ic l” nti sychotic evelo e ; it is rove for the tre t ent of tients w ith refr ctory schizo hreni n h s lso e onstr te effic cy in re u cing suici lity in schizo hreni or schizo ffective isor er. Bu ro ion (B) is n nti e ress nt th t is believe to block reu t ke of o ine in the br in. Althou gh ll nti e ress nt e ic tions h ve si il r effic cy, inclu ing nortri tyline (H) (tricyclic nti e ress nt), roxetine (I) n sertr line (L) (serotonin-

281

s ecific reu t ke inhibitors), n venl f xine (N ) (co bine serotonin-nore ine hrine reu t ke inhibitor), only bu ro ion h s re u ce risk of sexu l ysfu nction, n in f ct y be use to tre t nti e ress nt-in u ce sexu l ysfu nction. While no sychotro ic e ic tions re roven s fe in regn ncy, nti nic gents su ch s c rb ze ine, lithiu (G), n v l roic ci (M) re ssocite w ith signific nt fet l bnor lities n shou l be voi e . H lo eri ol (E) h s been u se s fely in regn nt tients w ith bi ol r isor er. L otrigine (F) is n ntie ile tic e ic tion in ic te for the inten nce tre t ent of bi ol r isor er (to el y the ti e to occurrence of oo e iso es) n for the cute tre t ent of bi ol r e ression. Ari i r zole (A) n ris eri one (J) re secon -gener tion nti sychotic e ic tions use for schizo hreni , schizo ffective isor er, n bi ol r isor er. 90. (E) Stu ies th t fin no st tistic l ifference betw een grou s w hen there ctu lly is ifference resu lt in ty e II error. In this c se, the ositive fin ings t the other u niversities ke the fin ing t u niversity A less likely to be correct. University A y h ve nee e l rger nu ber of su bjects in the stu y to fin the tre t ent effect. Ty e I errors occu r ore often th n ty e II errors n c u se the rese rcher to conclu e th t there is ifference betw een grou s w hen in f ct there is none. The v ri nce is the su of the ifferences of e ch t oint fro the e n. Positive re ictive v lu e is the ro ortion of bnor l test resu lts th t re tru e ositive, w hile neg tive re ictive v lu e is the ro ortion of nor l test resu lts th t re true neg tive. St n r error is the egree to w hich the e ns of sever l ifferent s les w oul v ry if they w ere t ken re e te ly fro the s e o u l tion. 91. (E) This tient h s signs ost consistent w ith t r ive yskinesi , n extr yr i l synro e th t usu lly ffects erior l or li b u scu l tu re n c u ses choreifor oveents. Its onset is u su lly sever l ye rs fter t king nti sychotics n it is ore likely to

282

8: Pra c tic e Te s t 2

ffect ol er tients. Ak thisi is escribe s sycho otor restlessness th t y h ve n onset of ys to w eeks fter beginning n nti sychotic. Dystoni is n cu te re ction to nti sychotics (neu role tics) in w hich rticu l r uscle grou s (neck or ocu l r uscles co only) contr ct involu nt rily. It c n be infu l n shou l be tre te i e i tely w ith nticholinergic e ic tions. N MS is otenti lly leth l bu t r re e ic l e ergency in w hich tients y h ve glob l rigi ity, ent l st tu s ch nges, fever, c r iov scu l r inst bility, elev te cre tine hos hokin ses, n risk of rh b o yolysis. P rkinsonis y look i entic l to P rkinson ise se (tre or, br ykinesi , ske f ces) n usu lly h s n onset w ithin ys to w eeks fter beginning n nti sychotic e ic tion. 92. (A) The b s l g ngli , res onsible for the tr ns ission of thought to ction n for controlling the initi tion n qu lity of otor ction, is centr l to the tho hysiology of extr yr i l syn ro es, inclu ing ystoni , rkinsonis , k thisi , n t r ive yskinesi . The cerebellu is i ort nt in controlling the coor in tion of otor oveents n ostu re. The front l cortex is gener lly consi ere to be i ort nt for ecision king, i u lse control, n ffect regu l tion. The i br in cont ins nuclei th t hel to ensu re centr l nervou s syste ho eost sis by regu l ting neu roveget tive, u tono ic, n rou s l functions. The otor cortex serves s the l st st ge of cerebr l rocessing of otor infor tion before it escen s into the s in l cor . An int ct otor cortex is requ ire for initi tion of ove ent. 93. (D ) This tient is suffering fro chronic osttr u tic stress isor er (PTSD) sy to s, only rti lly tre te es ite co li nce w ith e i tions n bstinence. Serotonins ecific reu t ke inhibitors (SSRIs) re consi ere first-line tre t ent bec u se of their effic cy n toler bility. Pr zosin, n l h -1 renergic rece tor blocker h s e onstr te signific nt effic cy in tre ting ny of the sy to s of PTSD (e.g., night res, fl shb cks, hy er rou s l, voi nce). Bu ro ion

is n nti e ress nt u se to tre t jor e ressive isor er, but its ctiv ting ro erties ke it not s u sefu l for the tre t ent of PTSD. Anti sychotics like h lo eri ol shou l only be use in severe c ses of PTSD w ith ro inent sychotic sy to s or git tion/ ggression. Lor ze n other benzo i zeines shou l be voi e in tients w ith PTSD ue to their l ck of effic cy n otenti l for iction. Moo st bilizers su ch s v l roic ci y be benefici l for so e tients w ith PTSD, bu t re not likely s effic ciou s s SSRIs or l h -blockers like r zosin. 94. (D) Erik Erikson w s sycho n lyst best known for his escri tion of eight st ges of hu n sychologic l ex erience s nning the entire life s n. He believe th t successful co letion of ll e rlier st ges w s necess ry for successful co letion of future st ges. The tient in this ex le is ble to ro uctively work but h s ifficulty with inti cy in his rel tionshi , hence the conflict between inticy n isol tion. All of Erikson’s eight st ges center on st gero ri te evelo ent l conflicts: b sic trust versus istrust (birth to 1 ye r); utono y versus sh e n oubt (1–3 ye rs); initi tive versus guilt (3–5 ye rs); in ustry versus inferiority (6–11 ye rs); i entity versus role iffusion (11 ye rs to en of olescence); inti cy versus isol tion (21–40 ye rs); gener tivity versus st gn tion (40–65 ye rs); n integrity versus es ir (65 n ol er). 95. (B) H y erreligiou s thinking or reoccu tion w ith or l beh vior, ltere sexu l beh viors, hy ergr hi or over el bor tive co unic tion styles ( lso referre to s viscosity), n heightene ex erience of e otions for cl ssic constell tion of erson lity tr its ssoci te w ith co lex rti l e ile sy (or te or l lobe seizu res). De ression, obsessive-co u lsive sy to s, n slee isorers y be ssoci te w ith structur l br in inju ry in front l n su bcortic l re s, bu t they h ve not been escribe s being rel te s ecific lly to co lex rti l e ile sy. Urin ry incontinence is ore often ssoci te w ith nor l ressu re hy roce h lu s or jor neu rocognitive isor er ( e enti ).

Answe rs : 92–102

96. (E) This tient ost likely w oul e onstr te seizure ctivity in his te or l lobes, consistent w ith te or l lobe e ile sy. 97. (B) The Gl sgow Co Sc le e su res level of rou s l n r nges fro sc le of 3 ( ee co ) to 14 (fu lly lert). The c tegories ssesse re eye o ening, best otor res onse, n best verb l res onse. Eye o ening r nges fro score of 4 (o ening s ont neou sly) to 1 (not o ening t ll). In this c se, the eyes o en to in, so score of 2 is given. Best otor res onse is s follow s: obeys co n s, 5; loc lizes in, 4; flexion, 3; extension, 2; n no res onse, 1. In this c se, score of 3 is given for the res onse. Best verb l res onse is lso b se on sc le r nging fro 5 to 1. An oriente tient receives score of 5 n no res onse receives score of 1, s in this c se. The scores re then e , n in this c se re 2 + 3 + 1 = 6. 98. (E) When ch nging fro n SSRI to n MAOI, one u st llow w shout erio of bout five h lf-lives for the SSRI to be ore th n 90% eli in te ; in the c se of flu oxetine, this is roxi tely 5 w eeks. This rec u tion voi s the serotonin syn ro e, w hich y occur w hen signific nt bloo levels of both ru gs re resent. Serotonin syn ro e, w hich c n be f t l, is rke by sy to s of tre or, i horesis, rigi ity, yoclonu s, u tono ic ysregu l tion, hy erther i , rh b o yolysis, ren l f ilu re, n co . The other choices w ou l ose signific nt risk of evelo ing serotonin syn ro e. 99. (C) MAOIs revent the eri her l bre k ow n of tyr ine, w hich y be ingeste w ith cert in foo s su ch s liver, beer, re w ine, ge cheeses, s oke fish, ry s u s ge, n f v be ns. If too u ch tyr ine bu il s u in the bloo , it y s tu r te eri her l sy thetic nerve rece tors n c use hy ertensive crisis, otenti lly f t l con ition. The other choices o not contribu te to hy ertensive crisis. 100. (D ) Thiori zine is ty ic l nti sychotic, r rely u se now u e to the risk of c r i c rrhyth i . At oses of 1,000 g/ or higher,

283

thiori zine h s been ssoci te w ith ig ent ry retino thy, lso know n s retinitis igentos . Therefore, oses shoul not excee 800 g/ . Pig ent ry retino thy c n c use loss of retin l res onse to contr ction of the visu l fiel . An e rly sign y be noctu rn l confu sion. Consti tion, ry eyes, n urin ry retention re ll si e effects fro the nticholinergic ro erties of thiori zine, bu t ty ic lly re not so severe s to co ro ise he lth er nently. N e hrogenic i betes insi i u s is not ssoci te w ith thiori zine. 101. (B) This tient is ex eriencing n cu te ystonic re ction fro the h lo eri ol, high- otency first-gener tion (ty ic l) ntisychotic, ty ic lly seen hours to ys fter initi ting or incre sing the nti sychotic. Acu te ystonic re ctions y resent s u scle s s s, torticollis, oculogyric crisis, n l rynge l s s . Benztro ine 1 to 2 g IM is u seful in the tre t ent of cu te ystonic re ctions. Altern tively, i henhy r ine 50 g IM c n lso be u se . If the sy to s o not resolve w ithin 20 inu tes, l rger oses c n be given. Benzo i ze ines c n lso be u se bu t re not first-line tre t ent. For cu te l rynge l ystoni , l rger oses of benztro ine shou l be given i e i tely, follow e by lor ze IV if nee e . N one of the other choices re in ic te for the tre t ent of cute ystoni . While n lgesics su ch s cet inohen n ibu rofen y hel w ith in, they w on’t resolve the u n erlying etiology. Cyclobenz rine is u scle rel xer w hich w ill not equ tely tre t the ystoni . Pro r nolol is B-blocker, u se for hy ertension, bu t it c n lso be use to tre t k thisi , nother si e effect of nti sychotics, ch r cterize by n inner feeling of restlessness n nee to ove. 102. (C) The isor er escribe is ne hrogenic i betes insi i u s, ost likely c use by lithiu . Lithiu inhibits the effect of nti iu retic horone on the ki ney, c using olyuri , thirst, oly i si , n otenti l hy ern tre i n eliriu . Although c rb ze ine n v lroic ci (other oo st bilizers), h lo eri ol ( first-gener tion nti sychotic), n queti ine ( secon -gener tion nti sychotic) re ll

284

8: Pra c tic e Te s t 2

use to tre t bi ol r isor er, none re ssocite w ith i betes insi i us. 103. (B) Cloni ine, centr l l h -2- utorece tor gonist, h s been roven useful in the tre t ent of the utono ic hy er ctivity ssoci te with o ioi with r w l. A nt ine is use to tre t P rkinson ise se n lso s n nti-influenz gent. H lo eri ol, n nti sychotic, n lor ze , benzo i ze ine, y hel with signific nt git tion occ sion lly seen in with r w l, but they will not tre t the un erlying sy to s. N loxone, n o i te nt gonist, is use in o i te over ose n will signific ntly worsen o i te with r w l sy to s in this tient. 104. (D ) Meth one h s been roven to signific ntly re uce the use of heroin w hen use s inten nce e ic tion. P tients int ine on oses low er th n 40 g/ of eth one re f r ore likely to rel se th n those on higher oses. D ntrolene is uscle-rel x nt u se in the n ge ent of neu role tic lign nt syn ro e, r re si e effect of nti sychotic tre t ent. Disulfir is e ic tion u se to revent lcohol consu tion by c using n versive re ction if t ken in conju nction w ith lcohol. Flu zenil is in ic te for benzo i ze ine over ose. Sertr line is serotonin-s ecific reu t ke inhibitor, u se in the tre t ent of e ressive n nxiety isorers, ong other sychi tric illnesses. While it y be hel fu l for co orbi e ression or nxiety, it h s not been show n to ecre se o i te u se in in ivi u ls. 105. (B) I i r ine is tricyclic nti e ress nt (TCA). TCAs re consi ere cl ss 1 A nti rrhyth ics bec use they ossess quini ine-like effects th t ecre se con uction ti e through the bun le of His. They h ve been show n to incre se ort lity in c r i c tients. TCAs c n lso incre se the he rt r te, n so tient w ith co ro ise c r i c function y suffer fro incre se oxygen e n . Fin lly, TCAs re lso ssoci te w ith signific nt orthost tic hy otension, w hich y be further ex cerb te in tients w ith c r i c ise se. Wellbutrin, serotonin-nore ine hrine reu t ke

inhibitors (such s irt z ine n venl f xine), n serotonin-s ecific reu t ke inhibitors (SSRIs) like sertr line y ll be s fely use in tients w ith c r i c ise se; how ever, ttention shoul be i to otenti l hy ertension c use or w orsene by venl f xine n ny c r i c e icines et bolize through the P-450 cytochro e syste , s so e SSRIs (e.g., fluoxetine) y lter the e ic tion levels. 106. (D ) Acetylcholine is the neu rotr ns itter ost i lic te in cognitive fu nctioning. Anticholinergic effects of e ic tions re frequ ently i lic te in cognitive ecline n ru g-in u ce eliriu . Of the e ic tions liste , roxetine h s the ost nticholinergic effect, n therefore shou l be voi e in this tient w ith jor neu rocognitive isor er ( e enti ). 107. (A) Lithiu toxicity is ch r cterize e rly on by ys rthri , t xi , co rse tre or, n b o in l in. L ter nifest tions inclu e seizures, neuro uscul r irrit bility, n i ire consciousness ( eliriu to co ). Acute ystoni s re ssoci te w ith the use of ty ic l nti sychotics. Leg in n resthesi s re not ssoci te w ith lithiu . P r noi elusions re rt of the sy to rofile of schizo hreni n so eti es of bi ol r isor er. 108. (B) For ECT to be effective, the seizu re shou l l st t le st 15 secon s. Prolonge seizu res gre ter th n 180 secon s shou l be ter in te w ith n nesthetic (e.g., ro ofol) or intr venou s benzo i ze ines (e.g., i ze ). 109. (D ) Me ory i ir ent ( ostly u ring the tre t ent) is quite co on, lthou gh ost tients retu rn to their b seline by 6 onths. There is no in ic tion th t ECT c u ses br in ge. ECT y ctu lly be use to tre t tients w ith c t toni . Fr ctu res n vo iting re u nco only seen in ECT since the u se of u scle rel x nts n ntie etics h ve been i le ente . 110. (C) For extre e git tion, the benzo i ze ine lor ze c n be effective. Another ltern tive

Answe rs : 103–116

is the butyro henone nti sychotic h lo eri ol. Phenothi zine nti sychotics su ch s chlorro zine c n c u se u tono ic inst bility w hen given to tient w ith PCP intoxic tion. Tr zo one, use co only for inso ni , n fluoxetine, n SSRI, re nti e ress nts, n w ill not hel w ith git tion. Trihexy heni yl is use to co b t extr yr i l sy to s ssoci te w ith nti sychotic use n not for git te st tes. 111. (C) Su ortive c re is the best tre t ent for PCP intoxic tion th t is not co lic te by extre e git tion or violence. Lor ze , benzo i ze ine, or h lo eri ol, ty ic l nti sychotic, c n be u sefu l for extre e git tion. Aci ific tion of the u rine oesn’t signific ntly incre se the excretion of PCP. Des ite u rb n yths, neither cheese nor vit ins h ve rticu l r ther eu tic benefits for PCP intoxic tion. 112. (A) This tient is ost likely su ffering fro jor e ression w ith sychotic fe tu res. N either nti e ress nts nor nti sychotics lone w ill s r i ly or effectively tre t jor e ression w ith sychotic fe tu res s the co bin tion. The co bin tion of lor ze n cloz ine ( n ty ic l nti sychotic) w ill not ro ri tely tre t the e ression. Si il rly, the co bin tion of sertr line n lor ze w ill not equ tely tre t the sychotic sy to s. Lith iu n synthroi c n both effectively u g en t nti e ress nts su ch s roxetine n sertr line. Lor ze , benzo i ze ine, y hel w ith nxiety, bu t w ill not be effective g inst sychosis or e ression. 113. (L) PCP ( k “ ngel u st”) intoxic tion rouces git tion, vertic l nyst g us, n n lgesi s w ell s hy erther i , e ersonliz tion, n h llu cin tions in u itory, t ctile, n visu l o lities. 114. (B) Alcohol w ith r w l y begin w ithin 24 hou rs fter the l st rink, n c n rogress to seizures by 24 to 48 hou rs n eliriu tre ens (DTs) fter roxi tely 72 hou rs.

DTs re ssoci te ort lity.

w ith

285

signific nt risk of

115. (E) Sy to s ssoci te w ith coc ine w ithr w l ten to be il n l st sever l ys fter the l st u se. Occ sion lly, how ever, the e ression c n be severe enough to r ise the risk of self-inju rious beh vior or su ici e tte ts. 116. (G) H eroin w ith r w l e ks t bout 72 hou rs fter the l st u se in chronic u sers. It is ex erience s v riou s flu -like sy to s n g strointestin l co l ints, su ch s fever n chills, ru nny nose, n u se , bo y ches, i rrhe , n b o in l cr s. The e r nce of iloerection in the syn ro e g ve rise to the sl ng ter “col tu rkey” to escribe the ction of tot l cess tion of u se of the ru g. Alcohol intoxic tion (A) involves ch r cteristic beh vior l ch nges, s w ell s slu rre s eech, t xi , n other neurologic fin ings. It c n rogress to co if severe. C nn bis intoxic tion (C) heightens sensitivity to extern l sti u li n i irs otor skills, long w ith t chyc r i , conju nctiv l injection, n incre se etite (“the u nchies”). Coc ine intoxic tion (D ) is rke by el te oo , ecre se slee , h llu cin tions, n git tion. H eroin intoxic tion (F) is rke by n ltere oo , sycho otor ret r tion, n row siness, long w ith in oint u ils n constition. Intoxic tion by inh l nts (H) (vol tile subst nces su ch s g soline fu es) is rke by eu hori , isorient tion, fe r, n f ci l r sh, lthou gh the sy to s re short-live . The syn ro es of eth het ine (I) n MDMA (N ) intoxic tion re si il r to those of coc ine intoxic tion. MDMA u sers ty ic lly re ort n incre se “sense of closeness” w ith other eo le. N icotine w ith r w l (J) is ch r cterize by e resse oo , inso ni , irrit bility, ifficu lty concentr ting, ecre se he rt r te, n incre se etite. N itrou s oxi e intoxic tion (K) ro u ces eu hori n light-he e ness, n u su lly su bsi es w ithin hou rs w ithou t tre t ent. With r w l fro silocybin (M), h llu cinogen erive fro u shroo s, is not w ell escribe .

286

8: Pra c tic e Te s t 2

117. (B) Cloz ine ( n ty ic l nti sychotic) c uses gr nu locytosis in bou t 1% of tients. For this re son, w eekly bloo cou nts re e su re for the first 6 onths of tre t ent, every 2 w eeks for the next 6 onths, then every 4 w eeks there fter (u nless there is signific nt ro in w hite bloo cell cou nts).

118. (F) Lithiu is ssoci te w ith clinic l hy othyroi is in t le st 5% of tients, signific ntly ore often in w o en. Thyroi su le ent tion y be e to cou nter this si e effect. A roxi tely 25% to 30% of tients y show elev te thyroi -sti u l ting hor one levels.

Biblio g raphy

Adelson SL; American Academy of Ch ild and Adolescent Psych iatry (AACAP) Com m ittee on Qu ality Issues (CQI). Practice parameter on gay, lesbian, or bisexu al sexu al orientation, gend er noncon form ity, and gender d iscord ance in ch ild ren and adolescents. J A m A cad Child A dolesc Psychiatry. 2012;51(9):957–974. AMA’s Code of Med ical Ethics. Opinions on Confidentiality, A dvertising, and Communications M edia Relations. The American Med ical Association. Available at http:// w w w.am a-assn.org/ am a/ pub/ physician-resou rces/ med ical-ethics/ code-med ical-ethics/ opinion504.page? Accessed March 1, 2016. AMA’s Code of Med ical Ethics. Opinions on Confidentiality, A dvertising, and Communications M edia Relations. The American Med ical Association. Available at http:// w w w.am a-assn.org/ am a/ pub/ physician-resou rces/ med ical-ethics/ code-med ical-ethics/ opinion505.page? Accessed February 28, 2016. AMA’s Code of Med ical Ethics. Opinions on Confidentiality, A dvertising, and Communications M edia Relations. The American Med ical Association. Available at http:// w w w.am a-assn.org/ am a/ pub/ physician-resou rces/ med ical-ethics/ code-med ical-ethics/ opinion505.page? Accessed March 2, 2016. AMA’s Code of Med ical Ethics. Opinions on Confidentiality, A dvertising, and Communications M edia Relations. The American Med ical Association. Available at http:// w w w.am a-assn.org/ am a/ pub/ physician-resou rces/ med ical-ethics/ code-med ical-ethics/ opinion505.page? Accessed March 8, 2016 AMA’s Code of Med ical Ethics. Opinions on Confidentiality, A dvertising, and Communications M edia Relations. The American Med ical Association. Available at http:// w w w.am a-assn.org/ am a/ pub/ physician-resou rces/ med ical-ethics/ code-med ical-ethics/ opinion5055.page? Accessed February 26, 2016. AMA’s Code of Med ical Eth ics. Opinions on Practice M atters. The American Med ical Association. Available at http:/ / w w w.am a-assn.org/ am a/ pu b/ physician-resou rces/ m ed ical-eth ics/ cod e-m ed ical-eth ics.p age? Accessed Febru ary 14, 2016. AMA’s Code of Med ical Eth ics. Opinions on Practice M atters. The American Med ical Association. Available at http:// w w w.am a-assn.org/ am a/ pu b/ physician-resou rces/ med ical-eth ics/ cod e-med ical-eth ics/ opin ion 8115.page. Accessed March 13, 2016. AMA’s Code of Medical Ethics. Opinions on Practice M atters. The American Med ical Association. Available at http:// w w w.ama-assn.org/ ama/ pub/ physician-resou rces/ med ical-ethics/ code-med ical-ethics/ opinion808. page. Accessed February 29, 2016. AMA’s Code of Medical Ethics. Opinions on Practice M atters. The American Med ical Association. Available at http:// w w w.ama-assn.org/ ama/ pub/ physician-resou rces/ med ical-ethics/ code-med ical-ethics/ opinion808. page. Accessed March 12, 2016. AMA’s Code of Med ical Eth ics. Opinions on Practice M atters. The American Med ical Association. Available at http:// w w w.am a-assn.org/ am a/ pu b/ physician-resou rces/ med ical-eth ics/ code-med ical-eth ics/ opin ion808. page? Accessed Febru ary 28, 2016. AMA’s Code of Medical Ethics. Opinions on Practice M atters. The American Medical Association. Available at http:/ / w w w.am a-assn.org/ am a/ pub/ physician-resou rces/ med ical-eth ics/ code-med ical-eth ics/ opin ion8081. page. Accessed March 2, 2016. AMA’s Code of Med ical Ethics. Opinions on Practice M atters. The American Medical Association. Available at http:// w w w.ama-assn.org/ ama/ pub/ physician-resou rces/ med ical-ethics/ code-med ical-ethics/ opinion8121. page. Accessed February 29, 2016.

287

288

Bib liog ra p hy

AMA’s Code of Medical Ethics. Opinions on Practice M atters. The American Med ical Association. Available at http:// w w w.am a-assn.org/ ama/ pub/ physician-resou rces/ med ical-ethics/ code-med ical-ethics/ opinion814. page. Accessed March 7, 2016. AMA’s Code of Medical Ethics. Opinions on Practice M atters. The American Med ical Association. Available at http:// w w w.ama-assn.org/ ama/ pub/ physician-resou rces/ med ical-ethics/ code-med ical-ethics/ opinion9031. page. Accessed March 13, 2016. AMA’s Code of Med ical Eth ics. Opinions on Professional Rights and Responsibilities. The American Med ical Association. Available at http:// w w w.am a-assn.org/ am a/ pu b/ physician-resou rces/ med ical-eth ics/ codemed ical-eth ics/ opin ion905.page. Accessed March 12, 2016. AMA’s Code of Med ical Eth ics. Opinions on Professional Rights and Responsibilities. The American Med ical Association. Available at http:// w w w.am a-assn.org/ am a/ pu b/ physician-resou rces/ med ical-eth ics/ codemed ical-eth ics/ opin ion907.page. Accessed March 12, 2016. AMA’s Code of Med ical Eth ics. Opinions on Professional Rights and Responsibilities. The American Med ical Association. Available at http:// w w w.am a-assn.org/ am a/ pu b/ physician-resou rces/ med ical-eth ics/ codemed ical-eth ics/ opin ion9141.page. Accessed March 8, 2016. AMA’s Code of Med ical Eth ics. Opinions on Social Policy Issues. The American Med ical Association. Available at http:// w w w.am a-assn.org/ am a/ pu b/ physician-resou rces/ m ed ical-eth ics/ cod e-med ical-eth ics/ opin ion 223.page? Accessed Febru ary 26, 2016. AMA’s Code of Medical Ethics. Opinions on Social Policy Issues. The American Med ical Association. Available at http:// w w w.ama-assn.org/ ama/ pub/ physician-resou rces/ med ical-ethics/ code-med ical-ethics/ opinion2211. page? Accessed March 8, 2016. AMA’s Code of Med ical Ethics. Opinions on Social Policy. The American Med ical Association. Available at http:// w w w.am a-assn.org/ am a/ pu b/ physician-resou rces/ m ed ical-eth ics/ cod e-med ical-eth ics/ opin ion 2215.page? Accessed March 1, 2016. American Psych iatric Association. A PA Commentary on Ethics in Practice. Topic 3.2.6 Therap eutic Bou nd ary Keeping. 2015:6. American Psych iatric Association. Diagnostic and Statistical M anual of M ental Disorders. 5th ed. (DSM -5). 2013: 186–187. The American Psychiatric Association. Principles of Medical Ethics with Annotations Especially Applicable to Psychiatry. 2013:6 Axelson DA, Birm aher B. Relation between an xiety and d epressive d isorders in ch ild hood and adolescence. Depress A nxiety. 2001;14(2):67–78. Barsky AJ, Orav EJ, Bates DW. Som atization increases med ical utilization and costs independent of p sych iatric and med ical comorbid ity. A rch Gen Psychiatry. 2005;62(8):903–910. Bobes J, Rejas J, Garcia-Garcia M, et al; EIRE Study Group. Weight gain in patients w ith sch izoph ren ia treated w ith risperidone, olan zapine, quetiapine or haloperidol: resu lts of the EIRE study. Schizophr Res. 2003;62(1-2):77–88. Boud en A, Krebs MO, Loo H, Olie JP. Mu nchau sen synd rome by proxy: a challenge for med icine. Presse M ed. 1996;25:567–569. Garcia-Campayo J, Ald a M, Sobrad iel N, Olivan B, Pascu al A. Personality d isorders in som atization d isorder patients: a controlled study in Spain. J Psychosom Res. 2007;62:675–680. Gartleh ner G, H ansen RA, Th ied a P, et al. Comparative Effectiveness of Second-Generation A ntidepressants in the Pharmacologic Treatment of A dult Depression. Comparative Effectiveness Review No. 7. (Prepared by RTI International-Un iversity of North Carolina Evid ence-based Practice Center u nd er Contract No. 290-020016.) Rockville, MD: Agency for Healthcare Research and Qu ality. Accessed Janu ary, 2007. Habeas Corpu s. Legal Information Institute. Cornell University Law School. Available at https:// w w w.law.cornell. edu/ wex/ habeas_corpu s. Accessed March 12, 2016 Herxheimer A, Petrie KJ. Melaton in for the prevention and treatment of jet lag. Cochrane Database Syst Rev. 2002;(2):CD001520.

Bib liog ra p hy

289

Kellogg N; American Academy of Ped iatrics Com m ittee on Ch ild Abu se and Neglect. The evalu ation of sexu al abu se in child ren. Pediatrics. 2005;116(2):506–512. Kieler H, Artam a M, Engeland A, et al. Selective seroton in reuptake in hibitors du ring pregnancy and risk of persistent pu lmonary hyp erten sion in the new born: p opu lation based cohort study from the five Nord ic cou ntries. BM J. 2012;344:d8012. Lam b EJ, Steven s PE, Nashef L. Topiram ate increases biochem ical risk of neph rolith iasis. A nn Clin Biochem. 2004;41(Pt 2):166–169. Lee T, Fou ras G, Brow n R; American Academy of Ch ild and Adolescent Psych iatry (AACAP) Com m ittee on Qu ality Issues (CQI). Practice parameter for the assessment and m anagement of youth involved w ith the ch ild welfare system. J A m A cad Child A dolesc Psychiatry. 2015;54(6):502–517. Available at http:// w w w.jaacap. com/ article/ S0890-8567(15)00148-3/ pd f Lock J, La Via MC; American Academy of Ch ild and Adolescent Psych iatry (AACAP) Com m ittee on Qu ality Issues (CQI). Practice parameter for the assessment and treatment of child ren and adolescents w ith eating d isorders. J A m A cad Child A dolesc Psychiatry. 2015;54(5):412–425. Available at http:// w w w.jaacap.com/ article/ S0890-8567(15)00070-2/ p d f McClellan J, Stock S; American Academy of Child and Adolescent Psychiatry (AACAP) Com m ittee on Quality Issues (CQI). Practice parameter for the assessment and treatment of child ren and adolescents w ith schizophrenia. J Am Acad Child A dolesc Psychiatry. 2013;52(9):976–990. Available at http:// w w w.jaacap.com/ article/ S0890-8567(13)00112-3/ pd f Mu rphy TK, Lew in AB, Storch EA, Stock S; American Academy of Ch ild and Adolescent Psych iatry (AACAP) Com m ittee on Qu ality Issues (CQI). Practice parameter for the assessment and treatment of ch ild ren and adolescents w ith tic d isorders. J A m A cad Child A dolesc Psychiatry. 2013;52(12):1341–1359. Available at http:// w w w.jaacap.com/ article/ S0890-8567(13)00695-3/ pd f Mu rray J, Farrington DP. Risk factors for conduct d isorder and d elinquency: Key find ings from longitud inal stud ies. Can J Psychiatry. 2010;55(10): 633–642. Neveu s T, Eggert P, Evan s J, et al; International Ch ild ren’s Continence Society. Evalu ation of and treatment for monosymptom atic enu resis: a stand ard ization docu ment from the International Ch ild ren’s Continence Society. J Urol. 2010;183(2):441–447. Nicole B, Brahm NC, Kissack J. Which psychotropics carry the greatest risk of QTc prolongation? Curr Psychiatr. 2012;11(10):36–39. Available at http://www.currentpsychiatry.com/home/article/which-psychotropics-carry-the-greatest-risk-of-qtc-prolongation/60d07ce6dd7c885cae85d bb19b9dd3c4.html Olatu nji BO, Mim iaga MJ, O’Cleirigh C, Safren SA. A review of treatment stud ies of depression in H IV. Top HIV M ed. 2006;14(3):112–124. Rösner S, H ackl-Herrwerth A, Leucht S, Vecch i S, Srisu rapanont M, Soyka M. Opioid antagon ists for alcohol dep endence. Cochrane Database Syst Rev. 2010;(12):CD001867. Sadock BJ, Sadock VA, Ru iz P. Kaplan & Sadock’s Synopsis of Psychiatry: Behavioral Sciences/Clinical Psychiatry. 11th ed. Lippincott William s & Wilkin s; 2014. Serretti A, Ch iesa AJ. Treatment-emergent sexu al dysfu nction related to antidepressants: a meta-analysis. Clin Psychopharmacol. 2009;29(3):259–266. Singh T, William s K. Atypical Depression. Psychiatry (Edgmont). 2006;3(4);33–39. Stein hau sen HC. The outcome of anorexia nervosa in the 20th centu ry. A m J Psychiatry. 2002;159(8):1284–1293. Stone J, Smyth R, Carson A, et al. System atic review of m isd iagnosis of conversion symptom s and “hysteria.” BM J. 2005;331(7523):989. Tish ler CL, Reiss NS, Rhodes AR. Su icid al behavior in ch ild ren you nger than twelve: A d iagnostic challenge for emergency department person nel. A cad Emerg M ed. 2007;14(9):810–818.

This page intentionally left blank

Inde x

A Abnorm al Involu ntary Movem ent Scale (AIMS), 248, 269 Acetylcholine, 262, 265, 281, 284 Acetylcysteine, 70 Acu te stress d isord er, 42, 66 d ifferential d iagnosis, 158, 168, 187–188, 213, 233 Ad d erall, 24 ADH D. See Attention-d eficit/ hyp eractivity d isord er Ad ju stm ent d isord er, 39, 63 w ith d epressed m ood , 173, 192–193 d iabetes m ellitu s w ith, 6, 22 MDD d istinguished , 6, 22 treatm ent, 173, 193 Ad olescents antisocial personality d isord er, 13, 28 bip olar d isord er in, 3, 18 bulim ia nervosa, 20 d ep ression in, 1, 17 MDD in, 6, 18, 20 panic d isord er, 5, 19, 22, 27, 29 su icid e risk in, 1, 17 Ad ult psychop athology, 33–58 Ad vance d irective, 147, 159 Agitation, 266, 284–285 Agnosia, 28 Agorap hobia, panic d isord er w ith d ifferential d iagnosis of, 173–174, 193 treatm ent of, 174, 193 Agranu locytosis, 109, 113, 116, 120, 209, 228 AIMS. See Abnorm al Involu ntary Movem ent Scale Akathisia antip sychotics and , 104 risperid one and , 98, 118 Alcohol abu se d ep ression and , 179, 199 d isulfiram and , 221, 242 Alcohol d ep end ence, 66 Alcohol intoxication, 70, 79, 266, 285 Alcohol u se d isord er, 39, 63 Alcoholism evalu ation, 178–179, 198 m ed ications, 97, 118 thiam ine d eficiency and , 112, 116 Alcohol u se d isord er elevated / d epressed liver enzym es and , 206, 226 m ed ications for, 207, 226 Alcohol w ithd raw al, 267, 285 elevated / d epressed liver enzym es and , 206, 226 hallucinations d u e to, 221, 242 lorazepam for, 189, 199, 221, 241–242

Alcohol w ithd raw al (Cont.): seizu res and , 170, 189 treatm ent, 170, 189 Alexia, 13, 28 Alp ha-blocker, 208, 228 Alzheim er d isease d ifferential d iagnosis, 178, 197 treatm ent, 253–254, 274 AMA. See Am erican Med ical Association AMA Cod e of Med ical Ethics, 158, 159–160, 162–163 Am antad ine (Sym m etrel), 212, 232 Am erican Law Institute test (Mod el Penal Cod e), 142, 155–156. See also Legal issu es Am erican Med ical Association (AMA), 157 Am itrip tyline card iac effects of, 93, 114 m ania and , 50, 73 Am nesia, d issociative, 183, 202 anterograd e, 62 ECT and , 67 w ith fu gue, 213–214, 234 localized , 73 Am ok, 211, 231 Am phetam ine, 56, 79 Anom ia, 28 Anorexia nervosa com p lications, 258, 277 d ifferential d iagnosis, 175, 195 m ed ical com p lications, 219, 239 m ortality rate, 219, 240 p rognosis, 258, 277–278 Anterograd e am nesia, 62 Anticholinergic toxicity, 211, 232 Anticholinesterase, 220, 241 Antidepressants. See also specific antidepressants OCD and , 220, 240 Antim otivational synd rom e, 100 Antip sychotic m ed ications akathisia and , 104–105 bipolar d isord er, 167, 187 d opam ine recep tors and , 114 N MS and , 105 schizotypal p ersonality d isord er, 206, 226 sid e effects, 51, 75, 92, 113 TD and , 106 Antisocial p ersonality d isord er, 183, 203 ad olescent, 22, 28, 29, 203, 229 cond u ct d isord er and , 20, 22, 24, 28 Anxiety brain activity and , 55 p erform ance, 118 Anxiety d isord ers MDD and , 5, 20 Aphasia, 28 Apraxia, 28

Argyll–Robertson p u p il, 196 Arip ip razole, 262, 281 Arnold -Chiari synd rom e, 62 Ativan. See Lorazep am Atropine, 220, 241 Attention, assessing, 222, 242–243 Attention-d eficit/ hyperactivity d isord er (ADH D) d ifferential d iagnosis, 223, 243–244 treatm ent of, 223, 244 behavioral therapy, 127 sym p tom s, 15 m ethylp henid ate for, 8, 26, 181, 201 m ania v., 10, 26 Autistic d isord er, 15 Autonom y, 144, 156–157 Avoid ant p ersonality d isord er, 184, 203 B Basal ganglia, 263, 282 BDI. See Beck Dep ression Inventory Beck Dep ression Inventory (BDI), 207, 227, 254, 275 Behavioral therap y, enu resis, 259, 278 Benad ryl. See Dip henhyd ram ine Bend er Gestalt Test, 254, 275 Benzod iazep ine ad verse effects, 110 alcohol w ithd raw al, 116 bip olar d isord er, 167, 187 DTs, 253, 274 high-p otency, 105 Benztrop ine (Cogentin) anticholinergic toxicity and , 211, 232 torticollis and , 264, 283 Bereavem ent, 249, 269–270 d ifferential d iagnosis, 219, 240 MDE v., 173, 192 treatm ent, 173, 192, 220, 240 Bipolar d isord er genetics, 180–181, 200 lam otrigine for, 257, 277 m anic, 167, 187, 260, 279 treatm ent of, 260, 279 Bipolar I d isord er, 210, 218, 230, 239, 250–251, 271 Bipolar II d isord er, 37, 40, 61, 64, 105, 165, 185, 214, 234, 250–251, 271 Bipolar m ania, 113, 277 Blessed Rating Scale, 228, 245, 255, 275 Bord erline p ersonality d isord er (BPD), 184, 203, 208, 212, 227, 232 DBT, 258, 278 d ialectical behavioral therap y and , 171, 191, 212, 232 d ifferential d iagnosis, 171, 191, 209, 229 treatm ent, 258, 278

291

292

Ind ex

Boston Diagnostic Aphasia Exam ination, 219, 224, 239, 244–245, 254, 275 BPD. See Bord erline p ersonality d isord er Brain activity in, 55 d am age, Klü ver-Bucy synd rom e and , 62 frontal lobes, 209, 229–230 receptors antip sychotics and , 113 clozap ine and , 88 d op am ine, 94 haloperid ol and , 94, 115 m ed ications and , 60 Brief psychotic d isord er, 59, 62, 66, 71, 172, 191–192 Brow n-Peterson Task, 207, 227 Bu limia nervosa ad olescent, 4, 20 CBT, 20 com p lications, 277 m ed ical com plications, 219, 239 m ed ications, 113 p sychotherap y, 20 signs, 20 sym ptom s, 31 treatm ent, 219, 239 Bu p rop ion, 6, 22, 26, 37, 262, 281 Bu Spar. See Bu sp irone Bu spirone (Bu Sp ar), 51, 75, 107, 109, 167, 169, 187, 216, 236 C Caffeine, w ithd raw al, 217, 237 Cannabis, intoxication, 266, 285 Capgras synd rom e, 33, 54, 58, 77, 248, 269 Card iac arrhythm ia, TCAs and , 169, 189 Card iac effects am itriptyline, 93, 113 lithiu m , 64, 71 Catalepsy, 74, 235 Catap lexy, 18, 51, 69, 74, 200 Catap res. See Clonid ine Catatonia, 215, 235–236 Charcot, Jean-Martin, 131, 140 Child abu se, 157 legal issu es, 12, 30, 144–145, 157, 203 PTSD and , 19 Child cu stod y, 152, 163 Child ren anxiety d isord ers, 19 bipolar d isord er in, 3, 11, 19, 26 clinical interview , 4 d evelopm ent, 15 MDD in, 5 treatm ent parental refu sal of, 145–146, 158 Chlorp rom azine, 262, 281 Citalopram , 247, 268 bulim ia nervosa, 219, 239 Clonid ine (Catapres), 59 ad verse effects, 11, 27 alcohol w ithd raw al, 189 opiate w ithd raw al, 265, 284 Tou rette d isord er, 6, 11, 22, 176, 195 Clozapine (Clozaril), 262, 281 agranulocytosis and , 209, 228 schizop hrenia, 222, 243 sid e effects, 208–209, 228, 267, 286 Clozaril. See Clozap ine Cocaine intoxication, 211, 224, 230–231, 245, 266, 285 w ithd raw al, 267, 285

Cod e of Ethics, AMA, 158, 159–160, 162–163 Cogentin. See Benztrop ine Cognitive-behavioral therap y (CBT), OCD, 174, 194 Cognitive therap y, d ep ressive d isord er, 179, 199 Cold -caloric–ind u ced nystagm u s, 213, 233 Com p etency, 155, 251–252, 272 Com p etency to stand trial, 143, 149–150, 156, 160 Com p liance, m ed ication, 210, 230 Cond u ct d isord er, 16, 259, 278 antisocial personality d isord er and , 13 Confid entiality, 148, 158, 252, 272–273 m inors and , 145, 157 p hysician–p atient confid entiality, 146, 158, 255, 275 violating, 145, 157 w aivers, 156, 158 Continu ou s p ositive airw ay pressu re (CPAP), 220, 241 Conversion d isord er, 174–175, 194, 211, 230, 251, 272 Coprop raxia, 249, 270 Cou ntertransference, 250, 270. See also Patient-d octor relationship CPAP. See Continuou s p ositive airw ay p ressu re Crim inal resp onsibility, 142, 156 Cu ltu re-bou nd d elu sions, 77 Cu ltu re-bou nd synd rom es, 211, 231 Cyclothym ic d isord er, 43, 61 Cyp rohep tad ine, 103, 107 D DBT. See Dialectical behavioral therap y Dd avp . See Desm op ressin Defense m echanism s, 11, 12, 35, 59, 70, 80. See also specific defense mechanisms Déjà entend u , 223, 244 Delirium , 262, 281 d ifferential d iagnosis, 172, 177, 178, 190–191, 197, 198 m ed ications, 171, 191 Delirium trem ens (DTs), treatm ent, 253, 274 Delusional d isord er p resentation, 166, 186 treatm ent, 166, 186 Delusions Fregoli, 33 MDD, 180, 200 m ood congru ent, 180, 200 p aranoid , 142, 155 Dem entia d ifferential d iagnosis, 209, 217–218, 228–229, 238 Folstein MMSE, 209, 228–229 Lew y bod y, 178, 198 Dem erol. See Mep erid ine Denial, 213, 233 Dep akote. See Divalp roex sod iu m Dep end ent p ersonality d isord er, 183–184, 203 Dep ressed reflexes, 222, 243 Depression alcohol abu se and , 179, 199 electroconvu lsive therapy for, 214, 235 factitiou s d isord er by p roxy and , 208, 227 m ed ical cau ses of, 258, 277 w ith psychotic featu res, 266, 285

Depressive d isord ers d ifferential d iagnosis, 179, 199 m ed ically ill p atients w ith, 222, 242 treatm ent, 179, 199, 248, 269 Desm op ressin (Dd avp), 259, 278 Desyrel. See Trazod one Dialectical behavioral therap y (DBT) BPD and , 171, 212, 232, 258, 278 Diazep am (Valiu m ), 251, 272 p ostp artu m d epression, 185 Differential d iagnosis, 165–203 Diphenhyd ram ine (Benad ryl) confu sion and , 261, 280–281 insom nia, 181, 201 Discharging p atient, 149, 160 from care, 152, 162–163 Disulfiram , alcohol abu se and , 221, 242 Divalp roex sod iu m , sid e effect, 222, 243 Divalp roex sod iu m (Dep akote), bip olar d isord er, 260, 279 DTs. See Deliriu m trem ens Du loxetine, 119 Dusky v. United States, 143, 156, 160 E Echolalia, 249, 270 ECT. See Electroconvu lsive therap y Effexor. See Venlafaxine Ego-d ystonic, 249, 270 Ego-syntonic, 249, 270 Electroconvu lsive therap y (ECT) am nesia, 43, 67 d ep ression, 36, 60, 214, 235 p regnancy and , 94, 115 seizure length and , 265, 284 seizures, 266, 284 sid e effects, 284 Enu resis com orbid m ental illness, 8, 24 Dd AVP for, 24 m ed ication, 8, 24 second ary noctu rnal, 259, 278 treatm ent, 259, 278 Ep ilep sy com p lex partial, 263, 282 d ifferential d iagnosis, 176, 195–196 Erikson, Erik, 263, 282 Ethical issu es, 141–163 Extrap yram id al sid e effects (EPS) benztrop ine and , 211, 231 F Factitiou s d isord er d ifferential d iagnosis, 175, 194–195 by proxy, 13, 208, 227 (See also Child abu se) Fetishistic d isord er, 260, 279 Fire setting, 259, 278 Flat affect, 206, 226 Flu oxetine (Prozac), 247, 268 d ose, 255, 275–276 insom nia and , 172–173, 192 MDD, 172, 192, 256, 266, 276, 285 p ostp artu m d epression, 166, 185 p regnancy and , 256, 276 and risp erid one, 266, 285 sid e effects, 173, 192 Flu voxam ine (Lu vox) MDD, 221, 241 OCD, 221, 241

Ind ex Fregoli d elusion, 33 Freu d , Sigm u nd , 140 Frotteurism , 169, 188 G Galantam ine, 253–254, 274 Ganser synd rom e, 214, 234–235, 249, 270 Gend er, su icid e and , 17 Generalized anxiety d isord er (GAD) buspirone for, 216, 236 d ifferential d iagnosis, 174, 179, 194, 198–199 Genetics bip olar d isord er, 180–181, 200 schizop hrenia, 210, 230 Geod on. See Ziprasid one Gingival hyperp lasia, 210, 230 Glasgow Com a Scale, 264, 283 H H abeas corp u s, 151, 162 H allu cinations alcohol w ithd raw al and , 221, 242 com m and au d itory, 167, 181, 187, 200 lorazepam for, 221, 242 H aloperid ol (H ald ol), 205, 225, 262, 281 d elirium , 171, 191 d op am ine recep tors and , 94, 115 forcing treatm ent, 144, 156–157 lithiu m and , 168, 187 PCP intoxication, 285 postpartum d ep ression, 165, 185 pregnancy and , 223–224, 244 torticollis and , 264, 283 H ead ache, 217, 237 H em iballism u s, 250, 271 H epatic encephalopathy, 177, 196–197 H eroin d isord er of, 64 intoxication, 266, 285 overd ose, 46, 70 w ithd raw al, 267, 285 H istrionic personality d isord er, 184, 203 H oard ing d isord er, 182, 202 H om icid al threats, confid entiality and , 141, 154 H osp italization involuntary, 143–144, 156 p ostpartum d ep ression, 147–148, 159 partial hosp ital program s, 151, 162 H uffing, 222, 243 H um an im m unod eficiency viru s (H IV) d ifferential d iagnosis, 177, 197 inform ed consent and testing for, 145, 157–158 H untington d isease, 178, 198 H yperp lasia, gingival, 210, 230 H yperreligiou s thinking, 263, 282 H ypertensive crisis, 208, 216, 228, 236 H yperthyroid ism , 177, 197 H ypnotherap y, 123, 132 H ypoglycem ia, 177, 197 I Ibup rofen, 221, 242 Id entification, 215, 235 Im ipram ine (Tofranil) contraind ications, 265, 284 Inform ed consent, 143, 156 cap acity to give, 151–152, 162 H IV testing, 145, 157–158 obtaining, 151, 162

Inhalant, abu se, 222, 243 Inhalant intoxication, 266, 285 Inp atient facility, ad m ittance, 207, 226–227 Insanity, 144, 157 Insanity d efense, 142, 144, 155–156, 157 voluntary intoxication and , 150–151, 161 Insom nia, SSRIs and , 173, 192 Insu rance, 149, 160 Intellectu al d isability, 259, 278–279 Intelligence qu otient (IQ), calcu lation, 214, 234 Interm ittent exp losive d isord er, 183, 202–203 IQ. See Intelligence qu otient Iron d eficiency, restless legs synd rom e and , 183, 202 J Jehovah’s Witness, 146, 158 K Kayser–Fleischer rings, 250, 270–271 Klep tom ania, 249, 270 Korsakoff synd rom e, 180, 199–200 L La belle ind ifférence, 251, 272 Lam otrigine, 262, 281 bipolar d ep ression, 215, 236 bipolar d isord er, 257, 277 sid e effects, 257, 277 Learning d isord ers d ifferential d iagnosis, 214, 234 Legal issu es, 141–163 child abu se, 144, 157 p aranoid d elu sions, 142, 155 w ills, 142, 155 Lew y bod y d em entia, 178, 198 Lithiu m (Eskalith), 205, 225, 262, 281 haloperid ol and , 168, 187 ibup rofen, 221, 242 laboratory tests before treatm ent w ith, 218, 238 nep hrogenic d iabetes insip id u s, 264–265, 283–284 sid e effects, 267, 286 toxicity, 265, 284 Living w ill, 151–152, 162 Lorazep am (Ativan) agitation and , 266, 284–285 for alcohol u se d isord er, 207, 226 alcohol w ithd raw al, 170, 179, 189, 199, 221, 241–242 for hallucinations, 221, 242 Lu vox. See Flu voxam ine M Magical thinking, 250, 271 Major d ep ressive d isord er (MDD), 208, 227 w ith atyp ical featu res, 214–215, 235, 260, 280 bereavem ent v., 173, 192 d elu sions, 180, 200 d ifferential d iagnosis, 168, 188, 209, 229 ECT for, 214, 235 m irtazap ine for, 221–222, 242 OCD w ith, 221, 241 w ith p sychotic featu res, 171–172, 191 recu rrent ep isod es of, 256, 276 w ith seasonal pattern, 213, 233–234 treatm ent of, 172, 191, 255, 256, 275–276

293

Malingering, 152, 162, 252, 273 conscious p rod u ction of sym ptom s, 252, 273 d ifferential d iagnosis, 220, 240 Malp ractice harm or d am age in, 141, 154 p hysician’s p rotection against, 142, 155 “stand ard of care,” 142, 145 su icid e and , 150, 161 Managem ent, 165–203 Mariju ana, 224, 245 Med ical exp ert w itnesses, 151, 161–162 Mellaril. See Thiorid azine Mens rea, 142, 156 Meperid ine (Dem erol), 216, 236 Methad one, op iate w ithd raw al, 265, 284 Metham phetam ine intoxication, 266, 285 3,4-Methylened ioxym etham p hetam ine (MDMA) (Ecstasy) intoxication, 266, 285 Methylp henid ate (Ritalin) ADH D, 181, 201, 223, 244 d ep ressive d isord er, 222, 242 Microcep haly, 259, 279 Mini-m ental state exam ination (MMSE), 209, 224, 228–229, 244, 254, 255, 275 Minnesota Multip hasic Personality Inventory 2 (MMPI-2), 212, 233, 255, 275 Minors, confid entiality and , 145, 157 Mirtazap ine, 247, 268 MDD, 221–222, 242 MMPI-2. See Minnesota Mu ltip hasic Personality Inventory 2 MMSE. See Mini-m ental state exam ination Monoam ine oxid ase inhibitor (MAOI) MDD, w ith atyp ical featu res, 261, 280 p regnancy and , 216, 237 SSRIs and changing to, 264, 283 tyram ine and , 264, 283 N N aloxone opiate overd ose, 253, 274 su bstance d ep end ence and , 217, 238 N arcissistic p ersonality d isord er, 184, 203 N arcolepsy, d ifferential d iagnosis, 181, 200–201 N eologism s, 248, 269 N eu rolep tic m alignant synd rom e (N MS), 217, 237 characteristics, 166–167, 186–187 laboratory abnorm alities, 217, 237–238 treatm ent, 167, 187 N eu rosyp hilis, 176, 196 N icotine, intoxication, 224, 245 N igrostriatal D 2 recep tors, 258, 278 N itrous oxid e intoxication, 266, 285 N ortriptyline, 262, 281 N ot gu ilty by reason of insanity, 144, 157 O Obsessive-com p u lsive d isord er (OCD) antid ep ressants and , 220, 240 d ifferential d iagnosis, 174, 193–194 MDD w ith, 221, 241 treatm ent, 174, 194 Obsessive-com p u lsive p ersonality d isord er, 184, 203, 205, 225 Obstru ctive sleep ap nea hyp opnea, 220, 240–241

294

Ind ex

OCD. See Obsessive-com pu lsive d isord er Ocu logyric crisis, 168, 188 Olanzap ine (Zyp rexa) d elu sional d isord ers, 166, 186 hallucinations, 167, 187 Opiates. See also specific opiates intoxication, 170, 190, 224, 245 overd ose, treatm ent of, 253, 274 w ithd raw al, 170, 181–182, 190, 201 Opp ositional d efiant d isord er (ODD), 73 P Panic attacks d ifferential d iagnosis, 168–169, 188 p hysiologic pathology, 248, 268–269 Panic d isord er w ith agoraphobia d ifferential d iagnosis of, 173–174, 193 treatm ent of, 174, 193 d ifferential d iagnosis, 208, 227–228 sertraline, 207, 227 treatm ent of, 208, 228 Paranoid d elu sions, 142, 155 Paranoid personality d isord er, 184, 203 Paroxetine (Paxil), 247, 262, 268, 281 cognitive d ecline and , 265, 284 Partial hospital p rogram s, 151, 162 Patient-d octor relationship, 216–217, 237 p aym ent and , 150, 161, 252, 273 sexual, 148, 159 Paxil. See Paroxetine Personality d isord ers, 183, 203, 261, 280. See also specific personality disorders Phencyclid ine (PCP) intoxication, 224, 245, 267, 285 supportive care, 266, 285 treatm ent for, 266, 285 Pheochrom ocytom a, 176–177, 196 Physician-assisted su icid e, 148, 160, 254, 274 Physician–p atient confid entiality, 146, 158, 252, 255, 272–273, 275 Pick d isease. See also Dem entia d ifferential d iagnosis, 178, 198 Postp artu m d epression involuntary com m itm ent and , 147–148, 159 w ith p sychosis, 165–166, 185 treatm ent, 166, 185 Postp artu m psychosis, 165–166, 185 Posttraum atic stress d isord er (PTSD) treatm ent, 263, 282 Pow er of attorney, 147, 159 Prazosin, 263, 282 Pregnancy fluoxetine and , 256, 276 haloperid ol and , 223–224, 244 MAOI and , 216, 237 Priapism , 215–216, 236, 256, 276 Prion (Creu tzfeld t–Jacob) d isease, 198 Pseud od em entia, 178, 198. See also Dem entia Psilocybin w ithd raw al, 266, 285 Psychiatric illnesses, 205, 225 Psychogenic unresponsiveness, 213, 233 Psychosis, postpartum , 165–166, 185 Psychotherapy. See also specific psychotherapies ad ju stm ent d isord er, 173, 193 Psychotic d isord er, brief, 172, 191–192 Psychotic featu res, MDD w ith, 171, 191 treatm ent of, 172, 192

Q Qu etiapine (Seroqu el), 211–212, 232 p ostp artu m d ep ression, 166, 185 R Racing thou ghts, 210, 230 Rand om letter test, 252–253, 273 Reactive attachm ent d isord er, 259–260, 279 Refu sal of treatm ent, 143, 147, 156 child ren and , 145–146, 158 Jehovah’s Witness and , 146, 158 Rep orting im p aired colleagu es, 152–153, 163 Restless legs synd rom e d ifferential d iagnosis, 183, 202 treatm ent, 183, 202 Retinitis p igm entosa, 264, 283 Rey–Osterrieth Test, 218, 224, 238, 244, 254, 275 Risp erid one, 262, 281 and flu oxetine, 266, 285 Risp erid one Consta, 262, 281 Risp erid one (Risperd al) MDD w ith p sychotic featu res, 172, 191 Tourette d isord er, 259, 278 Rorschach Test, 210, 212, 230, 232, 255, 275 S Schizoaffective d isord er d iagnosis for, 261, 280 m ed ications, 165, 185 Schizoid p ersonality d isord er, 180, 184, 200, 203 Schizop hrenia flat affect and , 206, 226 genetics, 210, 230 negative sym p tom , 206, 226 risk of, 210, 230 treatm ent, 222, 243, 248, 269 Schizophreniform d isord er d ifferential d iagnosis, 169, 189, 210, 223, 230, 244 Schizotyp al p ersonality d isord er, 184, 203 d ifferential d iagnosis, 206, 209, 225–226, 229 treatm ent, 206, 226 Seizures alcohol w ithd raw al and , 170, 189 ECT, 266, 284 tem p oral lobe, 263, 283 Selective m u tism , 13–15 Serotonin-sp ecific reu p take inhibitor (SSRI), 167, 187 insom nia and , 173, 192 MAOIs and changing from , 264, 283 sexu al d ysfu nction and , 169, 188, 222–223, 243 sid e effects, 169, 188 su icid e and , 257, 276–277 Serotonin synd rom e, 218, 239 d ifferential d iagnosis, 182, 201–202 m anagem ent of, 182, 202 Sertraline (Zoloft) MDD w ith p sychotic featu res, 172, 191 p anic d isord er, 207, 227 w ith agorap hobia, 174, 193 for p anic d isord er, 208, 228 schizoaffective d isord er, 165, 185 Seru m liver fu nction stu d ies, 217, 238

Sexu al abu se, 273–274 Sexu al d ysfu nction, SSRIs and , 169, 188, 222–223, 243 SLE. See System ic lu pu s erythem atosus Sleep ap nea, 220, 240–241 Som atic sym p tom d isord er, 177, 197 Sonata. See Zalep lon SSRI. See Serotonin-specific reuptake inhibitor St. John w ort, 257, 277 Steroid -ind u ced p sychotic d isord er d ifferential d iagnosis, 182, 201 m anagem ent of, 182, 201 Stevens–Johnson synd rom e, 257, 277 Stress from w ork, 206, 226 Stroke, visual nonverbal m em ory and , 218, 238 Stroop Test, 254, 275 Subp oena to release m ed ical inform ation, 149, 160 Substance intoxication, insanity d efence and , 150–151, 161 Suicid e. See also Physician-assisted su icid e m alp ractice and , 150, 161 risk factors, 150, 161, 254, 258, 274, 277 SSRI and , 257, 276–277 Supportive care, for PCP intoxication, 266, 285 Sym m etrel. See Am antad ine Systemic lupus erythematosus (SLE), 177, 197 T Tarasoff v. Regents of University of California, 141, 154 Tard ive d yskinesia (TD), 262–263, 281–282 TD. See Tard ive d yskinesia Tem p oral Orientation Test, 211, 231 Testam entary cap acity, 142, 155, 251, 272 Test of Mem ory Malingering, 220, 240 Thiam ine, Wernicke encep halopathy and , 166, 180, 186, 199, 216, 237, 253, 274 Thiorid azine (Mellaril), 264, 283 Thou ght broad casting, 250, 271 Thyroid d isord er, 210, 230 Tofranil. See Im ip ram ine Torticollis, 264, 283 Tou rette d isord er, 259, 278 cop rop raxia, 249, 270 echolalia and , 249, 270 treatm ent, 176, 195 Toxicology screen, 170, 189–190 brief p sychotic d isord er and , 172, 192 u rine, 256, 276 Trazod one (Desyrel), 256, 276 p riap ism and , 215, 236, 256–257, 276 Triazolam (H alcion), 170, 190 Tricyclic antid ep ressants (TCAs) card iac arrhythm ia and , 169, 189 Type II error, 262, 281 V Valp roic acid , 262, 281 bip olar I d isord er, 251, 271 Venlafaxine (Effexor), 247, 262, 268, 281 W Wad a Test, 255, 275 WAIS-R. See Wechsler Ad u lt Intelligence Scale—Revised WCST. See Wisconsin Card Sorting Test Wechsler Ad ult Intelligence Scale—Revised (WAIS-R), 214, 234, 255, 275 Wechsler Intelligence Scale for Child ren (WISC), 175–176, 195, 224, 244

Ind ex Weight gain, 247, 268 Wernicke encephalop athy, 166, 186 m anagem ent, 166, 186 thiam ine and , 166, 180, 186, 199, 216, 237, 253, 274 Wernicke–Korsakoff synd rom e, 65 Wills legality, 142, 155 living w ill, 151–152, 162 Wilson d isease, 250, 270–271

WISC. See Wechsler Intelligence Scale for Child ren Wisconsin Card Sorting Test (WCST), 209, 229, 254, 275 Withd raw al alcohol, 267, 285 caffeine, 217, 237 cocaine, 267, 285 heroin, 267, 285 opiates, 170, 181–182, 190, 201

Z Zalep lon (Sonata), 216, 236 Zip rasid one (Geod on) schizoaffective d isord er, 165, 185 w eight gain and , 247, 268 Zoloft. See Sertraline Zolp id em (Am bien), 181, 201, 256, 276

295